Step 2 Cases

November 25, 2017 | Author: Sergio Villa | Category: Aorta, Diseases And Disorders, Clinical Medicine, Medical Specialties, Wellness
Share Embed Donate


Short Description

Download Step 2 Cases...

Description

A 36-year-old man presents to his physician complaining of right scrotal swelling. He states that the swelling has been present for 1 week. He initially noticed the swelling shortly after moving furniture for his new living room. He denies any nausea, vomiting, change in bowel habits, abdominal pain, or urinary tract symptoms. He has no other significant medical or surgical history. On examination, he has an enlarged right hemi-scrotum with a mass that appears to be originating at the level of the external inguinal ring. With the patient completely relaxed, the physician is able to reduce the mass by pushing it back through the external inguinal ring. With the mass reduced, the physician instructs the patient to perform a Valsalva maneuver, upon which a protrusion is felt at the external inguinal ring. Once the mass is reduced, the testicle appears normal in size and consistency. Question 1 of 4

Which of the following is the most likely diagnosis? / A. Hydrocele / B. Femoral hernia / C. Inguinal hernia / D. Testicular cancer / E. Varicocele Explanation - Q: 1.1

Close

The correct answer is C. This patient has a reducible inguinal hernia. A hernia is a protrusion of a structure, usually intestine, through tissue that normally contains it. Inguinal hernias are either direct or indirect. Indirect inguinal hernias occur through the internal inguinal ring in a protrusion of peritoneum along the spermatic cord in the internal spermatic fascia. Direct inguinal hernias occur through the floor of the inguinal canal, separate from the spermatic cord as a result of breakdown of the transversus abdominis aponeurosis and transversalis fascia. If the mass, i.e., hernia, is easily returned back to its normal position, then it is called reducible. If the mass is not reducible, then this is called incarcerated. And, if the mass becomes incarcerated and develops compromised blood supply, it is termed strangulated. Strangulated hernias require emergent repair because the intestinal contents will necrose and cause the patient to become sick. As long as the mass is reducible, surgical repair can be performed on an outpatient basis. A hydrocele (choice A) is a fluid collection contained within the tunica vaginalis that surrounds the testicle. It presents as a painless swelling of the scrotum, which transilluminates light when it is placed against the mass. A hydrocele cannot be reduced. Femoral hernias (choice B) are more common in women, presenting as swellings in the upper part of the thigh. The neck of the hernia sac lies at the

femoral ring, below and lateral to the pubic tubercle, distinguishing these from inguinal hernias, which are above and medial to the tubercle. A mass caused by testicular cancer (choice D) cannot be reduced either. Patients will complain of a dull, heavy, aching feeling within the testicle. The testicle itself is usually enlarged, hard, irregular, and nontender. A varicocele(choice E) is caused by dilatation of the pampiniform venous plexus of the spermatic cord. It is typically described as feeling like "a bag of worms" on physical examination. The dilatation is more easily appreciated with the patient standing or during the Valsalva maneuver. A 36-year-old man presents to his physician complaining of right scrotal swelling. He states that the swelling has been present for 1 week. He initially noticed the swelling shortly after moving furniture for his new living room. He denies any nausea, vomiting, change in bowel habits, abdominal pain, or urinary tract symptoms. He has no other significant medical or surgical history. On examination, he has an enlarged right hemi-scrotum with a mass that appears to be originating at the level of the external inguinal ring. With the patient completely relaxed, the physician is able to reduce the mass by pushing it back through the external inguinal ring. With the mass reduced, the physician instructs the patient to perform a Valsalva maneuver, upon which a protrusion is felt at the external inguinal ring. Once the mass is reduced, the testicle appears normal in size and consistency. Question 2 of 4

Which of the following nerves travels along the spermatic cord within the inguinal canal and may be damaged during a surgical procedure to correct this patient's condition? / A. IIiohypogastric / B. IIioinguinal / C. Lateral femoral cutaneous / D. Obturator / E. Pudendal Explanation - Q: 1.2

Close

The correct answer is B. Although it is now possible to perform inguinal hernia repairs laparoscopically, the traditional approach is through an incision over the inguinal canal. At the time of surgery, the inguinal canal is opened via sharp dissection through the external oblique aponeurosis (the anterior wall of the inguinal canal). The ilioinguinal nerve is then encountered as it runs on the anterior aspect of the spermatic cord. If not properly identified at time of surgery it is possible to transect the ilioinguinal nerve or to "trap" it during closure. If the ilioinguinal nerve is transected or entrapped in closure, the patient will complain of numbness over the nerve's

distribution, i.e., the upper medial aspect of the thigh and the anterior portion of the scrotum on the affected side. The iliohypogastric, lateral femoral cutaneous, obturator, and pudendal nerves are not encountered during traditional inguinal hernia repair. The iliohypogastric nerve(choice A) is derived from L1 (as is the ilioinguinal nerve) and runs with the ilioinguinal nerve as they both pierce the transversus abdominis muscle near the anterior superior iliac spine. They then pass through the internal and external oblique muscles to supply the skin of the suprapubic and inguinal regions and the abdominal musculature. The iliohypogastric sends a lateral branch to the skin of the gluteal region and then continues on, to pass through the superficial inguinal ring. The lateral femoral cutaneous nerve (choice C) originates from L2 and L3 and is a direct branch of the lumbar plexus. It enters the thigh deep to the lateral end of the inguinal ligament near the anterior superior iliac spine and supplies the skin on the anterior and lateral aspects of the thigh. The obturator nerve(choice D) is the nerve of the adductor muscles of the thigh. It arises from the lumbar plexus (L2, L3, L4), enters the pelvis minor, and then leaves the pelvis via the obturator foramen. The obturator nerve also sends a small cutaneous branch to the medial aspect of the mid thigh. The pudendal nerve (choice E) arises from the sacral plexus (S2, S3, S4), accompanies the internal pudendal artery, and leaves the pelvis between the piriformis and coccygeus muscles. The nerve hooks around the sacrospinous ligament to enter the perineum through the lesser sciatic foramen to supply the muscles of the perineum, including the external anal sphincter, and then ends as the dorsal nerve of the penis or clitoris. It also supplies some sensation to the external genitalia A 36-year-old man presents to his physician complaining of right scrotal swelling. He states that the swelling has been present for 1 week. He initially noticed the swelling shortly after moving furniture for his new living room. He denies any nausea, vomiting, change in bowel habits, abdominal pain, or urinary tract symptoms. He has no other significant medical or surgical history. On examination, he has an enlarged right hemi-scrotum with a mass that appears to be originating at the level of the external inguinal ring. With the patient completely relaxed, the physician is able to reduce the mass by pushing it back through the external inguinal ring. With the mass reduced, the physician instructs the patient to perform a Valsalva maneuver, upon which a protrusion is felt at the external inguinal ring. Once the mass is reduced, the testicle appears normal in size and consistency. Question 3 of 4

If a segment of terminal ileum becomes strangulated as a consequence of his condition, it may become infarcted and necrotic due to occlusion of a branch of which of the following vessels? / A. Celiac trunk / B. Inferior mesenteric artery / C. Middle colic artery / D. Right colic artery / E. Superior mesenteric artery Close Explanation - Q: 1.3 The correct answer is E. The small bowel is a derivative of the midgut and therefore receives its blood supply from the superior mesenteric artery. This artery emerges from the aorta 1 cm below the celiac trunk and passes ventral to the left renal vein to give off 12 to 15 jejunal and ileal arteries. As these arteries divide, they join with an adjacent branch to form arches. These arches may then communicate to form an arcade. Straight arteries also emerge from these arches to supply the bowel. Although there are variants, the ileal artery is usually a branch of the superior mesenteric artery, which supplies a branch to the terminal ileum. The celiac trunk (choice A) provides branches to supply the stomach, liver, pancreas, and duodenum. It is not responsible for supplying blood to the terminal ileum. The inferior mesenteric artery (choice B) provides branches that supply a limited part of the transverse colon near the splenic flexure and the descending and sigmoid colon. The middle colic artery (choice C) is a branch of the superior mesenteric artery. There are two branches, right and left. The right branch supplies the right half of the transverse colon and the left branch supplies the left half of the transverse colon. The right colic artery (choice D) is also a branch of the superior mesenteric artery. It is responsible for supplying the hepatic flexure as well as that part of the ascending colon not supplied by the ileocolic artery. A 36-year-old man presents to his physician complaining of right scrotal swelling. He states that the swelling has been present for 1 week. He initially noticed the swelling shortly after moving furniture for his new living room. He denies any nausea, vomiting, change in bowel habits, abdominal pain, or urinary tract symptoms. He has no other significant medical or surgical history. On examination, he has an enlarged right hemi-scrotum with a mass that appears to be originating at the level of the external inguinal ring. With the patient completely relaxed, the physician is able to reduce the mass by pushing it back through the external inguinal ring. With the mass reduced, the physician instructs the

patient to perform a Valsalva maneuver, upon which a protrusion is felt at the external inguinal ring. Once the mass is reduced, the testicle appears normal in size and consistency. Question 4 of 4

Which of the following pathological processes might cause the patient's underlying condition to occur in an infant? / A. Defect in the floor of the inguinal canal / B. Defect in the internal inguinal ring / C. Defect in the linea semilunaris / D. Patent processus vaginalis / E. Persistent lumen of the tunica vaginalis Explanation - Q: 1.4

Close

The correct answer is D. The pathologic process that causes hernias is different in neonates and infants than in an adult. Embryologically, in the seventh week, the testes begin descending from their location at the 10th thoracic level into the scrotum. After the eighth week, a peritoneal evagination called the processus vaginalis forms just anterior to the gubernaculum. The gubernaculum is a condensation of peritoneum that attaches superiorly to the gonad and inferiorly to the fascia that is developing between the external and internal oblique muscles in the region of the labioscrotal swellings. The processus vaginalis pushes out as a "sock-like" extension into the transversalis fascia, the internal oblique muscle, and the external oblique muscle, thus forming the inguinal canal. After the processus vaginalis has evaginated into the scrotum, the gubernacula shorten and simply pull the gonads through the canal. Within the first year after birth, the superior portion of the processus vaginalis is usually obliterated, leaving only a distal remnant sac, the tunica vaginalis, which lies anterior to the testis. During infancy, this sac wraps around most of the testis. Its lumen is normally collapsed, but under pathologic conditions it may fill with serous secretions, forming a testicular hydrocele (choice E). If the processus vaginalis remains patent, a connection between the abdominal cavity and scrotal sac will occur. Loops of intestine may herniate into this processus, resulting in an indirect inguinal hernia. In adults, hernias are caused by a protrusion of a structure, usually intestine, through a tissue that normally contains it. Direct inguinal hernias occur through the floor of the inguinal canal separate from the spermatic cord (choice A). Direct inguinal hernias occur because of a breakdown of the transversus abdominis aponeurosis and transversalis fascia. An indirect inguinal hernia occurs through the internal inguinal ring in a protrusion of peritoneum along the spermatic cord in the internal spermatic fascia. Therefore, a large indirect inguinal hernia will descend into the scrotum along the spermatic cord (choice B).

Spigelian hernias are rare and occur due to a weakness in the linea semilunaris (choice C), which is located at the lateral margin of the rectus sheath. A 2-year-old child is seen for a welI-child visit in a pediatric clinic. Abdominal examination demonstrates a palpable, non-tender mass on the Ieft side of the abdomen. The mother had no idea the mass was present and the pediatrician did not note the presence of the mass at the child's 18-month welI-child visit. Physical examination is otherwise unremarkable. Question 1 of 5

CT examination demonstrates an 8-cm diameter, roughly round, mass involving the lower pole of the kidney. Which of the following is the most likely cause of this mass? / A. Ewing sarcoma / B. Neuroblastoma / C. Renal cell carcinoma / D. Transitional cell carcinoma / E. Wilms tumor Explanation - Q: 2.1

Close

The correct answer is E. Wilms tumor is an embryonal malignancy of the kidney that most commonly affects children from birth to age 4, but can affect children up to about age 15. This tumor is the second most common extracranial solid tumor in children and tends to form large, round, solitary masses of the kidney, although bilaterality and multicentricity may also occur. Wilms tumors usually present with a palpable abdominal mass, with less common presentations including abdominal pain, hematuria (indicating invasion of the collecting system), hypertension, fever, nausea, and vomiting. Ewing sarcoma (choice A) most commonly involves the long bones of older children and young adolescents. Neuroblastoma (choice B) may also effect very young children, and typically arises in the adrenal gland. Extensions into the kidney can occur, but usually involve the upper pole first. Renal cell carcinoma (choice C) and transitional cell carcinoma (choice D) would be much more likely to involve the kidney of an adult. A 2-year-old child is seen for a welI-child visit in a pediatric clinic. Abdominal examination demonstrates a palpable, non-tender mass on the Ieft side of the abdomen. The mother had no idea the mass was present and the pediatrician did not note the presence of the mass at the child's 18-month welI-child visit. Physical examination is otherwise unremarkable. Question 2 of 5

If a CT guided biopsy of the mass were performed, which of the following histological patterns would be most suggestive of the likely

diagnosis? / A. Cords of clear cells with rounded or polygonal shape and abundant clear cytoplasm / B. Invasive papillary lesions with delicate connective tissue stalk covered with epithelium resembling that lining the bladder / C. Small dark cells embedded in a finely fibrillar matrix with formation of numerous rosettes / D. Triphasic pattern with tubule formation, spindle cells, and blastemal elements / E. Uniform sheets of small round cells with scanty cytoplasm Explanation - Q: 2.2

Close

The correct answer is D. Adequate sampling of most Wilms tumors will detect the three distinct growth patterns that are most characteristic of this tumor: less cellular tumor areas containing spindle cells; densely cellular (blastemal) areas with closely packed small cells with scanty cytoplasm and darkly blue nuclei; and areas with production of more mature-appearing epithelium that may form occasional tubules. The tissues present are similar to those present in the developing kidney, and may also include primitive glomeruli. Wilms tumors may additionally contain heterologous elements, such as skeletal muscle, smooth muscle, squamous or mucinous epithelium, adipose tissue, cartilage, bone, or neural tissue. Choice A suggests renal cell carcinoma. Choice B suggests transitional cell carcinoma. Choice C suggests neuroblastoma. Choice E suggests Ewing sarcoma; neuroblastoma and some lymphomas may also sometimes have a similar appearance.

A 2-year-old child is seen for a welI-child visit in a pediatric clinic. Abdominal examination demonstrates a palpable, non-tender mass on the Ieft side of the abdomen. The mother had no idea the mass was present and the pediatrician did not note the presence of the mass at the child's 18-month welI-child visit. Physical examination is otherwise unremarkable. Question 4 of 5

Sometimes this child's disease occurs in association with certain congenital anomalies. Children with which of the following abnormalities at birth should be monitored for the development of this disease? / A. Aniridia / B. Coarctation of the aorta / C. Cystic hygroma / D. Parathyroid hyperplasia / E. Simian crease Explanation - Q: 2.4

Close

The correct answer is A. The WAGR syndrome includes Wilms tumor, aniridia (lack or defect of the iris), genital anomalies (gonadal dysgenesis, hypospadias, cryptorchidism, or other genitourinary anomalies), and mental retardation. Wilms tumor can also occur as part of the Beckwith-Wiedemann syndrome with hemihypertrophy, and the Drash syndrome with nephropathy and ambiguous genitalia. Coarctation of the aorta (choice B) and cystic hygroma (choice C) are associated with Turner syndrome, which may predispose for later development of ovarian cancer. Parathyroid hyperplasia (choice D) can be a component of multiple endocrine neoplasia (MEN). Simian crease (choice E) suggests Down syndrome, which is associated with an increased incidence of leukemia. A 2-year-old child is seen for a welI-child visit in a pediatric clinic. Abdominal examination demonstrates a palpable, non-tender mass on the Ieft side of the abdomen. The mother had no idea the mass was present and the pediatrician did not note the presence of the mass at the child's 18-month welI-child visit. Physical examination is otherwise unremarkable. Question 5 of 5

Abnormalities of which of the following chromosomes have been linked to this disease? / A. 3 / B. 5 / C. 11 / D. 15 / E. 21 Explanation - Q: 2.5

Close

The correct answer is C. Inactivation of the WT-1 Wilms tumor gene (located at 11p13, and thought to encode a DNA-binding protein important in fetal kidney development) has been reported in the tumor cells from many

Wilms tumor cases. Also, the WAGR syndrome involves a deletion of chromosome 11p13 and the Beckwith-Wiedemann syndrome involves a rearrangement of chromosome 11p15. The genetic basis of the Drash syndrome has not been established. The other choices are distracters.

A 67-year-old man with an 18-year history of type 2 diabetes mellitus presents for a routine physical examination. His temperature is 36.9 C (98.5 F), his blood pressure is 158/98 mm Hg and his pulse is 82/minute and regular. On examination, the physician notes a nontender, pulsatile, mass in the mid-abdomen. A plain abdominal x-ray film with the patient in the lateral position reveals spotty calcification of a markedly dilated abdominal aortic walI. Question 1 of 5

Which of the following is most likely pathogenetically related to this patient's aortic disease? / A. Atherosclerosis / B. Cystic medial necrosis / C. Kawasaki disease / D. Mönckeberg arteriosclerosis / E. Syphilis

Explanation - Q: 3.1

Close

The correct answer is A. This patient has an abdominal aortic aneurysm (AAA); these are most commonly the result of atherosclerosis. Hypertension and cigarette smoking are also risk factors. AAA may be asymptomatic, or may be associated with pain. Some aortic aneurysms are palpable as pulsating masses in the abdomen. Abdominal aortic aneurysms typically begin below the renal arteries and may extend well into the iliac system bilaterally. Calcified atherosclerotic plaques on plain films of the abdomen can be seen in the majority of cases. Rupture of an aortic aneurysm is usually preceded by excruciating pain in the lower abdomen and back. The mortality rate for abdominal aortic aneurysm repair performed electively is about 2-5%, while that performed emergently after rupture of the aneurysm has begun is 50%. Many patients with atherosclerotic aneurysms also have severe coronary artery disease, and repair of the coronary arteries may be needed before the patient is subjected to the risk of aortic aneurysm repair. Cystic medial necrosis (choice B) is a risk factor for dissecting aneurysms that typically do not widen the lumen of the aorta. Kawasaki disease (choice C) can cause small aneurysms of the coronary

arteries. Mönckeberg's arteriosclerosis (choice D) can cause focal calcification of (usually medium-sized) arteries, but is not associated with aneurysm formation. Syphilis (choice E) is a now rare cause of aortic aneurysms that typically involve the aortic root and ascending aorta.

A 67-year-old man with an 18-year history of type 2 diabetes mellitus presents for a routine physical examination. His temperature is 36.9 C (98.5 F), his blood pressure is 158/98 mm Hg and his pulse is 82/minute and regular. On examination, the physician notes a nontender, pulsatile, mass in the mid-abdomen. A plain abdominal x-ray film with the patient in the lateral position reveals spotty calcification of a markedly dilated abdominal aortic walI. Question 2 of 5

Which of the following physiologic observations helps to account for the fact that 75% of the aneurysms of this patient's type are found in the abdomen and only 25% principally involve the thorax? / A. Diastolic pressure is greater in the abdominal aorta in the supine position / B. Negative intrathoracic pressure reduces aortic wall tension in the thorax / C. The average blood flow in the abdominal aorta is greater than that in the thoracic aorta / D. The average blood pressure in the abdominal aorta is higher than that in the thoracic aorta / E. The average degree of turbulence in the thoracic aorta is higher than that in the abdominal aorta Explanation - Q: 3.2

Close

The correct answer is D. Increased blood pressure is a strong risk factor for atherosclerosis, and humans pay a price for their erect sitting and standing postures. In these postures, the abdominal aorta experiences the weight of a column of blood added to the pressure produced by the heart. In the supine posture, the pressures in the thoracic and abdominal aorta are similar. So, if an average daily pressure is taken, the abdominal aorta tends to have a significantly higher pressure than does the thoracic aorta. Diastolic pressure (choice A) is actually greater in the thoracic aorta compared to the abdominal aorta in the supine position. However, the systolic blood pressure is greater in the abdominal aorta. A negative intrathoracic pressure (choice B) would tend to increase transmural pressure across the wall of the thoracic aorta, and thereby

increase wall tension and promote the development of aneurysms. Blood flow (choice C) in the abdominal aorta is less than that in the thoracic aorta, because some blood leaves the aorta through its thoracic branches. Higher turbulence (choice E) tends to predispose for atherosclerosis, and the flow in the abdominal aorta, with its many branches, tends to be more turbulent than that in the thoracic aorta; this problem is exacerbated as atherosclerotic disease advances.

A 67-year-old man with an 18-year history of type 2 diabetes mellitus presents for a routine physical examination. His temperature is 36.9 C (98.5 F), his blood pressure is 158/98 mm Hg and his pulse is 82/minute and regular. On examination, the physician notes a nontender, pulsatile, mass in the mid-abdomen. A plain abdominal x-ray film with the patient in the lateral position reveals spotty calcification of a markedly dilated abdominal aortic walI. Question 3 of 5

The patient is taken to surgery and the abdominal aorta and proximal common iliac arteries are replaced with a graft. Which of the following aneurysm diameters is usually considered the threshold above which elective surgery is recommended, unless contraindicated by other disease? / A. 1 cm / B. 2 cm / C. 6 cm / D. 10 cm / E. 15 cm Explanation - Q: 3.3

Close

The correct answer is C. This recommendation is made because larger aneurysms have a much greater chance of rupture, and emergency repair carries a high mortality rate. For aneurysms larger than 5 cm, the risk of rupture is 5-10% per year A 67-year-old man with an 18-year history of type 2 diabetes mellitus presents for a routine physical examination. His temperature is 36.9 C (98.5 F), his blood pressure is 158/98 mm Hg and his pulse is 82/minute and regular. On examination, the physician notes a nontender, pulsatile, mass in the mid-abdomen. A plain abdominal x-ray film with the patient in the lateral position reveals spotty calcification of a markedly dilated abdominal aortic walI. Question 4 of 5

Following surgery, the patient is placed on a low-fat diet to reduce the risk of continued progression of his atherosclerotic disease. A bile acid sequestrant is added to interrupt enterohepatic circulation of bile acids. Which of the following agents was most likely prescribed? / A. Atorvastatin / B. Cholestyramine / C. CIofibrate / D. Gemfibrozil / E. Lovastatin Close Explanation - Q: 3.4 The correct answer is B. Cholestyramine and colestipol are bile acid sequestrants that bind bile acids in the intestine, thereby interrupting enterohepatic circulation of bile acids. This has an indirect effect to enhance LDL clearance and lower lipids in the blood. Atorvastatin (choice A) and lovastatin (choice E) are lipid-lowering drugs that competitively inhibit HMG-CoA reductase, an early step in cholesterol biosynthesis. Clofibrate (choice C) and gemfibrozil (choice D) are fibric acid derivatives that may increase the activity of lipoprotein lipase. A 67-year-old man with an 18-year history of type 2 diabetes mellitus presents for a routine physical examination. His temperature is 36.9 C (98.5 F), his blood pressure is 158/98 mm Hg and his pulse is 82/minute and regular. On examination, the physician notes a nontender, pulsatile, mass in the mid-abdomen. A plain abdominal x-ray film with the patient in the lateral position reveals spotty calcification of a markedly dilated abdominal aortic walI. Question 5 of 5

The table shows values of vessel radius (r), intraluminal pressure (P), and wall thickness (w) for both the normal aorta and an aortic aneurysm.

What effect does the aneurysm have on wall stress? / A. Wall stress decreases 10 fold / B. Wall stress decreases 2.5 fold / C. Wall stress decreases 5 fold / D. Wall stress increases 10 fold / E. Wall stress increases 2.5 fold / F. Wall stress increases 5 fold

Explanation - Q: 3.5

Close

The correct answer is E. According to the Law of LaPlace for a cylindrical structure such as the aorta: wall stress (s) = (P x r)/w. Because P and w are shown in the table to be unaffected by the aneurysm and because radius is increased by 2.5 fold at the site of the aneurysm, it is clear that wall stress has increased by 2.5 fold at the site of the aneurysm. This relationship between vessel radius and wall tension can explain why the probability of rupture increases as the aneurysm becomes larger. A 3-year-old girl is seen in the emergency department with acute abdominal pain. She has a 5-day history of vomiting and abdominal distension. She has not passed stool during this time, and during the past day, has been vomiting bilious materiaI. On physical examination, she is lethargic, with a firm and tender abdomen, and peritoneal signs are present. She is immediately referred for laparotomy for suspected diagnoses of intussusception vs. volvulus. At surgery, approximately 20 cm of small intestine is found to be markedly distended, and is resected. The section contains a tightly knotted ball of nematodes that are about 15 to 35 cm in length. The worms have tapered ends without hooks. Question 1 of 5

The worms are most likely which of the following?

Explanation - Q: 4.1

Close

The correct answer is B. Ascaris lumbricoides is the only parasitic worm that is likely to cause intestinal obstruction, and then only if the worm burden is high. The description given of the worms in the case presentation is typical. All of the other worms listed in the choices are also nematodes or roundworms. Ankylostoma duodenale(choice A) is a small (approximately 1 cm) hookworm that inhabits the small intestine and clings to the mucosa. Enterobius vermicularis(choice C) is the pinworm. This is an approximately 1 cm long worm that inhabits the large bowel (and appendix); the female deposits eggs on the perianal skin.

Strongyloides stercoralis(choice D), or threadworm, is a 2.5 mm worm that lives in the crypts of the small bowel and may cause chronic infection due to autoinfection. It is the only nematode capable of increasing its numbers in a host. In immunocompromised hosts, it can cause life-threatening disseminated infection. Trichuris trichiura(choice E), or whipworm, is a 3-5 cm worm that lives on the colorectal mucosa. A 3-year-old girl is seen in the emergency department with acute abdominal pain. She has a 5-day history of vomiting and abdominal distension. She has not passed stool during this time, and during the past day, has been vomiting bilious materiaI. On physical examination, she is lethargic, with a firm and tender abdomen, and peritoneal signs are present. She is immediately referred for laparotomy for suspected diagnoses of intussusception vs. volvulus. At surgery, approximately 20 cm of small intestine is found to be markedly distended, and is resected. The section contains a tightly knotted ball of nematodes that are about 15 to 35 cm in length. The worms have tapered ends without hooks. Question 2 of 5

Which of the following best describes what would have been seen under the microscope if the patient's stool had been analyzed for ova and parasites? / A. Large oval eggs with a lateral spine / B. Large round to oval eggs, with a thick mammillated shells / C. Lemon-shaped eggs, with bipolar plugs, / D. Round eggs and proglottids filled with eggs / E. Small larvae Explanation - Q: 4.2

Close

The correct answer is B. This is the correct description of fertilized Ascaris eggs. Both fertilized and somewhat more elongated unfertilized eggs may be seen; fertilized eggs may contain multiple cells under the thick wavy shell. Choice A describes the eggs of the trematode (fluke) Schistosoma mansoni. Choice C describes the eggs of Trichuris trichiura. Choice D describes the eggs of the cestodes (tapeworms) Taenia saginata and Taenia solium. Choice E describes the larvae of Strongyloides stercoralis. Strongyloides stercoralis has a rapid life cycle in its host. It is the only nematode whose diagnostic form is a larva, not an egg.

A 3-year-old girl is seen in the emergency department with acute abdominal pain. She has a 5-day history of vomiting and abdominal distension. She has not passed stool during this time, and during the past day, has been vomiting bilious materiaI. On physical examination, she is lethargic, with a firm and tender abdomen, and peritoneal signs are present. She is immediately referred for laparotomy for suspected diagnoses of intussusception vs. volvulus. At surgery, approximately 20 cm of small intestine is found to be markedly distended, and is resected. The section contains a tightly knotted ball of nematodes that are about 15 to 35 cm in length. The worms have tapered ends without hooks. Question 3 of 5

If this child acquired her infection in the United States, in what region of the nation does she most likely reside? / A. Desert Southwest / B. Midwest / C. Northeast / D. Pacific Northwest / E. Southeast Explanation - Q: 4.3

Close

The correct answer is E. It is easy for medical students, residents, and physicians to neglect parasitic diseases because they assume that the American population is not likely to have them. However, it is thought that more than 4 million individuals in the Untied States, most of whom are immigrants from developing countries, are infected with Ascaris species, predominately Ascaris lumbricoides. Most of these people have asymptomatic infections. Ascaris can also be acquired in rural areas of the Southeastern United States, where it is endemic. Worldwide, 1.4 billion people are estimated to be infected. A 3-year-old girl is seen in the emergency department with acute abdominal pain. She has a 5-day history of vomiting and abdominal distension. She has not passed stool during this time, and during the past day, has been vomiting bilious materiaI. On physical examination, she is lethargic, with a firm and tender abdomen, and peritoneal signs are present. She is immediately referred for laparotomy for suspected diagnoses of intussusception vs. volvulus. At surgery, approximately 20 cm of small intestine is found to be markedly distended, and is resected. The section contains a tightly knotted ball of nematodes that are about 15 to 35 cm in length. The worms have tapered ends without hooks. Question 4 of 5

Which of the following medications would be the most appropriate pharmacotherapy for this patient? / A. Bithionol / B. Mebendazole

/ C. Metronidazole / D. Niclosamide / E. Praziquantel

Explanation - Q: 4.4

Close

The correct answer is B. Mebendazole is a systemically absorbed broadspectrum anthelminthic agent effective against Ascaris species, hookworm, tapeworm, liver fluke, and pinworms. Bithionol (choice A) is used to treat infections caused by Fasciola hepatica, a tissue fluke. Metronidazole (choice C) is used to treat infections caused by anaerobic organisms, including the intestinal protozoa, Giardia lamblia, and Entamoeba histolytica. Niclosamide (choice D) is not available in the United States. It can be used to treat tapeworm infections caused by T. saginata and D. latum. It is less expensive than praziquantel. Praziquantel (choice E) has broad-spectrum activity against most trematodes and cestodes, with the exception of F. hepatica.

A 3-year-old girl is seen in the emergency department with acute abdominal pain. She has a 5-day history of vomiting and abdominal distension. She has not passed stool during this time, and during the past day, has been vomiting bilious materiaI. On physical examination, she is lethargic, with a firm and tender abdomen, and peritoneal signs are present. She is immediately referred for laparotomy for suspected diagnoses of intussusception vs. volvulus. At surgery, approximately 20 cm of small intestine is found to be markedly distended, and is resected. The section contains a tightly knotted ball of nematodes that are about 15 to 35 cm in length. The worms have tapered ends without hooks. Question 5 of 5

Part of the life cycle of this patient's parasite is a filarial stage in which larva, hatched in the duodenum, penetrate the wall of the small intestine. The passage of migrating larvae most commonly produces symptomatic disease in which of the following organs? / A. Heart / B. Liver / C. Lungs

/ D. Pancreas / E. Stomach

Explanation - Q: 4.5

Close

The correct answer is C. Ascaris infection is acquired by ingestion of the eggs, usually from contaminated soil. The ingested eggs hatch in the duodenum to produce larvae, which cross the small intestinal wall to enter the blood stream. They then pass through the heart to lodge in the lungs. They leave the lung capillary bed to enter the airspaces. They then ascend the bronchial tree into the oropharynx, where they are again swallowed. This time, they return to the small intestine where they develop into mature worms that can live in the host for up to two years. A gravid female worm may produce up to 250,000 eggs daily, which are shed in stool. The passage of the larvae through the lungs often produces cough and/or wheezing, and may, in severe cases, produce fever, dyspnea, fleeting patchy pulmonary infiltrates (Loeffler pneumonitis with prominent eosinophilia), and rarely hemoptysis. Passage through the heart (choice A) is usually asymptomatic, as the larvae are small. A single adult worm, but not usually larval forms, can migrate into the biliary tree (choice B), leading to biliary colic, cholangitis, or gallstone formation; obstructive jaundice uncommonly occurs. The pancreas (choice D) and stomach (choice E) are not common sites for complications of ascariasis.

A 27-year-old woman goes to an emergency room with severe abdominal pain. She had previously experienced similar episodes of pain that Iasted several hours to a few days, but this episode is the most severe. She has also been experiencing nausea, vomiting, and constipation. The physician is left with the impression that she is agitated and somewhat confused, and an accurate history is difficult to elucidate. The patient is sent for emergency laparotomy, but no pathology is noted at surgery. Following the unrevealing surgery, an older surgeon comments that he had once seen a similar case that was actually due to porphyria. Question 1 of 5

The porphyrias are biochemical abnormalities in which of the following pathways? / A. GIycogen degradation / B. Heme synthesis / C. Lipoprotein degradation / D. Nucleotide degradation / E. Urea cycle Explanation - Q: 1.1

Close

The correct answer is B. The porphyrias are a group of rare, related diseases that have in common a block in the heme synthesis pathway. The block is usually partial rather than complete, and thus many of these patients have only intermittent symptoms. Most cases of porphyria present with either a neurovisceral pattern (including both psychiatric symptoms and abdominal pain) or with photosensitive skin lesions. These two patterns are associated with different forms of porphyria. Associate abnormalities of glycogen degradation (choice A) with the glycogen storage diseases, such as von Gierke disease, Pompe disease, and Forbes disease. Associate abnormalities of lipoprotein degradation (choice C) with some forms of hyperlipoproteinemia (notably Type I). Associate abnormalities of nucleotide degradation (choice D) with gout and Lesch-Nyhan syndrome. Associate abnormalities of the urea cycle (choice E) with congenital hyperammonemia, citrullinemia, and argininosuccinic acidemia. A 27-year-old woman goes to an emergency room with severe abdominal pain. She had previously experienced similar episodes of pain that Iasted several hours to a few days, but this episode is the most severe. She has also been experiencing nausea, vomiting, and constipation.

The physician is left with the impression that she is agitated and somewhat confused, and an accurate history is difficult to elucidate. The patient is sent for emergency laparotomy, but no pathology is noted at surgery. Following the unrevealing surgery, an older surgeon comments that he had once seen a similar case that was actually due to porphyria. Question 2 of 5

Following the surgery, the decision is made to screen for the porphyrias that cause acute neurovisceral symptoms. Which of the following tests would be most likely to be used? / A. Erythrocyte porphyrins / B. Total fecal porphyrins / C. Total plasma porphyrins / D. Total urinary porphyrins / E. Urinary porphobilinogen Explanation - Q: 1.2

Close

The correct answer is E. The acute neurovisceral porphyrias are those that tend to present with severe abdominal pain, often accompanied by neuropsychiatric symptoms. The best tests to use for screening of these diseases are urinary porphobilinogen (PBG, either random or 24 hour) and urinary delta-aminolevulinic acid (ALA, either random or 24 hour). Erythrocyte porphyrins (choice A) are used for follow-up in the photosensitive types of porphyria. Total fecal porphyrins (choice B) are used for follow-up evaluation after screening tests for either the photosensitive porphyrias or the acute neurovisceral porphyrias are positive. Total plasma porphyrias (choice C) are useful for first line screening of the photosensitive porphyrias, and are used for further evaluation after screening in the acute neurovisceral porphyrias. Total urinary porphyrins (choice D) are used for further evaluation after screening for acute neurovisceral porphyrias. A 27-year-old woman goes to an emergency room with severe abdominal pain. She had previously experienced similar episodes of pain that Iasted several hours to a few days, but this episode is the most severe. She has also been experiencing nausea, vomiting, and constipation. The physician is left with the impression that she is agitated and somewhat confused, and an accurate history is difficult to elucidate. The patient is sent for emergency laparotomy, but no pathology is noted at surgery. Following the unrevealing surgery, an older surgeon

comments that he had once seen a similar case that was actually due to porphyria. uestion 3 of 5

Which of the following are the three most common forms of porphyria? / A. Acute intermittent porphyria, erythropoietic protoporphyria, and porphyria cutanea tarda / B. Acute intermittent porphyria, hepatoerythropoietic porphyria, and variegate porphyria / C. Congenital erythropoietic porphyria, delta-aminolevulinic acid dehydratasedeficient porphyria, and hepatoerythropoietic porphyria / D. Erythropoietic protoporphyria, hereditary coproporphyria, and porphyria cutanea tarda / E. Hereditary coproporphyria, variegate porphyria, and X-Iinked sideroblastic anemia Explanation - Q: 1.3

Close

The correct answer is A. The porphyrias are complex diseases that can easily appear overwhelming. A very useful point to know (both clinically and for the USMLE) is that the three most common forms are acute intermittent porphyria, erythropoietic protoporphyria, and porphyria cutanea tarda. Acute intermittent porphyria tends to present with acute neurovisceral symptoms. Erythrocytic protoporphyria tends to present acutely with painful skin lesions. Porphyria cutanea tarda tends to present with chronic blistering skin lesions. The other types listed in various choices are also porphyrias, but are less common.

A 27-year-old woman goes to an emergency room with severe abdominal pain. She had previously experienced similar episodes of pain that Iasted several hours to a few days, but this episode is the most severe. She has also been experiencing nausea, vomiting, and constipation. The physician is left with the impression that she is agitated and somewhat confused, and an accurate history is difficult to elucidate. The patient is sent for emergency laparotomy, but no pathology is noted at surgery. Following the unrevealing surgery, an older surgeon comments that he had once seen a similar case that was actually due to porphyria. uestion 4 of 5

This patient is found to have increased levels of both delta-aminolevulinic acid (ALA) and porphobilinogen (PBG) in blood. Follow-up studies demonstrate low PBG deaminase in erythrocytes. AIso, additional history is elicited, revealing that the woman had started a very low carbohydrate diet about one week before being admitted to the hospitaI. Which of the following is the most likely diagnosis? / A. Acute intermittent porphyria

/ / / /

B. Congenital erythropoietic porphyria C. Erythropoietic protoporphyria D. Porphyria cutanea tarda E. X-Iinked sideroblastic anemia Explanation - Q: 1.4

Close

The correct answer is A. These laboratory findings are most consistent with acute intermittent porphyria, which is due to PBG deaminase deficiency. Patients usually, but not always, have a deficiency of erythrocyte PBG deaminases. (Some cases have also been described in which the enzyme deficiency is limited to liver.) The condition is an autosomal dominant disorder that typically becomes symptomatic in women after puberty, and then often only if a precipitating event (dieting, use of certain drugs, premenstrual) is also present. Symptoms during the attacks can include abdominal symptoms (pain, nausea, vomiting, constipation, diarrhea, abdominal distension, ileus), which are thought to be due to the effects of this condition on visceral nerves. Other symptoms that may be mediated neurologically include incontinence, urinary retention, tachycardia, diaphoresis, hypertension, muscle weakness, psychiatric symptoms, seizures, and rarely, severe paralysis, respiratory insufficiency, and death. Both intravenous glucose (oral is often inadequate due to poor intestinal function) and exogenous heme supplementation can suppress the heme biosynthetic mechanism, and tend to ameliorate the acute attack. Patients should be cautioned to diet gently, as intense dieting can precipitate attacks. Congenital erythropoietic porphyria (choice B) is characterized by severe skin blistering that usually begins after birth, pink to dark-brown urine, normal ALA and PBG, and increased porphyrins (primarily uroporphyrin I and coproporphyrin I) in urine, plasma, and erythrocytes. Erythropoietic protoporphyria (choice C) is characterized by cutaneous photosensitivity that begins early in life and high protoporphyrin in erythrocytes and bone marrow. Porphyria cutanea tarda (choice D) is characterized by photosensitivity with skin blistering, elevated plasma porphyrins, and elevated urine porphyrins (mostly uroporphyrin and heptacarboxylporphyrin). The very rare X-linked sideroblastic anemia (choice E), due to a deficiency of delta-aminolevulinic acid synthase, can clinically resemble acute intermittent porphyria, and is characterized by elevated levels of urinary ALA and coproporphyrin.

A 27-year-old woman goes to an emergency room with severe abdominal pain. She had previously experienced similar episodes of pain that Iasted several hours to a few days, but this episode is the most severe. She has also been experiencing nausea, vomiting, and constipation. The physician is left with the impression that she is agitated and somewhat confused, and an accurate history is difficult to elucidate. The patient is sent for emergency laparotomy, but no pathology is noted at surgery. Following the unrevealing surgery, an older surgeon comments that he had once seen a similar case that was actually due to porphyria. Question 5 of 5

Which of the following drugs would be most likely to induce an attack of abdominal pain in this patient? / A. Acetaminophen / B. Aspirin / C. Barbiturate / D. GIucocorticoid / E. Insulin Explanation - Q: 1.5

Close

The correct answer is C. Some symptomatic episodes of acute porphyria (including acute intermittent porphyria, hereditary coproporphyria, variegate porphyria, and aminolevulinic acid dehydratase porphyria) are triggered by drug ingestion, and administration of drugs to undiagnosed patients can cause an acute exacerbation of an ongoing attack of acute porphyria. Drugs considered unsafe for use in these patients notably include alcohol, anticonvulsants, barbiturates, many other sedatives, and sulfonamide antibiotics. Of particular concern are the sedative agents, since it may be very tempting to give an obviously agitated patient a sedative to allow easier examination of the patient. Many other drugs are also on the lists of potentially dangerous drugs in these patients. Once the diagnosis is established, the patient should be instructed to always inform her/his physician of her condition, and ask that the safety of drugs prescribed in patients with porphyria be checked. Many of the drugs that can induce or exacerbate an attack of porphyria do so by increasing the activity of the cytochrome P450 system, which indirectly triggers an increase in heme biosynthesis. The other medications listed in the choices are "safe" in these patients.

A 47-year-old woman presents to the emergency department with cramping/colicky abdominal pain. The current episode of pain began

several hours ago, following a fatty meaI. The pain began slowly, and rose in intensity to a plateau over the course of several hours. The patient reports that she had had several other episodes of similar pain during the past several months, with long intervening periods of freedom from pain. On physical examination, she is noted to have tenderness to deep palpation in the right upper quadrant of the abdomen near the rib cage. The patient also reports that she is experiencing shoulder/back pain at a site she identifies near the right lower scapula, but no tenderness can be elicited during the back and shoulder examination. Question 1 of 7

Which of the following organs is the most likely source of this woman's pain? / A. Appendix / B. Diaphragm / C. Esophagus / D. Gallbladder / E. Stomach Explanation - Q: 2.1

Close

The correct answer is D. This woman most likely has gallstones. Cholelithiasis, or the formation of calculi (gallstones) within the gallbladder, is very common in the United States, with over 500,000 cholecystectomies being performed yearly. While many cases of gallstone disease are symptomatic, right upper quadrant pain with referral of the pain to the lower right scapula should specifically suggest gallbladder disease. The pattern of episodes of several hours of pain followed by long periods of freedom from pain is also typical of symptomatic gallstone disease. The appendix (choice A) would most likely cause lower abdominal pain. Pain from irritation of the diaphragm (choice B) can cause right upper quadrant pain and referred pain in the supraclavicular area (rather than the subscapular pain of biliary colic). The absence of right upper quadrant tenderness to palpation, and the history of pain after a fatty meal also argue against this diagnosis. Esophageal pain (choice C) related to regurgitation of gastric contents (heartburn) can occur postprandially, but tends to radiate into the neck, throat, or even face. Peptic ulcer pain of gastric origin (choice E) is usually described as causing burning, gnawing, or hunger, and may be relieved by eating. A 47-year-old woman presents to the emergency department with cramping/colicky abdominal pain. The current episode of pain began several hours ago, following a fatty meaI. The pain began slowly, and rose in intensity to a plateau over the course of several hours. The

patient reports that she had had several other episodes of similar pain during the past several months, with long intervening periods of freedom from pain. On physical examination, she is noted to have tenderness to deep palpation in the right upper quadrant of the abdomen near the rib cage. The patient also reports that she is experiencing shoulder/back pain at a site she identifies near the right lower scapula, but no tenderness can be elicited during the back and shoulder examination. Question 2 of 7

Which of the following techniques would be most appropriate to demonstrate the patient's most likely diagnosis? / A. Colonoscopy / B. CT scan of the abdomen / C. Esophagoduodenoscopy / D. MRI scan of the abdomen / E. UItrasonography Close Explanation - Q: 2.2 The correct answer is E. Real-time ultrasonography, with 98% sensitivity and 95% specificity, is considered the method of choice for diagnosing possible gallbladder stones. Colonoscopy (choice A) and esophagoduodenoscopy (choice C) might be helpful for excluding alternative diagnoses, but would not themselves establish a diagnosis of gallstone disease. CT (choice B) and MRI (choice D) scans of the abdomen are expensive tests whose use is not warranted, since real-time ultrasonography performs as well or better. A 47-year-old woman presents to the emergency department with cramping/colicky abdominal pain. The current episode of pain began several hours ago, following a fatty meaI. The pain began slowly, and rose in intensity to a plateau over the course of several hours. The patient reports that she had had several other episodes of similar pain during the past several months, with long intervening periods of freedom from pain. On physical examination, she is noted to have tenderness to deep palpation in the right upper quadrant of the abdomen near the rib cage. The patient also reports that she is experiencing shoulder/back pain at a site she identifies near the right lower scapula, but no tenderness can be elicited during the back and shoulder examination. Question 3 of 7

Following appropriate diagnostic studies, the patient is taken to the surgical suite. During the surgery, the surgeon inserts his fingers from right to left behind the hepatoduodenal ligament. As he does so, his fingers enter which of the following? / A. Ampulla of Vater

/ / / /

B. Common bile duct C. Epiploic foramen D. Greater peritoneal sac E. Portal vein Explanation - Q: 2.3

Close

The correct answer is C. The space behind the stomach, hepatoduodenal ligament, and hepatogastric ligament is the omental bursa. This space can be entered by passing through the epiploic foramen of Winslow, as described in the question stem. The common bile duct enters the duodenum through the ampulla of Vater (choice A). The hepatoduodenal ligament contains the common bile duct (choice B), the portal vein (choice E), and the hepatic artery. The greater peritoneal sac (choice D) lies anterior to the stomach and hepatoduodenal ligament. A 47-year-old woman presents to the emergency department with cramping/colicky abdominal pain. The current episode of pain began several hours ago, following a fatty meaI. The pain began slowly, and rose in intensity to a plateau over the course of several hours. The patient reports that she had had several other episodes of similar pain during the past several months, with long intervening periods of freedom from pain. On physical examination, she is noted to have tenderness to deep palpation in the right upper quadrant of the abdomen near the rib cage. The patient also reports that she is experiencing shoulder/back pain at a site she identifies near the right lower scapula, but no tenderness can be elicited during the back and shoulder examination. Question 4 of 7

During the cholecystectomy, the surgeon ligates the cystic artery. This is typically a branch of which of the following? / A. Gastroduodenal artery / B. Left gastroepiploic artery / C. Right gastroepiploic artery / D. Right hepatic artery / E. Superior pancreaticoduodenal artery Close Explanation - Q: 2.4 The correct answer is D. The cystic artery is generally a branch of the right hepatic artery. The gastroduodenal artery (choice A) is a branch of the (common) hepatic

artery. The left gastroepiploic artery (choice B) is a branch of the splenic artery. The right gastroepiploic artery (choice C) is a branch of the gastroduodenal artery. The superior pancreaticoduodenal artery (choice E) is a branch of the gastroduodenal artery. A 47-year-old woman presents to the emergency department with cramping/colicky abdominal pain. The current episode of pain began several hours ago, following a fatty meaI. The pain began slowly, and rose in intensity to a plateau over the course of several hours. The patient reports that she had had several other episodes of similar pain during the past several months, with long intervening periods of freedom from pain. On physical examination, she is noted to have tenderness to deep palpation in the right upper quadrant of the abdomen near the rib cage. The patient also reports that she is experiencing shoulder/back pain at a site she identifies near the right lower scapula, but no tenderness can be elicited during the back and shoulder examination. Question 5 of 7

Pathologic examination of the specimen removed by the surgeon demonstrates the presence of numerous yellow stones (shown above). These are most likely composed primarily of which of the following? / A. Bilirubinate / B. Calcium phosphate / C. Cholesterol / D. Cystine

/ E. Struvite

Explanation - Q: 2.5

Close

The correct answer is C. The stones are gallstones, and their yellow color indicates that they are composed of cholesterol. Cholesterol stones are the most common form of gallstones. Risk factors include female sex, multiparity, obesity, increased age (female, fat, forty, and fertile) and North American Indian race. Bilirubinate (choice A) gallstones, which are usually associated with hemolytic anemias, are less common, brown, rather than yellow, and often faceted. Calcium phosphate (choice B), cystine (choice D), and struvite (choice E) composition can be seen in kidney stones. A 47-year-old woman presents to the emergency department with cramping/colicky abdominal pain. The current episode of pain began several hours ago, following a fatty meaI. The pain began slowly, and rose in intensity to a plateau over the course of several hours. The patient reports that she had had several other episodes of similar pain during the past several months, with long intervening periods of freedom from pain. On physical examination, she is noted to have tenderness to deep palpation in the right upper quadrant of the abdomen near the rib cage. The patient also reports that she is experiencing shoulder/back pain at a site she identifies near the right lower scapula, but no tenderness can be elicited during the back and shoulder examination. Question 6 of 7

If this patient had a small stone lodge near the ampulla of Vater, which of the following complications would be most likely to occur? / A. Crohn disease / B. Diabetes mellitus / C. Pancreatitis / D. Peptic ulcer / E. Polyarteritis nodosa

Explanation - Q: 2.6 The correct answer is C. A small gallstone obstructing the pancreatic outflow is a well-known cause of acute pancreatitis. The other conditions listed are not caused by gallstones.

Close

A 47-year-old woman presents to the emergency department with cramping/colicky abdominal pain. The current episode of pain began several hours ago, following a fatty meaI. The pain began slowly, and rose in intensity to a plateau over the course of several hours. The patient reports that she had had several other episodes of similar pain during the past several months, with long intervening periods of freedom from pain. On physical examination, she is noted to have tenderness to deep palpation in the right upper quadrant of the abdomen near the rib cage. The patient also reports that she is experiencing shoulder/back pain at a site she identifies near the right lower scapula, but no tenderness can be elicited during the back and shoulder examination. Question 7 of 7

If this patient had refused surgical treatment, which of the following would be the most appropriate pharmacotherapy to provide definitive treatment and thereby relieve associated pain? / A. Ampicillin / B. CIofibrate / C. Meperidine / D. Oxycodone / E. Ursodiol Explanation - Q: 2.7

Close

The correct answer is E. The question is asking, "Which of the following will eradicate a gallstone?" When a gallstone is eliminated the pain will subsequently be eliminated. This question is NOT asking, "which of the following is the most appropriate form of pain control?". Ursodiol (ursodeoxycholic acid) is a hydrophilic bile acid that is used to dissolve small (< 20 mm), non-calcified, radiolucent cholesterol gallstones in patients with functioning gallbladders who cannot undergo (or refuse) cholecystectomy. Analgesics and antibiotics, such as ampicillin (choice A), are administered when appropriate, but do not help eradicate the stones. Clofibrate (choice B) is an antihyperlipidemic that is associated with the development of gallstones. High-risk patients, such as diabetics and the elderly, should be watched closely. As a side note, if this question were asking: "which of the following is the most appropriate form of pain control in this patient", the most appropriate answer would be meperidine. Meperidine (choice C) is the narcotic of choice since it causes the least amount of spasm of the sphincter of Oddi. In other words, meperidine is preferred over oxycodone (choice D).

A 64-year-old man with a history of coronary artery disease (CAD) comes to the emergency department with the acute onset of severe, constant, Iower abdominal pain and rectal bleeding. He reports that he previously has had several episodes of similar, but less severe pain. About 12 hours after the onset of pain, the patient began passing copious bright red blood per rectum. He denies nausea, vomiting, sick contacts, or foreign traveI. Initial physical examination reveals a distressed man, who is afebrile, but tachypneic, with scant diffuse abdominal tenderness to palpation. Rectal examination is positive for blood. Laboratory studies reveal a metabolic acidosis with an elevated serum Iactate. Question 1 of 5

Which of the following is the most likely diagnosis? / A. Colon carcinoma / B. Infectious colitis / C. Inflammatory bowel disease / D. Ischemic colitis / E. Necrotizing enterocolitis Explanation - Q: 3.1

Close

The correct answer is D. A patient with severe abdominal pain and rectal bleeding with an unremarkable physical examination is likely suffering from ischemic colitis. "Pain out-of-proportion to examination" is a classic finding for ischemic colitis. The previous episodes of less severe pain represent ischemic angina. An infarction has occurred, as indicated by the rise in serum lactate secondary to the colon's anaerobic metabolism. The history of coronary artery disease also suggests this diagnosis, as the atherosclerotic processes that contribute to his CAD are also likely present in his abdominal vasculature. Colon cancer (choice A) would produce less acute symptoms, but occasionally, colon cancer may present acutely with obstructive symptoms. Patients may have bleeding and abdominal pain, but the pain is typically intermittent and accompanied by nausea, vomiting, abdominal distention, and absence of flatus. Infectious colitis (choice B) is incorrect. While patients may have bleeding and abdominal pain, nothing in the history suggests a disease of infectious origin (no sick contacts or foreign travel). The acute onset also suggests a vascular event, rather than an infectious one. Inflammatory bowel disease (IBD) (choice C) is incorrect because while the patient reports previous episodes, an elderly man with IBD would likely have

a chronic history of abdominal pain and bleeding. Necrotizing enterocolitis (choice E) affects premature infants and would not be relevant in this setting.

A 64-year-old man with a history of coronary artery disease (CAD) comes to the emergency department with the acute onset of severe, constant, Iower abdominal pain and rectal bleeding. He reports that he previously has had several episodes of similar, but less severe pain. About 12 hours after the onset of pain, the patient began passing copious bright red blood per rectum. He denies nausea, vomiting, sick contacts, or foreign traveI. Initial physical examination reveals a distressed man, who is afebrile, but tachypneic, with scant diffuse abdominal tenderness to palpation. Rectal examination is positive for blood. Laboratory studies reveal a metabolic acidosis with an elevated serum Iactate. Question 2 of 5

The lactate produced from the anaerobic metabolism in the infarcted gut will likely be which of the following? / A. Exhaled as a fruity odor / B. Incorporated into glycogen in the liver / C. Incorporated into myoglobin in muscle / D. Incorporated into urea in the urine / E. Secreted by the kidneys unchanged Explanation - Q: 3.2

Close

The correct answer is B. Lactate is converted into glucose, and then glycogen in the liver by a process know as the Cori cycle. Choice A is incorrect, as lactate would not be exhaled. A fruity odor on the breath would be a sign of ketoacidosis. While some of the carbon from the lactate may be incorporated into peptides via Krebs intermediates (e.g., choice C), the vast majority would be left as carbohydrate. Urea (choice D) represents a means of eliminating nitrogenous waste. Choice E is wrong, as the kidneys would retain the lactate, rather than excreting it. A 64-year-old man with a history of coronary artery disease (CAD) comes to the emergency department with the acute onset of severe,

constant, Iower abdominal pain and rectal bleeding. He reports that he previously has had several episodes of similar, but less severe pain. About 12 hours after the onset of pain, the patient began passing copious bright red blood per rectum. He denies nausea, vomiting, sick contacts, or foreign traveI. Initial physical examination reveals a distressed man, who is afebrile, but tachypneic, with scant diffuse abdominal tenderness to palpation. Rectal examination is positive for blood. Laboratory studies reveal a metabolic acidosis with an elevated serum Iactate. Question 3 of 5

If this patient's disease were drug-induced, which of the following agents would most likely be responsible? / A. Acetaminophen / B. Amiodarone / C. Cocaine / D. Dexamethasone / E. Nitroglycerin Explanation - Q: 3.3

Close

The correct answer is C. Cocaine is a sympathomimetic drug that indirectly acts on both the alpha and beta adrenergic receptors on the vasculature. As such, cocaine may cause vasospasm in the abdominal vasculature leading to infarction and ischemic colitis. Similar vasospastic events may occur in the coronary vasculature, leading to myocardial infarction. Acetaminophen (choice A) is an analgesic, and would not play a role in producing ischemic colitis. Amiodarone (choice B) is an antiarrhythmic, and would not contribute to ischemic colitis. Dexamethasone (choice D) is a steroidal anti-inflammatory drug. Not only would this medication not cause ischemic colitis, it might mask the symptoms due to its potent anti-inflammatory properties. Nitroglycerin (choice E) is a venodilator, and would not contribute to ischemic colitis. As a venodilator, nitroglycerin is used to treat coronary ischemia by reducing cardiac preload. A 64-year-old man with a history of coronary artery disease (CAD) comes to the emergency department with the acute onset of severe, constant, Iower abdominal pain and rectal bleeding. He reports that he previously has had several episodes of similar, but less severe pain.

About 12 hours after the onset of pain, the patient began passing copious bright red blood per rectum. He denies nausea, vomiting, sick contacts, or foreign traveI. Initial physical examination reveals a distressed man, who is afebrile, but tachypneic, with scant diffuse abdominal tenderness to palpation. Rectal examination is positive for blood. Laboratory studies reveal a metabolic acidosis with an elevated serum Iactate. Question 4 of 5

While the patient is in the emergency department, the pain becomes increasingly severe. Several hours after his initial examination, the patient becomes febrile and is now exquisitely tender to palpation. He writhes in pain when the physician jostles the bed. Air is seen under the diaphragm in an upright chest x-ray film. These new findings suggest which of the following? / A. Abdominal aortic aneurysm / B. Bowel obstruction / C. Cholecystitis / D. Hypovolemia / E. Perforation with peritonitis Explanation - Q: 3.4

Close

The correct answer is E. This patient has experienced a bowel perforation. Air under the diaphragm in an upright chest film provides definitive evidence that a hollow viscus has ruptured. Air near the liver on a left lateral decubitus (patient lays with the left side down) is an alternative study to demonstrate perforation. Spillage from the perforated bowel has irritated and inflamed the peritoneum, resulting in peritonitis. Symptoms of peritonitis include extreme, sharp pain exacerbated by jostling (patients often report that the bumpy ride to the emergency department caused extreme pain). Patients will be exquisitely tender to palpation and percussion and may have abdominal rigidity. Fever typically accompanies peritonitis. While an abdominal aortic aneurysm or AAA (choice A) presents as acute abdominal pain, this pain is described as tearing and may radiate to the back. A pulsatile abdominal mass may be palpated. The air on the chest film is also inconsistent with AAA. This patient does not have bowel obstruction (choice B). Signs and symptoms of bowel obstruction include: nausea, vomiting, intermittent abdominal pain, hypovolemia, abdominal distention, absence of flatus, and a "step ladder" bowel pattern on abdominal films. Cholecystitis (choice C) typically presents as right upper quadrant (RUQ) pain, fever, and jaundice. Patients usually have a history of colicky RUQ

pain. While the patient is at risk for hypovolemia (choice D), none of the symptoms listed typify hypovolemia. Signs and symptoms of mild to moderate hypovolemia include malaise, dry mouth, thirst, decreased skin turgor, tachycardia, hypotension, and decreased urine output. A 64-year-old man with a history of coronary artery disease (CAD) comes to the emergency department with the acute onset of severe, constant, Iower abdominal pain and rectal bleeding. He reports that he previously has had several episodes of similar, but less severe pain. About 12 hours after the onset of pain, the patient began passing copious bright red blood per rectum. He denies nausea, vomiting, sick contacts, or foreign traveI. Initial physical examination reveals a distressed man, who is afebrile, but tachypneic, with scant diffuse abdominal tenderness to palpation. Rectal examination is positive for blood. Laboratory studies reveal a metabolic acidosis with an elevated serum Iactate. Question 5 of 5

Upon surgical exploration of the abdomen, the colon is dull and dusky from the mid transverse colon to the rectum. The patient has occluded which of the following vessels? / A. Celiac trunk / B. Cystic artery / C. External iliac artery / D. Inferior mesenteric artery / E. Superior mesenteric artery Explanation - Q: 3.5

Close

The correct answer is D. The inferior mesenteric artery distributes blood to the embryologic hindgut. This includes the distal 1/3 of the transverse colon to the rectum. The rectum is spared because it receives circulation from the inferior rectal artery (not mesenteric). The celiac trunk (choice A) supplies the embryologic foregut. The first three branches include the splenic artery, the left gastric artery, and the common hepatic artery. This patient has no findings in this distribution. The cystic artery (choice B) supplies the gall bladder. There are no gall bladder findings in this case. The external iliac artery (choice C) gives rise to the vessels of the lower extremity. Symptoms of occlusion or stenosis might include buttock and thigh pain exacerbated by walking. Severe stenosis might give patients buttock and thigh pain, even at rest.

The superior mesenteric artery (choice E) supplies the embryologic hindgut. This extends from the duodenum to the proximal 2/3 of the transverse colon.

A 45-year-old man goes to an emergency department because he is experiencing severe abdominal pain, which is radiating straight through to his back. The pain began several hours after an admitted alcoholic binge, and has not changed in position, although it has become worse. Question 1 of 5

Which of the following would be the most likely cause of this type of pain? / A. Acute appendicitis / B. Acute hepatitis / C. Acute pancreatitis / D. Chronic hepatitis / E. Myocardial infarction Explanation - Q: 4.1

Close

The correct answer is C. The typical pain described occurs in approximately 50% of patients with acute pancreatitis. Other patients may have milder pain or even, uncommonly, pain first felt in the lower abdomen. The pain of acute appendicitis (choice A) is often felt first as referred pain near the umbilicus, with tenderness on palpation in the left lower quadrant. Acute hepatitis (choice B) can cause pain referred to the right shoulder. Chronic hepatitis (choice D) does not usually cause pain. Myocardial infarction (choice E) can cause substernal pain and pain radiating to the left shoulder. A 45-year-old man goes to an emergency department because he is experiencing severe abdominal pain, which is radiating straight through to his back. The pain began several hours after an admitted alcoholic binge, and has not changed in position, although it has become worse. Question 2 of 5

In addition to alcohol use, which of the following is a common predisposing factor for this patient's disease? / A. Biliary tract stones / B. Duodenal cancer / C. Gastric carcinoma / D. Kidney stones / E. Peptic ulcer

Explanation - Q: 4.2

Close

The correct answer is A. The overwhelmingly most common predisposing factors for acute pancreatitis are gallstones (more specifically tiny ones that lodge in the extrahepatic bile duct system) and alcohol abuse. Rarely, nearby cancers (choices B and C) can occlude the pancreatic duct system and cause a secondary acute pancreatitis. Kidney stones (choice D) have no relationship with pancreatitis. Peptic ulcers (choice E) that erode into the pancreas can uncommonly secondarily inflame the pancreas.

A 45-year-old man goes to an emergency department because he is experiencing severe abdominal pain, which is radiating straight through to his back. The pain began several hours after an admitted alcoholic binge, and has not changed in position, although it has become worse. Question 3 of 5

Marked serum elevation of which of the following markers would most strongly substantiate the likely diagnosis? / A. Acid phosphatase / B. Amylase / C. Aspartate aminotransferase / D. AIkaline phosphatase / E. Creatinine kinase Explanation - Q: 4.3

Close

The correct answer is B. The usual markers for pancreatitis are amylase and lipase. Marked elevation of amylase usually means either pancreatic disease or salivary gland disease; lipase will be elevated in pancreatic disease but not salivary gland disease. If you see elevated amylase on a USMLE question, you should think of pancreatitis or salivary gland disease (mumps, salivary gland stone). However, you should be aware, for your general medical knowledge, that modest elevations of amylase can be seen in a much wider variety of settings (often reflecting either subclinical pancreatic damage or hemoconcentration of pancreatic enzymes), including GI obstruction, mesenteric thrombosis and infarction, macroamylasemia (a genetic condition with abnormal amylase), renal disease, ruptured tubal pregnancy, lung cancer, acute alcohol ingestion, and following abdominal surgery. Associate acid phosphatase (choice A) with diseases involving the prostate

and, to lesser degrees, bone, the heart, platelets, and the liver. Associate aspartate aminotransferase (choice C) with diseases of the heart, muscle, liver, pancreas (though not as important for diagnosis as amylase and lipase), and brain. Associate alkaline phosphatase (choice D) with diseases of bone, liver, and to lesser degrees, lung and heart. Associate creatinine kinase (choice E) with diseases of the heart, muscle, brain, and the general body (trauma, surgery). A 45-year-old man goes to an emergency department because he is experiencing severe abdominal pain, which is radiating straight through to his back. The pain began several hours after an admitted alcoholic binge, and has not changed in position, although it has become worse. Question 4 of 5

The patient has a severe course that requires treatment in an ICU. CIinically, he appears similar to patients with sepsis, with fever, elevated white count, hypotension, increased pulse rate, shallow and rapid breathing, oliguria, and a blunted sensorium, in addition to his pain and abdominal tenderness. These clinical findings are most likely related to which of the following? / A. Activation of the inflammatory cascade / B. AIcohol withdrawal symptoms / C. AIIergic reaction to alcohol / D. Drug toxicity effect / E. Secondary infection with mixed flora gut bacteria Close Explanation - Q: 4.4 The correct answer is A. Acute pancreatitis can either be relatively mild, or a severe condition that may cause death. It is thought that, in severe cases, leakage of enzyme-containing pancreatic secretions into the tissues/and or blood stream causes cleavage of precursors, thus strongly activating the complement and inflammatory cascades. These, in turn, produce abundant cytokines, which worsen the symptoms. The clinical result is similar to sepsis, with risk of multi-organ failure and death. The treatment of acute pancreatitis is primarily supportive, and may include careful attention to fluid resuscitation, oxygen supplementation, cardiovascular support, dialysis, management of electrolyte abnormalities, pain control, and total parenteral nutrition. Alcohol allergy (choice C) or withdrawal (choice B) do not play any additional part in most of these symptoms once the pancreatitis has developed. Infection (choice E) and drug toxicity (choice D) are also not a necessary

part of the clinical picture, although physicians may worry that the patient's general clinical status is masking other, potentially more treatable, problems. A 45-year-old man goes to an emergency department because he is experiencing severe abdominal pain, which is radiating straight through to his back. The pain began several hours after an admitted alcoholic binge, and has not changed in position, although it has become worse. Question 5 of 5

The patient's condition resolves in about two weeks, but he continues to drink after leaving the hospitaI. When seen several years later, he has had a number of similar episodes, and now has chronic severe abdominal pain. CT scan demonstrates a single, smooth-walled, fluid filled space in the tail of the pancreas, which can be reached by the radiologist for CT-guided aspiration with an approach from the back. The fluid aspirated is yellowish, clear, and acellular. Which of the following is the most likely diagnosis? / A. Pancreatic microcystic adenoma / B. Pancreatic mucinous cystadenocarcinoma / C. Pancreatic mucinous cystadenoma / D. Pancreatic pseudocyst / E. Pancreatic solid-cystic tumor Explanation - Q: 4.5

Close

The correct answer is D. Pancreatic pseudocyst is a fairly common complication of both acute and chronic pancreatitis, and appears to develop when trapping of pancreatic digestive juices (containing amylase, lipase, and proteases) causes a "digestion" of part of the pancreas, leaving a fluid filled cystic space. The term "pseudocyst", rather than "cyst", is used by purists because the space does not have an epithelial lining, and is hence not a "true cyst". Pseudocysts are usually solitary and typically measure 5-10 cm in diameter. They can be surgically excised (and the surrounding tissue will typically show evidence of chronic pancreatitis in long-standing cases) or sometimes, if the anatomy is favorable, drained into adjacent hollow viscera. Some are medically managed if small. Most true neoplasms of the pancreas contain (often large numbers of) smaller, multiple, cysts. These tumors can be benign or malignant, and the ones with mucus-secreting epithelium (choices B and C) are more common than those with a serous lining (choices A and E). A 17-year-old boy is taken to the emergency department because he has developed severe abdominal pain. The pain began abruptly several hours previously, and was felt initially in the periumbilical region, but later shifted to the right lower quadrant. The boy had initially felt

somewhat nauseous, but this has passed. On physical examination, he is noted to have localized pain on cough and to be running a lowgrade fever. Question 1 of 5

Examination of the abdomen demonstrates right lower quadrant tenderness at the junction of the middle and outer thirds of the line joining the umbilicus to the anterior superior spine of the iliac. This location is known as which of the following? / A. Gubernaculum / B. Langer's line / C. Linea alba / D. McBurney's point / E. Tunica albuginea Explanation - Q: 5.1

Close

The correct answer is D. The point described is McBurney's point, which overlies the location of the appendix in most individuals. The gubernaculum (choice A) is the fibrous cord that connects the primordial testis or ovary to the anterolateral abdominal wall. Langer's lines (choice B) are the cleavage lines of the skin. The linea alba (choice C) is a sheet-like aponeurosis that covers the anterior abdominal wall. The tunica albuginea (choice E) is a tough fibrous coat that covers the testis. A 17-year-old boy is taken to the emergency department because he has developed severe abdominal pain. The pain began abruptly several hours previously, and was felt initially in the periumbilical region, but later shifted to the right lower quadrant. The boy had initially felt somewhat nauseous, but this has passed. On physical examination, he is noted to have localized pain on cough and to be running a lowgrade fever. Question 2 of 5

Which of the following is the most likely diagnosis? / A. Appendicitis / B. Diverticulitis / C. Gallstones / D. Rectal ulcer / E. Renal colic Explanation - Q: 5.2 The correct answer is A. This patient has a typical presentation for

Close

appendicitis, and the diagnosis is confirmed by the presence of localized tenderness at McBurney's point. Diverticulitis (choice B) is usually a disease of middle-aged or older individuals and most commonly affects the left-lower quadrant. Symptomatic gallstone disease (choice C) causes pain and tenderness in the right upper quadrant. Rectal ulcer (choice D) causes pain with stool movement, but does not usually produce tenderness identifiable on abdominal examination. Renal colic (choice E) usually produces flank or lower back pain. A 17-year-old boy is taken to the emergency department because he has developed severe abdominal pain. The pain began abruptly several hours previously, and was felt initially in the periumbilical region, but later shifted to the right lower quadrant. The boy had initially felt somewhat nauseous, but this has passed. On physical examination, he is noted to have localized pain on cough and to be running a lowgrade fever. Question 3 of 5

The patient also exhibits an increase in pain in the right lower quadrant from the passive extension of the right hip joint. This finding suggests that the inflammation also involves which of the following? / A. BIadder / B. External oblique muscle / C. Femur / D. IIiopsoas muscle / E. Transverse abdominal muscle Explanation - Q: 5.3

Close

The correct answer is D. This patient has a "positive psoas sign," which is an increase in pain from passive extension of the right hip joint. This maneuver stretches the iliopsoas muscle, which lies behind the appendix and can become secondarily inflamed when the appendiceal inflammation extends through the serosa. The psoas sign is clinically useful in both confirming the appendix as the probable origin of the patient's pain, and indicating that the inflammation is transmural and that the risk of rupture and peritonitis is increased. The bladder (choice A) is located more medially, and is usually not affected by appendicitis. The external oblique (choice B) and transverse abdominal (choice E)

muscles are in the anterior and lateral abdominal walls, and do not usually become inflamed with appendicitis. The femur (choice C) is moved during the extension of the right hip joint, but is not the source of the pain. A 17-year-old boy is taken to the emergency department because he has developed severe abdominal pain. The pain began abruptly several hours previously, and was felt initially in the periumbilical region, but later shifted to the right lower quadrant. The boy had initially felt somewhat nauseous, but this has passed. On physical examination, he is noted to have localized pain on cough and to be running a lowgrade fever. Question 4 of 5

The patient is prepared for immediate surgery. Cefotaxime is administered before, during, and after surgery. The specimen, once removed, is sent to the laboratory for pathology and bacteriologic culture. A malodorous pus surrounds the serosa of the surgical specimen, and a mixed gram-negative flora is cultured. Rapid enzyme tests for beta-Iactamase production are positive. Which of the following drugs should be added to the initial cefotaxime regimen? / A. Bacitracin / B. CIavulanic acid / C. CIindamycin / D. Isoniazid / E. Vancomycin Explanation - Q: 5.4

Close

The correct answer is B. Clavulanic acid is a beta-lactamase inhibitor, which when administered with beta lactam agents, irreversibly binds and inactivates bacterial beta-lactamases, thereby permitting the companion drug to disrupt bacterial cell wall synthesis. Suspected appendicitis is usually treated with prompt appendectomy, since delay is associated with increased risk of potentially life-threatening peritonitis and sepsis. Bacitracin (choice A) is not correct, since this drug inhibits bacterial cell wall synthesis by binding to and inhibiting the dephosphorylation of a membranebound lipid pyrophosphate. Gram-negative bacteria are resistant to this agent, and it would not have a synergistic effect if administered with a third generation cephalosporin. Clindamycin (choice C) is not correct, because this drug blocks protein elongation by binding to the 50S ribosome. Although it is effective against anaerobic gram-negative bacilli, it would not have a complementary effect

when administered with a third generation cephalosporin. Isoniazid (choice D) is not correct because it inhibits the synthesis of mycolic acids for the cell wall of actively dividing Mycobacteria. It would not be effective in the flora of this patient's gut, nor would it act synergistically with third generation cephalosporins. Vancomycin (choice E) is not correct because it disrupts cell wall synthesis in growing gram-positive bacteria. It would not be effective against the flora of this patient's gut, nor would it act synergistically with third generation cephalosporins. A 17-year-old boy is taken to the emergency department because he has developed severe abdominal pain. The pain began abruptly several hours previously, and was felt initially in the periumbilical region, but later shifted to the right lower quadrant. The boy had initially felt somewhat nauseous, but this has passed. On physical examination, he is noted to have localized pain on cough and to be running a lowgrade fever. Question 5 of 5

The patient's postoperative recovery is uneventfuI, but 10 days after discharge, he returns to his physician complaining of continuous lowgrade fever. An abscess is drained transrectally, and organisms are cultured from the pus. Which of the following is an attribute of this organism that makes it an important abscess former? / A. It is an anaerobe / B. It is an intracellular pathogen / C. Its endotoxin lacks 2,3-ketodeoxyoctonate / D. Mycolic acid / E. Prodigious capsule Explanation - Q: 5.5

Close

The correct answer is E. Prevotella (Bacteroides) is a frequent cause of abscesses in the intestinal tract because it is a normal flora organism and produces a large capsule, which impedes phagocytosis. Although the genus is anaerobic (choice A), it is not this attribute which causes its formation of abscesses. Prevotella is extracellular, not an intracellular pathogen (choice B). Although Prevotella does indeed have this type of endotoxin (choice C), the absence of this molecule decreases the toxicity of the toxin, and does not contribute to its proclivity toward abscess formation. Mycobacteria, and not other genera such as Prevotella, are known for their

long-chain fatty acids (mycolic acids; choice D).

The parents of a 13-month-old boy are contacted by a local social agency because the day care they are using has reported them for suspected child abuse. The abuse was suspected because the day care personnel thought that the toddler had "too many bruises." The parents deny the abuse, and promptly arrange to have their child seen by the chairman of the pediatrics department in a clinic run by a university medical center. On physical examination, the child is noted to have multiple large bruises on his legs, arms, and buttocks. No skin abrasions are seen, and no "pattern marks" (suggestive of being hit by an object such as a belt buckle or rod) are seen. A blood smear isunremarkable. PTT is prolonged, and PT and bleeding times are normaI. Question 1 of 6 Which of the following is the most likely diagnosis? / A. Acute myelogenous leukemia / B. Disseminated intravascular coagulation / C. Hemophilia / D. Thrombocytopenia / E. Von Willebrand's disease

Explanation - Q: 1.1

Close

The correct answer is C. This child has hemophilia, which is an X-linked recessive disorder of clotting. The severity of the symptoms experienced can vary with the degree of enzymatic block in the production of the affected clotting factor. Unfortunately, some cases may reach medical attention only when the child begins to experience large numbers of falls when learning to walk, and the parents are suspected of child abuse. The severity of the symptoms can vary markedly from case to case, since even as little as 2-5% of the normal levels of the missing clotting factor can markedly reduce symptoms. Symptoms can include bruising, spontaneous bleeding, bleeding into joints with associated pain and swelling, gastrointestinal and urinary tract hemorrhage, and prolonged bleeding from cuts, tooth extractions, or surgery. Acute myelogenous leukemia (choice A) and thrombocytopenia (choice D) can also produce a tendency to bruise, but would not be associated with normal peripheral smears. Disseminated intravascular coagulation (choice B) is usually seen in patients with other severe diseases, such as sepsis or complicated pregnancy. Von Willebrand's disease (choice E) tends to cause oozing blood after injury

rather than bruising, and is associated with a long bleeding time.

The parents of a 13-month-old boy are contacted by a local social agency because the day care they are using has reported them for suspected child abuse. The abuse was suspected because the day care personnel thought that the toddler had "too many bruises." The parents deny the abuse, and promptly arrange to have their child seen by the chairman of the pediatrics department in a clinic run by a university medical center. On physical examination, the child is noted to have multiple large bruises on his legs, arms, and buttocks. No skin abrasions are seen, and no "pattern marks" (suggestive of being hit by an object such as a belt buckle or rod) are seen. A blood smear is unremarkable. PTT is prolonged, and PT and bleeding times are normaI. Question 2 of 6

Deficiency of which of the following has been linked to this patient's condition? / A. Factor VII / B. Factor VIII / C. Factor X / D. Factor XII / E. Factor XIII Explanation - Q: 1.2

Close

The correct answer is B. Hemophilia can be caused by a deficiency of either factor VIII (classic hemophilia or hemophilia A) or factor IX (Christmas disease or hemophilia B). The two variations are very similar clinically, although the appropriate factor must be replaced when treating an acute bleeding episode. Both of these factors are in the intrinsic coagulation pathway. The other factors listed in the choices are not involved.

The parents of a 13-month-old boy are contacted by a local social agency because the day care they are using has reported them for suspected child abuse. The abuse was suspected because the day care personnel thought that the toddler had "too many bruises." The parents deny the abuse, and promptly arrange to have their child seen by the chairman of the pediatrics department in a clinic run by a university medical center. On physical examination, the child is noted to have multiple large bruises on his legs, arms, and buttocks. No skin abrasions are seen, and no "pattern marks" (suggestive of being hit by an object such as a belt buckle or rod) are seen. A blood smear is unremarkable. PTT is prolonged, and PT and bleeding times are normaI.

Question 3 of 6

This child's condition is due to a defective enzyme encoded on which of the following chromosomes? / A. 13 / B. 18 / C. 21 / D. X / E. Y Explanation - Q: 1.3

Close

The correct answer is D. Both the hemophilia A and hemophilia B genes are located on the X chromosome. While almost all affected individuals are boys (rare exceptions are girls with a carrier mother and an affected father), the Y chromosome (choice E) is not involved. Associate chromosome 13 (choice A) with the trisomy known as Patau syndrome. Associate chromosome 18 (choice B) with the trisomy known as Edwards syndrome. Associate chromosome 21 (choice C) with the trisomy known as Down syndrome. The parents of a 13-month-old boy are contacted by a local social agency because the day care they are using has reported them for suspected child abuse. The abuse was suspected because the day care personnel thought that the toddler had "too many bruises." The parents deny the abuse, and promptly arrange to have their child seen by the chairman of the pediatrics department in a clinic run by a university medical center. On physical examination, the child is noted to have multiple large bruises on his legs, arms, and buttocks. No skin abrasions are seen, and no "pattern marks" (suggestive of being hit by an object such as a belt buckle or rod) are seen. A blood smear is unremarkable. PTT is prolonged, and PT and bleeding times are normaI. Question 4 of 6

Which of the following can temporarily increase secretion of the needed factor in some patients with a milder form of this disease? / A. Desmopressin / B. Epsilon-aminocaproic acid / C. Factor VIII concentrate / D. Factor IX concentrate

/ E. Fresh frozen plasma

Explanation - Q: 1.4

Close

The correct answer is A. Desmopressin can temporarily raise factor VIII levels in a patient with mild classic hemophilia by causing the release of factor VIII stored in the body. The use of desmopressin before dental work or small surgical procedures may allow such procedures to be performed without using factor VIII concentrates. Epsilon-aminocaproic acid (choice B) is a anti-fibrinolytic agent used to slow the degradation of blood clots, particularly in patients with hemophilia B. Fresh frozen plasma (choice E) contains both factor VIII (choice C) and factor IX (choice D), both of which are also available as concentrates. These products are used to raise clotting factor levels, usually in patients who are either actively bleeding or who are expected to be actively bleeding because of surgery or dental procedures. The parents of a 13-month-old boy are contacted by a local social agency because the day care they are using has reported them for suspected child abuse. The abuse was suspected because the day care personnel thought that the toddler had "too many bruises." The parents deny the abuse, and promptly arrange to have their child seen by the chairman of the pediatrics department in a clinic run by a university medical center. On physical examination, the child is noted to have multiple large bruises on his legs, arms, and buttocks. No skin abrasions are seen, and no "pattern marks" (suggestive of being hit by an object such as a belt buckle or rod) are seen. A blood smear is unremarkable. PTT is prolonged, and PT and bleeding times are normaI. Question 5 of 6

The patient continues to experience persistent bleeding. Which of the following would be the most appropriate pharmacotherapy? / A. Aminocaproic acid / B. Aspirin / C. Desmopressin / D. High-dose IV immune globulin / E. Prednisone Explanation - Q: 1.5

Close

The correct answer is A. Aminocaproic acid is a synthetic hemostatic with a specific antifibrinolysis action. The medication inhibits plasminogen activator substance, which is concerned with the destruction of clots. This medication does not control bleeding caused by a loss of vascular integrity.

Aminocaproic acid is indicated for control of excessive bleeding resulting from systemic hyperfibrinolysis, well as for prevention of bleeding in hemophiliacs undergoing a surgical procedure. It is contraindicated for use in patients with severe renal impairment or active disseminated intravascular clotting. Aspirin (choice B) is absolutely contraindicated for use in this patient since it would result in a more severe bleed. Desmopressin is a synthetic analog of arginine vasopressin (choice C), the naturally occurring human antidiuretic hormone (ADH). This agent is generally used only in milder forms of the disease. High-dose IV immune globulin (choice D) is most commonly used in the treatment of ITP (idiopathic thrombocytopenic purpura) to rapidly raise the platelet count. Prednisone (choice E) is a glucocorticoid indicated for the treatment of idiopathic thrombocytopenic purpura (IV dosing only) as well as in the treatment of a variety of inflammatory conditions. It would be ineffective in this patient. The parents of a 13-month-old boy are contacted by a local social agency because the day care they are using has reported them for suspected child abuse. The abuse was suspected because the day care personnel thought that the toddler had "too many bruises." The parents deny the abuse, and promptly arrange to have their child seen by the chairman of the pediatrics department in a clinic run by a university medical center. On physical examination, the child is noted to have multiple large bruises on his legs, arms, and buttocks. No skin abrasions are seen, and no "pattern marks" (suggestive of being hit by an object such as a belt buckle or rod) are seen. A blood smear is unremarkable. PTT is prolonged, and PT and bleeding times are normaI. Question 6 of 6

In approximately what year did effective screening of blood products begin to limit the transmission of HIV and hepatitis B in these patients? / A. 1955 / B. 1965 / C. 1975 / D. 1985 / E. 1995 Explanation - Q: 1.6

Close

The correct answer is D. This is clinically important information, because hemophiliac children who were diagnosed after 1985 have had very little chance of developing AIDS or hepatitis as a complication of their disease (they may still get exposures by other routes, e.g., sexual), while these were major problems in patients who were treated earlier. Fortunately, recently developed recombinant DNA factor VIII products are coming into increasing use, and offer the chance of providing truly adequate hemostatic control without any increased viral risk. Following a viral illness, a previously healthy 3-year-old child develops multiple petechiae and purpura, and is noted to be oozing blood from the mouth. Prothrombin and activated partial thromboplastin times are within normal limits. Review of a peripheral smear demonstrates that platelet counts are markedly decreased, but all the platelets that are present appear morphologically normaI. The remainder of the blood smear and additional bone marrow studies are normaI. Question 1 of 4

Which of the following is the most likely diagnosis? / A. Disseminated intravascular coagulation / B. Hemophilia A / C. Hemophilia B / D. Idiopathic thrombocytopenic purpura / E. Von Willebrand disease Explanation - Q: 1.1

Close

The correct answer is D. Idiopathic thrombocytopenic purpura (ITP), also known as immune thrombocytopenic purpura, now that the immunologic basis of the disease has been better defined, is a hemorrhagic disorder that can present in two very different forms in children and adults. Children (peak ages 2-4 years) tend to present with acute, self-limited thrombocytopenia that often follows a viral (or immunization) trigger. In contrast, adults (peak ages 20-50 years) tend to have an insidious development of thrombocytopenia that may persist for long periods. In both types of ITP, the hemorrhagic diathesis is the result of a pure thrombocytopenia that does not affect prothrombin (PT) and activated partial thromboplastin (PTT) times because the blood clotting factors are present in normal quantities. In disseminated intravascular coagulation (choice A), platelets are often decreased as a result of consumption, but PT and PTT are prolonged. In hemophilia A and B (choices B and C), platelets are present in normal numbers, PT is normal, and PTT is long.

In von Willebrand disease (choice E), platelet counts are normal, but platelet function is decreased because of decreased or abnormal von Willebrand factor. Following a viral illness, a previously healthy 3-year-old child develops multiple petechiae and purpura, and is noted to be oozing blood from the mouth. Prothrombin and activated partial thromboplastin times are within normal limits. Review of a peripheral smear demonstrates that platelet counts are markedly decreased, but all the platelets that are present appear morphologically normaI. The remainder of the blood smear and additional bone marrow studies are normaI. Question 2 of 4

Which of the following tests would be most helpful in confirming the probable diagnosis? / A. D-dimer levels / B. Factor VIII concentration / C. Factor IX concentration / D. IgG autoantibodies on platelet surfaces / E. Total plasma von Willebrand factor antigen Explanation - Q: 1.2

Close

The correct answer is D. While ITP is mostly a diagnosis of exclusion, the presence of IgG autoantibodies on the platelet surface can be an important confirmatory test result. High D-dimer levels (choice A) suggest disseminated intravascular coagulation. Low factor VIII (choice B) suggests hemophilia A. Low factor IX (choice C) suggests hemophilia B. Low or abnormal von Willebrand factor antigen (choice E) suggests von Willebrand disease.

Following a viral illness, a previously healthy 3-year-old child develops multiple petechiae and purpura, and is noted to be oozing blood from the mouth. Prothrombin and activated partial thromboplastin times are within normal limits. Review of a peripheral smear demonstrates that platelet counts are markedly decreased, but all the platelets that are present appear morphologically normaI. The remainder of the blood smear and additional bone marrow studies are normaI.

Question 3 of 4

Most patients with this condition have antibodies to which of the following? / A. PIatelet ADP / B. PIatelet membrane cholesterol / C. PIatelet membrane glycoproteins / D. PIatelet membrane lipids / E. PIatelet thromboxane A2 Explanation - Q: 1.3

Close

The correct answer is C. The autoantibodies are usually directed against platelet membrane glycoproteins. ADP (choice A) and thromboxane A2 (choice E) are involved in platelet aggregation. Cholesterol (choice B) and lipids (choice D) are components of virtually all mammalian plasma membranes. Following a viral illness, a previously healthy 3-year-old child develops multiple petechiae and purpura, and is noted to be oozing blood from the mouth. Prothrombin and activated partial thromboplastin times are within normal limits. Review of a peripheral smear demonstrates that platelet counts are markedly decreased, but all the platelets that are present appear morphologically normaI. The remainder of the blood smear and additional bone marrow studies are normaI. Question 4 of 4

Approximately what percentage of children with this condition will undergo spontaneous remission? / A. Less than 5% / B. 15% / C. 30% / D. 60% / E. More than 80% Explanation - Q: 1.4

Close

The correct answer is E. More than 80% of childhood cases of ITP undergo spontaneous remission, which frequently occurs within 2 months. Spontaneous remission is uncommon (2%) in adults, but more than 60% will eventually recover with treatment. Treatment in children is controversial; treatment in adults can involve splenectomy, glucocorticoids, and intravenous IgG. Approximately 1% of children with ITP and 5% of adults

with ITP die from hemorrhagic complications, including intracranial hemorrhage. A 23-year-old man is admitted to the hospital through the emergency department with probable appendicitis. He has been having right lower quadrant abdominal pain for several days, which has been becoming increasingly worse. His temperature 39.2 C (102.6 F), blood pressure is 80/40 mm Hg, pulse is 120/min, and respirations are 35/min. The abdomen is rigid with guarding. Multiple petechiae and purpura are present, and the patient is oozing blood from his oral mucosa. According to the patient's wife, he has not had bleeding problems in the past. Question 1 of 5

The fact the abdomen is rigid with guarding suggests which of the following? / A. Colon cancer / B. Diverticulitis / C. Liver failure / D. Peritonitis / E. Shock Explanation - Q: 2.1

Close

The correct answer is D. The usual reason for a patient to have a rigid abdomen is that peritonitis is present, and is causing severe pain related to peritoneal nerve fiber stimulation. The probable cause of the peritonitis is a ruptured appendix. This patient is also probably in shock (choice E), as indicated by the hypotension with increased respirations and heart rate, but this would not cause the abdominal guarding. The other answers are distracters. A 23-year-old man is admitted to the hospital through the emergency department with probable appendicitis. He has been having right lower quadrant abdominal pain for several days, which has been becoming increasingly worse. His temperature 39.2 C (102.6 F), blood pressure is 80/40 mm Hg, pulse is 120/min, and respirations are 35/min. The abdomen is rigid with guarding. Multiple petechiae and purpura are present, and the patient is oozing blood from his oral mucosa. According to the patient's wife, he has not had bleeding problems in the past. Question 2 of 5

Given this patient's presentation, which of the following is the most likely cause of his petechiae, purpura, and mucosal blood oozing? /

A. Disseminated intravascular coagulation / B. Hemophilia A

/ C. Idiopathic thrombocytopenic purpura / D. Von Willebrand disease / E. Wiskott-AIdrich syndrome

Explanation - Q: 2.2

Close

The correct answer is A. You should suspect disseminated intravascular coagulation (DIC) in patients who are seriously ill with other disease who then develop evidence of a coagulopathy. In this case, sepsis is the probable inciting event. DIC is thought to occur in 30-50% of patients with sepsis, and may, in many cases, be triggered by a reaction to gram-negative or staphylococcal cell wall material. Other settings in which DIC can be a complication include major trauma, obstetric complications, acute myelocytic leukemias, disseminated carcinomas, burns, massive transfusions, acute hepatic failure, myocardial infarction, and inflammatory conditions (e.g., ulcerative colitis, rheumatoid arthritis, Crohn disease, sarcoidosis). The other conditions listed in the choices would not be expected to be specifically associated with sepsis, although for medical/legal and safety reasons, supporting diagnostic tests should be ordered to exclude previously undiagnosed disease. Hemophilia (choice B) is a hereditary coagulopathy with X-linked genetics that typically presents in early childhood with extensive tissue hemorrhages and hemoarthroses following minor trauma. Idiopathic thrombocytopenic purpura (choice C) is an acquired autoimmune disease characterized, in adults, by chronic coagulopathy with petechiae, purpura, and mucosal bleeding. It is not usually associated with any other systemic diseases, such as this patient's appendicitis and probable sepsis. Von Willebrand disease (choice D) is hereditary, with usually autosomal dominant genetics. It typically causes prolonged menstrual bleeding and prolonged bleeding from small cuts and surgical procedures. Petechiae and ecchymoses are less often seen. Wiskott-Aldrich syndrome (choice E) is a hereditary disease with X-linked genetics, thrombocytopenia, eczema, and recurrent infection that tends to present in children with bloody diarrhea. A 23-year-old man is admitted to the hospital through the emergency department with probable appendicitis. He has been having right lower quadrant abdominal pain for several days, which has been becoming increasingly worse. His temperature 39.2 C (102.6 F), blood pressure is 80/40 mm Hg, pulse is 120/min, and respirations are 35/min. The abdomen is rigid with guarding. Multiple petechiae and purpura are

present, and the patient is oozing blood from his oral mucosa. According to the patient's wife, he has not had bleeding problems in the past. Question 3 of 5

Which of the following findings on screening laboratory tests would be most consistent with the probable diagnosis? / A. Decreased platelet count, increased prothrombin time, increased activated partial thromboplastin time / B. Increased platelet count, increased prothrombin time, normal activated partial thromboplastin time / C. Increased platelet count, normal prothrombin time, increased activated partial thromboplastin time / D. Normal platelet count, increased prothrombin time, increased activated partial thromboplastin time / E. Normal platelet count, normal prothrombin time, normal activated partial thromboplastin time Explanation - Q: 2.3

Close

The correct answer is A. Platelet count is invariably decreased (and usually obvious on peripheral smear) in DIC, and this is one of the more reliable screening tests for the condition (although it does not exclude many other diagnoses). Both prothrombin time and activated partial thromboplastin times are often prolonged, but you should be aware that they might each be shortened or normal as well in DIC. Thus, these two commonly performed tests are actually not as useful as they might appear. The other choices listed are wrong because the platelet counts are not decreased. A 23-year-old man is admitted to the hospital through the emergency department with probable appendicitis. He has been having right lower quadrant abdominal pain for several days, which has been becoming increasingly worse. His temperature 39.2 C (102.6 F), blood pressure is 80/40 mm Hg, pulse is 120/min, and respirations are 35/min. The abdomen is rigid with guarding. Multiple petechiae and purpura are present, and the patient is oozing blood from his oral mucosa. According to the patient's wife, he has not had bleeding problems in the past. Question 4 of 5

For confirmation of the probable diagnosis, the emergency department physician orders a D-dimer test. D-dimer is which of the following? / A. A clotting factor in the common coagulation pathway / B. A clotting factor in the extrinsic coagulation pathway / C. A clotting factor in the intrinsic coagulation pathway / D. A factor found in the complement pathway / E. A fibrin degradation product Explanation - Q: 2.4

Close

The correct answer is E. A variety of specialized tests have been developed to help in the diagnosis of disseminated intravascular coagulation. Unfortunately, many of these are only available in major medical centers or through reference laboratories, which markedly limits their usefulness in emergency rooms elsewhere. One of the more available and useful of these tests is the D-dimer tests, which measures a fibrin degradation product that is made when a cross-linked clot (indicating the presence of thrombin to form the clots) is lysed by plasmin. One way to understand disseminated intravascular coagulation is to think of it as primarily a thrombotic problem, with excessive clot formation and then lysis, which leads secondarily to a hemorrhagic problem as platelets and clotting factors are consumed. High Ddimer levels indicate that clots are forming and then lysing at an unusual rate in the body, and thus, together with a low platelet count, suggest that DIC is present. Functionally, DIC can be subdivided into acute and chronic presentations, which tend to occur in somewhat different settings. In the acute presentations, the hemorrhagic processes often dominate the clinical difficulties, while in the chronic presentations, the clotting tendencies often dominate the picture. The D-dimers are not in themselves factors (e.g., substances to be consumed) in either the coagulation or complement pathways. A 23-year-old man is admitted to the hospital through the emergency department with probable appendicitis. He has been having right lower quadrant abdominal pain for several days, which has been becoming increasingly worse. His temperature 39.2 C (102.6 F), blood pressure is 80/40 mm Hg, pulse is 120/min, and respirations are 35/min. The abdomen is rigid with guarding. Multiple petechiae and purpura are present, and the patient is oozing blood from his oral mucosa. According to the patient's wife, he has not had bleeding problems in the past. Question 5 of 5

Which of the following would be most effective in diminishing the rate at which clot formation is occurring? / A. Aminocaproic acid / B. Heparin / C. PIatelet transfusion / D. Red cell transfusion / E. Tranexamic acid Explanation - Q: 2.5

Close

The correct answer is B. The therapy of disseminated intravascular coagulation is complex and problematic, and, frankly, not always successful. The patients are often already critically ill, and the DIC they have developed often contributes to major organ failure. The basic problem in therapy is how to treat simultaneous bleeding and clotting tendencies. Paradoxically, while

the bleeding tendency may be appearing to dominate the clinical picture, treatment with the anticoagulant heparin may actually help the patient by decreasing the rate at which the clotting factors are being consumed. This must, of course, be done very carefully, since an "overshoot" may exacerbate the patient's problems. Blood components including platelet transfusions (choice C), red cell transfusions (choice D), and cryoprecipitate are also sometimes used to restore the blood to a closer to normal state. As a last resort when other measures have failed, some physicians will also try antifibrinolytic agents such as aminocaproic acid (choice A) and tranexamic acid (choice E) to try to keep clots from dissolving and exacerbating the bleeding tendency.

A 7-year-old girl is evaluated by a pediatrician for a possible bleeding disorder. The girI's father and grandmother have both had life-Iong histories of easy bruising. On physical examination, the child is noted to have multiple small bruises over her body, and the mother reports that she will frequently bleed for most of an hour following a small cut. The child has also had numerous nosebleeds that were stopped only when the nose was packed with facial tissue. CBC is normal on all measured values. The peripheral smear shows the usual number and morphology of platelets. Template bleeding time is longer than normaI. Prothrombin time is within the reference range. Activated partial thromboplastin time is slightly longer than normaI. Question 1 of 5

Which of the following is the most likely diagnosis? / A. Disseminated intravascular coagulation / B. Hemophilia A / C. Hemophilia B / D. Idiopathic thrombocytopenic purpura / E. Von Willebrand disease Explanation - Q: 3.1

Close

The correct answer is E. Von Willebrand disease is a usually autosomal dominant condition in which there is either a quantitative or qualitative abnormality of von Willebrand factor (VWF) production. You should also be aware that the disease can uncommonly be autosomal recessive or acquired (associated with Wilms tumor, congenital heart disease, systemic lupus erythematosus, seizures treated with valproic acid, and hypothyroidism). The presentation and laboratory findings illustrated are typical. The disease is life-long, and patients may also present with menorrhagia, post-operative bleeding, postpartum bleeding, or gingival bleeding. Activated partial thromboplastin time may be either normal or prolonged, depending on the degree to which the intrinsic blood coagulation pathway has been inhibited

by decreased binding of factor VIII to von Willebrand factor. Disseminated intravascular coagulation (choice A) and idiopathic thrombocytopenic purpura (choice D) are both acquired conditions. Hemophilia A and B (choice B and C) have X-linked genetics. A 7-year-old girl is evaluated by a pediatrician for a possible bleeding disorder. The girI's father and grandmother have both had life-Iong histories of easy bruising. On physical examination, the child is noted to have multiple small bruises over her body, and the mother reports that she will frequently bleed for most of an hour following a small cut. The child has also had numerous nosebleeds that were stopped only when the nose was packed with facial tissue. CBC is normal on all measured values. The peripheral smear shows the usual number and morphology of platelets. Template bleeding time is longer than normaI. Prothrombin time is within the reference range. Activated partial thromboplastin time is slightly longer than normaI. Question 2 of 5

Multimers of all sizes of von Willebrand's factor form complexes in plasma with which of the following? / A. Factor V / B. Factor VI / C. Factor VIII / D. Factor IX / E. Factor X Close Explanation - Q: 3.2 The correct answer is C. Normal plasma levels of clotting factor VIII are maintained by complexing with multimers of Von Willebrand factor. This protects factor VIII from degradation and helps to deliver it to the site of injury. This is the contribution VWF makes to the intrinsic pathway of coagulation. The other factors listed do not complex with VWF.

A 7-year-old girl is evaluated by a pediatrician for a possible bleeding disorder. The girI's father and grandmother have both had life-Iong histories of easy bruising. On physical examination, the child is noted to have multiple small bruises over her body, and the mother reports that she will frequently bleed for most of an hour following a small cut. The child has also had numerous nosebleeds that were stopped only when the nose was packed with facial tissue. CBC is normal on all measured values. The peripheral smear shows the usual number and morphology of platelets. Template bleeding time is longer than normaI. Prothrombin time is within the reference range. Activated partial thromboplastin time is slightly longer than normaI.

Question 3 of 5

Very large von Willebrand's factor multimers contribute most to which of the following platelet activities? / A. Normal adherence of platelets to damaged blood vessels / B. Normal budding of platelets from megakaryocytes / C. Normal digestion of platelets during clot lysis / D. Normal discharge of granules from platelets / E. Normal transport of platelets in the blood Explanation - Q: 3.3

Close

The correct answer is A. The typical platelet life span begins in the bone marrow when platelets bud off from megakaryocytes, and are released to float freely in the blood stream. When a damaged blood vessel is encountered, VWF acts as an adhesive bridge between the platelets and the damaged subendothelium at the site of vascular injury. The platelets subsequently contract and release granules to help in formation of the clot. Eventually, the clot lyses, and macrophages consume the platelet debris. A 7-year-old girl is evaluated by a pediatrician for a possible bleeding disorder. The girI's father and grandmother have both had life-Iong histories of easy bruising. On physical examination, the child is noted to have multiple small bruises over her body, and the mother reports that she will frequently bleed for most of an hour following a small cut. The child has also had numerous nosebleeds that were stopped only when the nose was packed with facial tissue. CBC is normal on all measured values. The peripheral smear shows the usual number and morphology of platelets. Template bleeding time is longer than normaI. Prothrombin time is within the reference range. Activated partial thromboplastin time is slightly longer than normaI. Question 4 of 5

Which of the following is the binding site for the von Willebrand's factor multimers on the platelets? / A. ADP / B. Factor VIII / C. GIycoprotein 1b / D. Thromboxane A2 / E. Prostaglandin l2 Explanation - Q: 3.4 The correct answer is C. Von Willebrand factor binds on platelets to the specific receptor glycoprotein 1b when acting as a bridge between the platelets and the site of vascular injury. ADP (choice A) is released by damaged erythrocytes and helps VWF to adhere to the exposed collagen. It is not, however, the binding factor that

Close

connects the VWF to the platelets. Factor VIII (choice B) does not participate directly in the platelet portion of blood clot formation. Thromboxane A2(choice D) promotes subendothelial exposure after vascular injury and induces platelet aggregation. Prostaglandin I2(choice E) inhibits subendothelial exposure after minor vascular injury. A 7-year-old girl is evaluated by a pediatrician for a possible bleeding disorder. The girI's father and grandmother have both had life-Iong histories of easy bruising. On physical examination, the child is noted to have multiple small bruises over her body, and the mother reports that she will frequently bleed for most of an hour following a small cut. The child has also had numerous nosebleeds that were stopped only when the nose was packed with facial tissue. CBC is normal on all measured values. The peripheral smear shows the usual number and morphology of platelets. Template bleeding time is longer than normaI. Prothrombin time is within the reference range. Activated partial thromboplastin time is slightly longer than normaI. Question 5 of 5

In an individual with one of the milder forms of the disease presented in this case, which of the following medications could be used to increase von Willebrand's factor and factor VIII after treatment? / A. Aspirin / B. Aminocaproic acid / C. Desmopressin / D. Ibuprofen / E. Naproxen Explanation - Q: 3.5

Close

The correct answer is C. Desmopressin is a synthetic analogue of antidiuretic hormone that can be administered intravenously, intranasally, or subcutaneously, and is frequently used as the primary treatment for bleeding in individuals with mild von Willebrand disease. Its function in this disease is to cause release of VWF from endothelial storage sites, so its use is reserved for acute treatment of bleeding symptoms (including home treatment with the high dose intranasal preparation), rather than as a routine medication. Factor VIII concentrates that also contain VWF in high molecular weight form are used in the treatment of patients with the more severe forms of Von Willebrand disease. With treatment, the majority of patients, particularly those with milder forms of Von Willebrand disease, have normal

life spans. Aminocaproic acid (choice B) is an inhibitor of fibrinolysis that is used to maintain clots in some individuals with von Willebrand disease. Drugs with platelet inhibiting activities, including aspirin (choice A), ibuprofen (choice D), and naproxen (choice E), should be avoided in patients with disorders of hemostasis.

A child is taken to a pediatrician because the mother is concerned that the child has been losing, rather than gaining weight, and has been having numerous infections. After a complete physical examination with screening serum chemistry and blood studies, the physician suspects that the child has a malignancy. Question 1 of 5

Which of the following is the most common malignancy of childhood? / A. Ewing sarcoma / B. Leukemia / C. Neuroblastoma / D. Retinoblastoma / E. Wilms tumor Explanation - Q: 1.1

Close

The correct answer is B. Leukemia is the most common malignancy of childhood. The other malignancies listed are also characteristically childhood diseases, but occur less frequently. Ewing sarcoma (choice A) can involve bone or soft tissues; Neuroblastoma (choice C) can involve the adrenal gland or the extraadrenal sympathetic chain. Retinoblastoma (choice D) involves the eye, and Wilms tumor (choice E) involves the kidney. A child is taken to a pediatrician because the mother is concerned that the child has been losing, rather than gaining weight, and has been having numerous infections. After a complete physical examination with screening serum chemistry and blood studies, the physician suspects that the child has a malignancy. Question 2 of 5

The complete blood count shows anemia, thrombocytopenia, and a white count of 110,000/mm3. The peripheral smear contains numerous blasts, which are smalI, intensely blue cells with homogeneous chromatin, regular nuclear shape, small or absent nucleolus, and scanty blue cytoplasm. This patient's disease most likely will be classified as which of the following types? / A. L1 / B. L2 / C. L3 / D. M3 / E. M7 Explanation - Q: 1.2

Close

The correct answer is A. These blasts cells are the lymphoid L1 cells, and the child has ALL, type L1. L1 is one of the French-American-British classifications for lymphoblastic leukemia, which also includes L2 and L3 types. L2 (choice B) cells are large and heterogenous blast class with heterogeneous chromatin, irregular nuclear shape, and large nucleolus. L3 (choice C) cells are large and homogeneous cells whose most prominent feature is cytoplasmic vacuolization that may overlie the nucleus. A French-American-British classification has also been used for acute myeloid leukemias, and different types are given different M designations. M3 (choice D) is acute promyelocytic leukemia, and the cells have abundant cytoplasm that often contains numerous Auer rods (small rod like structures). M7 (choice E) is megakaryoblastic leukemia, and the cells are very large and maybe multinucleated, resembling megakaryocytes.

A child is taken to a pediatrician because the mother is concerned that the child has been losing, rather than gaining weight, and has been having numerous infections. After a complete physical examination with screening serum chemistry and blood studies, the physician suspects that the child has a malignancy. Question 3 of 5

Which of the following is the most common form of leukemia in childhood? / A. Acute lymphoblastic leukemia / B. Acute myeloblastic leukemia / C. Chronic lymphocytic leukemia / D. Chronic myelogenous leukemia / E. Erythroleukemia Explanation - Q: 1.3

Close

The correct answer is A. Acute lymphoblastic leukemia is the most common form of leukemia in children, and accounts for nearly 1/3 of all pediatric cancers. A child is taken to a pediatrician because the mother is concerned that the child has been losing, rather than gaining weight, and has been having numerous infections. After a complete physical examination with screening serum chemistry and blood studies, the physician

suspects that the child has a malignancy. Question 4 of 5

What is the peak age of incidence of the most common form of this disease in children? / A. Less than 6 months / B. 8 -14 months / C. 2-5 years / D. 10-12 years / E. More than 15 years Explanation - Q: 1.4

Close

The correct answer is C. While acute lymphoblastic leukemia can occur at any age, its peak age of incidence is 2 to 5 years. A child is taken to a pediatrician because the mother is concerned that the child has been losing, rather than gaining weight, and has been having numerous infections. After a complete physical examination with screening serum chemistry and blood studies, the physician suspects that the child has a malignancy. Question 5 of 5

What is the cure rate of the most common form of this disease in children? / A. Less than 5% / B. 10-25% / C. 35-50% / D. 70-80% / E. More than 95% Explanation - Q: 1.5

Close

The correct answer is D. With modern chemotherapeutic regimens, we have now achieved a 70-80% cure rate of acute lymphoblastic leukemia. The medical regimens with which we treat these children may include corticosteroids (prednisone, dexamethasone), anti-neoplastic agents (vincristine, asparaginase, daunorubicin, 6-mercaptopurine, cytarabine, etoposide, cyclophosphamide), and prophylactic antimicrobials (sulfamethoxazole and trimethoprim, nystatin, clotrimazole, itraconazole). Radiotherapy is also sometimes used in these patients. Complications of acute lymphoblastic leukemia or its treatment can include suppression of normal bone marrow elements (predisposing for anemia, neutropenia, and thrombocytopenia), tumor lysis syndrome (which produces hyperuricemia and electrolyte abnormalities and can be prevented or ameliorated with hydration and allopurinol), neurotoxicity (which may cause a mild mental retardation and poor emotional control), and secondary malignancies. A 22-year-old man has been having fatigue and headaches. On physical examination, the patient is noted to be pale and has a petechial

rash. A complete blood count and peripheral smear show markedly decreased numbers of platelets, red blood cells, granulocytes, monocytes, and reticulocytes. The erythrocyte morphology is normaI. Bone marrow aspiration and biopsy show a marrow with 10% cellularity, the rest of the marrow being occupied principally by fat cells. Question 1 of 5

Which of the following is the most likely diagnosis? / A. Aplastic anemia / B. Folate deficiency anemia / C. Iron deficiency anemia / D. Sickle cell anemia / E. Thalassemia Explanation - Q: 2.1

Close

The correct answer is A. This patient most likely has aplastic anemia, which is defined as an anemia that involves all cell lines produced in the marrow. The underlying problem is an impaired ability of the most primitive stem cells in the bone marrow (from which all the blood cell lines except lymphocytes are derived) to reproduce. Patients may present with symptoms related to anemia (pallor, headache, palpitations, dyspnea, or fatigue), thrombocytopenia (mucosal bleeding or petechiae), or neutropenia (infections, mouth and pharyngeal ulcerations). There is a double peak of incidence, in the early 20's and then in older adulthood. Folate deficiency (choice B) produces a megaloblastic anemia, and altered neutrophils with hypersegmentation of the nucleus may be seen, but the marrow will not be hypocellular. Iron deficiency anemia (choice C) tends to be microcytic, and does not affect cell lines other than erythrocytes. Sickle cell anemia (choice D) has bizarre-shaped erythrocytes and affects only the erythrocyte line. Thalassemia (choice E) is characterized by hypochromic erythrocytes that may have a "target" appearance, and does not affect cell lines other than the erythrocytic line. A 22-year-old man has been having fatigue and headaches. On physical examination, the patient is noted to be pale and has a petechial rash. A complete blood count and peripheral smear show markedly decreased numbers of platelets, red blood cells, granulocytes, monocytes, and reticulocytes. The erythrocyte morphology is normaI. Bone marrow aspiration and biopsy show a marrow with 10% cellularity, the rest of the marrow being occupied principally by fat cells. Question 2 of 5

Which of the following is noted for sometimes causing this patient's condition? / A. Asbestos / B. Benzene / C. Lead / D. Mercury / E. Methyl alcohol Explanation - Q: 2.2

Close

The correct answer is B. Approximately 80% of cases of aplastic anemia are acquired, with the remainder being congenital. Among the acquired cases, often no inciting cause can be identified, and an autoimmune basis appears probable. However, in some cases, an inciting cause can be identified, and in these, it is important to recognize and if possible remove the cause. Benzene is the chemical most commonly cited as being associated with development of aplastic anemia. Associate asbestos (choice A) with pulmonary fibrosis, mesothelioma, and lung cancer. Associate lead poisoning (choice C) with a hypochromic, microcytic anemia, abdominal discomfort, and CNS effects. Associate mercury poisoning (choice D) with (depending upon the route of exposure) GI disturbance, pneumonitis, renal failure, and CNS involvement. Associate methyl alcohol (choice E) with visual system and central nervous system damage.

A 22-year-old man has been having fatigue and headaches. On physical examination, the patient is noted to be pale and has a petechial rash. A complete blood count and peripheral smear show markedly decreased numbers of platelets, red blood cells, granulocytes, monocytes, and reticulocytes. The erythrocyte morphology is normaI. Bone marrow aspiration and biopsy show a marrow with 10% cellularity, the rest of the marrow being occupied principally by fat cells. Question 3 of 5

Which of the following medications has been associated with the development of this patient's condition? / A. Amyl nitrate / B. Aspirin / C. Chloramphenicol / D. CIofazimine / E. Dapsone

Explanation - Q: 2.3

Close

The correct answer is C. Drugs noted for causing aplastic anemia include chloramphenicol, phenylbutazone, and gold. Cases triggered by idiosyncratic reaction to drugs appear not to involve an immune mechanism, but may be a result of direct toxicity, possibly involving altered detoxification. Amyl nitrate (choice A) does not have any hematologic side effects. Aspirin (choice B) can cause leukopenia, thrombocytopenia, and prolongation of bleeding time, but not aplastic anemia. Clofazimine (choice D) does not have any hematologic side effects. Dapsone (choice E) can cause dose-related hemolysis in patients with or without glucose-6-phosphatase deficiency. A 22-year-old man has been having fatigue and headaches. On physical examination, the patient is noted to be pale and has a petechial rash. A complete blood count and peripheral smear show markedly decreased numbers of platelets, red blood cells, granulocytes, monocytes, and reticulocytes. The erythrocyte morphology is normaI. Bone marrow aspiration and biopsy show a marrow with 10% cellularity, the rest of the marrow being occupied principally by fat cells. Question 4 of 5

With supportive care alone, this patient's condition is estimated to have which of the following mortality rates? / A. Less than 5% / B. 10-25% / C. 30-40% / D. 50-60% / E. More than 70% Explanation - Q: 2.4

Close

The correct answer is E. Supportive care consists primarily of appropriate transfusions to replace life-threateningly low red cell counts or platelet counts. Supportive care alone is no longer considered adequate therapy, since aplastic anemia has a greater than 70% mortality rate if that is the only therapy offered. A 22-year-old man has been having fatigue and headaches. On physical examination, the patient is noted to be pale and has a petechial rash. A complete blood count and peripheral smear show markedly decreased numbers of platelets, red blood cells, granulocytes,

monocytes, and reticulocytes. The erythrocyte morphology is normaI. Bone marrow aspiration and biopsy show a marrow with 10% cellularity, the rest of the marrow being occupied principally by fat cells. Question 5 of 5

If treated with immunosuppression, the five year survival rate is approximately which of the following? / A. 5% / B. 25% / C. 50% / D. 75% / E. 95% Explanation - Q: 2.5

Close

The correct answer is D. Patients treated with immunosuppression have an estimated 75% 5-year-survival rate, and those lucky individuals who have matched sibling donor bone marrow transplant have a more than 90% 5year-survival. Colony stimulating cytokines (G-CSF and GM-CSF) are now commercially available, and can also have great value in stimulating any residual growth capacity the stem cells have. A complete blood count (CBC) and a blood smear are taken from a patient who has been complaining of fatigue. The CBC shows a white count of 200,000/mm3, with differential count including 70% segmented neutrophils and 15% band forms. The absolute numbers of basophils and eosinophils are increased somewhat. Review of the peripheral smear does not show any blast forms. Question 1 of 8

Which of the following is the most likely diagnosis? / A. Acute lymphocytic leukemia / B. Acute myelogenous leukemia / C. Chronic lymphocytic leukemia / D. Chronic myelogenous leukemia / E. Hodgkin disease Explanation - Q: 3.1

Close

The correct answer is D. This patient most likely has chronic myelogenous leukemia (CML). This is a cancer of pluripotent marrow stem cells characterized by a proliferation, predominantly of granulocytes, although other cell lines (megakaryocytes, monocytes, erythrocytes, and even some T and B cell lines) may also be increased. Blast cells would be seen in the peripheral smear in acute myelogenous leukemia (choice B) and acute lymphocytic leukemia (choice A). In chronic lymphocytic leukemia (choice C), the numbers of lymphocytes are

increased and the numbers of other blood cells may be decreased. Hodgkin disease (choice E) may show a modest increase in neutrophils and eosinophils in the blood, but would not have the dramatic increase seen in this patient. A complete blood count (CBC) and a blood smear are taken from a patient who has been complaining of fatigue. The CBC shows a white count of 200,000/mm3, with differential count including 70% segmented neutrophils and 15% band forms. The absolute numbers of basophils and eosinophils are increased somewhat. Review of the peripheral smear does not show any blast forms. Question 2 of 8

Which of the following is the median age for presentation of this disease? / A. 1 year / B. 5 years / C. 25 years / D. 45 years / E. 75 years Explanation - Q: 3.2

Close

The correct answer is D. Chronic myelogenous leukemia is primarily a disease of middle age, with a median age of 45 years. The comparatively few cases occurring in the elderly are more likely to be aggressive than those occurring in middle-aged adults. A complete blood count (CBC) and a blood smear are taken from a patient who has been complaining of fatigue. The CBC shows a white count of 200,000/mm3, with differential count including 70% segmented neutrophils and 15% band forms. The absolute numbers of basophils and eosinophils are increased somewhat. Review of the peripheral smear does not show any blast forms. Question 3 of 8

This patient's disease can be confused in some settings with polycythemia vera. Which of the following tests would be most helpful in distinguishing between these two diseases? / A. Leukocyte alkaline phosphatase / B. Myelin basic protein / C. Protein C activity / D. ScI-70 / E. Tartrate resistant acid phosphatase Explanation - Q: 3.3 The correct answer is A. While to the novice, polycythemia vera and

Close

chronic myelogenous leukemia appear to be obviously different diseases, both cause an increase in many marrow cell lines, and ambiguous cases do exist. Leukocyte alkaline phosphatase is a helpful test to perform in these settings, because it is characteristically low in chronic myelogenous leukemia, but high in polycythemia vera. Also, polycythemia vera does not usually have the Philadelphia chromosome. Myelin basic protein (choice B) is used in the diagnosis of multiple sclerosis. Protein C (choice C) is involved in the clotting mechanism. Scl-70 (choice D) is an antibody often present in scleroderma. Associate tartrate-resistant acid phosphatase (choice E) positivity with hairy cell leukemia. A complete blood count (CBC) and a blood smear are taken from a patient who has been complaining of fatigue. The CBC shows a white count of 200,000/mm3, with differential count including 70% segmented neutrophils and 15% band forms. The absolute numbers of basophils and eosinophils are increased somewhat. Review of the peripheral smear does not show any blast forms. Question 4 of 8

Which of the following organs is most commonly significantly enlarged in these patients? / A. Liver / B. Pancreas / C. Stomach / D. Spleen / E. Thymus Explanation - Q: 3.4

Close

The correct answer is D. Splenomegaly is common in patients with chronic myelogenous leukemia, and in more than half of the patients, is palpable more than 5 cm below the costal margin at the time of diagnosis. Splenomegaly may be a very helpful sign on physical examination, since it is uncommon in reactive leukocytosis. Hepatomegaly (choice A) can also occur, but is less common. The other answers are distracters. A complete blood count (CBC) and a blood smear are taken from a patient who has been complaining of fatigue. The CBC shows a white count of 200,000/mm3, with differential count including 70% segmented neutrophils and 15% band forms. The absolute numbers of basophils and eosinophils are increased somewhat. Review of the peripheral smear does not show any blast forms.

Question 5 of 8

Which of the following translocations is very common in this patient's disease? / A. t(8;14) / B. t(8;21) / C. t(9;22) / D. t(12;21) / E. t(15;17) Explanation - Q: 3.5

Close

The correct answer is C. The characteristic chromosomal translocation of chronic myelogenous leukemia is t(9;22)(q34;q11). This translocation results in a distinctive abnormal chromosome sometimes called the Philadelphia chromosome, and is seen in 90-95% of CML cases. The Philadelphia translocation is also seen in 20-25% of adults with acute lymphoblastic leukemia, 2% of adults with acute myeloid leukemia, and 2-10% of childhood acute lymphoblastic leukemia. Some authors think that these cases may represent variants of chronic myelogenous leukemia in which the "blast transformation" (which commonly occurs 3-5 years after diagnosis in untreated patients with chronic myelogenous leukemia) either develops very early in the disease, or later, but in a patient in whom the chronic phase had been missed clinically. The Philadelphia translocation is not usually encountered in children with acute myeloid leukemia or in adults with chronic lymphocytic leukemia. T(8;14) (choice A) is a translocation with unfavorable prognosis found in some cases of acute lymphoblastic (L3 form) leukemia. T(8;21) (choice B) is a translocation with favorable prognosis found in some cases of acute myelogenous leukemia (M2 form). T(12;21) (choice D) is a translocation with favorable prognosis found in some cases of acute lymphoblastic (L1 form) leukemia. T(15;17) (choice E) is a translocation with favorable prognosis found in some cases of acute myelogenous leukemia (M3 form). A complete blood count (CBC) and a blood smear are taken from a patient who has been complaining of fatigue. The CBC shows a white count of 200,000/mm3, with differential count including 70% segmented neutrophils and 15% band forms. The absolute numbers of basophils and eosinophils are increased somewhat. Review of the peripheral smear does not show any blast forms. Question 6 of 8

This translocation joins which of the following two genes? / A. BCR and ABL / B. E2A and PBX1

/ C. myc and lgH / D. myc and kappa light chain / E. myc and lambda light chain

Explanation - Q: 3.6

Close

The correct answer is A. The t(9;22) Philadelphia translocation links the Abelson oncogene (c-ABL) (normally on chromosome 9) to the BCR region of chromosome 22. The resulting gene product is a new, chimeric, protein called BCR-ABL, which has tyrosine kinase activity. In some manner still not well understood, this gene product then produces the clinical problems we diagnosis as CML. E2A and PBX1 (choice B) describes t(1;19), seen in some cases of pre-Bcell ALL, which links the E2A gene product (a basic helix-loop-helix transcription factor) with the homeobox gene PBX1. The myc oncogene and the immunoglobulin heavy chain IgH (choice C) are joined in a t(8;14) translocation commonly seen in Burkitt's lymphoma and type L3 acute lymphoblastic leukemia. The translocations described in choices D and E also occur with lower frequency in Burkitt's lymphoma and L3 acute lymphoblastic leukemia; in these cases, the translocations produced are, respectively, t(2;8) and t(8;22).

A complete blood count (CBC) and a blood smear are taken from a patient who has been complaining of fatigue. The CBC shows a white count of 200,000/mm3, with differential count including 70% segmented neutrophils and 15% band forms. The absolute numbers of basophils and eosinophils are increased somewhat. Review of the peripheral smear does not show any blast forms. Question 7 of 8

Which of the following specifically inhibits the abnormal gene product produced by the translocation seen in this disease? / A. Busulfan / B. Hydroxyurea / C. Imatinib mesylate / D. Interferon alfa-2a / E. Interferon alfa-2b Explanation - Q: 3.7

Close

The correct answer is C. Imatinib mesylate (Gleevec) is an important new drug that specifically targets the abnormal gene product (a tyrosine kinase) of the BCR-ABL translocation. This new drug is well absorbed after oral

administration and can be used to treat both the chronic phase and blast crisis of CML. It also shows promise in potentially being used in autologous (self-to-self) bone marrow transplant, where it may be possible to clear the transplant of leukemic cells in vitro with chemotherapy before reinjecting the transplant. The other agents listed in the choices are also used in the chemotherapy of CML. Busulfan (choice A) is an alkylating agent, hydroxyurea (choice B) inhibits DNA synthesis, and recombinant forms of interferons (choices D and E) can help induce remissions in CML. A complete blood count (CBC) and a blood smear are taken from a patient who has been complaining of fatigue. The CBC shows a white count of 200,000/mm3, with differential count including 70% segmented neutrophils and 15% band forms. The absolute numbers of basophils and eosinophils are increased somewhat. Review of the peripheral smear does not show any blast forms. Question 8 of 8

The cure rate with patients who have this condition and undergo bone marrow transplant is now estimated to be which of the following? / A. 5% / B. 15% / C. 25% / D. 50% / E. 80% Explanation - Q: 3.8

Close

The correct answer is D. In the past, the average life expectancy of CML patients has been 3-5 years, with most patients dying after a blast crisis develops. The blast crisis may produce either clinical acute lymphoblastic leukemia or acute myeloid leukemia. This was historically true despite chemotherapy and any apparent improvements in blood counts with chemotherapy. Bone marrow transplant is becoming an increasingly preferred treatment modality in those cases in which a suitable donor can be found, because it now has a cure rate of approximately 50%. The mortality rate of this procedure is 10-20% or less if a matched sibling donor is used and increases to 30-40% if an unrelated donor is used.

A 17-year-old African American with type 1 diabetes spends a weekend camping with friends. A day after returning, he is brought into the emergency department because he is in diabetic ketoacidosis. In the emergency room, he is noted to be jaundiced. The laboratory calls the emergency department physician because the plasma has a red color.

Question 1 of 5

Which of the following enzymatic deficiencies would most likely cause a hemolytic episode such as this man is experiencing? / A. Acid sphingomyelinase deficiency / B. GIucocerebrosidase deficiency / C. GIucose-6-phosphate dehydrogenase deficiency / D. Ferrochelatase deficiency / E. Phytanic acid alpha-oxidase deficiency Explanation - Q: 4.1

Close

The correct answer is C. Of the enzyme deficiencies listed, only glucose-6phosphate dehydrogenase deficiency (G-6-PD) would cause hemolysis. The cells most likely to be affected are usually the older erythrocytes. Most patients with this condition have long asymptomatic intervals and only undergo hemolysis when a metabolic stress occurs. While elementary discussions of this condition usually stress the ingestion of fava beans or the taking of medicines (e.g., sulfonamides, antimalarial drugs) as triggers of hemolysis, you should be aware that in real life in the United States, the most common triggers are fever, acute viral and bacterial infections, and diabetic ketoacidosis. Once the diagnosis of G6PD deficiency is suspected, the actual enzyme activity can be assayed in the laboratory to establish the diagnosis. Acid sphingomyelinase deficiency (choice A) produces the lysosomal storage disease Niemann-Pick disease. Glucocerebrosidase deficiency (choice B) produces the lipid storage disease Gaucher disease. Ferrochelatase deficiency (choice D) produces erythropoietic protoporphyria. Phytanic acid alpha-oxidase deficiency (choice E) produces the storage disease Refsum disease. A 17-year-old African American with type 1 diabetes spends a weekend camping with friends. A day after returning, he is brought into the emergency department because he is in diabetic ketoacidosis. In the emergency room, he is noted to be jaundiced. The laboratory calls the emergency department physician because the plasma has a red color. Question 2 of 5

Which of the following is the approximate incidence of this enzymatic defect in African-American males? / A. Less than 0.1% / B. 1% / C. 10% / D. 30%

/ E. 90%

Explanation - Q: 4.2

Close

The correct answer is C. The incidence of glucose-6-phosphate deficiency in American black males is thought to be about 12%. The condition can also be found in lower frequency in black females (who either have both X chromosomes involved or who have one affected X chromosome with the other inactivated as a Barr body in a high percentage of the body). G6PD deficiency has a worldwide distribution, but occurs most frequently in tropical and subtropical areas of the Eastern hemisphere. Populations that have a significant frequency of G6PD deficiency include Brazilian Blacks, people from the lowlands of the Mediterranean basin, especially those of Greek or Sardinian descent, Kurdish Jews and the Southern Chinese. Most affected individuals are asymptomatic. However, individuals from ethnic backgrounds with high frequencies of G6PD deficiency should be checked for it if treatment of an illness requires a medication known to precipitate hemolysis. G6PD deficiency should be also be considered in the differential diagnosis of any nonimmune hemolytic anemia. A 17-year-old African American with type 1 diabetes spends a weekend camping with friends. A day after returning, he is brought into the emergency department because he is in diabetic ketoacidosis. In the emergency room, he is noted to be jaundiced. The laboratory calls the emergency department physician because the plasma has a red color. Question 3 of 5

Which of the following is a product of the enzyme for which the patient has a deficiency? / A. FADH2 / B. NAD+ / C. NADH / D. NADP+ / E. NADPH Explanation - Q: 4.3

Close

The correct answer is E. Glucose-6-phosphate dehydrogenase is the committed step of the hexose monophosphate shunt pathway (pentose phosphate pathway). The enzymes of this pathway are present in the cytoplasm of all cells, but are especially prevalent in adipose and liver cells. The hexose monophosphate shunt pathway makes ribose-5-phosphate for nucleotide synthesis, interconverts 6 carbon sugars and 5 carbon sugars, and produces NADPH. NADPH, the reduced form of nicotinamide adenine dinucleotide phosphate, is used in reductive biosynthetic reactions such as fatty acid synthesis, and in the regeneration of glutathione by glutathione

reductase. Glutathione has three main roles in a cell: it keeps the protein sulfhydryl groups of cysteines reduced, it reduces peroxides, and it maintains hemoglobin in the reduced state. NADP+ (choice D) is also involved in this reaction, but as a substrate, rather than a product. The various forms of NAD (choices B and C) and FAD (choice A) do not take part. A 17-year-old African American with type 1 diabetes spends a weekend camping with friends. A day after returning, he is brought into the emergency department because he is in diabetic ketoacidosis. In the emergency room, he is noted to be jaundiced. The laboratory calls the emergency department physician because the plasma has a red color. Question 4 of 5

Regeneration of which of the following in a reduced form is impaired in individuals with this disorder? / A. Acetaldehyde / B. AcetyI CoA / C. Biotin / D. GIutathione / E. GIycogen Explanation - Q: 4.4

Close

The correct answer is D. Reduced glutathione is an effective scavenger of dangerous oxidative metabolites; regeneration of reduced glutathione is accomplished by glutathione reductase with the aid of NADPH, a product of the hexose monophosphate shunt pathway. Red blood cells that are deficient for G6PD, and are exposed to oxidants, can be depleted of their store of glutathione. The lack of glutathione leads to the oxidation of the sulfhydryl groups on hemoglobin, causing the formation of methemoglobin and then denatured globin, which forms insoluble masses called Heinz bodies that are attached to the RBC membrane. The overall outcome is that the red blood cells become rigid, making them susceptible to destruction by the reticuloendothelial system. The other answers are distracters. A 17-year-old African American with type 1 diabetes spends a weekend camping with friends. A day after returning, he is brought into the emergency department because he is in diabetic ketoacidosis. In the emergency room, he is noted to be jaundiced. The laboratory calls the emergency department physician because the plasma has a red color.

Question 5 of 5

In a Nigerian population, the frequency of males with G6PD deficiency is 0.2. Assuming the population is in Hardy-Weinberg equilibrium at this locus, what is the frequency of homozygous females? / A. 0.02 / B. 0.04 / C. 0.2 / D. 0.32 / E. 0.45 Explanation - Q: 4.5

Close

The correct answer is B. Glucose 6-phosphate dehydrogenase deficiency is an X-linked recessive trait. A male is hemizygous for the X chromosome, and thus has only one copy of each trait located on that chromosome. The frequency of an X-linked recessive in males is thus equal to the frequency of the allele in the population. From this, we know that q= 0.2 and p = 0.8. A woman has two copies of each gene on the X chromosome, so the equation for Hardy Weinberg equilibrium is the same as for autosomal traits. In this case, a homozygous recessive female would occur at a frequency of q2 or 0.04. Choice A, 0.02, is incorrect. If a person remembered that G6PD was more frequent in males, but didn't know how to use the equations to get the true estimate, he or she might guess this answer. Choice C, 0.2, makes the assumption that the trait is autosomal, and so the frequency of affected males and affected females are equal. Choice D, 0.32, assumes that q=0.01, and then calculates the frequency of carrier females (2pq). Choice E, 0.45 is a distracter A follow-up physical examination is performed on a 7-year-old child with known celiac disease, which has been difficult to controI. A complete blood count is done as part of the evaluation, which demonstrates a moderate anemia. Review of the peripheral blood smear shows numerous enlarged, hypochromic erythrocytes. Some of the neutrophils show hypersegmentation of the nuclei, with up to 8 Iobes, rather than the usuaI 3. Question 1 of 5

Deficiency of which of the following would most likely cause this child's anemia? / A. Iron / B. Folate / C. Vitamin A

/ D. Vitamin C / E. Vitamin K

Explanation - Q: 5.1

Close

The correct answer is B. The combination of enlarged, hypochromic erythrocytes with hypersegmented neutrophils indicates that the patient has a megaloblastic anemia. Of the causes listed in the choices, only folate deficiency will produce these changes. Folate deficiency can also be caused by inadequate intake (elderly, alcoholics, infants), increased requirements (pregnancy, infancy, high metabolic states), impaired utilization (methotrexate therapy), or increased loss (hemodialysis). Iron deficiency (choice A) usually produces a microcytic, hypochromic anemia. Vitamin A deficiency (choice C) causes night blindness. Vitamin C deficiency (choice D) causes scurvy and poor wound healing. Vitamin K deficiency (choice E) causes a bleeding tendency. A follow-up physical examination is performed on a 7-year-old child with known celiac disease, which has been difficult to controI. A complete blood count is done as part of the evaluation, which demonstrates a moderate anemia. Review of the peripheral blood smear shows numerous enlarged, hypochromic erythrocytes. Some of the neutrophils show hypersegmentation of the nuclei, with up to 8 Iobes, rather than the usuaI 3. Question 2 of 5

Deficiency of which the following can also cause this patient's type of anemia? / A. Calcium / B. Magnesium / C. Vitamin B12 / D. Vitamin D / E. Vitamin E Explanation - Q: 5.2 The correct answer is C. Vitamin B12 deficiency can also cause megaloblastic anemia. Calcium and vitamin D deficiencies (choices A and D) can cause osteoporosis and rickets. Magnesium deficiency (choice B) is usually seen in settings in which

Close

patients are very ill for other reasons, but appears to cause GI symptoms, personality change, and tetany. Vitamin E (choice E) is an antioxidant. Deficiency may produce mild hemolytic anemia or neurologic (spinocerebellar) disease. A follow-up physical examination is performed on a 7-year-old child with known celiac disease, which has been difficult to controI. A complete blood count is done as part of the evaluation, which demonstrates a moderate anemia. Review of the peripheral blood smear shows numerous enlarged, hypochromic erythrocytes. Some of the neutrophils show hypersegmentation of the nuclei, with up to 8 Iobes, rather than the usuaI 3. Question 3 of 5

The substance most likely deficient in this child is normally absorbed primarily at which of the following sites? / A. Colon / B. Distal ileum / C. Duodenum and jejunum / D. Esophagus / E. Stomach Explanation - Q: 5.3

Close

The correct answer is C. Folate is absorbed in the proximal small bowel and vitamin B12 is absorbed in the ileum (choice B). Thus poorly controlled celiac disease, which principally affects the proximal small intestine, is more likely to cause folate deficiency than B12 deficiency. The esophagus (choice D) and stomach (choice E) are not major absorptive sites; and the colon (choice A) principally absorbs fluid and electrolytes. A follow-up physical examination is performed on a 7-year-old child with known celiac disease, which has been difficult to controI. A complete blood count is done as part of the evaluation, which demonstrates a moderate anemia. Review of the peripheral blood smear shows numerous enlarged, hypochromic erythrocytes. Some of the neutrophils show hypersegmentation of the nuclei, with up to 8 Iobes, rather than the usuaI 3. Question 4 of 5

The two substances that can cause this type of anemia if deficient, are necessary for synthesis of which of the following? / A. AcetyI CoA / B. ADP / C. ATP / D. NADPH / E. Tetrahydrofolate (THF)

Explanation - Q: 5.4

Close

The correct answer is E. Vitamin B12 is a cofactor in the activation of folic acid to tetrahydrofolate (THF). The THF is necessary for DNA base synthesis, and careful evaluation will demonstrate megaloblastic changes in all 3 hematopoietic lines. The other answers are distracters. A follow-up physical examination is performed on a 7-year-old child with known celiac disease, which has been difficult to controI. A complete blood count is done as part of the evaluation, which demonstrates a moderate anemia. Review of the peripheral blood smear shows numerous enlarged, hypochromic erythrocytes. Some of the neutrophils show hypersegmentation of the nuclei, with up to 8 Iobes, rather than the usuaI 3. Question 5 of 5

Deficiency of vitamin B12 can produce damage to which of the following? / A. Cardiovascular system / B. Central nervous system / C. Female reproductive system / D. Hepatobiliary system / E. Urinary system Explanation - Q: 5.5

Close

The correct answer is B. Vitamin B12 functions in transfer of 1-carbon fragments, and has a role in other pathways besides DNA synthesis. Vitamin B12 deficiency can also cause demyelination and myelopathy, with resultant CNS degeneration. The other choices are distracters.

_.

A 2-year-old African American child is evaluated by a pediatrician because the mother noticed that the child had been behaving listlessly for

several months. Office hematocrit is 29%. A peripheral blood smear is shown above. Question 1 of 6

Which of the following is the most likely diagnosis? / A. AIpha thalassemia major / B. Beta thalassemia major / C. Beta thalassemia minor / D. Sickle cell anemia / E. Sickle cell trait Explanation - Q: 6.1

Close

The correct answer is D. This child has sickle cell anemia. This is an autosomal recessive, chronic hemolytic anemia characterized by sickle- or crescent-shaped erythrocytes. Almost all cases are individuals with at least some African ancestry. In African Americans, the prevalence of sickle cell anemia is about 0.3%. The various thalassemias (choices A, B, and C) can be excluded because the peripheral smear findings would include a reference to or a description of target cells. In questionable cases, or when the possibility of coexisting disease is present, hemoglobin electrophoresis can distinguish between the various diseases noted in the choices. Sickle cell trait (choice E), or the heterozygote and milder form of sickle cell disease, can be excluded because the patient is frankly anemic with a very low hematocrit. Sickle cell trait has a prevalence in African Americans of about 8-13%. These individuals are usually asymptomatic, but may develop clinical disease in unusual situations (such as with severe infections).

Question 2 of 6

The disease is caused by a nucleotide substitution at position +70 in the gene region. The mutation also destroys a restriction endonuclease site from +66 to +72 in the gene. Using a probe that binds just upstream from the mutation, restriction patterns from his mother and from his father were obtained and are shown at the left on the diagram below.

Which of the patterns on the right most likely represents that of the child? / A. A / B. B / C. C / D. D / E. E Explanation - Q: 6.2

Close

The correct answer is E. The mutation that causes the disease also destroys a restriction enzyme site. Loss of a restriction site would produce a longer restriction fragment that would be retained higher on the gel. Thus, in both parents' patterns, the upper band must be cut from the chromosome bearing the mutation. The affected child would be homozygous for this mutation and show only the upper band in his pattern. The band is thicker, indicating more material. The same size restriction fragment is cut from both his chromosomes. Choice A shows a homozygous normal individual. Choice B shows an individual with a band representing a shorter fragment not seen in either parent. Choice C shows an individual with a band representing a longer fragment not seen in either parent. If the mutation involved a triplet repeat expansion, a pattern like this might be generated. It is not correct for the mutation described in the stem. Choice D shows a carrier. Question 3 of 6

The prevalence of this disease in African Americans is about 1/625. The boy's uncle has been tested and knows that he is a carrier of the mutation involved. If he marries an African American woman, what is the risk their first child will be affected with this disease?

/ / / / /

A. 1/2500 B. 1/1200 C. 1/125 D. 1/100 E. 1/50 Explanation - Q: 6.3

Close

The correct answer is E. If the prevalence of the disease (q2) were 1/625, then the carrier frequency (2pq) in the same population would be 2(1/25) or 2/25

The risk of having an affected child is then: 2/25 x 1/4 = 2/100 or 1/50. The risk is the same for each child, regardless of birth order. Question 4 of 6

Compared to the prevalence in African Americans (1/625), the prevalence of this disease in Africans may be as high as 1/50. Which of the following genetic mechanisms contributes to this disparity? / A. Founder effect / B. Gene flow / C. Genetic drift / D. Incomplete penetrance / E. Increased mutation rate in Africans as compared to African Americans. Explanation - Q: 6.3b

Close

The correct answer is E. If the prevalence of the disease (q2) were 1/625, then the carrier frequency (2pq) in the same population would be 2(1/25) or 2/25

The risk of having an affected child is then: 2/25 x 1/4 = 2/100 or 1/50. The risk is the same for each child, regardless of birth order.

Question 4 of 6

Compared to the prevalence in African Americans (1/625), the prevalence of this disease in Africans may be as high as 1/50. Which of the following genetic mechanisms contributes to this disparity? / A. Founder effect / B. Gene flow / C. Genetic drift / D. Incomplete penetrance / E. Increased mutation rate in Africans as compared to African Americans. Explanation - Q: 6.4

Close

The correct answer is B. Gene flow refers to the exchange of genes among populations. When two populations, previously isolated reproductively, begin to intermarry, gene frequencies within both populations will change due to this flow, eventually reaching new equilibria. In either of the populations, some gene frequencies will increase and some will decrease. Another cause of the higher prevalence of sickle cell disease in Africans is the continued selective pressure of malaria, not present in the U.S. Natural selection is not given as an option in this question. A founder effect (choice A) refers to the introduction of a mutation into a rapidly expanding population by an individual who was part of a small ancestral group. Both populations in this question share the same genetic origin (Africa) and carry the same mutation. A founder effect would not account for this disparity in gene frequency. Genetic drift (choice C) refers to the random fluctuations of gene frequencies in small populations. Some genes can increase in frequency, others may decrease, when compared to the gene frequencies seen in larger populations. Gene frequencies are statistical measurements, and small

populations are more likely to vary from the statistical mean than large populations. Consider a family in which both parents are carriers of an autosomal recessive disease-producing gene. Statistics indicate that if the family has enough children, 25% will be AA, 50% will be Aa, and 25% will be aa. If the family only has 4 children, there is some random chance that 2 will be affected (e.g. 50% rather than 25%) Incomplete penetrance (choice D) refers to the fact that not all individuals with the disease-producing genotype have the disease (phenotype). The same mutation should show nearly identical penetrance in two populations, and would not account for the differences described in this question. Mutation rates in large populations tend to be the same (compare to choice E), and do not account for major differences in gene frequencies. Question 5 of 6

Which of the following viruses most commonly causes aplastic crises in patients with this disease? / A. BK virus / B. Human herpes virus 8 / C. Human papilloma virus 18 / D. JC virus / E. Parvovirus B19 Explanation - Q: 6.5

Close

The correct answer is E. Sickle cell patients are vulnerable to a variety of potentially life-threatening clinical crises. Important causes of aplastic crises (also called hematologic crises), in which the marrow fails to produce adequate numbers of erythrocytes, include parvovirus B19 infection and folate deficiency. For your personal information, although the question is usually asked about sickle cell anemia, these two inciting agents can also induce exacerbation of anemia in individuals with thalassemia and a variety of other hereditary red cell disorders. Simple transfusions can be used to treat aplastic crises. BK virus (choice A) causes renal disease in immunocompromised hosts. Human herpes virus 8 (choice B) has been implicated in Kaposi sarcoma. Human papilloma virus 18 (choice C) is an important cause of cervical dysplasia and cancer. JC virus (choice D) causes progressive multifocal leukoencephalopathy. Question 6 of 6

At age 5, the boy and his mother take a trip in a friend's small plane. On landing, the boy complains of very severe pain involving his bones and abdomen. Which of the following problems most likely developed? / A. Aplastic crisis / B. Hematologic crisis / C. Infectious crisis / D. Osteomyelitis / E. Vasoocclusive crisis Explanation - Q: 6.6

Close

The correct answer is E. This patient probably developed a vasoocclusive crisis, which occurs when there is an exacerbation of red cell sickling. The increased sickling leads to obstruction of the microcirculation with sickled erythrocytes, which in turn produces ischemia with its accompanying severe pain (analogous to the pain of angina and myocardial infarction). The pain most frequently involves bones, abdomen, and/or chest. Involvement of the nervous system can produce focal neurologic signs and symptoms. Involvement of the chest can produce acute chest syndrome (due to microvascular occlusion in the pulmonary capillary bed), and is the major cause of death in children less than 5 years of age who have sickle cell disease. Precipitating causes for vasooclusive crises include cold weather (which induces vasospasm), hypoxia (as in this patient who flew in an unpressurized airplane), infection, dehydration, acidosis, alcohol intoxication, emotional stress, and pregnancy. Vasoocclusive crises are usually treated with hydration and analgesia. In severe cases, or if acute chest syndrome develops, exchange blood transfusion is sometimes used to reduce the number of circulating sickled cells. Important triggers for aplastic or hematologic crises (choices A and B) are discussed in the explanation to the preceding question. Infections (choice C), which can include osteomyelitis (choice D), are usually related to these patients' functional asplenia (due to splenic ischemia and infarction) and resultant defective immunity. Encapsulated organisms (Haemophilus influenzae and Streptococcus pneumoniae) are a particular vulnerability, but there is also an increased vulnerability (apparently related to altered serum IgM levels, impaired opsonization, and poor complement pathway function) to some other organisms including Mycoplasma pneumoniae, Salmonella typhimurium, Staphylococcus aureus, and Escherichia coli. A 62-year-old woman consults her physician because she has been having headaches, dizziness, and tinnitus. The patient also reports that her skin seems to be a great deal more itchy than several years ago, and that the episodes of itchiness are sometimes triggered by a hot shower. Physical examination demonstrates hypertension, splenomegaly, and a ruddy complexion seen best in the face, palms, and nail beds. The patient did not appear to be dehydrated.

Question 1 of 4

The patient has an erythrocyte count of 6.7 million/mm3. Which of the following studies would be helpful in establishing that an absolute increase in erythrocyte mass has occurred? / A. Free erythrocyte protoporphyrin / B. Hematocrit / C. Hemoglobin / D. Partial thromboplastin time / E. Radioactive chromium labeling Explanation - Q: 7.1

Close

The correct answer is E. The dilution of small numbers of 51Cr-labelled erythrocytes in the patient's blood can be used to calculate the patient's red cell mass. This is important in cases of erythrocytosis, because it allows the exclusion of hemoconcentration effects (relative erythrocytosis) not related to a true increase in red cell mass. Free erythrocyte protoporphyrin (choice A) is used in the evaluation of the porphyrias. Hematocrit (choice B) and hemoglobin (choice C) cannot distinguish between a relative and an absolute erythrocytosis. Partial thromboplastin time (choice D) is used in the evaluation of clotting disorders. A 62-year-old woman consults her physician because she has been having headaches, dizziness, and tinnitus. The patient also reports that her skin seems to be a great deal more itchy than several years ago, and that the episodes of itchiness are sometimes triggered by a hot shower. Physical examination demonstrates hypertension, splenomegaly, and a ruddy complexion seen best in the face, palms, and nail beds. The patient did not appear to be dehydrated. Question 2 of 4

If the erythrocytosis is related to ectopic production of erythropoietin, which of the following tumors would be the most likely source of the erythropoietin? / A. Adenocarcinoma of the breast / B. Endometrial adenocarcinoma / C. Hepatocellular carcinoma / D. Renal cell carcinoma / E. Serous cystadenoma of the ovary Explanation - Q: 7.2

Close

The correct answer is D. Erythropoietin is normally produced in the kidneys, and renal cell carcinoma can produce it ectopically, producing a form of secondary erythrocytosis. The other answers are distracters. A 62-year-old woman consults her physician because she has been having headaches, dizziness, and tinnitus. The patient also reports that her skin seems to be a great deal more itchy than several years ago, and that the episodes of itchiness are sometimes triggered by a hot shower. Physical examination demonstrates hypertension, splenomegaly, and a ruddy complexion seen best in the face, palms, and nail beds. The patient did not appear to be dehydrated. Question 3 of 4

In addition to the increased red cell count, this patient has a leukocytosis of 15 x 103per microliter, with increases in neutrophils, eosinophils, and basophils. Her platelet count is 450 x 103per microliter. Which of the following is most likely to be contributing to this woman's pruritus? / A. Basophils / B. Eosinophils / C. Erythrocytes / D. Neutrophils / E. PIatelets Explanation - Q: 7.3

Close

The correct answer is A. This patient's disease is closely related to chronic myelogenous leukemia, and increases in a variety of cell lines are seen, although the most striking increase is in the erythrocyte line. Basophils and mast cells are interrelated cell types that can both release histamine and a variety of vasoactive chemicals and cytokines. An occasionally helpful clinical clue is that itchiness not related to an obvious rash can be due to increased numbers of these cell lines. High eosinophil (choice B) counts might be associated with itchiness, but in non-allergic diseases, the high mast cell and basophil counts are more likely to cause symptoms. Erythrocytes (choice C), neutrophils (choice D), and platelets (choice E) do not contain granules that would be likely to cause itchiness. A 62-year-old woman consults her physician because she has been having headaches, dizziness, and tinnitus. The patient also reports that her skin seems to be a great deal more itchy than several years ago, and that the episodes of itchiness are sometimes triggered by a hot shower. Physical examination demonstrates hypertension, splenomegaly, and a ruddy complexion seen best in the face, palms, and nail beds. The patient did not appear to be dehydrated. Question 4 of 4

If this patient has a normal arteriaI O2 saturation level and low erythropoietin, which of the following is the most likely diagnosis? / A. Chronic lung disease / B. Chronic myelogenous leukemia / C. Essential thrombocythemia / D. Polycythemia vera / E. Tumor-associated erythrocytosis Explanation - Q: 7.4

Close

The correct answer is D. Polycythemia vera is an idiopathic chronic myeloproliferative disorder that is characterized by an increased red cell mass. For the diagnosis to be formally made, the patient must have either all major criteria for diagnosis, or the first two major criteria plus any two minor criteria. Major criteria include (1) a red cell mass equal to or greater than 36 mL/kg in men and 32 mL/kg in women; (2) an arterial O2 saturation greater than 92%; and (3) splenomegaly (related to extramedullary hematopoiesis). Minor criteria include (1) thrombocytosis greater than 400 x 103/microliter; leukocytosis greater than 12 x 103/microliter; leukocyte alkaline phosphatase activity greater than 100 in the absence of fever and infection (this helps exclude chronic myelogenous leukemia, choice B); and serum B12 greater than 900 pg/mL or unsaturated B12-binding capacity greater than 2200 pg/mL (B12 abnormality is common in polycythemia vera). This condition is closely related to both chronic myelogenous leukemia and essential thrombocythemia (choice C), in that all cell lines produced in the marrow tend to be increased, although in the case of polycythemia vera, the predominant increase is in the erythrocyte line. Patients are usually treated with recurrent phlebotomy (removal of blood as for a donation) to prevent complications related to hyperviscosity of the blood (venous thrombosis, stroke, hepatic portal vein thrombosis, angina pectoris, intermittent claudication). These patients are also vulnerable to bleeding complications (related to abnormal platelets), peptic ulcer disease (related to high histamine levels from basophils and mast cells), and pruritus. Some cases of polycythemia vera eventually "burn out" and the patient becomes anemic and requires transfusions. Patients are also vulnerable to developing secondary leukemias as either a direct complication of their disease or as a complication of therapy. Chronic lung disease (choice A) severe enough to cause erythrocytosis would not show a normal O2 saturation. Tumor-associated erythrocytosis (choice E) would show a high erythropoietin.

A 3-year-child is taken to a physician for a routine physical examination. On questioning about any problems she may have noticed that concern her, the mother comments that the child does not seem to be feeling well much of the time. She has noticed that the child is irritable, sleeps poorly, and is often constipated. Physical examination is unremarkable. Office hematocrit shows a mild anemia. Question 1 of 6

The physician considers the possibility that the child has an environmental illness. Which of the following is the most common environmental illness of children in the United States? / A. Arsenic poisoning / B. Bismuth poisoning / C. Copper poisoning / D. Insecticide poisoning / E. Lead poisoning Explanation - Q: 8.1

Close

The correct answer is E. Lead poisoning is thought to be the most common environmental illness of children in this country. A high index of suspicion is warranted, because no pathognomic symptoms exist and the physical findings are usually negative. Symptoms that can be seen are often vaguely neurologic (irritability, either sleeplessness or excessive lethargy, poor appetite, headaches, vague changes in activity level). Abdominal pain with or without vomiting can also be seen. Arsenic poisoning (choice A) is less common than lead poisoning in this country, and is mostly likely to be the result of either ground-water contaminated by flowing through arsenic bearing rocks, or ingestion of rat poison. Bismuth (choice B) is used industrially and is not usually found in environments to which children are exposed. Copper (choice C) is relatively nontoxic, although it can contribute to liver disease in patients with abnormal copper metabolism (e.g., Wilson's disease). Insecticide poisoning (choice D) can occur in children with some frequency since insecticides are common in households, but this is less frequent than lead poisoning. A 3-year-child is taken to a physician for a routine physical examination. On questioning about any problems she may have noticed that concern her, the mother comments that the child does not seem to be feeling well much of the time. She has noticed that the child is irritable,

sleeps poorly, and is often constipated. Physical examination is unremarkable. Office hematocrit shows a mild anemia. Question 2 of 6

The child's family is poor, and lives in an apartment built in the 1930's and never substantively remodeled. Rodents and insects are common in the building, as are pesticides used to try to eliminate them. If the child does have the most common environmental illness in US children, which of the following would be the most likely source of the poison? / A. Crumbling sidewalk fragments / B. OId metal doorknob / C. OId paint chips / D. Rat poison / E. Roach poison Explanation - Q: 8.2

Close

The correct answer is C. In the past, the interior paint used on houses often contained lead, which provided a very white color to the paint (to which other colors could be added to produce vibrant paints). These paints are no longer in use, but there is enough very old housing still present in this country that lead poisoning remains a significant problem. The old paint often chips off and the chips may be ingested by toddlers. The affected children have often been bottle-fed for protracted periods and have developed some degree of pica (ingestion of non-food substances). Lead was also commonly used in plumbing pipes in early houses as well. In black, non-Hispanic children living in homes built before 1946, the prevalence of lead poisoning is almost 22%. The rates drop to 13.7% in those living in homes built from 1946 to 1973, and then to 3.4% in homes built after 1973. Rates of lead poisoning are lower among Mexican-American children (13%, 2.3% and 1.6%, respectively) and white, non-Hispanic children (5.6%, 1.4%, and 1.5%, respectively). Lead poisoning from paints also is occasionally a problem when decorative dishes are used to contain acid foods (the classic example is an imported Mexican pitcher used for orange juice) that leak the lead. Also, exposures can come from some folk remedies and from adults working in lead-related occupations. The crumbling sidewalk fragments (choice A) and the old metal doorknob (choice B) are probably non-toxic. Rat poison (choice D) can contain arsenic and roach poison (choice E) can contain organophosphate insecticides. A 3-year-child is taken to a physician for a routine physical examination. On questioning about any problems she may have noticed that concern her, the mother comments that the child does not seem to be feeling well much of the time. She has noticed that the child is irritable,

sleeps poorly, and is often constipated. Physical examination is unremarkable. Office hematocrit shows a mild anemia. Question 3 of 6

Which of the following is traditionally associated with this child's disease, but is actually now uncommonly seen? / A. Aphthous ulcer on inside of cheek / B. Dark line on the gums / C. Fissures on the tongue / D. Hairy leukoplakia of the tongue / E. Pyogenic granuloma of gums Explanation - Q: 8.3

Close

The correct answer is B. The "lead-line," which is due to lead deposition along the gums, is traditionally associated with lead poisoning, but has become uncommon because we are picking up most cases of lead poisoning earlier, when the body load of lead is smaller. The physician should look for it in suspected cases of lead poisoning, even though he or she probably won't find it. Aphthous ulcers (choice A), commonly called "canker sores," are idiopathic benign oral ulcers that can occur with emotional distress, or unrelated to any obvious inciting causes. Fissures on the tongue (choice C) can be seen with a variety of nutritional deficiencies. Hairy leukoplakia (choice D) is a distinctive oral lesion (fuzzy white tongue) seen in some AIDS patients. Pyrogenic granuloma (choice E) produces a ball-like red mass lesion, and can occur sporadically or in association with pregnancy. A 3-year-child is taken to a physician for a routine physical examination. On questioning about any problems she may have noticed that concern her, the mother comments that the child does not seem to be feeling well much of the time. She has noticed that the child is irritable, sleeps poorly, and is often constipated. Physical examination is unremarkable. Office hematocrit shows a mild anemia. Question 4 of 6

This child's disease often produces an anemia that on peripheral smear most closely resembles which of the following? / A. Folate deficiency / B. Hereditary spherocytosis / C. Iron deficiency / D. Sickle cell anemia / E. Vitamin B12 deficiency

Explanation - Q: 8.4

Close

The correct answer is C. The anemia of lead toxicity is a microcytic, hypochromic anemia that closely resembles that of iron deficiency on the peripheral blood smear. While the gold standard for diagnosing lead poisoning is the whole blood lead level, the free erythrocyte protoporphyrin level is often used to demonstrate the degree of biological abnormalities that exists. Lead interferes with ferrochelatase, the enzyme helps to incorporate iron into the protoporphyrin molecule in hemoglobin synthesis. This is why the anemia of lead poisoning clinically resembles that of iron deficiency - the iron is present, but the body cannot use it. Folate deficiency (choice A) and vitamin B12 deficiency (choice E) both produce macrocytic anemia. In hereditary spherocytosis (choice B), all of the erythrocytes are small spheres that lack the biconcave shape typical of normal erythrocyte. In sickle cell anemia (choice D), even when the patient is not in a sickling crisis, at least a few markedly deformed sickle cells will be seen. Question 5 of 6

A peripheral blood smear is examined (above). Which of the following diagnostic features is present in the smear? / A. Acanthocytes / B. Auer rods / C. Basophilic stippling / D. Hairy cells / E. Sickle cells

Explanation - Q: 8.5

Close

The correct answer is C. Basophilic stippling causes round, dark-blue granules in red blood cells on smears stained with brilliant cresyl blue (the granules are more red-purple on Wright/Giemsa stains). The granules are precipitated ribosomes and mitochondria, and are usually caused by an impairment of maturation in the bone marrow. Causes include lead poisoning, exposure to some drugs, severe burns, anemias, and septicemia. Acanthocytes (choice A) are "star-like" red cells with irregularly spaced projections that can be found in abetalipoproteinemia and some liver diseases. Auer rods (choice B) are elongated, bluish-red rods composed of fused lysosomal granules. They are found in the cytoplasm of cells in the myelocytic and monoblastic lines in patients with acute myelogenous leukemia. Hairy cells (choice D) are seen only in hairy cell leukemia and are white cells with fine, irregular pseudopods ("hairs") and immature nuclear features. Sickle cells (choice E) are seen in sickle cell anemia, not lead poisoning. A 3-year-child is taken to a physician for a routine physical examination. On questioning about any problems she may have noticed that concern her, the mother comments that the child does not seem to be feeling well much of the time. She has noticed that the child is irritable, sleeps poorly, and is often constipated. Physical examination is unremarkable. Office hematocrit shows a mild anemia. Question 6 of 6

Which of the following is an oral drug that can be used to treat this disorder in children? / A. Amantadine / B. Succimer / C. Sulfasalazine / D. Tolbutamide / E. Tubocurarine Explanation - Q: 8.6

Close

The correct answer is B. Lead poisoning is treated with lead chelating agents to increase urinary secretion. Succimer is the only one of these agents which has been approved for oral use in children; D-penicillamine is also commonly used, but has never been formally approved for this use by

the FDA. Other chelating agents that are sometimes used in severe acute lead poisoning include dimercaprol (IM) and calcium disodium edetate (IM or IV). Amantadine (choice A) is an antiviral agent active against influenza A virus. Sulfasalazine (choice C) is used in rheumatoid arthritis. Tolbutamide (choice D) is a sulfonylurea used as an oral hypoglycemic agent. Tubocurarine (choice E) is a skeletal muscle relaxant.

A 55-year-old man consults a physician because he has been experiencing chronic weakness and fatigue. Physical examination is notable for the presence of petechiae and a massively enlarged spleen. A peripheral blood smear was sent, which on review demonstrates anemia, thrombocytopenia, modest neutropenia, and the presence of small numbers of unusual white blood cells. These blood cells are small cells with a moderate amount of cytoplasm that are covered with distinctive cytoplasmic projections that appear to be long microvilli. Question 1 of 5

Which of the following is the most likely diagnosis? / A. Chronic myelogenous leukemia / B. Erythroleukemia / C. Hairy cell leukemia / D. Hodgkin's lymphoma / E. Metastatic oat cell carcinoma Explanation - Q: 9.1

Close

The correct answer is C. Hairy cell leukemia is a chronic leukemia that often has an indolent course. It can present with weakness and fatigue due to anemia, bleeding due to thrombocytopenia, or fever and infections due to neutropenia. Almost every patient has splenomegaly, which is massive in 4/5 of cases. The male to female ratio is 5:1 and the patients are usually middle aged with a median age of 52. The other conditions listed would not have "hairy cells" in the peripheral blood. A 55-year-old man consults a physician because he has been experiencing chronic weakness and fatigue. Physical examination is notable for the presence of petechiae and a massively enlarged spleen. A peripheral blood smear was sent, which on review demonstrates anemia, thrombocytopenia, modest neutropenia, and the presence of small numbers of unusual white blood cells. These blood cells are small cells

with a moderate amount of cytoplasm that are covered with distinctive cytoplasmic projections that appear to be long microvilli. Question 2 of 5

Which of the following tests could confirm this diagnosis? / A. 5-hydroxyindoleacetic acid / B. Free erythrocyte protoporphyrin / C. Homovanillic acid / D. Myelin basic protein / E. Tartrate-resistant acid phosphatase Explanation - Q: 9.2

Close

The correct answer is E. Strong positivity for tartrate-resistant acid phosphatase (TRAP) is a classic finding in hairy cell leukemia cells, and you may see an item about this on a USMLE question. However, you should also be aware that this test is no longer available at many centers. 5-hydroxyindoleacetic acid (choice A) is a serotonin derivative used in diagnosing carcinoid tumors. Free erythrocyte protoporphyrin (choice B) is used in the evaluation of porphyrias. Homovanillic acid (choice C) is a urinary catecholamine derivative that is used in diagnosis of pheochromocytomas. Myelin basic protein (choice D) is used in diagnosing multiple sclerosis. A 55-year-old man consults a physician because he has been experiencing chronic weakness and fatigue. Physical examination is notable for the presence of petechiae and a massively enlarged spleen. A peripheral blood smear was sent, which on review demonstrates anemia, thrombocytopenia, modest neutropenia, and the presence of small numbers of unusual white blood cells. These blood cells are small cells with a moderate amount of cytoplasm that are covered with distinctive cytoplasmic projections that appear to be long microvilli. Question 3 of 5

Which of the following immunologic markers is considered to have high sensitivity and specificity for this patient's disease? / A. BIy-7 / B. CD3 / C. CD19 / D. CD20 / E. CD21

Explanation - Q: 9.3

Close

The correct answer is A. Immunophenotyping of peripheral blood has mostly replaced the (less sensitive) TRAP test for diagnosis, and is thought to be capable of making the correct diagnosis in this setting in over 90% of cases, even when only very small numbers of circulating leukemia cells are present. Monoclonal Bly-7 is both sensitive and specific for hairy cell leukemia cells. These cells also express CD19 (choice C) and CD20 (choice D), but these are common pan-B cell antigens and are not specific for hairy cell leukemia. They do not express the late B cell antigen CD21 (choice E) or the T-cell antigen CD3 (choice B). A last useful tip is that they also stain for CD22 at a higher intensity than normal B cells. A 55-year-old man consults a physician because he has been experiencing chronic weakness and fatigue. Physical examination is notable for the presence of petechiae and a massively enlarged spleen. A peripheral blood smear was sent, which on review demonstrates anemia, thrombocytopenia, modest neutropenia, and the presence of small numbers of unusual white blood cells. These blood cells are small cells with a moderate amount of cytoplasm that are covered with distinctive cytoplasmic projections that appear to be long microvilli. Question 4 of 5

The cells described are abnormal cells in which of the following cell lines? / A. B-Iymphocytes / B. Eosinophils / C. Macrophages / D. Neutrophils / E. T-Iymphocytes Explanation - Q: 9.4

Close

The correct answer is A. Hairy cell leukemia cells are an abnormal type of mature B lymphocytes. Unlike many leukemias, these cells are often present in circulating blood in small numbers, although involvement of the spleen is more prominent. Bone marrow aspiration is often unsuccessful due to a dry tap; core biopsy will show the tumor cells with abundant cytoplasm infiltrating a fine fibrillar network. The hairy cell leukemia cells are sensitive to purine analogues (cladribine) and interferons. Granulocyte colony stimulating factor (filgrastim) is used to reduce the neutropenia that may produce lifethreatening susceptibility to infections (gram-negative rods, atypical mycobacteria, disseminated fungus, and Pneumocystis). Patients often experience long-term remissions with chemotherapy, and late relapses (after 5-10 years) often respond a second time to the same chemotherapy. The other choices listed are distracters.

A 55-year-old man consults a physician because he has been experiencing chronic weakness and fatigue. Physical examination is notable for the presence of petechiae and a massively enlarged spleen. A peripheral blood smear was sent, which on review demonstrates anemia, thrombocytopenia, modest neutropenia, and the presence of small numbers of unusual white blood cells. These blood cells are small cells with a moderate amount of cytoplasm that are covered with distinctive cytoplasmic projections that appear to be long microvilli. Question 5 of 5

Which of the following is the most appropriate pharmacotherapy? / A. CIadribine / B. Cyclophosphamide / C. FIudarabine / D. Methotrexate / E. 6-Mercaptopurine Explanation - Q: 9.5

Close

The correct answer is A. The treatment of hairy cell leukemia (HCL) has dramatically improved with the development of newer, more effective antineoplastic agents, such as cladribine. Cladribine is considered to be the treatment of choice for this condition. This is a relatively nontoxic drug that provides benefit in approximately 95% of all cases and a complete remission in more than 80% of these cases. In general, response rates are long-lasting, with few patients relapsing in the first several years. Interferon and splenectomy are rarely used today to treat this condition. With respect to the pharmacology of this agent, cladribine is indicated for the treatment of active HCL as defined by clinically significant anemia, neutropenia, thrombocytopenia, or disease related symptoms. Non-labeled uses include treatment of advanced cutaneous T-cell lymphoma, chronic lymphocytic leukemia, non-Hodgkin lymphomas, acute myeloid leukemia, and autoimmune hemolytic anemia. Cyclophosphamide (choice B) is an antineoplastic commonly used in the treatment of a variety of malignant diseases, such as Hodgkin disease as well as treatment of several types of leukemia. This medication is an alkylating agent related to the nitrogen mustards. It suppresses cell-mediated and humoral immunity as to interfere with "recall responses", causes lymphopenia of B and T cells, and reduces serum immunoglobulin levels. Fludarabine (choice C) is an antimetabolite antineoplastic agent indicated for the treatment of chronic lymphocytic leukemia (CLL). Although not approved for hairy cell leukemia, this agent has been used with some success in the treatment of this condition. Fludarabine is specifically a fluorinated nucleotide analog of the antiviral agent, vidarabine. This agent appears act by inhibiting DNA polymerase alpha, ribonucleotide reductase,

and DNA primase, thus inhibiting DNA synthesis. Methotrexate (choice D) is an agent that competitively inhibits dihydrofolic acid reductase and is also used in antineoplastic therapeutic treatment regimens, such as ALL, cancers of the head, neck, and breast, usually in combination with other therapies. It is also used to treat severe, active, classic or definite rheumatoid arthritis in adults who have an insufficient response with conventional therapies, and for severe, recalcitrant, disabling psoriasis. 6-Mercaptopurine (choice E), otherwise known as mercaptopurine or 6-MP, is a purine analog antimetabolite indicated for the treatment of remission induction and maintenance therapy of acute lymphocytic leukemia (ALL) as well as treatment of acute myelogenous (and acute myelomonocytic) leukemia.

A college student is brought to the student health center at the urging of his roommate. He has been missing class because he needs to check the room lock many times before he can leave. Once he starts to ride his bicycle to class, he frequently returns several times to lock the door. He repeats this ritual every morning and often when he leaves the house. He misses his appointments and his academic performance suffers. His hands are chafed. Question 1 of 4

Which of the following is the most likely diagnosis? / A. Generalized anxiety disorder / B. Obsessive compulsive disorder / C. Panic disorder / D. Paranoid personality disorder / E. Posttraumatic stress disorder Explanation - Q: 1.1

Close

The correct answer is B. Patients with obsessive compulsive disorder (OCD) suffer from obsessive thoughts and compulsive behaviors that impair everyday function. Obsessions are defined as recurrent and persistent thoughts, impulses, or images that are intrusive, inappropriate, and cause anxiety and distress. Patients realize that these thoughts and images are a product of their own mind and they will attempt to suppress them. In addition to obsessions, patients experience compulsions. Compulsions are repetitive behaviors or mental acts that a person performs in accordance with an obsession. The behaviors are aimed at reducing distress or preventing some dreaded event or situation. Locking doors is a common compulsion, and thus this patient meets the diagnostic criteria for obsessive compulsive disorder. Generalized anxiety disorder (choice A) is characterized by excessive anxiety and apprehensive expectation for a period greater than 6 months. Patients experience anxiety, cognitive vigilance, autonomic hyperactivity, motor tension, irritability, and poor concentration. Compulsions are not a part of this disorder. Panic disorder (choice C) is characterized by episodes of panic. Patients have a discrete period of intense fear with tachycardia, palpitations, sweating, trembling, shortness of breath, chest pain and tightening, abdominal discomfort, fear of dying, and paresthesias. Paranoid personality disorder (choice D) is characterized by enduring patterns of personality characterized by mistrust and suspiciousness of people. Posttraumatic stress disorder (choice E) is an anxiety disorder that develops around a traumatic event. Symptoms revolve around the event and include

reexperiencing of the trauma, psychic numbing, and increased autonomic arousal. A college student is brought to the student health center at the urging of his roommate. He has been missing class because he needs to check the room lock many times before he can leave. Once he starts to ride his bicycle to class, he frequently returns several times to lock the door. He repeats this ritual every morning and often when he leaves the house. He misses his appointments and his academic performance suffers. His hands are chafed. Question 2 of 4

The patient's incessant door locking is an example of which of the following? / A. Compulsion / B. Delusion / C. Magical thinking / D. Obsession / E. Paranoid ideation Explanation - Q: 1.2

Close

The correct answer is A. Compulsions are repetitive behaviors or mental acts that patients perform in accordance with an obsession (choice D). It is important to realize that obsessions are the mental processes and that compulsions are the actions or behaviors. Delusions (choice B) are fixed false beliefs that are not culturally accepted. (People that believe in Santa Claus are not deluded.) Magical thinking (choice C) is a mental process, not a behavior, like door locking. Patients with magical thinking believe they have special, "magical" capacities that others do not have. Thus door locking does not exemplify magical thinking. Door locking does not exemplify paranoid ideation (choice E). However, paranoid ideation, or the patient's belief that others are out to harm him/her, may provide the source of anxiety to drive the compulsion. A college student is brought to the student health center at the urging of his roommate. He has been missing class because he needs to check the room lock many times before he can leave. Once he starts to ride his bicycle to class, he frequently returns several times to lock the door. He repeats this ritual every morning and often when he leaves the house. He misses his appointments and his academic performance suffers. His hands are chafed. Question 3 of 4

Which of the following is the most appropriate pharmacotherapy for this patient? / A. CIozapine / B. Desipramine

/ C. FIuoxetine / D. Haloperidol / E. Lorazepam

Explanation - Q: 1.3

Close

The correct answer is C. The effectiveness of selective serotonin reuptake inhibitors (SSRIs) in treating OCD has contributed significant indirect evidence to the role of the serotonergic system in the pathophysiology of OCD. It is hypothesized that dysregulation of this neurotransmitter could contribute to the repetitive obsessions and ritualistic behaviors. This hypothesis is also supported by the relative ineffectiveness of noradrenergic antidepressants, such as desipramine. Clozapine (choice A) is an atypical antipsychotic, and would not play a role in the treatment of OCD unless psychotic features were noted. Desipramine (choice B) is a noradrenergic antidepressant, and as noted above, has no effect on OCD symptoms. Haloperidol (choice D) is an antipsychotic drug that has no effect on OCD symptoms unless psychotic features were noted. Lorazepam (choice E) is a benzodiazepine that is used to treat acute agitation. It would not be used to treat OCD. A college student is brought to the student health center at the urging of his roommate. He has been missing class because he needs to check the room lock many times before he can leave. Once he starts to ride his bicycle to class, he frequently returns several times to lock th door. He repeats this ritual every morning and often when he leaves the house. He misses his appointments and his academic performance suffers. His hands are chafed. Question 4 of 4

According to psychoanalytic theory, this patient's disorder develops when defense mechanisms fail to contain the patient's anxiety. One defense mechanism employed in this patient's constellation of symptoms is reaction formation. Which of the following is an example of reaction formation? / A. A man ignores the fact that his spouse is cheating on him and they invest in a house together / B. A man in the intensive care unit becomes infantile and unruly / C. A man in the intensive care unit tells his nurse, "You that are the best nurse l have ever seen-unlike those horrible nurses yesterday who made me wait for pain medicine. Those nurses were horrible and l never want to see them again."

/ D. A man who is extremely angry with his spouse treats her gently and kindly / E. A promiscuous man accuses his spouse of being unfaithful to him

Explanation - Q: 1.4

Close

The correct answer is D. Obsessional patients often show the defense mechanism of reaction formation. Reaction formation is when affects are transformed into their opposites and ambivalence is resolved in the opposite manner from which it arises. This man resolves his anger with his wife by creating the opposite affect. Choice A exemplifies the defense mechanism known as denial. Denial is the invalidation of an unpleasant or unwanted piece of information. He denies that his marriage is compromised, and continues investing in it. Choice B exemplifies regression. When regression is employed, patients return to an earlier level of functioning. This patient's infantile behavior represents regression. Regression is often seen in medically ill patients. Choice C exemplifies splitting. In splitting, aspects of mental content are kept separate. The man has overidealized those who met his needs, and devalued those who frustrate him. This defense is often seen in patients with borderline personality disorder. Choice E exemplifies projection. In projection, a person rids him/herself of unacceptable thoughts by attributing them to others. While this rids the affected individual of the unwanted affect, he/she then lives in a world of others who harbor the unacceptable material. This is often seen in paranoid patients.

A 32-year-old married lawyer presents to the emergency department with a complaint of "having a heart attack." He explains he was "doing nothing particular" at home about 45 minutes ago when he began having chest pain with shortness of breath and nausea. His symptoms peaked within ten minutes, and he "knew this was the big one." His wife noted he was "shaking and sweaty." His wife immediately brought him to the hospitaI. He has no significant past medical history, takes no medications, and denies substance use. His family medical history is significant for a paternal grandfather that "died of a massive heart attack" at age 56. Physical examination reveals an anxious diaphoretic man taking short, shallow breaths. Vital signs, cardiac auscultation, ECG, and cardiac enzymes are completely normaI. The patient has been to the emergency department 5 times in the last 6 weeks and apologizes for "the million dollar workup," but explains "every time it happens l just know l am doomed to die."

Question 1 of 6

Which of the following is the most likely diagnosis? / A. Agoraphobia / B. Generalized anxiety disorder / C. Malingering / D. Panic disorder / E. Phobia Explanation - Q: 2.1

Close

NONE AVAILABLE A 32-year-old married lawyer presents to the emergency department with a complaint of "having a heart attack." He explains he was "doing nothing particular" at home about 45 minutes ago when he began having chest pain with shortness of breath and nausea. His symptoms peaked within ten minutes, and he "knew this was the big one." His wife noted he was "shaking and sweaty." His wife immediately brought him to the hospitaI. He has no significant past medical history, takes no medications, and denies substance use. His family medical history is significant for a paternal grandfather that "died of a massive heart attack" at age 56. Physical examination reveals an anxious diaphoretic man taking short, shallow breaths. Vital signs, cardiac auscultation, ECG, and cardiac enzymes are completely normaI. The patient has been to the emergency department 5 times in the last 6 weeks and apologizes for "the million dollar workup," but explains "every time it happens l just know l am doomed to die." Question 2 of 6

Which of the following criteria would serve to exclude the most likely diagnosis? / A. Panic attacks beginning during sleep / B. Panic attacks beginning while driving / C. Panic attacks occurring at work and home / D. Panic attacks occurring "out of the blue" / E. Panic attacks occurring with caffeine intake Explanation - Q: 2.2 The correct answer is E. Panic attacks resulting from substances (especially stimulants) or general medical conditions are not considered panic disorder. Myocardial infarction, hypothyroidism, and carcinoid syndrome should be ruled out. Panic attacks may begin while driving, or awaken patients from sleep (choices A and B).

Close

Panic attacks may occur anywhere, including work and home (choice C). Panic attacks ONLY occurring with the trigger of being in open spaces are diagnostic of agoraphobia (a phobia). In panic disorder, panic attacks are unprecipitated ("out of the blue"; choice D). Panic attacks triggered by a feared event or object are seen in phobias. A 32-year-old married lawyer presents to the emergency department with a complaint of "having a heart attack." He explains he was "doing nothing particular" at home about 45 minutes ago when he began having chest pain with shortness of breath and nausea. His symptoms peaked within ten minutes, and he "knew this was the big one." His wife noted he was "shaking and sweaty." His wife immediately brought him to the hospitaI. He has no significant past medical history, takes no medications, and denies substance use. His family medical history is significant for a paternal grandfather that "died of a massive heart attack" at age 56. Physical examination reveals an anxious diaphoretic man taking short, shallow breaths. Vital signs, cardiac auscultation, ECG, and cardiac enzymes are completely normaI. The patient has been to the emergency department 5 times in the last 6 weeks and apologizes for "the million dollar workup," but explains "every time it happens l just know l am doomed to die." uestion 3 of 6

Which of the following would be the most appropriate pharmacotherapy? / A. CIozapine / B. Disulfiram / C. FIuoxetine / D. Lithium / E. Risperidone Explanation - Q: 2.3

Close

The correct answer is C. The selective serotonin reuptake inhibitors (SSRIs) are considered first line treatment for panic disorder. Buspirone and the benzodiazepines are also used. Clozapine (choice A) is an atypical antipsychotic reserved for treatment of refractory schizophrenia, due to the risk of agranulocytosis and myocarditis. Disulfiram (choice B) is used as an adjunct treatment to maintain sobriety. Patients ingesting alcohol while taking this medicine become ill, due to accumulation of acetaldehyde. Lithium (choice D) is a first line treatment for bipolar disorder, and can be used to augment antidepressant medicines.

Risperidone (choice D) is an atypical antipsychotic medication with prominent D2 blockade. It is used to treat psychotic conditions, such as schizophrenia A 32-year-old married lawyer presents to the emergency department with a complaint of "having a heart attack." He explains he was "doing nothing particular" at home about 45 minutes ago when he began having chest pain with shortness of breath and nausea. His symptoms peaked within ten minutes, and he "knew this was the big one." His wife noted he was "shaking and sweaty." His wife immediately brought him to the hospitaI. He has no significant past medical history, takes no medications, and denies substance use. His family medical history is significant for a paternal grandfather that "died of a massive heart attack" at age 56. Physical examination reveals an anxious diaphoretic man taking short, shallow breaths. Vital signs, cardiac auscultation, ECG, and cardiac enzymes are completely normaI. The patient has been to the emergency department 5 times in the last 6 weeks and apologizes for "the million dollar workup," but explains "every time it happens l just know l am doomed to die." Question 4 of 6

Years later, the patient develops a "fear of flying" and is started in once-weekly therapy for systematic desensitization. Which of the following forms of psychotherapy is systematic desensitization? / A. Behavioral psychotherapy / B. Cognitive psychotherapy / C. Family psychotherapy / D. Group psychotherapy / E. Psychoanalytic psychotherapy Explanation - Q: 2.4

Close

The correct answer is A. Behavioral therapies are based on the learning theory (operant and classical conditioning). If the anxiety is uncoupled from the situation, the avoidant behavior will decrease. In systematic desensitization, the patient constructs a hierarchy of images, and gradually works to tolerate imagining the most fearful situation. Behavioral psychotherapy explains behavior as being shaped by reward or punishment, unlike cognitive psychotherapy which posits behavior as secondary to the way a person thinks. Cognitive psychotherapy (choice B) is based on the premise that behavior can be changed by challenging errors in thinking (cognitive distortions). "Homework" is used to ascertain the underlying (negative) assumptions. Family psychotherapy (choice C) is based on the theory that a family is a

system striving to maintain homeostasis, which leads to behaviors. Group psychotherapy (choice D) is based on many theories, and techniques include identification and universalization. Psychoanalytic psychotherapy (choice E) is an intensive type of therapy, usually 4-5 times per week; the goal is for the patient to develop insight into unconscious conflicts, and become more aware of the underlying causes of behavior. Also, it is not uncommon for a patient to develop more than one anxiety disorder (panic disorder and a phobia). A 32-year-old married lawyer presents to the emergency department with a complaint of "having a heart attack." He explains he was "doing nothing particular" at home about 45 minutes ago when he began having chest pain with shortness of breath and nausea. His symptoms peaked within ten minutes, and he "knew this was the big one." His wife noted he was "shaking and sweaty." His wife immediately brought him to the hospitaI. He has no significant past medical history, takes no medications, and denies substance use. His family medical history is significant for a paternal grandfather that "died of a massive heart attack" at age 56. Physical examination reveals an anxious diaphoretic man taking short, shallow breaths. Vital signs, cardiac auscultation, ECG, and cardiac enzymes are completely normaI. The patient has been to the emergency department 5 times in the last 6 weeks and apologizes for "the million dollar workup," but explains "every time it happens l just know l am doomed to die." Question 5 of 6

A full cardiac workup of this patient is most likely to reveal which of the following? / A. Coronary vasospasm / B. Ebstein's anomaly / C. Mitral valve prolapse / D. Myocarditis / E. QTc prolongation Explanation - Q: 2.5

Close

The correct answer is C. As many as 50% of patients with panic disorder also have mitral valve prolapse. A beta blocker may alleviate some symptoms. Coronary vasospasm (choice A) occurs with cocaine use and usually presents with the signs and symptoms of a myocardial infarction (with a positive urine toxicology screen for cocaine). Ebstein's anomaly (choice B) is a rare, albeit notorious, congenital defect

associated with prenatal exposure to lithium. Myocarditis (choice D) has occurred with clozapine, which received a "black box" warning in the PDR in 2002. QTc prolongation (choice E) can occur with antipsychotic medicines and tricyclic antidepressants. Torsades may develop. A 32-year-old married lawyer presents to the emergency department with a complaint of "having a heart attack." He explains he was "doing nothing particular" at home about 45 minutes ago when he began having chest pain with shortness of breath and nausea. His symptoms peaked within ten minutes, and he "knew this was the big one." His wife noted he was "shaking and sweaty." His wife immediately brought him to the hospitaI. He has no significant past medical history, takes no medications, and denies substance use. His family medical history is significant for a paternal grandfather that "died of a massive heart attack" at age 56. Physical examination reveals an anxious diaphoretic man taking short, shallow breaths. Vital signs, cardiac auscultation, ECG, and cardiac enzymes are completely normaI. The patient has been to the emergency department 5 times in the last 6 weeks and apologizes for "the million dollar workup," but explains "every time it happens l just know l am doomed to die." Question 6 of 6

The patient returns to the emergency department in a "bizarre state." Friends report he has been "Iaughing like a loon at all the wrong things," "Iooking over his shoulder in public places," and repeatedly questioning their motives. Examination reveals tachycardia, scleral injection, and a dry cough. Use of which of the following substances is most likely to explain this patient's symptoms? / A. AIprazolam / B. Caffeine / C. Cannabis / D. Methamphetamine / E. Pseudoephedrine Explanation - Q: 2.6

Close

The correct answer is C. Inappropriate laughter, paranoia, and tachycardia, scleral injection, and a dry cough are all associated with cannabis use. Alprazolam (choice A) may cause sedation, slurred speech, disinhibition and ataxia. Patients seem "drunken." Caffeine use (choice B) may initially present with panic attacks, but when carefully questioned, the patient will report caffeine intake (coffee, tea, chocolate, cocoa, over-the-counter cold medications) and usually develops

headaches (during withdrawal) from caffeine. Caffeine intoxicated patients could develop paranoia and tachycardia, but cannabis use explains all the listed symptoms. Methamphetamine (choice D) and pseudoephedrine (choice E) are stimulants. Patients may initially present with panic attacks, and can develop paranoia and tachycardia, but the additional history of inappropriate laughter and scleral injection indicates cannabis use. Amphetamine abuse mimics symptoms of schizophrenia. A 30-year-old man is brought to the emergency department by police, who arrested him because he was in the parking lot of a local malI, yelling "I am a golden god" as he stepped in front of moving cars. When questioned about his identity, he talks incessantly in a rapid fashion and threatens to "unleash God's wrath on those who do not submit." He reports that he has not slept in a week, and does not need sleep. He has spent the last week preparing for a secret government mission that only the president knows about. When asked if he ever hears God talking to him, he says, "Of course not, I am God!" Police report that he has had several prior arrests for reckless driving and lewd acts. On examination, he is disheveled and malodorous. It is nearly impossible to get any further history, as he rapidly paces about and mutters, "I must go, I must go" over and over again. Question 1 of 7

Which of the following is the most likely preliminary diagnosis? / A. Antisocial personality disorder / B. Bipolar disorder / C. Borderline personality disorder / D. Post traumatic stress disorder / E. Schizophrenia Explanation - Q: 3.1

Close

The correct answer is B. This patient is displaying many of the diagnostic criteria for bipolar disorder. He has had a distinct period of abnormally and persistently elevated mood lasting at least one week. In addition, his thoughts are grandiose ("I am a golden god"), he has a decreased need for sleep, and he has pressured and excessive speech. His arrest history indicates possible previous sexual indiscretions, which are a hallmark of bipolar disorder (excessive involvement in pleasurable activities that have a high potential for painful consequences). His pacing indicates psychomotor agitation, also a hallmark of bipolar disorder. This patient probably has a history of severe depressive episodes, but the occurrence of a single manic episode allows the diagnosis of bipolar disorder to be made. Antisocial personality disorder (choice A) refers to a long-standing pattern of

socially irresponsible behavior that reflects a disregard for the rights of others. These individuals were formerly called psychopaths, and generally lack a conscience. Many people with this disorder engage in unlawful acts. The most pervasive characteristic is a lack of remorse for the harm they cause others. Borderline personality disorder (choice C) refers to a lifelong pattern of unstable affect and self-image with erratic behavior. Borderlines have interpersonal relationships that are intense, but very unstable ("love-hate" relationships). They are prone to self-mutilation and the defense mechanism of splitting. But they have a life of chaos, not a sense of grandeur and power, so the diagnosis does not fit this case. Posttraumatic stress disorder (choice D) is an anxiety disorder that develops surrounding a traumatic event. Symptoms revolve around the event and include reexperiencing of the trauma, avoidance of associated stimuli, psychic numbing, and increased autonomic arousal. While the psychotic symptoms of mania and schizophrenia (choice E) may, at times, be difficult to distinguish, the grandiose content of this man's symptoms suggest that he is having a manic episode. Note that his comments that God does not talk to him can be taken as a denial of auditory hallucinations. A 30-year-old man is brought to the emergency department by police, who arrested him because he was in the parking lot of a local malI, yelling "I am a golden god" as he stepped in front of moving cars. When questioned about his identity, he talks incessantly in a rapid fashion and threatens to "unleash God's wrath on those who do not submit." He reports that he has not slept in a week, and does not need sleep. He has spent the last week preparing for a secret government mission that only the president knows about. When asked if he ever hears God talking to him, he says, "Of course not, I am God!" Police report that he has had several prior arrests for reckless driving and lewd acts. On examination, he is disheveled and malodorous. It is nearly impossible to get any further history, as he rapidly paces about and mutters, "I must go, I must go" over and over again. Question 2 of 7

Before a definitive diagnosis can be made, which of the following should be performed? / A. Cranial nerve exam / B. CT scan of the head / C. EIectroencephalogram / D. Magnetic resonance imaging of the head / E. Toxicological screen Explanation - Q: 3.2

Close

The correct answer is E. Intoxication with a number of agents such as amphetamines, cocaine, or other sympathomimetics can mimic mania seen in bipolar disorder. Other possibilities to consider are antidepressant medications, thyroid hormone replacements, hyperthyroidism, and other neurologic conditions. It is often difficult to distinguish the mania of bipolar disorder from the mania of substance abuse. Often the two are comorbid conditions as patients "treat" their condition. If a manic episode is substanceinduced, it cannot contribute to a diagnosis of bipolar disorder. Cranial nerve exam (choice A) cannot be performed on an uncooperative patient and would likely not contribute to reaching a diagnosis in this patient. CT scan of the head (choice B) would not likely aid in the diagnosis of this patient unless something in the patient's medical history suggests an organic cause (e.g., frontal neoplasm). An electroencephalogram (choice C) would not contribute to the diagnosis of this patient as there is no evidence of seizures. MRI (choice D) would not contribute to the diagnosis for the same reasons a CT scan would not. A 30-year-old man is brought to the emergency department by police, who arrested him because he was in the parking lot of a local malI, yelling "I am a golden god" as he stepped in front of moving cars. When questioned about his identity, he talks incessantly in a rapid fashion and threatens to "unleash God's wrath on those who do not submit." He reports that he has not slept in a week, and does not need sleep. He has spent the last week preparing for a secret government mission that only the president knows about. When asked if he ever hears God talking to him, he says, "Of course not, I am God!" Police report that he has had several prior arrests for reckless driving and lewd acts. On examination, he is disheveled and malodorous. It is nearly impossible to get any further history, as he rapidly paces about and mutters, "I must go, I must go" over and over again. Question 3 of 7

This patient is started on lithium to stabilize his mood. BIood levels of lithium need to be closely monitored as therapeutic levels are close to toxic levels. The ratio of toxic dose to therapeutic dose is known as which of the following? / A. Fractional elimination constant / B. Half life / C. Loading dose / D. Therapeutic index / E. Volume of distribution

Explanation - Q: 3.3

Close

The correct answer is D. The therapeutic index of a drug is the ratio of the toxic dose to therapeutic dose. For a drug with a small therapeutic index, care must be taken not to overdose the patient. Small, stepwise increases in dosing can aid in finding the therapeutic dose without experiencing toxicity. A 30-year-old man is brought to the emergency department by police, who arrested him because he was in the parking lot of a local malI, yelling "I am a golden god" as he stepped in front of moving cars. When questioned about his identity, he talks incessantly in a rapid fashion and threatens to "unleash God's wrath on those who do not submit." He reports that he has not slept in a week, and does not need sleep. He has spent the last week preparing for a secret government mission that only the president knows about. When asked if he ever hears God talking to him, he says, "Of course not, I am God!" Police report that he has had several prior arrests for reckless driving and lewd acts. On examination, he is disheveled and malodorous. It is nearly impossible to get any further history, as he rapidly paces about and mutters, "I must go, I must go" over and over again. Question 4 of 7

Which of the following is a known adverse effect of lithium? / A. Agranulocytosis / B. AItered judgement / C. Aplastic anemia / D. Hypothyroidism / E. Male infertility Explanation - Q: 3.4

Close

The correct answer is D. Patients on lithium chronically may develop iatrogenic hypothyroidism. (Approx. 5% of patients taking the drug > 18 months.) Lithium exerts this effect by interfering with the synthesis and release of thyroid hormone. Thus any patient experiencing prolonged depressive symptoms while taking lithium needs to have his/her thyroid assessed. Agranulocytosis (choice A) is commonly reported for patients taking the atypical antipsychotic drug clozapine or carbamazepine, which is sometimes employed as a second-line treatment for bipolar disorder. Choice B is incorrect. Neurological side effects of lithium include tremor, choreoathetosis, ataxia, motor hyperactivity, dysarthria, and aphasia. Lithium is not known to cause aplastic anemia (choice C). Aplastic anemia is

a serious potential adverse effect of the mood stabilizer carbamazepine. Carbamazepine would present an alternative to lithium in this patient, but a periodic blood count must be performed to assess for aplastic anemia. Lithium is not known to affect fertility in males (choice E) or females. However, lithium has strong teratogenic effects (Ebstein's anomaly of the tricuspid valve) and should NOT be given to any woman who is pregnant or even thinking about becoming pregnant. A 30-year-old man is brought to the emergency department by police, who arrested him because he was in the parking lot of a local malI, yelling "I am a golden god" as he stepped in front of moving cars. When questioned about his identity, he talks incessantly in a rapid fashion and threatens to "unleash God's wrath on those who do not submit." He reports that he has not slept in a week, and does not need sleep. He has spent the last week preparing for a secret government mission that only the president knows about. When asked if he ever hears God talking to him, he says, "Of course not, I am God!" Police report that he has had several prior arrests for reckless driving and lewd acts. On examination, he is disheveled and malodorous. It is nearly impossible to get any further history, as he rapidly paces about and mutters, "I must go, I must go" over and over again. Question 4 of 7

Which of the following is a known adverse effect of lithium? / A. Agranulocytosis / B. AItered judgement / C. Aplastic anemia / D. Hypothyroidism / E. Male infertility Explanation - Q: 3.5

Close

The correct answer is B. Benzodiazepines are frequently used to treat acute agitation and can help manage acute mania until lithium can exert its effects. Benzodiazepines potentiate the inhibitory effect on the CNS neurons by binding to GABA receptors and increasing the frequency of the opening of chloride channels in response to GABA stimulation. The net effect is CNS depression and reduction in the patient's agitation. Barbiturates bind the GABA receptor and prolong the duration of opening of chloride channels in response to GABA (choice A). This acts to suppress the CNS. Lorazepam has no effect on dopamine receptors. Many antipsychotic medications act by inhibiting dopamine receptors (choice C). Lorazepam does not effect the release of epinephrine from the adrenal

medulla (choice D). As noted above, lorazepam has no effect on dopamine receptors (choice E). A 30-year-old man is brought to the emergency department by police, who arrested him because he was in the parking lot of a local malI, yelling "I am a golden god" as he stepped in front of moving cars. When questioned about his identity, he talks incessantly in a rapid fashion and threatens to "unleash God's wrath on those who do not submit." He reports that he has not slept in a week, and does not need sleep. He has spent the last week preparing for a secret government mission that only the president knows about. When asked if he ever hears God talking to him, he says, "Of course not, I am God!" Police report that he has had several prior arrests for reckless driving and lewd acts. On examination, he is disheveled and malodorous. It is nearly impossible to get any further history, as he rapidly paces about and mutters, "I must go, I must go" over and over again. Question 6 of 7

The patient later reaches a steady state level of lithium that produces toxic side effects. If he decides to discontinue his medication, how long would it take for his lithium blood levels to reach 25% of his original steady state levels assuming a half life of 22 hours for lithium? / A. 11 hours / B. 22 hours / C. 33 hours / D. 44 hours / E. 55 hours / F. 66 hours / G. 77 hours / H. 88 hours Explanation - Q: 3.6

Close

The correct answer is D. Lithium, like most drugs follows first-order kinetics, which means a constant percent of the drug is eliminated per unit time. His drug levels will decrease by 50% every half-life. Therefore, they will be 50% of original levels after one half-life, 25% after two half-lives, 12.5% after three half-lives, etc. Two half-lives is 22 x 2 = 44 hours. A 30-year-old man is brought to the emergency department by police, who arrested him because he was in the parking lot of a local malI, yelling "I am a golden god" as he stepped in front of moving cars. When questioned about his identity, he talks incessantly in a rapid fashion

and threatens to "unleash God's wrath on those who do not submit." He reports that he has not slept in a week, and does not need sleep. He has spent the last week preparing for a secret government mission that only the president knows about. When asked if he ever hears God talking to him, he says, "Of course not, I am God!" Police report that he has had several prior arrests for reckless driving and lewd acts. On examination, he is disheveled and malodorous. It is nearly impossible to get any further history, as he rapidly paces about and mutters, "I must go, I must go" over and over again. Question 7 of 7

A screening test is developed for assessing vulnerability to developing this condition. A sample of 10,000 people between the ages of 18 to 24 is recruited from the general population and given the screening test. Of this sample, 200 individuals are identified as likely to develop the condition. The sample is tracked over the next twenty years. Forty of the original sample, although none of those selected by the test, are lost to follow-up. A total of 100 people from the sample eventually were diagnosed with this condition, of which 90 were correctly identified by the screening test. Based on this study, the positive predictive value of the screening test is best estimated as which of the following? / A. 45% / B. 60% / C. 75% / D. 90% / E. 100% Explanation - Q: 3.7

Close

The correct answer is A. Positive predictive value assesses the proportion of those identified as having the condition that actually end up developing the condition. In this case, 90 of the original 200 identified by the test developed the disorder. (90/200 = 45%). Note the test has a sensitivity (ability to detect disease) of 90%. (90 of the 100 people who actually developed the condition were correctly identified by the screening test) A 25-year-old man presents to the emergency department with multiple lacerations to both wrists. He says he has a history of "every diagnosis in the book, Doc." The patient reports several recent stressors, including being fired from his job after "Iosing it" with a "rotten customer," financial problems, and a fight with his girlfriend three hours ago. When asked about suicidal ideation, the patient responds with "would you want this life?" With permission, the doctor speaks with the man's psychiatrist. His psychiatrist describes a long-standing pattern of unstable relationships, career changes, and extreme mood swings with erratic sleep patterns. The patient has taken multiple overdoses in

the past, usually when his psychiatrist is out of town. After the doctor listens several minutes to the patient empathetically, the man responds with multiple compliments, proclaiming the doctor is "the best doctor ever." An hour later the doctor is summoned by the nurses to again see the patient. The man is sullen and angry "You don't even care what happens to me. You never did; none of you do, and you just let me sit here forever. What kind of lousy doctor are you? You're an embarrassment to your profession." Question 1 of 6

Which of the following is the most likely diagnosis? / A. Antisocial personality disorder / B. Borderline personality disorder / C. Histrionic personality disorder / D. Narcissistic personality disorder / E. Paranoid personality disorder Explanation - Q: 4.1

Close

The correct answer is B. Borderline personality disorder is characterized by a pattern of instability in 1) relationships (fights with girlfriend and customers), 2) self image (career changes may be one), 3) affect (mood swings), AND marked impulsivity (multiple overdoses). Another clue is the frantic efforts to avoid abandonment (overdoses precede separation from doctor, wrist slashing after break up with girlfriend). Persons with borderline personality are prone to rages and complain of chronic feelings of emptiness. Antisocial personality disorder (choice A) is characterized by a pervasive pattern of disregard for the rights of others. Histrionic personality disorder (choice C) is characterized by a pervasive pattern of excessive emotionality and attention-seeking. These individuals are usually the "life of the party," and may be associated with "creating a scene." They are not self-destructive, like persons with borderline personality disorder. Narcissistic personality disorder (choice D) is characterized by a pervasive pattern of grandiosity, need for admiration, and lack of empathy. They exhibit a stable self-image and are not self-destructive like persons with borderline personality disorder. Paranoid personality disorder (choice E) is characterized by a pervasive pattern of distrust and suspiciousness. A 25-year-old man presents to the emergency department with multiple lacerations to both wrists. He says he has a history of "every diagnosis in the book, Doc." The patient reports several recent stressors, including being fired from his job after "Iosing it" with a "rotten

customer," financial problems, and a fight with his girlfriend three hours ago. When asked about suicidal ideation, the patient responds with "would you want this life?" With permission, the doctor speaks with the man's psychiatrist. His psychiatrist describes a long-standing pattern of unstable relationships, career changes, and extreme mood swings with erratic sleep patterns. The patient has taken multiple overdoses in the past, usually when his psychiatrist is out of town. After the doctor listens several minutes to the patient empathetically, the man responds with multiple compliments, proclaiming the doctor is "the best doctor ever." An hour later the doctor is summoned by the nurses to again see the patient. The man is sullen and angry "You don't even care what happens to me. You never did; none of you do, and you just let me sit here forever. What kind of lousy doctor are you? You're an embarrassment to your profession." uestion 2 of 6

The doctor repeatedly reassures the patient that he deserves the best care and his doctors are working on it. The patient refuses to accept the reassurance, and begins yelling loudly. The doctor screams at the patient to be quiet. Later, he says " I Iet him have it, because what he REALLY needed was some tough love for once." The doctor is using which of the following defense mechanisms? / A. Denial / B. Isolation / C. Rationalization / D. Sublimation / E. Suppression Explanation - Q: 4.2

Close

The correct answer is C. Rationalization, which is providing a logical reason (what the patient needs) for a behavior, is usually employed to avoid being blamed (e.g., for unprofessional behavior). Denial (choice A) is usually used to avoid awareness of a painful reality, and is often seen in patients given the news of a fatal illness, or in patients confronted about substance abuse. Isolation (choice B) splits the thought from the feeling, and can be seen when doctors discuss "interesting cases" or "severe pathology" in completely intellectual terms. Sublimation (choice D) is replacing an unacceptable wish with a more acceptable one. In this example, the doctor might have "taken out his aggression" in the exercise room. Suppression (choice E) is consciously deciding to remove an idea or feeling

from awareness. "I'm not going to think about that." A 25-year-old man presents to the emergency department with multiple lacerations to both wrists. He says he has a history of "every diagnosis in the book, Doc." The patient reports several recent stressors, including being fired from his job after "Iosing it" with a "rotten customer," financial problems, and a fight with his girlfriend three hours ago. When asked about suicidal ideation, the patient responds with "would you want this life?" With permission, the doctor speaks with the man's psychiatrist. His psychiatrist describes a long-standing pattern of unstable relationships, career changes, and extreme mood swings with erratic sleep patterns. The patient has taken multiple overdoses in the past, usually when his psychiatrist is out of town. After the doctor listens several minutes to the patient empathetically, the man responds with multiple compliments, proclaiming the doctor is "the best doctor ever." An hour later the doctor is summoned by the nurses to again see the patient. The man is sullen and angry "You don't even care what happens to me. You never did; none of you do, and you just let me sit here forever. What kind of lousy doctor are you? You're an embarrassment to your profession." Question 3 of 6

Which coping mechanism is illustrated by the statement, "You are the best doctor ever, the rest of the staff is cold and heartless."? / A. Denial / B. Projection / C. Repression / D. Splitting / E. Suppression Explanation - Q: 4.3

Close

The correct answer is D. Splitting is a primitive defense that oversimplifies all relationships into "good" and "bad." The self, others, and situations are completely polarized into one category or the other. Persons with borderline personality disorder have difficulty tolerating ambivalence (concurrent positive and negative feelings). Denial (choice A) is the outright rejection of information. "I do not have cancer." Projection (choice B) is attributing one's traits/feelings to another person. "You're mad at me." When, in fact, I am mad at you. Repression (choice C) is unconscious exclusion of thoughts/feelings, "What anger? I was never angry." Suppression (choice E) is active exclusion of thoughts/feelings from

consciousness, "I'm not going to think about that right now." A 25-year-old man presents to the emergency department with multiple lacerations to both wrists. He says he has a history of "every diagnosis in the book, Doc." The patient reports several recent stressors, including being fired from his job after "Iosing it" with a "rotten customer," financial problems, and a fight with his girlfriend three hours ago. When asked about suicidal ideation, the patient responds with "would you want this life?" With permission, the doctor speaks with the man's psychiatrist. His psychiatrist describes a long-standing pattern of unstable relationships, career changes, and extreme mood swings with erratic sleep patterns. The patient has taken multiple overdoses in the past, usually when his psychiatrist is out of town. After the doctor listens several minutes to the patient empathetically, the man responds with multiple compliments, proclaiming the doctor is "the best doctor ever." An hour later the doctor is summoned by the nurses to again see the patient. The man is sullen and angry "You don't even care what happens to me. You never did; none of you do, and you just let me sit here forever. What kind of lousy doctor are you? You're an embarrassment to your profession." Question 4 of 6

This patient is started on trazodone to help with sleep. Common side effects of trazodone include which of the following? / A. Drowsiness, dizziness, fatigue, and fatal liver failure / B. Drowsiness, dizziness, hypertension, and nervousness / C. Drowsiness, dizziness, hypotension, and priapism / D. Drowsiness, dizziness, nervousness, and seizures / E. Drowsiness, nervousness, GI distress and sexual dysfunction Explanation - Q: 4.4

Close

The correct answer is C. This question illustrates the fact that many antidepressant drugs have similar side effects, but often have a certain particular side effect worth knowing. The clue to choice C (trazodone) is priapism, a painful sustained erection. It is a medical emergency! The clue to choice A (nefazodone) is fatal liver failure; this drug now has a "black box" warning in the PDR. The clue to choice B (venlafaxine) is hypertension, specifically diastolic hypertension. The clue to choice D (bupropion) is seizures. DO NOT give this medicine to patients at risk for seizures (e.g., metabolic derangement, head injury). The clues to choice E (any and all SSRIs) are GI distress and sexual dysfunction, which are very troublesome side effects. Inquire about sexual

dysfunction in all patients taking SSRIs. A 25-year-old man presents to the emergency department with multiple lacerations to both wrists. He says he has a history of "every diagnosis in the book, Doc." The patient reports several recent stressors, including being fired from his job after "Iosing it" with a "rotten customer," financial problems, and a fight with his girlfriend three hours ago. When asked about suicidal ideation, the patient responds with "would you want this life?" With permission, the doctor speaks with the man's psychiatrist. His psychiatrist describes a long-standing pattern of unstable relationships, career changes, and extreme mood swings with erratic sleep patterns. The patient has taken multiple overdoses in the past, usually when his psychiatrist is out of town. After the doctor listens several minutes to the patient empathetically, the man responds with multiple compliments, proclaiming the doctor is "the best doctor ever." An hour later the doctor is summoned by the nurses to again see the patient. The man is sullen and angry "You don't even care what happens to me. You never did; none of you do, and you just let me sit here forever. What kind of lousy doctor are you? You're an embarrassment to your profession." Question 5 of 6

The doctor calls the insurance company to authorize admission for "mood stabilization." When asked, the patient says he does not want to harm himself at this point. The insurance company denies authorization for an inpatient admission. The doctor determines the patient's presentation is too despondent and hopeless to be safe. The most appropriate intervention is for the doctor to call the insurance company back and do which of the following? / A. Say "The patient is a danger to himself in my opinion, get your supervisor on the phone now." / B. Say "The patient is a danger to himself in my opinion, Iet me explain my reasoning." / C. Say "The patient says he is actively suicidal now.", even though he did not / D. Tell the patient to say he is suicidaI, then say "The patient says he is actively suicidal now." / E. "This is unacceptable. Get your supervisor on the phone now, or you will be exposing yourself to legal liability." Explanation - Q: 4.5

Close

The correct answer is B. The FIRST intervention is for the doctor to calmly explain the reasoning involved in the assessment. Choice A is an unnecessary beginning to the conversation; calmly asking for

the supervisor later (if needed) would be the appropriate response. Choice C is lying. It's illegal, for one. Contacting the attending, the board, the administrator, etc., may help the doctor get the patient's needs met through honest means. Choice D is illegal and unethical, and the doctor will ultimately suffer for modeling to the patient "we can make deals." Choice E is a threat and implies coercion, at the very least it could damage the doctor's professional reputation. A 25-year-old man presents to the emergency department with multiple lacerations to both wrists. He says he has a history of "every diagnosis in the book, Doc." The patient reports several recent stressors, including being fired from his job after "Iosing it" with a "rotten customer," financial problems, and a fight with his girlfriend three hours ago. When asked about suicidal ideation, the patient responds with "would you want this life?" With permission, the doctor speaks with the man's psychiatrist. His psychiatrist describes a long-standing pattern of unstable relationships, career changes, and extreme mood swings with erratic sleep patterns. The patient has taken multiple overdoses in the past, usually when his psychiatrist is out of town. After the doctor listens several minutes to the patient empathetically, the man responds with multiple compliments, proclaiming the doctor is "the best doctor ever." An hour later the doctor is summoned by the nurses to again see the patient. The man is sullen and angry "You don't even care what happens to me. You never did; none of you do, and you just let me sit here forever. What kind of lousy doctor are you? You're an embarrassment to your profession." Question 6 of 6

The nurse calls to tell the doctor that the patient has taken an overdose in the emergency department. When the doctor arrives, the patient has slurred speech and is sedated. Within five minutes the patient is sleeping, his respiratory rate is 10/min and he responds minimally to painful stimuli. The family tells the doctor that the man takes clonazepam at home for nocturnal myoclonus. The most appropriate pharmacologic intervention is to administer which of the following? / A. Dextrose / B. FIumazenil / C. Naloxone / D. Naltrexone / E. Thiamine Explanation - Q: 4.6

Close

The correct answer is B. The history is consistent with respiratory depression secondary to benzodiazepine overdose. Flumazenil is a benzodiazepine receptor antagonist. Dextrose (choice A) treats hypoglycemia. Naloxone (choice C), an opioid antagonist, reverses opioid overdose. Thiamine, naloxone, and dextrose are often given to patients who are "found down" and present to the emergency department in a coma without any history. Naltrexone (choice D) blocks the effects of opiates via opioid antagonism (people cannot "get high"). It is used to prevent relapse in a previously opioid dependent individual. Thiamine (choice E) prevents Wernicke-Korsakoff syndrome. A 69-year-old white man visits a physician in the outpatient clinic for the first time. Over the past 5 months, he reports increasing lethargy, weight loss, and crying "for no reason." The patient had always been an optimistic person, but today he feels "detached from everything." He describes his mood "as if there was a pane of glass between me and the rest of the world and l don't think I'm going to make it. My family and friends are like cardboard cutouts." His speech is slow and methodicaI, punctuated by frequent sighs. The patient has also lost interest in watching movies, which had been his favorite pastime. He also admits that his drinking has become a problem over the past few weeks, and he currently consumes a bottle of white wine every evening. He recently lost his job and is currently filing for divorce, which would end a seventeen-year marriage. His son has attention-deficit disorder, for which he is prescribed amphetamine. The patient was hospitalized for major depression three years ago. The patient's previous doctor had started him on a medication, but he is unable to recall the name or anything about it except that he is not supposed to eat cheese, aged meats, or chocolate while taking it. On physical examination, the patient appears emaciated. His abdomen is very distended, with hepatomegaly 3 cm below the right costal margin. He also has a slight tremor. His gait is normaI. His latest calcium level is 10.3 mg/dL. Question 1 of 5

Which of the following is the most likely diagnosis at this time? / A. Acute stress disorder / B. Adjustment disorder / C. AIcohol abuse / D. Bipolar disorder / E. Hypercalcemia / F. Unipolar disorder

Explanation - Q: 5.1

Close

The correct answer is F. The patient's symptoms of weight loss, lethargy, tearfulness, hopelessness ("I'm not going to make it"), and depersonalization ("world through a pane of glass; family and friends are like cardboard cutouts") for at least one month meet the criteria for unipolar disorder (major depression). Even more specific are his loss of interest in favorite activities, as well as the feelings of sadness. The symptoms have been going on for at least two weeks and represent a change from previous functioning, which defines depression. Three stressors in his life are job loss, the upcoming divorce. and his son's attention-deficit disorder. The lifetime rate of major depression in men in the US is about 10%. Acute stress disorder (choice A) is the consequence of the experience of a traumatic event outside the realm of normal human experience. Neither divorce nor job loss fits this criterion. Symptoms of acute stress disorder (ASD) must also include reexperiencing the event as dreams, recollections of flashback, and avoidance of associated stimuli, along with diffuse other symptoms such as irritability, sleep disruption, and difficulty concentrating. Adjustment disorder (choice B) is a dysfunctional change in behavior within three months of an identifiable stressor. The dysfunction can only last 6 months after the stressor has ended. An adjustment disorder cannot be a grief response AND the diagnosis only applies if no other Axis I diagnosis can be used. Given the patient's current symptoms and previous treatment history, the criteria for this diagnosis are not met. Alcohol abuse (choice C) is probably occurring as a coping mechanism for the depression, and is unlikely to be a specific cause of the depressive symptoms. Ascites, hepatomegaly, tremor, macrocytosis (raised mean corpuscular volume, and increased liver enzymes are suggestive of chronic alcohol use. Bipolar disorder (choice D) can have a similar presentation to unipolar disorder when in the depressive phase. The differential is based on being able to identify one or more manic episodes in the patient's past. Because none are presented here, there is no evidence for this diagnosis. Hypercalcemia (choice E) may present with symptoms of confusion, polyuria, polydipsia, and abdominal pain. This patient has none of these symptoms and, in addition, his calcium levels are actually normal. A 69-year-old white man visits a physician in the outpatient clinic for the first time. Over the past 5 months, he reports increasing lethargy,

weight loss, and crying "for no reason." The patient had always been an optimistic person, but today he feels "detached from everything." He describes his mood "as if there was a pane of glass between me and the rest of the world and l don't think I'm going to make it. My family and friends are like cardboard cutouts." His speech is slow and methodicaI, punctuated by frequent sighs. The patient has also lost interest in watching movies, which had been his favorite pastime. He also admits that his drinking has become a problem over the past few weeks, and he currently consumes a bottle of white wine every evening. He recently lost his job and is currently filing for divorce, which would end a seventeen-year marriage. His son has attention-deficit disorder, for which he is prescribed amphetamine. The patient was hospitalized for major depression three years ago. The patient's previous doctor had started him on a medication, but he is unable to recall the name or anything about it except that he is not supposed to eat cheese, aged meats, or chocolate while taking it. On physical examination, the patient appears emaciated. His abdomen is very distended, with hepatomegaly 3 cm below the right costal margin. He also has a slight tremor. His gait is normaI. His latest calcium level is 10.3 mg/dL. Question 2 of 5

Which of the following medications is the patient most likely currently taking? / A. Amitriptyline / B. Chlorpromazine / C. FIuoxetine / D. Lorazepam / E. Phenelzine Explanation - Q: 5.2

Close

The correct answer is E. Monoamine oxidase inhibitors (MAOIs) include phenelzine and tranylcypromine. MAOIs have the potential for severe side effects when taken with sympathomimetic medications or tyraminecontaining foods (e.g., cheeses, red wines, beers, meats, fruits, beans, liver, yeast extracts) because they can cause a hypertensive crisis. MAOIs can also cause orthostatic hypotension, nausea, insomnia, and sexual dysfunction. Tricyclic antidepressants include amitriptyline (choice A) and nortriptyline. They take anywhere from 2 to 6 weeks to take effect. They have anticholinergic side-effects including dry mouth, blurred vision, constipation, ileus, urinary retention, and even delirium. Neuroleptics such as chlorpromazine (choice B) reduce psychotic symptoms that result from a number of illnesses, including schizophrenia, bipolar disorder, major depressive disorder with psychotic features, psychosis secondary to stimulant drugs, and organic psychoses from Alzheimer

disease or Huntington disease. Selective serotonin reuptake inhibitors (SSRIs) include fluoxetine (choice C). They have a reduced side-effect profile and are effective in depression treatment. The risk of overdose is low. Their main pharmacologic effect is to block the presynaptic serotonin uptake site. Increasing the availability of serotonin in the synaptic cleft is thought to improve depressive symptoms. A benzodiazepine such as lorazepam (choice D) is primarily used for anxiety rather than depression. Adverse reactions include sedation, dizziness, weakness, unsteady gait, headache, and sleep disturbance. A 69-year-old white man visits a physician in the outpatient clinic for the first time. Over the past 5 months, he reports increasing lethargy, weight loss, and crying "for no reason." The patient had always been an optimistic person, but today he feels "detached from everything." He describes his mood "as if there was a pane of glass between me and the rest of the world and l don't think I'm going to make it. My family and friends are like cardboard cutouts." His speech is slow and methodicaI, punctuated by frequent sighs. The patient has also lost interest in watching movies, which had been his favorite pastime. He also admits that his drinking has become a problem over the past few weeks, and he currently consumes a bottle of white wine every evening. He recently lost his job and is currently filing for divorce, which would end a seventeen-year marriage. His son has attention-deficit disorder, for which he is prescribed amphetamine. The patient was hospitalized for major depression three years ago. The patient's previous doctor had started him on a medication, but he is unable to recall the name or anything about it except that he is not supposed to eat cheese, aged meats, or chocolate while taking it. On physical examination, the patient appears emaciated. His abdomen is very distended, with hepatomegaly 3 cm below the right costal margin. He also has a slight tremor. His gait is normaI. His latest calcium level is 10.3 mg/dL. Question 3 of 5

To gather more details about the patient's current state of mind, which of the following questions would be most appropriate for the physician to ask at this time? / A. " Are you taking your antidepressant medications as your previous doctor instructed?" / B. " Have you ever made plans to kill yourself?" / C. " Is there any history of depression in your family?" / D. " On a depression scale of one to ten, how depressed do think you feel right now?" / E. " On average, how much alcohol do you drink in one day?" Explanation - Q: 5.3

Close

The correct answer is B. At this stage, it is crucial to ask a direct question about suicidal ideation, particularly because this is the first time the physician is meeting the patient. In addition, the patient is a white male over the age of 50 who has already been hospitalized once for major unipolar depression, which places him at a greater risk for suicide. The suicide rate among people who have been hospitalized at least once for unipolar depression has been estimated to be 15%. A physician is the last person patients have seen before taking their own life in 10% of suicides. Asking this question as part of the mental status examination also gives information on how severe the depression is. If the patient has a plan about how they will take their own life, then this suggests that they are more likely to follow through with their plan. Compliance with medication (choice A) is an important cause of depression relapse. It has been estimated that fewer than 10% of people suffering from unipolar major depression are likely to be receiving a full therapeutic dose of medication. More than 20% of patients fail to fill the first prescription they receive for major depression, and the majority of patients who do begin treatment discontinue the medication within 14 weeks, usually due to side effects. Depression in the family (choice C) is a risk factor for recurrent depression. Pursuing this line of questioning is useful for the long-term management of depression but does not address any major life-threatening issues. Other risk factors for recurrent depression include a history of multiple episodes (patients with 3 or more prior episodes have at least a 90% recurrence rate), depression associated with dysthymia, onset after age 60, long duration of individual episodes, poor symptom control during therapy, comorbid anxiety disorder, or substance abuse. Objective analysis of depression such as a depression score of 1 to 10 (choice D) is also useful in the long term care of patients with depression. In doing so, the physician is able to objectively document the effectiveness of the patient's antidepressant therapy. Another useful method is counting the number of improvements in the quality of the patient's life, e.g., more family interaction, being able to go to church, returning back to work. For this to be useful, it is important to get an idea of where the patient is at in the initial visit. It does not, however, take precedence over asking about suicidal ideation. Alcoholic intake (choice E) is an important question to ask since the patient appears to have signs of alcoholism (hepatomegaly, increasing abdominal girth as an indication of ascites, weight loss, and tremor). However, it does not necessarily have to be addressed on the initial visit. Suicidal risk is a more ominous life-threatening event in this instance. Accurate answers can be derived when asking the patient to describe their drinking habits during

the course of the day rather than asking about alcohol intake as a single numerical quantity. A 69-year-old white man visits a physician in the outpatient clinic for the first time. Over the past 5 months, he reports increasing lethargy, weight loss, and crying "for no reason." The patient had always been an optimistic person, but today he feels "detached from everything." He describes his mood "as if there was a pane of glass between me and the rest of the world and l don't think I'm going to make it. My family and friends are like cardboard cutouts." His speech is slow and methodicaI, punctuated by frequent sighs. The patient has also lost interest in watching movies, which had been his favorite pastime. He also admits that his drinking has become a problem over the past few weeks, and he currently consumes a bottle of white wine every evening. He recently lost his job and is currently filing for divorce, which would end a seventeen-year marriage. His son has attention-deficit disorder, for which he is prescribed amphetamine. The patient was hospitalized for major depression three years ago. The patient's previous doctor had started him on a medication, but he is unable to recall the name or anything about it except that he is not supposed to eat cheese, aged meats, or chocolate while taking it. On physical examination, the patient appears emaciated. His abdomen is very distended, with hepatomegaly 3 cm below the right costal margin. He also has a slight tremor. His gait is normaI. His latest calcium level is 10.3 mg/dL. Question 4 of 5

The physician decides to discontinue his current medication and prescribes sertraline instead. Sertraline directly affects which of the following neurotransmitters? / A. Acetylcholine / B. Dopamine / C. Epinephrine / D. Norepinephrine / E. Serotonin Explanation - Q: 5.4

Close

The correct answer is E. Sertraline specifically blocks the reuptake of serotonin into the presynaptic axon terminal. This enhances serotonin activation and brings about a cascade of events ultimately resulting in a reduced sensitivity of presynaptic autoreceptors for serotonin and reduced serotonin synthesis. The most common adverse reactions to the SSRIs are gastrointestinal (especially nausea), neuropsychiatric (particularly headache and tremor), and changes in sexual functioning. SSRIs also treat anxious depression, dysthymia, and atypical depression.

Acetylcholine (choice A) has been cited as the main neurotransmitter involved in Alzheimer dementia. Drugs that utilize this concept are tacrine and donepezil, which work in the brain as cholinesterase inhibitors at the neuronal synapse. By inhibiting the cholinesterase enzyme, they increase the level of acetylcholine and aid in maintaining mental function, although they do not stop the degeneration of cholinergic cells. Sertraline has no effect on the levels of acetylcholine. Dopamine (choice B) is found in both small and large neuronal pathways in the CNS. The latter include the nigrostriatal pathway involved, in the etiology of Parkinson disease and mesolimbic/mesocortical pathways, implicated in psychosis. L-dopa, which is converted to dopamine, is the primary replacement therapy in Parkinson disease. Dopamine antagonists are used in the treatment of psychosis. Dopaminergic input to the chemoreceptor trigger zone is the basis of the use of the agonist apomorphine as an emetic to treat poisoning, and the use of antagonists as antiemetics. Hypothalamic dopaminergic neurons inhibit prolactin secretion and lead to the use of agonists in inhibiting lactation. There are a number of drugs that interact presynaptically with dopamine terminals including reserpine, amphetamine, MAO inhibitors, and cocaine. Epinephrine (choice C) is a neurotransmitter, and a hormone. It stimulates alpha1-, alpha2-, beta1-, and beta2-adrenergic receptors in a dose-related fashion. It is the initial drug of choice for treating bronchoconstriction and hypotension resulting from anaphylaxis as well as all forms of cardiac arrest. It is useful in managing reactive airway disease, but beta-adrenergic agents are often used initially because of their convenience and oral inhalation route. Epinephrine is not a neurotransmitter specifically affected by any antidepressants currently available. Norepinephrine (choice D), like serotonin, is a neurotransmitter that may induce depression if depleted. There are several antidepressants that increase norepinephrine levels:

A 69-year-old white man visits a physician in the outpatient clinic for the first time. Over the past 5 months, he reports increasing lethargy, weight loss, and crying "for no reason." The patient had always been an optimistic person, but today he feels "detached from everything." He describes his mood "as if there was a pane of glass between me and the rest of the world and l don't think I'm going to make it. My family and friends are like cardboard cutouts." His speech is slow and methodicaI, punctuated by frequent sighs. The patient has also lost interest in watching movies, which had been his favorite pastime. He also admits that his drinking has become a problem over the past few weeks, and he currently consumes a bottle of white wine every evening. He recently lost his job and is currently filing for divorce, which would end a seventeen-year marriage. His son has attention-deficit disorder, for which he is prescribed amphetamine. The patient was hospitalized for major depression three years ago. The patient's previous doctor had started him on a medication, but he is unable to recall the name or anything about it except that he is not supposed to eat cheese, aged meats, or chocolate while taking it. On physical examination, the patient appears emaciated. His abdomen is very distended, with hepatomegaly 3 cm below the right costal margin. He also has a slight tremor. His gait is normaI. His latest calcium level is 10.3 mg/dL. Question 5 of 5

Should this particular patient commit suicide, the likeliest method of suicide is which of the following? / A. A drug overdose / B. By running his car in a closed garage for an extended period of time / C. Cutting his wrists / D. Hanging himself / E. With a firearm

Explanation - Q: 5.5

Close

The correct answer is E. Firearms were the commonest method of suicide used by persons aged 65 years or older representing 71% of suicides. In 1998, firearms were the most common method of suicide by both males and females, accounting for 78% of male and 35% of female suicides in this age group. Overdose with liquids, pills or gas (choice A), 17%, and suffocation (choice D),11%, represents the other two most common methods of suicide used by persons aged 65 years or older. Elderly patients make fewer attempts per completed suicide, have a higher-male-to-female ratio than other groups, have often visited a health-care provider before their suicide, and have more physical illnesses. It is estimated that 20% of elderly (over 65 years) persons who commit suicide visited a physician within 24 hours of their act, 41% visited within a week of their suicide and 75% have been seen by a physician within one month of their suicide. The suicide rate of white males rises dramatically after age 65 and accounts for the majority of all suicides in the elderly. In 2000, the death rate by car exhaust fumes (choice B) was less than 1 per 100 000 per year. Between the age group of 65 to 70 years, the mortality rate from suicide by a cut or stab (choice C) was 0.32 deaths per 100 000 per year.

A 62-year-old man is seen in an emergency department because of severe pain in his lower back, Ieft buttock and the posterolateral aspect of his left leg. The pain began acutely after he lifted his 8-year-old granddaughter into the back of a truck, and he describes it as "shooting" in nature. When examined by the physician, he has curled into a fetal position, which he says relieves the pain. Attempts to extend the man's spine cause a marked exacerbation of the pain. Question 1 of 5

On physical examination, the physician notes that spasm is occurring in the large muscles on the posterior aspect of the thigh. Which of the following muscles is most likely involved? / A. Adductor longus / B. Biceps femoris / C. Gastrocnemius / D. Sartorius / E. Soleus Explanation - Q: 1.1

Close

The correct answer is B. The biceps femoris is the only muscle listed that is in the posterior compartment of the thigh. The adductor longus (choice A) and the sartorius (choice D) are in the anterior compartment of the thigh. The gastrocnemius (choice C) and the soleus (choice E) are in the posterior compartment of the leg below the knee.

A 62-year-old man is seen in an emergency department because of severe pain in his lower back, Ieft buttock and the posterolateral aspect of his left leg. The pain began acutely after he lifted his 8-year-old granddaughter into the back of a truck, and he describes it as "shooting" in nature. When examined by the physician, he has curled into a fetal position, which he says relieves the pain. Attempts to extend the man's spine cause a marked exacerbation of the pain. Question 2 of 5

More detailed examination demonstrates foot drop with weakness of the anterior tibiaI, posterior tibiaI, and peroneal muscles. Sensory loss is demonstrated over the anterior shin and dorsal foot. These findings suggest a radiculopathy at which of the following cord levels? / A. C-7 / B. L-5 / C. S-3

/ D. T-9 / E. T-12

Explanation - Q: 1.2

Close

The correct answer is B. The findings are typical for involvement at the L-5 level. Most ruptured disks involve either the lumbar area (L-5 and S-1 most common) or the cervical area (C-6 and C-7 most commonly). Compression by a disk of the cervical cord can produce a spastic paresis of the lower limbs; compression of the cauda equina can cause urine retention or incontinence. C-7 (choice A) supplies the back of the shoulders, down the back of the arm, past the elbow, to the back of the second and third fingers. S-3 (choice C) supplies a small area on the buttocks. T-9 (choice D) supplies a band on the abdomen and back that passes above the umbilicus. T-12 (choice E) supplies a similar band that passes below the umbilicus.

A 62-year-old man is seen in an emergency department because of severe pain in his lower back, Ieft buttock and the posterolateral aspect of his left leg. The pain began acutely after he lifted his 8-year-old granddaughter into the back of a truck, and he describes it as "shooting" in nature. When examined by the physician, he has curled into a fetal position, which he says relieves the pain. Attempts to extend the man's spine cause a marked exacerbation of the pain. Question 3 of 5

PIain spinal x-ray films show disk space narrowing in the involved area of the spine, and herniation with protrusion of the nucleus pulposus of one disk is demonstrated by CT scan. The normal location of the disk is between which of the following? / A. The body of the superior vertebrae and the body of the inferior vertebrae / B. The inferior articular process of the superior vertebrae and the superior articular process of the inferior vertebrae / C. The pedicle of the superior vertebrae and the pedicle of the inferior vertebrae / D. The superior articular process of the superior vertebrae and the inferior articular process of the inferior vertebrae / E. The vertebral canal of the superior vertebrae and the vertebral canal of the inferior vertebrae

Explanation - Q: 1.3

Close

The correct answer is A. The disk lies between the bodies of the vertebrae, cushioning the bones to allow for smooth motion. When too much pressure is placed on the spine, a disk can bulge or rupture (herniated disk). The pain of disk herniation may begin abruptly after an obvious inciting event (as in this case) or may be insidious in onset. In many patients, the pain is worsened by Valsalva maneuver, coughing, laughing, or straining at stool. Patients without symptoms of spinal cord compression, cauda equina compression, or severe neurologic deficits related to peripheral nerve root involvement are often treated conservatively, since up to 95% recover without surgery. Surgical repair is used in symptomatic patients, or those with more severe or persistent cases. The superior and inferior articular processes (choices B and D) form the surfaces for the joints between the posterior aspects of the vertebrae. The vertebral canal (choice E) contains the spinal cord, and the pedicle (choice C) contributes to its posterior margin. A 62-year-old man is seen in an emergency department because of severe pain in his lower back, Ieft buttock and the posterolateral aspect of his left leg. The pain began acutely after he lifted his 8-year-old granddaughter into the back of a truck, and he describes it as "shooting" in nature. When examined by the physician, he has curled into a fetal position, which he says relieves the pain. Attempts to extend the man's spine cause a marked exacerbation of the pain. Question 4 of 5

The nucleus pulposus is composed of which of the following? / A. Cancellous bone / B. Compact bone / C. EIastic fibrocartilage / D. Hyaline cartilage / E. White fibrocartilage Explanation - Q: 1.4

Close

The correct answer is E. The disk contains a soft, centrally located nucleus pulposus, surrounded by the protective shell of the annulus fibrosis. The nucleus pulposus is composed of white fibrocartilage. The disk contains no bone (choices A and B).

Elastic fibrocartilage (choice C) is found in the larynx, epiglottis, and ear. The disk has a thin outer layer of hyaline cartilage (choice D), but this does not involve the nucleus pulposus. A 62-year-old man is seen in an emergency department because of severe pain in his lower back, Ieft buttock and the posterolateral aspect of his left leg. The pain began acutely after he lifted his 8-year-old granddaughter into the back of a truck, and he describes it as "shooting" in nature. When examined by the physician, he has curled into a fetal position, which he says relieves the pain. Attempts to extend the man's spine cause a marked exacerbation of the pain. Question 5 of 5

A principal component of the intervertebral disk is collagen. Cross-Iinking of collagen to form a more stable structure requires the use of a vitamin C-dependent enzyme that can hydroxylate which of the following amino acids in procollagen? / A. Asparagine / B. Cysteine / C. Proline / D. Serine / E. Valine Explanation - Q: 1.5

Close

The correct answer is C. The amino acids proline and lysine in collagen are hydroxylated by two vitamin C-dependent enzymes, prolyl- and lysyl hydroxylase, respectively. Hydroxyproline plays an essential role in stable triple helix formation. Cysteine residues form disulfide bonds also important for a stable collagen structure, but hydroxylation plays no role in this process. The other amino acids listed are not hydroxylated in the process of collagen cross-linking. A 40-year-old man consults a physician because of chronic lumbar discomfort. On physical examination, Iarge tender bilateral masses are felt deep in his lower back, below his rib cage, and to either side of the vertebral column. Urinalysis demonstrates mild proteinuria and microscopic hematuria without red cell casts. CT scan demonstrates massive bilateral enlargement of the kidneys with near total replacement of the renal cortex by roughly spherical cysts of varying sizes. Question 1 of 5

Which of the following is the most likely diagnosis? / A. Autosomal dominant polycystic kidney disease / B. Autosomal recessive polycystic kidney disease / C. Cystic renal dysplasia

/ D. Medullary sponge kidney / E. Nephronophthisis-uremic medullary cystic disease complex

Explanation - Q: 2.1

Close

The correct answer is A. This patient most likely has the autosomal dominant (adult) form of polycystic kidney disease. This relatively common condition causes massive bilateral enlargement of the kidneys by midadulthood, and affects approximately 1 in 1000 individuals. It may present as an incidental finding on ultrasound (even in children) or with symptoms related to the effects of the cysts (lumbar discomfort, hematuria, urinary tract infection) or uremia. The cysts arise from dilatation of tubules throughout the nephron, are characteristically spherical, unless compressed by adjacent cysts, and are separated by residual functioning renal tissue. In contrast to the autosomal dominant form of polycystic kidney disease, the autosomal recessive form (choice B) tends to present in childhood, causes less massive renal enlargement, and tends to have a combination of small spherical cysts and elongated dilated channels that occur radially at right angles to the cortical surface. Cystic renal dysplasia (choice C) is an often unilateral developmental malformation of the kidney that is characterized by a fairly small number of large cysts with intervening tissue that may contain abnormal tissues such as cartilage and undifferentiated mesenchyme. Medullary sponge kidney (choice D) is characterized by multiple cystic dilations confined to the renal medulla, rather than cortex. The nephronophthisis-uremic medullary cystic disease complex (choice E) is a group of progressive renal disorders that are usually identified in childhood and present with medullary cysts with tubular atrophy and fibrosis in the cortex. A 40-year-old man consults a physician because of chronic lumbar discomfort. On physical examination, Iarge tender bilateral masses are felt deep in his lower back, below his rib cage, and to either side of the vertebral column. Urinalysis demonstrates mild proteinuria and microscopic hematuria without red cell casts. CT scan demonstrates massive bilateral enlargement of the kidneys with near total replacement of the renal cortex by roughly spherical cysts of varying sizes. Question 2 of 5

Which of the following sites would be most likely to additionally contain cysts in this patient's condition? / A. Adrenal gland / B. Cerebral cortex / C. Liver

/ D. Ovary / E. Thyroid

Explanation - Q: 2.2

Close

The correct answer is C. In addition to the massive cystic replacement of the kidneys, patients with autosomal dominant polycystic disease also have a tendency to form smaller numbers of cysts at other sites. Approximately 40% of these patients have one or more (usually asymptomatic) hepatic cysts; less frequently, cysts are seen in the spleen, pancreas, and lungs. The sites listed in the other choices do not have a notable predilection for developing cysts. A 40-year-old man consults a physician because of chronic lumbar discomfort. On physical examination, Iarge tender bilateral masses are felt deep in his lower back, below his rib cage, and to either side of the vertebral column. Urinalysis demonstrates mild proteinuria and microscopic hematuria without red cell casts. CT scan demonstrates massive bilateral enlargement of the kidneys with near total replacement of the renal cortex by roughly spherical cysts of varying sizes. Question 3 of 5

There is a specific link between this patient's disease and which of the following conditions? / A. Berry aneurysm / B. Lymphedema / C. Polyarteritis nodosa / D. Temporal arteritis / E. Thromboangiitis obliterans Explanation - Q: 2.3

Close

The correct answer is A. Up to 1/3 of patients with adult polycystic kidney disease may have berry aneurysms (roughly spherical dilatations) of the arteries of the circle of Willis at the base of the brain. These may bleed intracranially, with significant risk of death. It is thought that the same defect (possibly an abnormally weak basement membrane) that predisposes for the organ cysts also predisposes for berry aneurysm formation. Other cardiovascular anomalies that these patients may have include aortic root dilation and mitral valve prolapse. Lymphedema (choice B) is an accumulation of excess lymphatic fluid, which may occur in a congenital form or secondary to malignant or other disease that occludes lymphatic channels. Polyarteritis nodosa (choice C) is probably an autoimmune disease, with focal inflammation of medium sized muscular arteries. It can be seen

idiopathically, or following infection or medication use. Temporal arteritis (choice D) is a chronic inflammatory disease of large blood vessels, particularly those of the head. Temporal arteritis is often associated with polymyalgia rheumatica. Thromboangiitis obliterans (choice E) is a vascular disease of small and medium-sized arteries and veins that is most commonly associated with smoking. A 40-year-old man consults a physician because of chronic lumbar discomfort. On physical examination, Iarge tender bilateral masses are felt deep in his lower back, below his rib cage, and to either side of the vertebral column. Urinalysis demonstrates mild proteinuria and microscopic hematuria without red cell casts. CT scan demonstrates massive bilateral enlargement of the kidneys with near total replacement of the renal cortex by roughly spherical cysts of varying sizes. Question 4 of 5

5 years after the diagnosis is established, the patient develops persistent hypertension secondary to his renal disease. The hypertension is treated with lisinopriI. This agent acts at which of the following steps? / A. CIeavage of angiotensin l / B. Increase of sodium and fluid retention / C. Induction of aldosterone secretion / D. Secretion of angiotensin substrate / E. Secretion of rennin Explanation - Q: 2.4

Close

The correct answer is A. Hypertension is very common among patients with autosomal dominant polycystic disease, and apparently develops, in large part, because the cysts impair blood flow to enough renal glomeruli to trigger the angiotensin-renin system. ACE inhibitors (e.g., captopril, lisinopril, enalapril, benazepril, fosinopril, moexipril, perindopril, quinapril, ramipril, and trandolapril) are often effective anti-hypertensive agents in these patients because they interrupt the stimulation of this pathway. More specifically, ACE-inhibitors block the conversion of angiotensin I to angiotensin II, a step that is normally performed by angiotensin converting enzyme (ACE) as blood circulates through the lung. The renin-angiotensin system is activated when reduced blood pressure or distal tubular sodium triggers the secretion of renin (choice E) by the juxtaglomerular cells of the kidney. The circulating renin cleaves angiotensin substrate (secreted by the liver, choice D) to angiotensin I. Following cleavage of angiotensin I to angiotensin II by ACE, angiotensin II triggers both vasoconstriction and aldosterone secretion

(choice C), which then causes increased sodium and fluid retention (choice B). A 40-year-old man consults a physician because of chronic lumbar discomfort. On physical examination, Iarge tender bilateral masses are felt deep in his lower back, below his rib cage, and to either side of the vertebral column. Urinalysis demonstrates mild proteinuria and microscopic hematuria without red cell casts. CT scan demonstrates massive bilateral enlargement of the kidneys with near total replacement of the renal cortex by roughly spherical cysts of varying sizes. Question 5 of 5

Fifteen years after diagnosis, the patient develops chronic renal failure. This would be indicated most directly by rising serum levels of which of the following? / A. Amylase / B. Bilirubin / C. Creatinine / D. Potassium / E. Sodium Explanation - Q: 2.5

Close

The correct answer is C. Chronic renal failure is common among older patients with autosomal dominant polycystic kidney disease, and if it becomes sufficiently severe, may be treated with hemodialysis or kidney transplant. Renal failure is usually monitored clinically with serum levels of creatinine and urea. Amylase (choice A) is a marker for pancreatic damage. Bilirubin (choice B) is a marker for biliary tree damage and hemolysis. Serum potassium (choice D) may be either normal or moderately elevated in renal failure, but can be altered by many conditions and is a less useful monitoring tool than creatinine. Serum sodium (choice E) can be normal or reduced in renal failure, but can also be altered by many other conditions, and so is a less useful monitoring tool than creatinine. A 20-year-old man presents to his family physician with a 1-year history of low back pain and stiffness that is worse in the morning. He reports that the pain is improved with activity and a warm shower. He now pinpoints most of his symptoms to his sacroiliac joints. Physical examination reveals decreased chest expansion on inspiration. X-ray films reveal bilateral symmetric sacroiliac erosion and bony fusion (bamboo spine) of part of the lumbar spine. Question 1 of 5

Which of the following is the most likely diagnosis? / A. Ankylosing spondylitis / B. Enteropathic arthritis / C. Juvenile arthritis / D. Psoriatic arthritis / E. Reiter syndrome Explanation - Q: 3.1

Close

The correct answer is A. This patient most likely has ankylosing spondylitis, an inflammatory arthropathy of the vertebral column. This disorder is most common in young men, and its peak incidence is at 20 years of age. Greater than 90% of patients are positive for HLA-B27. Patients usually complain of pain that is helped with activity and/or a warm shower. There is classically bilateral sacroiliac involvement. Enteropathic arthritis (choice B) is seen in patients with inflammatory bowel diseases such as ulcerative colitis and Crohn disease. Juvenile arthritis (choice C) is a general term for chronic inflammatory arthritis in children. Psoriatic arthritis (choice D) is seen in 7% of patients with psoriasis. HLAB27 is positive in many patients with psoriatic arthritis affecting the spine. Reiter syndrome (choice E) is a triad of seronegative polyarthritis, conjunctivitis and nonspecific urethritis. It is most commonly seen in men, and is associated with venereal disease exposure or bacillary dysentery. Approximately 90% of patients are HLA-B27 positive. A 20-year-old man presents to his family physician with a 1-year history of low back pain and stiffness that is worse in the morning. He reports that the pain is improved with activity and a warm shower. He now pinpoints most of his symptoms to his sacroiliac joints. Physical examination reveals decreased chest expansion on inspiration. X-ray films reveal bilateral symmetric sacroiliac erosion and bony fusion (bamboo spine) of part of the lumbar spine. Question 2 of 5

Which of the following genetic markers is most commonly associated with this condition? / A. HLA-B5 / B. HLA-B8 / C. HLA-B18 / D. HLA-B27 / E. HLA-DR5 Close Explanation - Q: 3.2

The correct answer is D. Over 90% of patients with ankylosing spondylitis are HLA-B27 positive. HLA-B27 is also found in patients with Reiter syndrome and psoriatic arthritis. HLA-B5 (choice A) is associated with congenital adrenal hyperplasia. HLA-B8 (choice B) is associated with autoimmune adrenalitis, celiac disease, Graves disease, and Hashimoto thyroiditis. HLA-B18 (choice C) is associated with Hodgkin disease. HLA-DR5 (choice E) is associated with Hashimoto thyroiditis and juvenile arthritis. A 20-year-old man presents to his family physician with a 1-year history of low back pain and stiffness that is worse in the morning. He reports that the pain is improved with activity and a warm shower. He now pinpoints most of his symptoms to his sacroiliac joints. Physical examination reveals decreased chest expansion on inspiration. X-ray films reveal bilateral symmetric sacroiliac erosion and bony fusion (bamboo spine) of part of the lumbar spine. Question 3 of 5

What cardiac condition is most commonly associated with this patient's diagnosis? / A. Aortic regurgitation / B. Aortic stenosis / C. Ischemia / D. Mitral regurgitation / E. Mitral stenosis Explanation - Q: 3.3

Close

The correct answer is A. Approximately 10% of patients with ankylosing spondylitis develop a characteristic aortic valve lesion. The aortic ring becomes dilated and the valve leaflets become scarred. Inflammatory lesions develop on the valve ring, resulting in aortic regurgitation. The other diagnoses are possible, but are not the most common form of cardiac involvement in these patients. A 20-year-old man presents to his family physician with a 1-year history of low back pain and stiffness that is worse in the morning. He reports that the pain is improved with activity and a warm shower. He now pinpoints most of his symptoms to his sacroiliac joints. Physical examination reveals decreased chest expansion on inspiration. X-ray films reveal bilateral symmetric sacroiliac erosion and bony fusion (bamboo spine) of part of the lumbar spine.

Question 4 of 5

The patient later falls, and then complains of low back pain. X-ray films reveal a fracture of the lumbar spine. On physical examination, he has Iow back tenderness and is unable to extend his great toe on the right. Which of the following nerve roots was most likely injured? / A. L2 / B. L3 / C. L4 / D. L5 / E. S1 Close Explanation - Q: 3.4 The correct answer is D. This patient is unable to extend his great toe, which indicates impairment of the action of the long toe extensors, specifically the extensor hallucis longus (EHL). The EHL is innervated by the L5 nerve root. The L2 nerve root (choice A) innervates the hip flexors, for example the iliopsoas. The L3 nerve root (choice B) innervates the knee extensors, such as the quadriceps. The L4 nerve root (choice C) innervates the ankle dorsiflexors, such as the tibialis anterior. The S1 nerve root (choice E) innervates the ankle plantar flexors, such as the gastrocnemius and soleus. A 20-year-old man presents to his family physician with a 1-year history of low back pain and stiffness that is worse in the morning. He reports that the pain is improved with activity and a warm shower. He now pinpoints most of his symptoms to his sacroiliac joints. Physical examination reveals decreased chest expansion on inspiration. X-ray films reveal bilateral symmetric sacroiliac erosion and bony fusion (bamboo spine) of part of the lumbar spine. Question 5 of 5

A 24-year-old man is seen later that day in the same clinic with complaints of dysuria and pain in his knee and ankle joints. On examination, he has a mild conjunctivitis bilaterally. Which of the following genetic markers is most commonly associated with this patient's disease? / A. HLA-B5 / B. HLA-B8 / C. HLA-B18 / D. HLA-B27 / E. HLA-DR5

Explanation - Q: 3.5

Close

The correct answer is D. The patient most likely has Reiter syndrome, another spondyloarthropathy that affects young men. Reiter syndrome is characterized classically by the triad of seronegative polyarthritis, conjunctivitis and urethritis. HLA-B27 is found in 60-85% of patients with Reiter syndrome and is also found in patients with psoriatic arthritis. HLA-B5 (choice A) is associated with congenital adrenal hyperplasia. HLA-B18 (choice C) is associated with Hodgkin disease. HLA-B8 (choice B) is associated with autoimmune adrenalitis, celiac disease, Graves disease and Hashimoto thyroiditis. HLA-DR5 (choice E) is associated with Hashimoto thyroiditis and juvenile arthritis.

A 67-year-old woman is brought to the hospital complaining of severe back pain after falling out of a chair. A Iateral film of the spine is obtained; the T6-T8 region is shown above. Laboratory studies demonstrate a normocytic, normochromic anemia with a normal white blood cell count, but a markedly elevated erythrocyte sedimentation rate (ESR). A peripheral blood smear demonstrates rouleaux formation. Question 1 of 5

Rouleaux formation suggests which of the following? / A. High calcium content in serum / B. High globulin content in serum / C. High sodium content in serum / D. Low potassium content in serum / E. Low urea content in serum Explanation - Q: 4.1

Close

The correct answer is B. Rouleaux formation refers to the stacking of erythrocytes (distinguished from agglutination) seen when there is a relative or absolute hypergammaglobulinemia. It can be seen when there is excess globulin production, as in multiple myeloma and Waldenström's macroglobulinemia, and also in liver disease, when there is a loss of enough albumin to leave a relative hypergammaglobulinemia. The increased rouleaux formation in this patient also led to a markedly elevated erythrocyte sedimentation rate (ESR), since the rouleaux settle out of blood much faster than do individual erythrocytes. Rouleaux formation is unrelated to the electrolyte or urea content of the serum. Hypercalcemia (choice A) is a frequent finding in multiple myeloma due to the numerous osteolytic bone lesions, and the resulting release of calcium into the blood, but it does not contribute to rouleaux formation. Sodium (choice C) and potassium (choice D) concentrations in the serum do not affect the ESR or rouleaux formation. Urea levels are often elevated, rather than decreased (choice E), in this condition, secondary to glomerulopathy or tubulointerstitial nephritis. Question 2 of 5

Serum electrophoresis demonstrates a spike of protein in the gamma zone. When the electrophoresed proteins are then reacted with specific antisera, the product is characterized. Which of the following antisera are most likely to positively react with the excess proteins being produced? / A. Anti-alpha and anti-gamma isotypes / B. Anti-epsilon and anti-kappa isotypes

/ C. Anti-gamma and anti-Iambda isotypes / D. Anti-kappa and anti-Iambda isotypes / E. Anti-mu and anti-delta isotypes

Explanation - Q: 4.2

Close

The correct answer is C. The most common antibody produced in multiple myeloma is IgG, and the most common light chain produced is lambda light chain (normal immunoglobulin light chains may be of the kappa or lambda types). Therefore, this product would react with antisera prepared against the gamma isotype (heavy chain constant domain determinants) and the lambda isotype (light chain constant domain determinants). In multiple myeloma, the antibodies are monoclonal. Thus, it is possible for anti-alpha OR anti-gamma isotype antisera to react with the product, but not both (choice A). Although it is possible for a monoclonal plasma cell myeloma to produce IgE heavy chains and kappa light chains simultaneously (choice B), it is not the most likely type of antibody to be produced. A plasma cell myeloma cannot produce two light chains simultaneously, e.g., kappa and lambda isotypes (choice D). Mu and delta isotypes (choice E) are both heavy chain isotypes, and only one would be produced by a monoclonal plasma cell myeloma.

Question 3 of 5

Biopsy of one of the lytic bone lesions reveals nodules with the microscopic appearance shown above. Which of the following cellular

markers would most likely characterize these cells? / / / / /

A. CD5 and surface immunoglobulin M B. CD10 and surface immunoglobulin M C. CD38 and cytoplasmic immunoglobulin D. TdT and CD1 E. TdT and CD19 Explanation - Q: 4.3

Close

The correct answer is C. The photomicrograph shows a typical area of myeloma. The typical appearance of a myeloma is that of sheet-like masses of cells resembling normal plasma cells, intermixed with other cytologic variants such as plasmablasts and multinucleated cells. The markers that identify plasma cells in tissues are surface CD38 molecules and cytoplasmic immunoglobulin. CD5 and surface immunoglobulin M (choice A) are the most likely markers to be found in the malignant cells of chronic lymphocytic leukemia. CD10 and surface immunoglobulin M (choice B) describes the most likely markers found in the malignant cells of Burkitt lymphoma. TdT and CD1 (choice D) are the most likely markers to be found in the neoplastic cells of acute lymphoblastic leukemia of the T cell type. Remember that TdT is a marker only found on the most primitive of lymphocyte precursors. It would never be found in a fully differentiated cell tumor such as multiple myeloma. TdT and CD19 (choice E) are the most likely markers to be found in the neoplastic cells of acute lymphoblastic leukemia of the B cell type.

Question 4 of 5

The presence of Bence-Jones protein in the urine of this patient would put him at risk for development of disease that is most closely related to which of the following? / A. AIzheimer disease / B. Familial amyloidotic neuropathy / C. Hemodialysis-associated amyloidosis / D. Primary amyloidosis / E. Reactive systemic amyloidosis Explanation - Q: 4.4

Close

The correct answer is D. Multiple myeloma is usually accompanied by the

presence of free immunoglobulin light chains (Bence Jones protein) in the serum and urine. These proteins eventually build up to toxic levels and precipitate in the extracellular spaces as amyloid fibrils (AL type). This is considered a primary amyloidopathy, since its development is not secondary to an infectious/inflammatory process. Alzheimer disease (choice A) is the most common non-infectious amyloidopathy in the U.S. Amyloid in this disorder is formed of amyloid beta protein cleaved from amyloid precursor protein, and thus is not closely related to the proteinaceous fibrils of AL (amyloid of light chain immunoglobulin molecules). In familial amyloidotic neuropathy (choice B), the amyloid material is composed of transthyretin. In hemodialysis-associated amyloidosis (choice C), the precursor to the amyloid fibril is beta-2 microglobulin. Reactive systemic amyloidosis (choice E) is a secondary amyloidosis resulting from chronic inflammatory conditions. The fibril precursor in this case would be SAA (serum amyloid A protein). Question 5 of 5

Which of the following conditions is likely to accompany this disease? / A. Decreased resistance to / B. Decreased resistance to lnfluenza A virus / C. Decreased resistance to / D. Decreased serum concentrations of C1, C4, and C2 / E. Increased incidence of bacteremia Explanation - Q: 4.5

Close

The correct answer is C. Patients with multiple myeloma generally suffer from a decrease of humoral immune responses, although their cell-mediated responses remain relatively normal. Of the options, the protective response to Streptococcus pneumoniae is the one that is most strongly mediated by antibodies, primarily of the IgA type. Resistance to fungi (e.g., Candida albicans, choice A) and parasites is primarily through cell-mediated mechanisms. Resistance to intracellular pathogens such as the enveloped influenza A virus (choice B) is primarily maintained by cell-mediated mechanisms. C1, C4, and C2 (choice D) are components of the classical complement cascade that are used up when complement is activated by complexes of

antigen and antibody (IgG and IgM). Since the myeloma protein reflects proliferation of a single clone, it is unlikely that the myeloma antibody would ever encounter the antigenic determinant that fits its idiotype and activate complement. Recurrent Neisseria bacteremia (choice E) usually results from deficiencies in the late components of the complement cascade (C5-8), which are genetic, and not present in multiple myeloma. A 32 year-old woman is brought to the emergency department by her husband. He states that his wife suddenly developed severe right back pain approximately three hours ago. The pain is sharp in nature, radiates towards her groin, and comes and goes. She vomits when she arrives at the emergency department. She has no fevers or chills, and states that she has had no prior similar episodes. She complains of urinary urgency and frequency, but denies dysuria or hematuria. Her last bowel movement was yesterday and normaI. She does not remember the date of her last menses. The patient states that when the pain is most severe, she is unable to catch her breath. As the physician enters the room to evaluate the patient, he observes that she is tossing and turning, and unable to get comfortable. On examination, her temperature is 37.8 C (100.1 F) with otherwise normal vital signs. Heart and lung examination are within normal limits. Abdominal examination reveals decreased bowel sounds diffusely with mild right-sided abdominal pain. There are no peritoneal signs. There is severe costovertebral angle tenderness on the right side only. Question 1 of 6

These symptoms are most consistent with which of the following? / A. Acute appendicitis / B. Acute cholecystitis / C. Biliary colic / D. Ectopic pregnancy / E. Renal colic Explanation - Q: 5.1

Close

The correct answer is E. This patient is exhibiting a classic picture of renal colic. This term describes the pain that is caused by the stretching of the collecting system or ureter, and/or distention of the renal capsule. Urinary obstruction (usually from a calcification) is the main mechanism responsible for renal colic. As the stone moves down the ureter, local pain is referred to the distribution of the ilioinguinal nerve and the genital branch of the genitofemoral nerve, thus explaining the pain she is feeling in her groin. Severity and location of the pain can vary from patient to patient due to stone size, location, degree of obstruction, and acuteness of obstruction. In contrast to renal colic, patients with acute appendicitis (choice A) have

peritoneal irritation and therefore prefer to remain perfectly still. Movement irritates the peritoneal inflammation, making their pain worse. Abdominal examination will reveal peritoneal signs. Acute cholecystitis (choice B) has an onset of pain that is similar to biliary colic (see below), however, the pain persists and may be unremitting for several days. Progression of inflammation causes gallbladder distention, leading to inflammation of the peritoneum; therefore the patient is most comfortable lying still. Biliary colic (choice C) is due to the presence of an impacted stone in the cystic duct, or Hartmann pouch, or from passage of a stone through the duct. It is characterized by a rapid increase in intensity of pain, a plateau of discomfort that lasts for several hours, and a gradual decrease in intensity. Pain is felt in the right upper quadrant or midepigastrium. This colic usually occurs after meals. There are no associated urinary complaints. Patients with an ectopic pregnancy (choice D) have lower abdominal pain and irregular or absent menses. They do not complaint of back/flank pain. On examination, there is discrete tenderness over the lower abdomen.

A 32 year-old woman is brought to the emergency department by her husband. He states that his wife suddenly developed severe right back pain approximately three hours ago. The pain is sharp in nature, radiates towards her groin, and comes and goes. She vomits when she arrives at the emergency department. She has no fevers or chills, and states that she has had no prior similar episodes. She complains of urinary urgency and frequency, but denies dysuria or hematuria. Her last bowel movement was yesterday and normaI. She does not remember the date of her last menses. The patient states that when the pain is most severe, she is unable to catch her breath. As the physician enters the room to evaluate the patient, he observes that she is tossing and turning, and unable to get comfortable. On examination, her temperature is 37.8 C (100.1 F) with otherwise normal vital signs. Heart and lung examination are within normal limits. Abdominal examination reveals decreased bowel sounds diffusely with mild right-sided abdominal pain. There are no peritoneal signs. There is severe costovertebral angle tenderness on the right side only. Question 2 of 6

If the source of this patient's pain is found to be obstructive, chemical analysis of the obstruction would most likely reveal which of the following? / A. Bilirubin / B. Calcium oxalate

/ C. Cholesterol / D. Cystine / E. Uric acid

Explanation - Q: 5.2

Close

The correct answer is B. The source of pain is a renal calcification. There are a variety of components that make up a renal stone, however, in the United States, calcium oxalate is the most common type of kidney stone. Bilirubin (choice A) is present in pigmented gallstones. These are the most common types of gallbladder calculi worldwide. They are characterized by their high concentration of bilirubin and their low cholesterol content. Cholesterol gallstones (choice C) are the most common form of gallbladder calculi in the United States. Neither cholesterol nor bilirubin are present in renal stones. Cystine stones (choice D) are relatively rare. They form in acidic urine and are secondary to an inborn error of metabolism (cystinuria) resulting in abnormal intestinal mucosal absorption and renal tubular absorption of dibasic amino acids including cystine, ornithine, lysine, and arginine. Uric acid stones (choice E) comprise less than 5% of all urinary calculi. There is a higher incidence in patients with gout or myeloproliferative diseases, and those treated for malignant conditions with cytotoxic drugs. Acidic pH of the urine is necessary for their formation. Question 3 of 6

If the patient instead was found to have a struvite stone, which of the following would be a likely associated finding? / A. Acidic pH of the urine / B. E. coli / C. EIevated uric acid levels / D. Low urinary ammonium levels / E. Urinary tract infection Close Explanation - Q: 5.3 The correct answer is E. Struvite stones are composed of magnesium ammonium phosphate (MAP). They are found most commonly in women, and may recur rapidly. They are associated with infection with urea-splitting organisms such as Proteus, Pseudomonas, Providencia, Klebsiella, Staphylococci, and Mycoplasma. The high ammonium concentration derived from the urea-splitting organisms results in an alkaline urinary pH. The urinary pH of a patient with a MAP stone ranges from 6.8 to 8.3 and is rarely below 7.0 (compare with choice A). The normal urinary pH is 5.85.

E. coli(choice B) is a frequent cause of urinary tract infections. However, it is not a urea-splitting organism and therefore is not associated with struvite stones. Elevated uric acid levels (choice C) are associated with uric acid stones. Uric acid stones compromise less than 5% of all urinary calculi. As previously stated, the ammonium levels of patients with struvite stones is elevated, not decreased (choice D). Question 4 of 6

Ketorolac is given intravenously to the patient. To which of the following drug classes does this agent belong? / A. Antibiotic / B. Antidiuretic / C. Nonsteroidal anti-inflammatory drug / D. Opioid / E. Steroid Explanation - Q: 5.4

Close

The correct answer is C. Ketorolac (Toradol) is a nonsteroidal antiinflammatory agent (NSAID) and is the only NSAID approved for parenteral use in the US. Ketorolac can dramatically improve the pain of renal colic and can be administered either IM or IV. In severe cases, ketorolac can be used in combination with opioid analgesics. Ketorolac has minimal side effects, however, it can increase prothrombin time when used in conjunction with anticoagulants, increase phenytoin levels and methotrexate toxicity, and is potentiated by probenecid. This agent is contraindicated in patients with renal failure, peptic ulcer disease, or recent GI bleeding. The use of antibiotics (choice A) in patients with kidney stones is controversial. Antibiotic use is justified if there is evidence of a UTI. Whereas only 3% of patients treated for renal colic have been shown to develop new UTIs, an infection can seriously complicate the patient's clinical outcome. The antidiuretic (choice B) desmopressin (DDAVP), a synthetic analogue of vasopressin, has been shown to substantially reduce the pain of patients with renal colic. Its mechanism of action is thought to be reduction of intraureteral pressure, but it may also relax ureteral and renal pelvic musculature. Opioids (choice D) are an important part of acute renal colic therapy. Morphine, meperidine, and butorphanol are the agents most commonly used. Prednisone, an oral corticosteroid (choice E), is a potent anti-inflammatory

agent that can reduce local edema in the ureter. Question 5 of 6

The patient continues to have nausea and vomiting. Which of the following is the most appropriate pharmacotherapy? / A. Diphenoxylate / B. Lansoprazole / C. Loperamide / D. Metoclopramide / E. Sucralfate Explanation - Q: 5.5

Close

The correct answer is D. Patients with acute renal colic frequently have nausea and vomiting, therefore antiemetics frequently have a role in the treatment of such patients. The antiemetic effect of metoclopramide results from its ability to block central dopamine receptors. Clinical trials have additionally shown metoclopramide to be effective in relief of the pain of renal colic. Other antiemetic medications are also used, such as promethazine, hydroxyzine, and prochlorperazine. Diphenoxylate (choice A) and loperamide (choice C) are antidiarrheal agents. Lansoprazole (choice B) decreases gastric acid production by irreversibly inhibiting the H+/K+ ATPase located in the luminal membrane of parietal cells. This agent, and similar agents such as omeprazole, are used to treat Zollinger-Ellison syndrome and gastroesophageal reflux disorder (GERD). Sucralfate (choice E), or aluminum sucrose sulfate, polymerizes in the acid environment of the stomach. The polymer then forms a protective coating over ulcer beds, thus accelerating the healing of peptic ulcers and reducing their rate of recurrence. Question 6 of 6

For this stone to pass spontaneously from this patient, it must travel the entire length of the ureter. Which of the following correctly describes the anatomical position of the right ureter? / A. It crosses underneath the common iliac artery / B. It lies medial to the inferior vena cava / C. It sits on the quadratus lumborum muscle / D. It travels between the inferior vena cava and the descending abdominal aorta / E. The right gonadal vessels cross over the ureter Explanation - Q: 5.6

Close

The correct answer is E. The gonadal vessels roughly parallel the ureter through much of its retroperitoneal extent. These vessels obliquely cross over the ureter from medial to lateral before entering the renal pelvis. Normally, the gonadal arteries are branches of the abdominal aorta. The right gonadal vein drains directly into the vena cava while the left gonadal vein drains into the left renal vein. At no point does either ureter go underneath the iliac vessels (choice A). The ureter crosses above the iliac vessels at approximately the bifurcation of the internal and external iliac arteries. The inferior vena cava and abdominal aorta are both retroperitoneal structures. The aorta runs slightly on the patient's left side and the inferior vena cava runs slightly on the patient's right side. Along its path, the right ureter remains lateral to the inferior vena cava (choices B and D). Rarely, the ureter may cross underneath the vena cava in its superior location. This is called a retrocaval ureter. In its retroperitoneal location, the ureter (on either side) is related posteriorly to the psoas muscle. The quadratus lumborum (choice C) runs lateral to the psoas muscle and is not related to the ureter.

A 23-year-old woman complains to her physician of chronic flatulence, abdominal cramping, and being bloated. She delivered a healthy baby 3 months previously, and she states that her symptoms began several months into her pregnancy. She had expected them to disappear after delivery, but they did not. On further questioning, the woman reports that she has been having alternating diarrhea and constipation. Her stool appears dark and oily to her, and floats in the toilet bowI. She feels hungry all of the time, and has been losing weight despite eating a great deaI. Question 1 of 5

This woman's symptoms are most suggestive of which of the following? / A. Biliary tract disease / B. Cancer of the gastrointestinal tract / C. Granulomatous infectious disease / D. Malabsorption / E. Psychiatric disease Explanation - Q: 1.1

Close

NONE AVAILABLE Question 2 of 5

The woman's physician initially suggests that the patient avoid milk products, but her symptoms fail to improve. This finding is consistent with which of the following conditions? / A. Adenomatous polyps / B. Appendicitis / C. Celiac disease / D. Cystic fibrosis / E. UIcerative colitis Explanation - Q: 1.2

Close

The correct answer is C. Flatulence, bloating, and abdominal cramping often reflect bacterial gas production as a result of utilization of unabsorbed nutrients by the bacteria within the gut. On a practical basis, lactose intolerance is the most common cause of malabsorption with flatulence, and a trial of avoidance of lactose-containing milk products was reasonable. If this fails to correct the problem, then more serious gastrointestinal (usually small intestinal) disease must be considered. Of the list given in the choices, only celiac disease and cystic fibrosis (choice D) commonly cause malabsorption, and cystic fibrosis almost always presents in childhood. Adenomatous polyps (choice A) are usually asymptomatic, but may cause

bleeding per rectum. Appendicitis (choice B) usually presents with fever and right lower quadrant abdominal pain. Ulcerative colitis (choice E) can cause alternating diarrhea and constipation, but would not usually cause significant malabsorption, since the small bowel is not significantly involved. Question 3 of 5

Which of the following is usually considered to be the most accurate way of establishing this patient's likely diagnosis? / A. BIood culture / B. Jejunal biopsy / C. Nasal biopsy / D. Rectal biopsy / E. Stool for ova and parasites Explanation - Q: 1.3

Close

The correct answer is B. Jejunal biopsy demonstrating flattened mucosal villi is considered the gold standard for demonstrating celiac disease. The patient should have active disease (e.g., be on a normal diet and symptomatic) at the time of biopsy in order for the diagnosis to be established. Some clinicians also simply do a trial of gluten-free diet, to avoid subjecting the patient to biopsy. You should also be aware that endomysial antibody (EMA) titers in serum are relatively sensitive and specific for celiac disease, and have been proposed (although still not widely accepted) as a screening tool for celiac disease. The disease does not appear to have an infectious basis, and so blood culture (choice A) and stool examination for ova and parasites (choice E) would not be helpful. Celiac disease is usually confined to the small intestine, so nasal (choice C) and rectal (choice D) biopsies would not be helpful. Question 4 of 5

Which of the following is an example of a food that the patient should avoid? / A. Apple / B. Bread / C. Carrot / D. Lettuce / E. Steak Close Explanation - Q: 1.4 The correct answer is B. Celiac disease appears to be the result of an immunologically mediated toxic reaction to gluten that occurs in genetically

susceptible individuals. The gluten proteins can be found in all forms of wheat (including durum, semolina, spelt, kamut, einkorn, and faro), as well as related grains including barley, triticale, rye, and possibly, oats. Breads are consequently an obvious food to avoid, but patients should be warned that the essentially complete avoidance of these grain products may require that they prepare most of their own foods, since a surprising variety of commercially prepared foods contain small amounts of flour or other grain products. Even products like vinegars, alcohols, and pharmaceuticals may have gluten-containing additives. Celiac disease often presents in childhood, but some individuals (such as this patient) remain either asymptomatic or undiagnosed until some event in adulthood (such as pregnancy, child birth, surgery, viral infection, or severe emotional stress) causes an acute exacerbation of their disease. Consequently, the absence of a life-long history of problems should not be used to exclude the possibility of celiac disease in a symptomatic patient. Fresh fruits (choice A), vegetables (choices C and D), and meats (choice E) prepared at home without additives can be eaten safely. Question 5 of 5

Several years later, the patient cheats on her dietary restrictions and develops a blistering, intensely itchy skin rash that has a symmetrical distribution and is found on her elbows, knees, and buttocks. This is most likely which of the following? / A. Dermatitis herpetiformis / B. Herpes simplex / C. Herpes zoster / D. Molluscum contagiosum / E. Psoriasis Close Explanation - Q: 1.5 The correct answer is A. Dermatitis herpetiformis is a skin condition known to be associated with celiac disease, and, interestingly, may also be the presenting complaint for celiac disease. The diagnosis can be confirmed by skin biopsy with demonstration of IgA in the biopsy tissues. More than 85% of patients with dermatitis herpetiformis also have (obviously symptomatic or not) gluten sensitivity, and removal of gluten from the diet may improve the skin disease. Untreated or poorly treated celiac disease is also associated with anemia (due to iron, folate, or vitamin B12 deficiency), osteoporosis, vitamin K deficiency with risk of hemorrhage, nervous system disorders secondary to nutrient deficiencies, pancreatic insufficiency, and intestinal lymphomas. Other disorders, in addition to dermatitis herpetiformis with which celiac disease is associated, include diabetes mellitus, thyroid disease, lupus, IgA nephropathy, primary biliary cirrhosis, and less commonly chronic active hepatitis, scleroderma, myasthenia gravis, Addison disease,

rheumatoid arthritis, Sjögren syndrome, and Down syndrome. Herpes simplex (choice B) and herpes zoster (choice C) are infectious causes of vesicular skin disease not specifically associated with celiac disease. Molluscum contagiosum (choice D) is due to a viral infection that induces the formation of raised papules with central craters ("small volcanoes"). Psoriasis (choice E) is a predominately autoimmune skin condition characterized by scaly patch formation.

During a routine examination of a 2-year-old child, the mother mentions to the pediatrician that the child has been bloated and has passed Iarge amounts of gas for the last several months. On further questioning, the mother also reports that the child has been having intermittent, but frequent episodes of watery malodorous diarrhea with abdominal cramps and sometimes fever, which began shortly after the child was moved to a new day care center. Other family members have also had similar symptoms, but of shorter duration, which have since resolved. Physical examination of the child is remarkable only for hyperactive bowel sounds. Question 1 of 5

Which of the following is the most likely pathogen?

Explanation - Q: 2.1

Close

The correct answer is D. All of the protozoa listed in the choices are intestinal parasites that can cause chronic diarrhea. Giardia lamblia is the most frequent cause of protozoal diarrhea in North America. The case history illustrates the typical course in children who acquire the infection in a day care setting and then go on to develop chronic infection. Giardia lamblia is an interesting protozoal parasite whose life cycle alternates between trophozoite and cyst stages. While most bacterial infections require inoculating doses of hundreds to thousands of organisms or more to produce disease, giardiasis requires ingestion of only as few as 10 or less organisms in the cyst form (research studies have shown infection after ingestion of a single cyst). In practice, this means that the infection is very highly communicable in a day

care or home setting, since it is difficult to remove all of the cysts from one's hands after handling a contaminated diaper. The use of gloves and taking care to keep children from playing with their diapers can be helpful in controlling the spread of the disease. Giardiasis can also be acquired through ingestion of contaminated water (it will resist chlorination of less than 8 hours but can be killed with boiling or removed with micropore filtration), and so can also be seen among hikers and as traveler's diarrhea. The reservoirs appear to be beavers, bears, dog, cats, and humans. Question 2 of 5

A thorough stool exam for ova and parasites is most likely to reveal which of the following? / A. Acid fast oocyst that are 5 microns in diameter / B. Acid fast oocysts that are 10 microns in diameter / C. Oval cysts with up to 4 nuclei / D. Round cysts containing no more than 4 nuclei / E. Round cysts, including some with 5 to 8 nuclei Explanation - Q: 2.2

Close

The correct answer is C. The cysts of Giardia lamblia are oval, have prominent cell walls, and up to 4 nuclei. A diagnosis of giardiasis can be made by finding either the trophozoites and/or the cysts in a stool sample. However, organisms are only found in 50% of cases if only one stool sample is used. If three stool samples obtained on three different days are used, the detection rate is approximately 90%. If stools are negative, Giardia can be detected in duodenal material that can be sampled using the Enterotest. A gelatin capsule that is attached to a nylon string is swallowed. After 4 to 6 hours, the string is removed and examined under the microscope for trophozoites. Choice A describes the oocysts of Cryptosporidium parvum, a common cause of mild diarrheal illness. It causes incurable, protracted diarrhea in AIDS patients. The diagnosis of cryptosporidiosis is made by finding oocysts in the feces using a modified acid fast or auramine stain. Choice B describes the oocysts of Cyclospora cayetanensis, a protozoa that has recently been identified as a cause of diarrheal illness. In 1995 and 1996, it caused major outbreaks in the United States that were later traced to contaminated raspberries from Guatemala. Diagnosis of cyclosporiasis can be made by finding the acid fast oocyst in stool. Its oocyst is much larger than that of Cryptosporidium parvum, and it is important to differentiate between them since Cyclospora can be treated with trimethoprimsulfamethoxazole and other antibiotics, whereas there is no effective

treatment for Cryptosporidium parvum. Choice D describes the cysts of Entamoeba histolytica. The diagnosis of intestinal infection with Entamoeba histolytica can be made by finding either trophozoites or cysts in the stool. The cysts of Entamoeba histolytica are round, have refractile walls and contain up to four nuclei. Choice E describes the cysts of Entamoeba coli, a nonpathogenic protozoan. Its cysts can be distinguished from the cysts of Entamoeba histolytica because they contain 5 to 8 nuclei. Question 3 of 5

How long, after exposure to this organism, do most people who develop clinical disease exhibit symptoms? / A. 1-2 days / B. 3-5 days / C. 1-3 weeks / D. 4-6 weeks / E. 2-3 months Explanation - Q: 2.3

Close

The correct answer is C. Most patients who develop clinical disease become symptomatic 1-3 weeks after cyst ingestion. Many patients have asymptomatic infections, and it appears that some degree of immunity is often acquired after exposure, since clinical infection is more common in children than in adults after known exposures. Many patients who develop clinical illness have disease that lasts only 1 to 2 weeks, but chronic cases also occur, and are an important source for spread of the infection. Symptoms can include diarrhea, abdominal cramps, pale and greasy stools, fatigue, bloating, and weight loss. The mechanism by which the organisms produce disease is still poorly understood, but they are known to colonize (sometimes very heavily) the surface of the small intestine without invasion into the mucosa. Some of the symptoms may be due to an acquired (and reversible) lactase deficiency related to malfunction of the intestinal epithelium. Question 4 of 5

Which of the following drugs would be most effective in the treatment of this child? / A. Diloxanide furoate / B. Iodoquinol / C. Metronidazole / D. Paromycin / E. Trimethoprim-sulfamethoxazole Explanation - Q: 2.4

Close

The correct answer is C. It is important to establish an accurate diagnosis in protozoal intestinal infections, because different organisms have markedly different drug sensitivities and no "shot-gun" therapy is likely to be effective. Metronidazole (Flagyl) is effective against Giardia (and also Entamoeba); it is not officially licensed for this use in the United States but is commonly used anyway. Alternative agents include furazolidone (less effective than metronidazole) and oral quinacrine (no longer available in the United States because of severe side effects). Diloxanide furoate (choice A) and iodoquinol (choice B) and are sometimes used in the treatment of amebiasis. Paromycin (choice D) can be used to treat cryptosporidiosis. Trimethoprim-sulfamethoxazole (choice E) can be used to treat isosporiasis and cyclosporiasis. Question 5 of 5

This disease would most likely contribute to the patient's death if which of the following diseases were also present? / A. AIDS / B. Crohn disease / C. Goodpasture's syndrome / D. Measles / E. Minimal change disease Close Explanation - Q: 2.5 The correct answer is A. Patients who have had rectal contact during sex with an infected partner may easily acquire giardiasis. If these patients also have AIDS, a particularly overwhelming infection may occur, with essentially "wall-to-wall" colonization of the intestinal mucosal surface with Giardia. In these severe cases, the malabsorption that is produced may be sufficiently severe to induce malnutrition, which will exacerbate the AIDS patient's already high vulnerability to other infectious disease. The diseases listed in the other choices are distracters that have no particular link to giardiasis.

A 62-year-old man complains to his physician that he has noticed a marked increase in the amount of flatulence he experiences. He has also

been experiencing intermittent, but increasing abdominal pain, and has had a 20 pound weight loss in the last month. On physical examination, he seems to be slightly jaundiced. AIkaline phosphatase and bilirubin are increased. CT scan demonstrates a mass in the head of the pancreas. Question 1 of 4

Approximately how many Americans in the United States are diagnosed with this disorder each year? / A. 280 / B. 2,800 / C. 28,000 / D. 280,000 / E. 2,800,000 Explanation - Q: 3.1

Close

The correct answer is C. Approximately 28,000 patients in the United States are diagnosed with pancreatic cancer yearly, and nearly that number also die of it yearly. Pancreatic cancer is a deadly disease, with one of the highest mortalities of any cancer, many patients dying within one year of diagnosis. The underlying problem is that pancreatic cancer tends to not produce symptoms until after metastases to nearby critical structures (lymph nodes, liver, celiac plexus, superior mesenteric vessels, Ligament of Treitz, portal vein) have occurred. At least some pancreatic cancers appear to have a genetic basis. The National Familial Pancreas Cancer Registry now has over 250 families with two or more members with pancreatic cancer. Also, the second familial breast cancer gene, BRCA2 (in both men and women), appears to be important, and may account for the increased incidence of pancreatic cancer seen in Ashkenazi Jews. Other rare syndromes that may have pancreatic cancer as a component include Peutz-Jeghers syndrome, familial melanoma, hereditary colon cancer (the form without polyps), and hereditary pancreatitis. Question 2 of 4

Which of the following is an important risk factor for developing this disorder? / A. Aflatoxin exposure / B. AIcohol use / C. Caffeine ingestion / D. Cigarette smoking / E. Hepatitis B infection Explanation - Q: 3.2

Close

The correct answer is D. Cigarette smoking appears to be a very important risk factor for pancreatic carcinoma, but interestingly, neither alcohol use (choice B) nor caffeine ingestion (choice C) have been implicated. Other risk factors include older age, race (more common in African-Americans and

some Jewish groups than Caucasians), gender (men more than women, but may just reflect smoking rates), chronic pancreatitis, diabetes mellitus, gastric resection, and diet (bad: meats, cholesterol, fried foods, nitrosamines; good: fruit, vegetables). Exposure to the fungal product aflatoxin (choice A) and hepatitis B (choice E) are risk factors for liver, but not pancreatic, cancers. Question 3 of 4

In patients with this man's condition with intractable pain referred to the back, the network of nerves around the aorta is sometimes blocked with alcohoI. This network is known as which of the following? / A. Celiac plexus / B. Choroid plexus / C. Esophageal plexus / D. Hepatic plexus / E. Mesenteric plexus Close Explanation - Q: 3.3 The correct answer is A. The celiac plexus contains the nerves around the aorta that may be stimulated by either pressure or direct involvement by tumor, and so produce pain. The choroid plexus (choice B) is a vascular plexus of the eye, and is also a vascular plexus extending into the ventricles. The esophageal plexus (choice C) refers to the nerve plexus around the esophagus. The hepatic plexus (choice D) is the division of the celiac plexus that accompanies the hepatic artery and portal vein to the liver. There are actually two mesenteric plexuses (choice E), which accompany the superior and inferior mesenteric arteries. Question 4 of 4

Which of the following surgical methods is used most often to treat patients with small tumors of the head of the pancreas with no evidence of metastasis? / A. Billroth l procedure / B. Billroth ll procedure / C. Roux-en-Y gastric bypass / D. Vertical banded gastroplasty / E. Whipple procedure Close Explanation - Q: 3.4 The correct answer is E. While you do not need to know a great deal about

surgery for the Step 1 USMLE examination, a few operations may be mentioned. Among these is the Whipple procedure, also known as a pancreaticoduodenectomy. In this surgical procedure, the duodenum, proximal pancreas, gallbladder, and sometimes, the distal stomach are resected. The remaining pancreas, biliary tree, and stomach are then reattached to the small intestine. The Whipple procedure is only performed in the relatively small percentage of patients who may have resectable disease. Tumors of the tail of the pancreas might, in theory, be more easily resected, but this area tends to not produce any signs or symptoms until late in the disease, after metastasis has occurred. Pancreatic cancers that are not resectable can be treated with chemotherapy and radiation therapy. Immunotherapy using a vaccine based on the patient's own cancer cells is also being tried experimentally, and shows promise. An important thing to remember when treating these patients (and other poor prognosis patients) is that long-term survivors have occurred, and that even if death eventually intervenes, a survival of several years may be very much worthwhile to both the patient and his family (particularly if young children are indirectly involved). The Billroth I and II procedures (choices A and B ) are used to treat gastric carcinomas and intractable peptic ulcers of the duodenum and stomach. While you could argue that the gastric resection part of the Whipple procedure resembles a Billroth II operation, you should pick the Whipple procedure if both answers are available, because it is much more specific for pancreatic carcinoma. The Roux-en-Y gastric bypass (choice C) and vertical banded gastroplasty (choice D) are used to treat obesity by reducing the effective stomach volume. A 14-year-old girl asks her pediatrician if there is anything she can do about the large amounts of gas she passes during class at schooI. She states that her flatulence is worse in the afternoon, and she often has diarrhea when she goes home from schooI. Her symptoms generally subside by morning, and she thinks she is better on weekends. She has been having these problems for four or five years, but has the impression that they are worse now than when she was younger. Question 1 of 5

The physician suspects food intolerance. Statistically, intolerance to which of the following would be most likely in this patient's age group and with her history? / A. Fructose / B. Galactose / C. Lactose / D. Maltose / E. Sucrose

Explanation - Q: 4.1

Close

The correct answer is C. Lactose intolerance is the most common form of intolerance to sugars. Some authors argue that it is often not even really a "disease," but rather a natural consequence of the maturation of the small intestine, with resulting reduction in the levels of mucosal lactase. Caucasians tend to have a lower incidence of symptomatic problems than many other races (e.g., Blacks, Asians and Native Americans). Impaired absorption of the other sugars listed in the choices can also occur, but is rare. Question 2 of 5

A consulting dietician identifies a major food group that should be avoided by the young patient and further cautions that avoiding these foods may put her at risk for a secondary dietary deficiency. If the patient removes the offending food from her diet, she is at greatest risk of developing a deficiency of which of the following? / A. Calcium / B. Chloride / C. Iron / D. Potassium / E. Sodium Explanation - Q: 4.2

Close

The correct answer is A. Lactose is found predominately in milk and milk products. Aged cheeses and butter have lower concentrations of lactose and may be tolerated by many individuals. Complete removal of milk products from the diet puts the individual at greatest risk for developing calcium deficiency, because milk products are a major source of calcium. Three easy ways to prevent the deficiency are to encourage the individual to drink fruit juice products supplemented with calcium, to use lactose-free milk products, and to chew calcium-containing antacid tablets. Lactase enzyme is also available in forms that can be added to milk (usually hours before drinking) or taken as tablets before eating a lactose-containing meal. Table salt is the usual source of sodium and chloride (choices B and E). Meats are the most important dietary source of iron (choice C). Dietary deficiencies of potassium (choice D) are usually only seen if severe restriction of all fruits and vegetables is present or if the patient is taking a Klosing diuretic. Question 3 of 5

This patient's diarrhea would be most accurately described as which of the following / A. Diarrhea related to increased intestinal motility

/ / / /

B. Diarrhea related to short gut syndrome C. Exudative diarrhea D. Osmotic diarrhea E. Secretory diarrhea Explanation - Q: 4.3

Close

The correct answer is D. While there is some overlap in mechanisms in some cases of diarrhea, it can be helpful to conceptualize diarrhea into categories based on the mechanism accounting for the diarrhea. In this case, the lactose sugar that is retained in the gut markedly increases the osmolarity of the gut contents, which provides a physiologic force that tends to hold fluid in the gut. Irritable bowel syndrome is a good example of a type of diarrhea due to increased intestinal motility (choice A). The diarrhea seen in infants after recovery from necrotizing enterocolitis is a good example of diarrhea related to a short gut (choice B). The diarrhea seen in patients with invasive amebiasis is an example of an exudative diarrhea (choice C). The toxin-mediated diarrhea seen in cholera is a good example of a secretory diarrhea (choice E). Question 4 of 5

The undigested carbohydrate passing into the colon will induce enzymes for its metabolism in resident E. Coli. The mechanism that induces gene expression for these enzymes most directly involves the carbohydrate / A. binding to a repressor protein / B. binding to an activator protein / C. binding to an enhancer element associated with the gene region / D. decreasing cAMP within the E. Coli / E. increasing cAMP within the E.Coli Explanation - Q: 4.4

Close

The correct answer is A. The enzymes required to metabolize lactose are encoded by the lactose operon in E. coli. Lactose induces gene expression by binding to a repressor protein and interfering with repressor binding to the operator region of the DNA. Lack of repression contributes to inducing expression of the genes. E. coli carries out lactose fermentation, producing hydrogen gas that may be exhaled and detected in the breath of individuals who are lactose intolerant.

Lactose has no direct effect on the cAMP concentration (choices D and E). cAMP concentrations are more directly controlled by the extracellular concentration of glucose. In response to a low extracellular glucose concentration, cAMP increases, binds to an activator protein (choice B) that in turn binds to a CAP region (similar to a eukaryotic enhancer) (choice C) in the DNA. This mechanism does not directly involve lactose. Question 5 of 5

The result of a large population study to determine the adult distribution of deficiency of the intestinal enzyme that would normally metabolize the nutrient in question is shown in the graph above. Given this data, if one wanted to determine the heritability of this deficiency, the most appropriate test would be which of the following? / A. A concordance study in twins / B. Calculation of the Hardy-Weinberg distribution / C. Calculation of the linkage dysequilibrium / D. Log of the odds (LOD) score calculation / E. Spectral karyotype Explanation - Q: 4.5

Close

The correct answer is A. The graph shows a distribution characteristic of a multifactorial trait. If one wanted to determine the contribution of genes (heritability) versus non-genetic factors (diet, environment, etc.), a concordance study in monozygotic (MZ) versus dizygotic (DZ) twins would be appropriate. Heritability is calculated as (CMZ - CDZ)/(1- CDZ ). It is not necessary to memorize the calculation but rather to understand the concept

of heritability and concordance studies in twins. The Hardy-Weinberg equation (choice B) is most commonly used to calculate carrier frequencies in recessive disease. It can also be used to calculate the disease prevalence from the carrier frequency. Although it can be applied to dominant genetic diseases, there is no carrier status in these conditions and the equation is not typically used in these cases. Linkage dysequilibrium exists if specific combinations of alleles at two loci are seen together more often than expected by chance. Calculation of linkage dysequilibrium (choice C) is most commonly used in mapping genes. A LOD score calculation (choice D) is used to identify linkage between two genetic loci. A common usage is in establishing linkage between a disease phenotype and a known chromosomal marker in an effort to map the gene involved. A spectral karyotype (choice E) is used to assess cytogenetic abnormalities such as trisomies, deletions, and translocations.

A 41-year-old woman comes to the physician 3 weeks after a vaginal delivery, complaining of a lump in her right breast. She states that over the past few days, she has noticed increasing redness in the area and that the site is painful and feels somewhat firm to her. She also complains of fever and chills. She is concerned because she is currently breastfeeding her child. She has no medical or surgical history. She uses acetaminophen occasionally for headaches and is allergic to sulfa drugs. Her temperature is 37.8 C (100.1 F), blood pressure is 110/70 mm Hg, pulse is 98/minute, and respirations are 12/minute. The right breast has an area of erythema and edema that is tender with no fluctuance. Question 1 of 4

Which of the following is the most likely diagnosis? / A. Breast carcinoma / B. Eczema / C. Fibroadenoma / D. Mastitis / E. Trauma Explanation - Q: 1.1

Close

The correct answer is D. Mastitis represents a parenchymatous infection of the mammary glands. It is most often seen in postpartum women who are breast-feeding, and the symptoms typically appear 3-4 weeks postpartum. Most women with mastitis complain of pain in the breast with an area of redness and "hardness." Women also often have fever and chills, myalgias and arthralgias, and tachycardia. Examination shows erythema, edema, and tenderness. Patients with breast carcinoma (choice A) often present with an asymptomatic mass. While it is possible for patients with breast cancer to have erythema and edema of the breast and tenderness on exam, this presentation in a postpartum, breast-feeding woman is most consistent with mastitis. Eczema (choice B) does not present with a lump, as this patient has. It is a superficial disorder involving the top part (epidermis and upper dermis) of the skin only. It is also characterized by scale with erythema and not associated with fever, chills, and systemic symptoms like this patient has. Fibroadenoma (choice C) represents a proliferation of fibrous tissue in the breast. It is the most common tumor in young women. Patients with fibroadenoma typically present with complaints of a breast lump. The mass is usually small, unilateral, firm, and freely mobile. Patients with fibroadenoma do not usually present with erythema, edema, pain in the breast, and

systemic signs of infection. Trauma (choice E) to the breast can lead to a condition called fat necrosis. Patients with breast trauma usually complain of a firm area of the breast, sometimes mobile, and occasionally with induration. However, patients with fat necrosis from breast trauma usually recall a prior incident of trauma. Question 2 of 4

Histological examination of diagnostic tissue from this patient would reveal which of the following? / A. A Iocalized area of acute inflammation / B. Cellular, fibroblastic stroma enclosing cystic spaces lined by epithelium / C. Edema fluid within the intercellular spaces of the epidermis / D. Hemorrhage enclosed within the tissue / E. Strands of infiltrating tumor cells Explanation - Q: 1.2

Close

The correct answer is A. This patient has a presentation that is most consistent with mastitis, which represents an acute localized infection. Histologic evaluation would, therefore, reveal a localized area of acute inflammation with edema and neutrophil emigration. Cellular, fibroblastic stroma enclosing cystic spaces lined by epithelium (choice B) would be the histology of a fibroadenoma and not mastitis. Edema fluid within the intercellular spaces of the epidermis (choice C) is the histologic finding in eczema. Hemorrhage enclosed within a tissue (choice D) describes a hematoma and would be the expected histologic finding in a patient with trauma to the breast with a resultant hematoma. This patient's presentation is more consistent with mastitis than trauma. Strands of infiltrating tumor cells (choice E) would be the expected finding in a patient with certain types of carcinoma of the breast. This is not the histologic finding in mastitis. Question 3 of 4

Which of the following is most likely responsible for this pathologic process?

Explanation - Q: 1.3

Close

The correct answer is E. Staphylococcus aureus is a catalase-positive, coagulase-positive, and beta-hemolytic organism that is the most common cause of mastitis. The source of the organism is almost always from the nursing infant's oropharynx. Enterotoxin F, or Toxic Shock Syndrome Toxin, has been reported to cause toxic shock syndrome in some patients with mastitis caused by Staphylococcus aureus. Blunt force injury (choice A) to the breast might be expected to cause a hematoma or fat necrosis. This patient has a presentation that is consistent with mastitis, and not traumatic injury to the breast. Chlamydia trachomatis(choice B) is an obligate intracellular organism. It is most commonly found in the genital tract and is associated with cervicitis and pelvic inflammatory disease in women, urethritis in men, and pneumonia and conjunctivitis in newborns. It is not normally associated with mastitis. Hormonal exposure (choice C) is not considered causative of mastitis. There is some evidence that hormonal exposure may contribute to the development of breast cancer. This patient, however, has a presentation more consistent with mastitis than breast cancer. Neisseria gonorrhoeae(choice D) is a gram-negative coccus that can cause cervicitis, pelvic inflammatory disease, arthritis, pharyngitis, and urethritis. It can also cause neonatal conjunctivitis. It is not commonly associated with mastitis. Question 4 of 4

The patient is started on dicloxacillin. This medication works via which of the following mechanisms? / A. BIocking cell wall synthesis / B. Inhibition of bacterial dihydrofolate reductase / C. Inhibition of bacteriaI DNA gyrase / D. Inhibition of protein synthesis / E. Inhibition of resorption of sodium and chloride Explanation - Q: 1.4

Close

The correct answer is A. Dicloxacillin belongs to the general class of penicillin antibiotics. Penicillins interfere with bacterial cell wall synthesis by binding to bacterial penicillin binding proteins, resulting in eventual bacterial cell lysis. Bacterial resistance to penicillins results when bacterial betalactamases disrupt the beta-lactam ring contained within these antibiotics. Dicloxacillin (like methicillin and nafcillin) is synthesized to be resistant to beta-lactamases. However, resistance to these antibiotics is increasing as well. If a patient with mastitis does not respond to dicloxacillin, bacterial resistance should be suspected and vancomycin should be used. Inhibition of bacterial dihydrofolate reductase (choice B) is the mechanism of action of trimethoprim and pyrimethamine. Inhibition of bacterial DNA gyrase (choice C) is the mechanism of action of the fluoroquinolones and quinolones. Inhibition of protein synthesis (choice D) is the mechanism of action of the lincosamines (clindamycin, lincomycin). These drugs bind the 50S subunit of ribosomes to inhibit the bacterial protein synthesis. Inhibition of resorption of sodium and chloride (choice E) is the mechanism of furosemide (a loop diuretic). A 37-year-old woman undergoes a routine breast examination. During the breast examination, the physician is aware that the skin of the breast moves together with the underlying breast tissue, rather than being obviously separate from it. Question 1 of 6

The breast tissue is normally attached to the overlying skin via which of the following? / A. Cooper's ligaments / B. Cruciate ligaments / C. Falciform ligament / D. Poupart's ligaments / E. Rhomboid ligaments Explanation - Q: 2.1

Close

The correct answer is A. The suspensory ligaments of Cooper are fibrous condensations of connective tissue stroma that attach the mammary gland to the dermis of the overlying skin. These are particularly prominent in the superior aspect of the breast, and help to support the breast tissue. The cruciate ligaments(choice B) are in the knee.

The falciform ligament (choice C) attaches the peritoneum to the liver. Poupart's ligament (choice D) is an alternative name for the inguinal ligament. The rhomboid ligament (choice E) is another name for the costoclavicular ligament. Question 2 of 6

Careful examination of the central depressed area of the nipple demonstrates multiple small openings. These openings are from which of the following? / A. Areola / B. Lactiferous ducts / C. Montgomery's glands / D. Sweat glands / E. Terminal ductules Explanation - Q: 2.2

Close

The correct answer is B. The lactiferous ducts open into the nipples. The area of each of these ducts immediately below the nipple is usually dilated, forming a lactiferous sinus, which can store a droplet of milk that helps to initiate the baby's sucking reflex during nursing. The areola (choice A) is the ring of darkly pigmented skin around the nipple. Montgomery's glands (choice C) are modified eccrine glands (described incorrectly by some authors as sebaceous glands) that provide oil and moisture for the skin of the nipple and areola. They open into the areola in small tubercles rather than the nipple. Sweat glands (choice D) are common in the skin of the breast generally, but are too small to be able to seen by the unaided eye. The terminal ductules (choice E) of the breast system are at the deep end of the duct system of the breasts, and receive milk from the lobular tissue. Question 3 of 6

The physician identifies a palpable mass in one breast, and the patient is scheduled for a "Iumpectomy." The reason that "Iumpectomies" for breast lumps can be safely performed in some patients is that the breast is divided into multiple lobes, each of which contains a separate duct system with connecting lobules. The normal breast usually contains how many lobes? / A. 2 to 3 / B. 5 to 7 / C. 10 to 12

/ D. 15 to 25 / E. 30 to 50

Explanation - Q: 2.3

Close

The correct answer is D. The normal breast contains 15 to 25 lobes. Each lobe can be thought of as having an "inverted tree" composed of a "trunk" made of the lactiferous duct, "branches" made of smaller ducts that feed into the lactiferous duct, and "leaves" made of lobular tissue. Depending on the clinical setting, surgeons will also sometimes excise the duct system under the nipple along with the lump in the breast. Question 4 of 6

The surgical specimen is sent fresh from the surgical suite to the laboratory for frozen section examination. Before cutting into the specimen, the pathologist makes a careful gross examination, which demonstrates that most of the specimen has replacement of the normally fatty breast tissue with strands of dense, white, firm tissue. In a few areas, roughly spherical lesions up to 3-cm diameter with a bluish hue to them are seen. Palpation of these areas produces a fluctuant sensation. Which of the following is the most likely diagnosis based on the gross evaluation? / A. Breast cancer / B. Changes of pregnancy / C. Fibroadenoma / D. Fibrocystic disease / E. Mastitis Explanation - Q: 2.4

Close

The correct answer is D. While a careful pathologist will wait for appropriate frozen or permanent tissue histologic examination, the description given in the question stem is most suggestive of fibrocystic disease. The fibrous part of the lesion forms the dense, white tissue strands, while the blue lesions are what are called "blue-domed cysts. The blue color comes from the presence of darkly colored fluid (which usually means old hemorrhage) within the cyst. The interior cyst wall is usually smooth. These blue-domed cysts are a favorite of examiners because they produce a distinctive gross picture and should be specifically associated with fibrocystic disease, but you should be aware that in real life they do not occur with anything near the frequency of fibrocystic disease without obvious large cysts grossly (but many smaller cysts on microscopic examination). Fibrocystic disease of the breast is a very common lesion, and a frequent source of palpable lumps in the breast, which may require further evaluation through either excisional biopsy (as in this case), Tru-cut needle biopsy (producing a thin core about 1 cm long), or

needle aspiration (producing fluid or cells for cytology). While a diagnosis of breast cancer should obviously be confirmed microscopically before telling the clinician, invasive breast cancer (choice A) can be suspected when there is a hard, white, nodular area of the breast that has irregular (but often reasonably well-defined - in contrast to the fibrotic areas of fibrocystic disease) boundaries. The changes of pregnancy (choice B) cannot be reliably picked up on gross examination of the breast. Fibroadenoma (choice C), like breast cancer, tends to produce a welldefined mass lesion, but it typically has smooth borders and may have a slightly gray and slightly mucoid (e.g., shiny or oily appearing) surface on cross-section. Long-standing mastitis (choice E) can cause breast fibrosis, but will not cause blue-domed cyst formation. More acutely, mastitis can produce abscesses (which appear white rather than blue). Question 5 of 6

Frozen section examination demonstrates fibrosis and cystic spaces. AIso seen are areas of compressed glands with a lobular orientation. The glands are lined by a single layer of epithelial cells with oval nuclei and regular arrangement. No true invasion of glands into the adjacent stroma is seen. This patient probably has which of the following? / A. Atypical ductal hyperplasia / B. Ductal carcinoma in situ / C. Lobular carcinoma in situ / D. Sclerosing adenosis / E. Usual ductal hyperplasia Explanation - Q: 2.5

Close

The correct answer is D. The lesion described is sclerosing adenosis. The tip-offs in the description are the references to compressed glands and lobular orientation. Sclerosing adenosis is a common component of fibrocystic disease, and occurs when fibrosis distorts the normal lobular architecture. The result can be some fairly bizarre, but completely benign, compressed glands that may mimic carcinoma on both frozen and permanent sections. Low power examination is often helpful, as this emphasizes the lobular character of the lesion. Lesions actually involving the epithelium of the duct system typically have multiple layers of cells and range from usual ductal hyperplasia (choice E, with low risk of invasive carcinoma and characterized by the presence of both myoepithelial cells and epithelial cells within the duct) through atypical

ductal hyperplasia (choice A, with medium risk of invasive carcinoma and characterized by ductal carcinoma in situ-like features only involving portions of a duct) to ductal carcinoma in situ (choice B, with relatively high risk of invasive carcinoma and characterized by clearly abnormal features such as loss of myoepithelial cells and formation of cribriform patterns involving complete cross-sections of ducts). Lobular carcinoma in situ (choice C) typically produces lobular units whose lumina are completely filled with epithelial cells. In practice, while you may be asked to distinguish classic examples of the different lesions mentioned in the choices on examinations, you should be aware that this whole area can be very problematic in real-life microscopic examinations of breast tissue, and one piece of breast tissue sent to different experts in breast pathology may be returned with a variety of diagnoses.

Question 6 of 6

Which of the following breast lesions is considered to have the greatest potential for eventual progression to a malignant lesion? / A. Apocrine metaplasia / B. BIue dome cyst / C. Epithelial hyperplasia / D. Fat necrosis / E. Fibrosis Explanation - Q: 2.6

Close

The correct answer is C. While fibrocystic disease may have many components, including cyst formation (choice B), apocrine metaplasia (choice A, a benign alteration of cyst epithelium to resemble that of apocrine sweat glands), sclerosing adenosis, and fibrosis (choice E), only the epithelial hyperplasia (usual, atypical, or carcinoma in situ) is thought to indicate significant premalignant (or malignant, for carcinoma in situ) potential. For this reason, most pathologists pay particular attention to the epithelial lining of the ducts and lobules when evaluating breast biopsy specimens with fibrocystic disease. Fibrocystic breasts without any evidence of epithelial changes do not appear to have any significant increased risk of progression to breast cancer. (You should, however, be aware that a fibrocystic breast may make both breast palpation and mammography more difficult and make it more likely to miss a small lesion.) Fat necrosis (choice D) is seen after breast trauma, and has no significant malignant potential.

A 47-year-old man presents to his physician because he has noticed that his right breast is noticeably larger than his left breast. Breast examination demonstrates that the right breast is diffusely enlarged. The breast tissue is freely mobile and without distinct masses. The patient reports that the breast has been slowly enlarging over a period of several years. Question 1 of 7

Which of the following is the most likely diagnosis? / A. Ductal carcinoma in situ / B. Fibroadenoma / C. Gynecomastia / D. Invasive ductal carcinoma / E. Lobular carcinoma in situ Explanation - Q: 3.1

Close

The correct answer is C. The most common cause for male breast evaluation in the United States is gynecomastia, or benign breast enlargement. Gynecomastia may involve one or both breasts. Male breasts can also develop ductal carcinoma in situ (choice A), invasive ductal carcinoma (choice D), and fibroadenoma (choice B), but these lesions are much less common than gynecomastia. The glands in male breasts do not have lobular tissue, so lobular carcinoma in situ (choice E) does not occur. Question 2 of 7

AIso noted on physical examination is the presence of multiple spider angiomata. These are most closely associated with disease of which of the following organs? / A. Liver / B. Prostate / C. Stomach / D. Testes / E. Thyroid Explanation - Q: 3.2

Close

The correct answer is A. Spider angiomas are small vascular lesions with small blood vessels radiating from a central point. They are specifically associated with liver disease, particularly due to alcohol abuse. They are not specifically associated with diseases of the other organs listed in the choices. Question 3 of 7

The presence of the spider angiomas should specifically trigger a question to the patient about his history of use of which of the following? / A. AIcohol / B. Caffeine / C. Cigarettes / D. Heroin / E. Marijuana Explanation - Q: 3.3

Close

The correct answer is A. Spider angiomas are most often associated with liver disease related to alcohol abuse. Abuse of the other items listed in the choices does not predispose for spider angioma formation. Question 4 of 7

Which of the following is the most likely mechanism causing a relative excess of hormone leading to the breast enlargement in this patient? / A. Decreased production of testosterone secondary to primary hypogonadism / B. Drug that inhibits testosterone synthesis / C. Drug with estrogen-Iike activity / D. Increased peripheral conversion of androgens to estrogens / E. Increased production of estrogen by a cancer Explanation - Q: 3.4

Close

The correct answer is D. Peripheral conversion of androgens (testosterone and androstenedione) to estrogens occurs mainly in adipose tissue, muscle, and skin. In patients with chronic liver disease, malnutrition, and hyperthyroidism, this peripheral conversion is increased, and may be associated with feminization (seen as changes in hair distribution, body fat distribution, and breast size). Conditions that cause primary or secondary hypogonadism (choice A) can cause gynecomastia by the mechanisms of decreased production and/or action of testosterone. These conditions can include Klinefelter syndrome, congenital anorchia, testicular trauma or torsion, viral orchitis (e.g., mumps), pituitary tumors, and renal failure. Drugs that can cause gynecomastia by inhibiting testosterone synthesis (choice B) or action include ketoconazole, metronidazole, cisplatin, spironolactone, and cimetidine. Drugs that can cause gynecomastia because of their estrogen-like activity (choice C) include diethylstilbestrol, digitalis, and estrogen-containing foods and cosmetics.

Gynecomastia can also be seen as a consequence of increased estrogen production by some tumors (choice E), including testicular tumors and cancers secreting ectopic hCG (from lung, kidney, GI tract, and extragonadal germ cell tumors). Gynecomastia can also occur as a normal physiologic variant, particularly during puberty and in older men. Question 5 of 7

Which of the following enzymes converts androgens to estrogens? / A. ALA synthase / B. Aromatase / C. Branching enzyme / D. MethylmalonyI CoA mutase / E. PRPP synthetase Explanation - Q: 3.5

Close

The correct answer is B. The enzyme aromatase is found in adipose tissue (and hence is increased in fat people), muscle, and skin. It acts on androgens to form estrogens by adding double bonds to make a benzenelike ring. It is this enzyme whose activity is relatively increased in liver disease. ALA synthase (choice A) is an important early enzyme in heme synthesis. Branching enzyme (choice C) is used in glycogen formation. Methylmalonyl CoA mutase (choice D) is involved in the propionic acid pathway leading to synthesis of succinyl CoA. PRPP synthetase (choice E) occurs in purine synthesis.

Question 6 of 7

Which of the following is a genetic syndrome associated with this patient's condition and a 10- to 20-fold increased incidence of breast cancer? / A. Cushing syndrome / B. Down syndrome / C. Hashimoto disease / D. KIinefelter syndrome / E. Turner syndrome

Explanation - Q: 3.6

Close

The correct answer is D. The genetic condition Klinefelter syndrome (47,XXY) is fairly unique among the causes of gynecomastia in that it is accompanied by an increased incidence of male breast cancer. In most types of patients with gynecomastia, the incidence of breast cancer is not increased, and there are no adverse medical complications of the gynecomastia (although social and psychological problems may occur). Gynecomastia is usually only treated (with surgery) in patients in whom the breast either continues to enlarge or is personally troubling to the patient. Gynecomastia may occur in Cushing syndrome (choice A) and hyperthyroidism related to Hashimoto disease (choice C), but these are not genetic diseases. Gynecomastia is not usually a feature of Down syndrome (choice B), or Turner syndrome (choice E). Question 7 of 7

The normal male breast differs from the normal female breast in which of the following ways? / A. Adipose tissue is absent / B. Dermis is absent / C. Ductal tissue is absent / D. Lobular tissue is absent / E. Muscle is absent Explanation - Q: 3.7

Close

The correct answer is D. The normal male breast (and the male breast with gynecomastia), unlike the normal female breast, lacks lobular tissue. Both male and female breasts contain ductal tissue (choice C), adipose tissue (choice A), dermis (choice B), and small amounts of smooth muscle tissue (choice E).

A 53-year-old woman consults a physician after discovering a mass in her breast. Physical examination demonstrates a 1.5-cm diameter, firm mass lesion in the upper, outer quadrant of her right breast. The mass is surgically removed and pathologic examination of tissue obtained at surgery reveals invasive breast cancer. Question 1 of 5

What percentage of breast masses are discovered by the patient, rather than by mammography or physician examination of the breasts? / A. 5%

/ / / /

B. 20% C. 50% D. 80% E. 95% Explanation - Q: 1.1

Close

The correct answer is D. Breast cancer accounts for the greatest number of new cancer cases in women each year. Mammography is the screening method used to detect subclinical breast cancer-the stage at which breast cancer is least likely to have spread, but about 80% of breast masses are discovered initially by the patient, which it is why it is important to continue to stress breast self-examination. Question 2 of 5

While about 75% of the lymphatic fluid from the breast drains first to the axilla, most of the remaining lymphatic fluid drains first to which of the following groups of lymph nodes? / A. Anterior internal thoracic nodes / B. Internal inferior thoracic nodes / C. Lateral intercostal nodes / D. Superior mediastinal nodes / E. Tracheobronchial nodes Explanation - Q: 1.2

Close

The correct answer is A. The anterior internal thoracic nodes, also known as the internal mammary nodes, are a pair of chained lymph nodes running superiorly to inferiorly along the chest wall near both sides of the sternum. They are inaccessible for surgical removal during mastectomy, but may contain metastatic breast cancer. Rarely, the lateral intercostal nodes (choice C) may contain metastatic breast cancer. The inferior internal thoracic nodes (choice B) drain the liver and diaphragm. The superior mediastinal nodes (choice D) drain the trachea, esophagus, and heart. The tracheobronchial nodes (choice E) drain the lung. Question 3 of 5

Which of the following will most likely be identified at pathologic examination of this woman's breast mass? / A. Ductal carcinoma / B. Lobular carcinoma

/ C. Medullary carcinoma / D. Paget disease / E. Tubular carcinoma

Explanation - Q: 1.2

Close

The correct answer is A. The anterior internal thoracic nodes, also known as the internal mammary nodes, are a pair of chained lymph nodes running superiorly to inferiorly along the chest wall near both sides of the sternum. They are inaccessible for surgical removal during mastectomy, but may contain metastatic breast cancer. Rarely, the lateral intercostal nodes (choice C) may contain metastatic breast cancer. The inferior internal thoracic nodes (choice B) drain the liver and diaphragm. The superior mediastinal nodes (choice D) drain the trachea, esophagus, and heart. The tracheobronchial nodes (choice E) drain the lung. Question 3 of 5

Which of the following will most likely be identified at pathologic examination of this woman's breast mass? / A. Ductal carcinoma / B. Lobular carcinoma / C. Medullary carcinoma / D. Paget disease / E. Tubular carcinoma Explanation - Q: 1.3

Close

The correct answer is A. There is a cumulative risk in women of developing breast cancer of 1 in 8 by age 95; 1/3 to 1/2 of the patients die of the disease. Breast cancer may occur in a ductal or a lobular pattern. Invasive ductal carcinoma, not otherwise specified, is the most common histological type of invasive breast cancer. Risk factors for breast cancer include positive family history, early menarche, late menopause, late first pregnancy, and history of in situ or invasive breast cancer. Women who develop breast cancer before age 35 tend to have more aggressive disease. Two breast cancer genes, BRCA1 and BRCA2, have been identified; 5% of women with breast cancer carry one or the other of these genes. Male breast cancer is much less common than female, but has a high mortality rate Lobular carcinoma (choice B) is the second most common type of breast cancer. It may occur bilaterally in some patients. Medullary carcinoma (choice C) and tubular carcinoma (choice E) are

histological variants of ductal carcinoma with statistically better prognoses than ductal carcinoma, not otherwise specified. Paget disease (choice D) is the presence of individual adenocarcinoma cells within the skin overlying a breast cancer. Question 4 of 5

Immunohistochemical examination of paraffin-embedded sections through the tumor demonstrates that it stains for estrogen receptors (ER) and progesterone receptors (PR). This finding specifically suggests that the tumor may respond to which of the following drugs? / A. 5-FIuorouracil / B. Cyclophosphamide / C. Doxorubicin / D. Methotrexate / E. Tamoxifen Explanation - Q: 1.4

Close

The correct answer is E. Treatment with adjuvant tamoxifen for 5 years in ER positive tumors can reduce the risk of death by 25% in both pre- and postmenopausal women with or without axial lymph node involvement. Breast cancers that express the Her-2/neu receptor may respond to a new drug, trastuzumab, which is a monoclonal antibody directed against the receptor. The other drugs listed are all used for breast cancer chemotherapy, but work because they are cytotoxic rather than because they affect the hormonal response of the cancer. Question 5 of 5

The patient's physician suggests that she immediately begin chemotherapy treatment. The patient dismisses his suggestion and says, "I do not need any medicine, all l need is bedrest." This statement is most consistent with which of the following responses to illness? / A. Acceptance / B. Anger / C. Bargaining / D. Denial / E. Grieving Explanation - Q: 1.5

Close

The correct answer is D. Denial is a coping mechanism to defend against overwhelming anxiety. Pathologic and extreme denial can interfere with accurate diagnosis, impede treatment, and consequently perpetuate the

disease state. Denial is common in the early stages of dealing with a terminal illness and is not necessarily pathologic. Less extreme forms of denial may even serve the patient in positive ways. "I'm as strong now as I was when I was 20 and I'm gonna make it." Notably, the stages of grieving over the loss of a loved one (bereavement) are very similar to grieving over the loss of one's health (stages of dying). Acceptance (choice A) is a realistic perspective concerning the consequences of illness. "Coming to terms" with the illness restores emotional equilibrium and patients appear to return to their baseline personality and emotional functioning. Anger (choice B) is often directed at fate, God, themselves, their caretakers, and their families and, if taken to the extreme, may result in isolation from much needed support. Bargaining (choice C) entails promises to buy additional time. Grieving (choice E) is a process of changing affective states over time and includes five stages as described by Elisabeth Kubler-Ross (denial, anger, bargaining, depression, and acceptance). Denial, anger, anxiety, depression, and dependence can all be abnormal responses to illness (when extreme).

A 23-year-old woman comes to the physician because of a lump in her right breast. She states that she first noted the lump about a year ago and that it has seemed to enlarge over the past year. She notes some occasional tenderness in the area, usually at the same time during her menstrual cycle. She has no medical problems. She had an appendectomy at the age of 18. She takes no medications and is allergic to penicillin. Examination of the breast demonstrates a freely mobile, smoothly contoured, discrete mass in the upper outer quadrant of the breast. UItrasonography demonstrates a smooth mass with circumscribed margins and homogeneous echo pattern, consistent with a solid Iesion. Question 1 of 4

Which of the following is the most likely diagnosis? / A. Breast abscess / B. Fibroadenoma / C. Fibrocystic breast changes / D. Mastitis / E. Pregnancy Explanation - Q: 2.1

Close

The correct answer is B. Fibroadenomas are the most common breast lesions found in women under 25 years of age. Fibroadenomas are the second most common cause of benign breast lesions (second only to fibrocystic changes) in women of all ages. Patients with a fibroadenoma typically present complaining of a palpable lump, often with some gradual growth. There may be some occasional cyclic tenderness. Management is with biopsy or close observation. If the lesion is palpable, increasing in size, or psychologically disturbing, biopsy should be performed. If the woman is less than 25 years of age with small fibroadenomas that appear "classic" by imaging, then expectant management with careful continued observation can be considered. A breast abscess (choice A) can also present as a lump in the breast. However, an abscess represents a localized collection of pus resulting from an infection. Therefore, patients with a breast abscess will often have erythema, edema, pain, and tenderness around the area of the mass. Such patients may also have systemic signs of infection, including fever and tachycardia. This patient has no evidence of infection. Fibrocystic breast changes (choice C) are the most common, benign condition of the breast. They can be present in young women, become more common as a woman approaches the menopause, and often regress during and after the menopause. The most common symptoms are pain and tenderness, and the masses are usually bilateral. Mammography and ultrasound of the breast often reveal the fibrocystic changes. Mastitis (choice D) is an infection of the breast. It can occur in any woman, but most often occurs in lactating women during the postpartum period. Patients with mastitis will often present with tenderness and erythema of the breast along with fever. Treatment is with antibiotics. Pregnancy (choice E) is associated with a number of changes in the breast, especially as the breast prepares for lactation. Fibroadenomas may grow rapidly during pregnancy, but the primary diagnosis, and hence the best answer, is still fibroadenoma Question 2 of 4

Histological examination of diagnostic tissue from this patient would reveal which of the following? / A. A classic cribriform pattern with neoplastic epithelial cells / B. Cystically dilated ducts plus stromal fibrosis / C. Irregular steatocytes and intervening necrotic material and inflammatory cells / D. Lobular hypertrophy

/ E. Proliferating ducts and stromal cells

Explanation - Q: 2.2

Close

The correct answer is E. Fibroadenomas have a typical microscopic appearance. The predominant feature is the fibroblastic stroma. This is a delicate, cellular, fibroblastic stroma resembling intralobular stroma. Within this fibroblastic stroma are seen proliferating ducts. These ducts are usually compressed and are lined by benign-appearing epithelium. If the margin includes surrounding tissue beyond the fibroadenoma, compressed breast connective tissue forming a "capsule" to the mass may be seen as well. A classic cribriform pattern with neoplastic epithelial cells (choice A) is what would be revealed by histological examination of an intraductal carcinoma of the breast. The epithelium in a fibroadenoma is benign-appearing. Histologic examination of a biopsy specimen from a patient with fibrocystic breast changes would demonstrate cystically dilated ducts plus stromal fibrosis (choice B). Irregular steatocytes and intervening necrotic material and inflammatory cells (choice C) describes the findings on pathologic evaluation of a biopsy specimen from a patient with fat necrosis. Fat necrosis is most commonly caused by trauma, but can also occur after surgery or radiation therapy. Lobular hypertrophy (choice D) is seen in pregnant women. This lobular hypertrophy occurs during the pregnancy to allow for lactation in the postpartum period. Question 3 of 4

If the histologic examination revealed similar findings as in this patient, but demonstrated increased cellularity, an elevated mitotic rate, stromal overgrowth, and infiltrative borders, then which of the following is the most likely diagnosis? / A. Fat necrosis / B. Fibrocystic changes / C. Mastitis / D. Normal breast tissue / E. Phyllodes tumor Explanation - Q: 2.2

Close

The correct answer is E. Fibroadenomas have a typical microscopic appearance. The predominant feature is the fibroblastic stroma. This is a delicate, cellular, fibroblastic stroma resembling intralobular stroma. Within this fibroblastic stroma are seen proliferating ducts. These ducts are usually

compressed and are lined by benign-appearing epithelium. If the margin includes surrounding tissue beyond the fibroadenoma, compressed breast connective tissue forming a "capsule" to the mass may be seen as well. A classic cribriform pattern with neoplastic epithelial cells (choice A) is what would be revealed by histological examination of an intraductal carcinoma of the breast. The epithelium in a fibroadenoma is benign-appearing. Histologic examination of a biopsy specimen from a patient with fibrocystic breast changes would demonstrate cystically dilated ducts plus stromal fibrosis (choice B). Irregular steatocytes and intervening necrotic material and inflammatory cells (choice C) describes the findings on pathologic evaluation of a biopsy specimen from a patient with fat necrosis. Fat necrosis is most commonly caused by trauma, but can also occur after surgery or radiation therapy. Lobular hypertrophy (choice D) is seen in pregnant women. This lobular hypertrophy occurs during the pregnancy to allow for lactation in the postpartum period. Question 3 of 4

If the histologic examination revealed similar findings as in this patient, but demonstrated increased cellularity, an elevated mitotic rate, stromal overgrowth, and infiltrative borders, then which of the following is the most likely diagnosis? / A. Fat necrosis / B. Fibrocystic changes / C. Mastitis / D. Normal breast tissue / E. Phyllodes tumor Explanation - Q: 2.3

Close

The correct answer is E. Phyllodes tumors are similar to fibroadenomas in that they arise from intralobular stroma. Furthermore, on pathologic evaluation, low-grade phyllodes tumors can resemble fibroadenomas. However, there are important differences. First, most phyllodes tumors present in the sixth decade, whereas fibroadenomas most commonly present in young women. Also, while most phyllodes tumors are low-grade tumors that only rarely metastasize, some are aggressive high-grade lesions that commonly recur locally and do metastasize hematogenously. These aggressive lesions are often called cystosarcoma phyllodes. Some phyllodes tumors are small, while others may be large enough to involve virtually the entire breast. Grossly, these tumors often have leaf-like projections off of them. On histologic evaluation, the keys to distinguishing between

fibroadenoma and phyllodes tumor are the increased cellularity, enhanced mitotic rate, stromal overgrowth, nuclear pleomorphism, and infiltrative borders that are seen in phyllodes tumors and are absent in fibroadenomas. Fat necrosis (choice A) demonstrates necrotic fat cells that are surrounded by lipid-filled macrophages and an infiltration of neutrophils. It does not resemble a fibroadenoma. Fibrocystic changes (choice B) are characterized by cysts, and do not closely resemble fibroadenomas, as low-grade phyllodes tumors do. Mastitis (choice C) is an infection of the breast, usually by Staphylococcus aureus. It is characterized by acute inflammation and does not resemble fibroadenoma. Normal breast tissue (choice D) is not characterized by a pattern similar to fibroadenoma, except with increased cellularity, elevated mitotic rate, stromal overgrowth, and infiltrative borders. These are characteristics of phyllodes tumors. Question 4 of 4

If the patient were instead found to have an aggressive breast carcinoma with a poor prognosis, the Kubler-Ross model predicts that she will go through which of the following stages? / A. Acceptance, anger, ambivalence, deniaI, depression / B. BIues, depression, psychosis, treatment, resolution / C. DeniaI, anger, bargaining, depression, acceptance / D. DeniaI, anger, psychosis, homicide, suicide / E. Realization, infantilization, socialization, condemnation Explanation - Q: 2.4

Close

The correct answer is C. Elisabeth Kubler-Ross is the psychiatrist who authored the groundbreaking "On Death and Dying" in 1969. She was born in Switzerland, but moved to the United States in the 1950s, where she worked with dying patients. Her model identified five stages that occur when individuals are confronted with death: 1) Denial, 2) Anger, 3) Bargaining, 4) Depression, and 5) Acceptance. While she originally described this process as it relates to facing death, others have used these 5 steps to describe reaction to grief or loss. While these stages are useful to understand and contemplate the experience of grief or dying, it is important to recognize that not all people will go through these steps. Those who do go through each of these steps may also not go through them in the "order" described by the Kubler-Ross model. No model is perfect for explaining the intricacies of every different human being's response to grief or death. The Kubler-Ross model does provide a nice framework and starting point for understanding these

difficult issues. None of the other choices correctly describes the stages proposed by Kubler-Ross.

A 45-year-old woman presents to her primary care physician complaining of fatigue, weight gain, and shortness of breath. She has always been an active athlete, but in the past 2 weeks, has found it impossible to jog for more than a few minutes, after which she feels tired and winded. She feels like her appetite is normal or has even declined, but she notices that she has gained 15 pounds and her pants and shoes no longer fit welI. She has very little energy, and is sleeping poorly, with occasional difficulty breathing at night. She denies any pain, fever, or chills. Review of her chart reveals an up-to-date health screening including a normal baseline mammogram, a normaI Pap smear in the last year, and total cholesterol of 165 mg/dL two years ago. On physical examination, she appears comfortable, has a temperature of 36.8 C (98.2. F), blood pressure of 135/68 mm Hg, pulse of 90/min, and respiratory rate of 24/min. She appears fatigued but not in acute distress, and her skin appears normaI. Expiratory wheezes are heard at the bases of both lungs. Her heart has a normaI-sounding S1 and S2, with a II/IV soft holosystolic murmur heard best at the apex of the heart. Her abdomen is modestly distended, and her ankles are edematous. A chest x-ray film reveals cardiomegaly as well as increased vascular markings in the lung beds and bilateral small pleural effusions. Laboratory studies show: Question 1 of 5

Which of the following is the most likely diagnosis? / A. Acute leukemia / B. Cardiomyopathy / C. Fibromyalgia / D. Hypothyroidism / E. Major depressive disorder Explanation - Q: 1.1

Close

The correct answer is B. This woman has many of the classic symptoms of heart failure, with symptoms of both poor forward cardiac output (fatigue, poor appetite) and of vascular congestion in both the right and left atria (edema, abdominal distension that may be ascites, cardiomegaly, pulmonary vascular congestion and effusions seen on chest x-ray, dyspnea with exertion, and paroxysmal nocturnal dyspnea.) Acute leukemia (choice A) is a potential cause of fatigue, poor energy, and poor nutritional status (which can cause edema and pleural effusion). Usually some abnormality will be apparent, most commonly pancytopenia, due to replacement of bone marrow with leukemic cells; the leukocyte count may be elevated due to the presence of leukemic cells in the peripheral blood. They often present with bleeding or infectious complications of pancytopenia. Anemia could potentially cause a murmur due to elevated cardiac output, but

an acute leukemia would not typically cause cardiomegaly or pulmonary edema. Fibromyalgia (choice C) is a potential cause of fatigue, poor energy, and poor sleep, especially in women ages 25-45: its principal sign, however, is diffuse musculoskeletal pain and stiffness, with characteristic tender trigger points. It is not consistent with this patient's chest x-ray abnormalities or cardiac and lung findings. Based on examination, this patient could certainly have hypothyroidism (choice D). Symptoms are usually insidious in onset and include fatigue, poor appetite with weight gain, poor sleep and possibly, obstructive sleep apnea. Patients often complain of constipation, cold intolerance, stiffness and muscle cramping, as well as decreased intellectual activity. Severe hypothyroidism can result in cardiomegaly, pericardial effusion, and symptoms of cardiac failure. The skin often appears dry, rough, and doughy in texture. The normal TSH, however, makes hypothyroidism in this patient very unlikely: The TSH is nearly always elevated, as most hypothyroidism is primary, which means the pituitary is secreting maximal TSH in an attempt to stimulate a hypofunctional thyroid gland. Rarely, TSH may be normal or depressed (even undetectable) in pituitary or hypothalamic failure. To rule this out, one might test first for T4 and T3 levels. Normal levels of these, in conjunction with the normal TSH, would rule out hypothyroidism as a cause of this clinical presentation. Major depression (choice E) should always be in the differential for a patient who presents with disturbances in sleep, appetite, and energy, and can also result in weight loss or gain. These "vegetative signs" of depression may be the presenting abnormality in a depressed patient who does not note a mood disturbance themselves. One should also ask about depressed mood, anhedonia (loss of interest in or inability to take pleasure in activities the person normally enjoys), an inability to concentrate and carry on usual intellectual activities, feelings of worthlessness or guilt, and suicidal ideation. Depression cannot, however, on its own, produce the physical findings this patient has, which taken together, are worrisome for some physiologic abnormality. Question 2 of 5

Which of the following is the most likely cause of the patient's murmur? / A. Aortic insufficiency / B. Aortic stenosis / C. High-output flow murmur / D. Mitral regurgitation / E. Mitral stenosis / F. Pulmonic insufficiency / G. Pulmonic stenosis / H. Tricuspid regurgitation

/ I.

Tricuspid stenosis Explanation - Q: 1.2

Close

The correct answer is D. Mitral regurgitation is characterized by a holosystolic murmur heard best at the apex, often with a blowing sound, which may radiate to the axilla. The murmur of aortic insufficiency (choice A) is a decrescendo diastolic murmur. Remember that the aortic valve is open during systole; a systolic murmur, then, cannot represent regurgitant aortic flow due to an improperly closed valve. Aortic stenosis (choice B) does produce a systolic murmur caused by turbulent flow across a narrowed aortic valve during systole. This murmur is usually a crescendo-decrescendo murmur, often with a harsh quality, and is characteristically heard best at the base of the heart; it may radiate to the carotids as well. High-output states (choice C) can cause a similar soft systolic murmur to that described here. However, this patient's history is most consistent with cardiac failure, which is a low-output state. Mitral stenosis (choice E) causes a murmur due to turbulent low-velocity flow during diastolic filling of the left ventricle through a narrowed mitral orifice. This results in a soft diastolic murmur heard best at the apex. Remember that the mitral valve is closed during systole, therefore, an abnormal mitral sound in systole must be the sound of abnormal regurgitant flow through a closed valve. The right-sided murmurs are less common, similar in quality, and usually less loud than the left-sided murmurs (given that pressures on the right are usually lower): Pulmonic insufficiency (choice F), when audible, therefore causes a soft diastolic murmur at the right upper sternal border. Pulmonic stenosis (choice G) causes a crescendo-decrescendo systolic murmur also heard at the base of the heart. Tricuspid regurgitation (choice H) causes a holosystolic murmur at the left lower or right lower sternal border. Tricuspid stenosis (choice I) when audible, is a diastolic murmur heard best at the same location Question 3 of 5

BIood in the pulmonary veins is at the same pressure (during all phases of the cardiac cycle) as blood in which of the following? / A. Aorta / B. Left atrium / C. Left ventricle / D. Right atrium / E. Right ventricle Explanation - Q: 1.3

Close

The correct answer is B. The pressures in two chambers, which are not separated by a closed valve, will be equal. The pulmonary vein empties into the left atrium, and no valve separates the two chambers, therefore the pressures are equal in all phases of the cardiac cycle. This patient's pulmonary vascular congestion is likely due to elevated pulmonary venous pressure, which is, in turn, likely due to elevated left atrial pressures. Pressures in the aorta (choice A) will be higher than pressures in the pulmonary veins during the cardiac cycle. The left ventricle (choice C) is separated from the left atrium and the pulmonary veins by the mitral valve. The pulmonary veins and the left atrium are at the same pressure as the left ventricle during diastole, when the mitral valve is open. With complete mitral insufficiency, the pulmonary veins are completely exposed to left ventricular pressures during systole, resulting in severe pulmonary edema. The right atrium (choice D) is not in communication with the pulmonary veins, being separated from them by, in sequence, the tricuspid valve, the right ventricle, the pulmonic valve, the pulmonary arterial system, and the pulmonary capillary bed. The right ventricle (choice E), during systole, is at the same pressure as the pulmonary artery, not the pulmonary veins. During diastole, the pulmonary arterial pressure exceeds right ventricular pressure, and the valve is closed.

Question 4 of 5

To improve her shortness of breath, the patient is given furosemide. What is the molecular mechanism and site of action of this drug? / A. ADH antagonism of in the collecting ducts / B. AIdosterone antagonism in the distal tubule / C. BIockade of sodium reabsorption in the proximal tubule / D. BIockade of sodium transport in the distal tubule / E. Inhibition of carbonic anhydrase in the proximal tubule / F. Inhibition of sodium-potassium-chloride cotransport in the loop of Henle

Explanation - Q: 1.4

Close

The correct answer is F. The Na-K-2Cl cotransporter in the loop of Henle operates via an ATP-dependent sodium-potassium exchange pump in the cell that creates a gradient for sodium diffusion from the urine space into the cell. This maintains the sodium concentration gradient of the renal medulla. Furosemide is the most commonly used loop diuretic; it acts by blocking the action of the cotransporter in the thick ascending limb of the loop of Henle. ADH antagonism (choice A) is not an important diuretic drug mechanism, however, certain drugs, most notably lithium, inhibit ADH's action, resulting in nephrogenic diabetes insipidus. Aldosterone promotes the reabsorption of sodium in the late distal tubule and collecting system and promotes the excretion of potassium. Aldosterone receptor antagonism (choice B) is the mechanism of action of potassiumsparing diuretics such as spironolactone. Sodium reabsorption in the proximal tubule (choice C) is a largely passive process, which is coupled to the transport of organic solutes and anions and also to chloride transport, via both transcellular and paracellular mechanisms. The thiazide diuretics work primarily by blocking sodium transport in the early portion of the distal tubule (choice D). Acetazolamide inhibits carbonic anhydrase (choice E), preventing the luminal transformation of bicarbonate into CO2, which diffuses back into the cell. Inhibition of this enzyme increases both bicarbonate and sodium concentrations in the urine, resulting in high urine pH and metabolic acidosis. Question 5 of 5

What important physiologic effect will starting this patient on an angiotensinconverting-enzyme inhibitor achieve? / A. Decrease in arteriolar resistance, resulting in less resistance to forward cardiac output / B. Decrease in cardiac filling pressures, resulting in less pulmonary congestion / C. Increase in arteriolar resistance, resulting in improved blood pressure / D. Increase in left-ventricular end-diastolic volume, improving stroke volume via Starling forces / E. Increase in myocardial contractility, resulting in improved stroke volume / F. Stabilization of myocardial membranes, resulting in reduced risk of arrhythmia Close Explanation - Q: 1.5 The correct answer is A. In cardiac failure, the juxtaglomerular apparatus

releases renin in response to low blood pressure or low flow states. Renin cleaves angiotensinogen into angiotensin I, which is then cleaved by angiotensin-converting enzyme (ACE) into angiotensin II. Angiotensin II is a potent vasoconstrictor and increases blood pressure. This, however, increases the resistance against which the heart must pump, thereby reducing cardiac output. By reducing angiotensin II activity, systemic vascular resistance (normally high in cardiac failure, in an attempt to maintain blood pressure in the presence of low flow) is reduced, permitting the heart to eject more volume against a lower aortic pressure. This is often described as "afterload reduction" and is the mainstay of therapy in congestive heart failure. Paradoxically, blood pressure may not change: the reduced resistance, by permitting increased flow, may result in no net change in pressure. This is most easily understood as a physiologic manifestation of Ohm's law: V = IR. In electricity, this law means that voltage is equal to current times resistance. Blood pressure is analogous to voltage, cardiac output to current flow, and the resistance in this case is the resistance of the systemic vasculature. Reduction of cardiac filling pressures (choice B) or "preload," is also an important aspect of the treatment of heart failure. In heart failure, the heart operates at high filling pressures and high left ventricular end-diastolic volume (LVEDV) because both aldosterone and ADH promote the retention of fluid in response to low forward flow and decreased effective circulating volume. The result is vascular congestion in the pulmonary veins due to the high LV diastolic pressure, resulting in symptomatic pulmonary edema. By reducing this preload, congestive symptoms can be relieved, and LVEDV can be reduced without significant loss of stroke volume. ACE inhibitors, however, do not reduce preload: drugs that do this are nitrates (which act as venodilators) and diuretics. Increasing arteriolar resistance (choice C) in heart failure increases the "afterload" against which the heart must eject and does not improve cardiac output. Increasing LVEDV (choice D) is usually helpful in hypovolemia or other states in which inadequate volume is available to the heart, thereby limiting cardiac output. This happens in the portion of the Starling curve at low LVEDV, where an increase in LVEDV results in a large increase in stroke volume. Patients in symptomatic heart failure like this patient operate at very high LVEDV and benefit from its reduction. Increasing myocardial contractility (choice E) is beneficial in heart failure, and is the mechanism of action of inotropic drugs. This is not a mechanism of ACE inhibitors. Prevention of arrhythmia (choice F) is also important in heart failure, as the

dilated heart is vulnerable to both atrial and ventricular arrhythmias. This is not a direct action of ACE inhibitors, however. A 23-year-old man presents to the urgent care clinic complaining of severe throat pain, fever, chills, and diffuse joint pains. He first developed symptoms two weeks ago and was evaluated by another physician at the same clinic. A throat culture was done, and the patient was given a prescription for antibiotics that he did not filI. He now returns with a worsening of his symptoms. He has since developed severe joint pain and swelling, which first affected his right wrist, then spread to both knees, and now has also affected his left ankle. He also complains of moderate to severe chest discomfort and shortness of breath. His temperature is 38.7 C (101.6 F), blood pressure is 118/86 mm Hg, pulse is 104/min, and respirations are 20/min. There is an exudate on his oropharynx and bilateral anterior cervical lymphadenopathy. On lung examination, there are bibasilar crackles, and the cardiac examination reveals tachycardia, but a normal rhythm and no murmurs or rubs. Examination of his joints reveals synovitis in his right wrist, Ieft ankle, and both knees. A c `.Í ... . ..Í. 1 .. Í . . ;.. 1 `....`.``-..--.....`..```..`-.-`.--.` Question 1 of 5 :

Which of the following is the most likely cause of this patient's cardiac findings? / A. Acute myocardial infarction / B. Aortic dissection / C. Mitral regurgitation / D. Myocarditis / E. Wolff-Parkinson-White (WPW) syndrome Explanation - Q: 2.1

Close

The correct answer is D. The patient has myocarditis, which is an inflammation of the cardiac muscle. It is most commonly the result of an infectious process. Signs and symptoms can range from an asymptomatic state to arrhythmias, heart failure, and death. The patient often has an antecedent infection, and in this case, he had an exudative pharyngitis. Acute myocardial infarction (choice A) usually presents with severe squeezing left-sided chest pain that can radiate down the left arm. Patients are generally middle-aged, and can have risk factors for cardiac disease such as hypertension, diabetes, hypercholesterolemia, or a history of tobacco use. The electrocardiogram can vary from nonspecific T wave changes to ST segment elevation. Aortic dissection (choice B) would present as sudden onset of severe chest pain, which often radiates to the back. Patients can have hypotension,

depending on the severity of the dissection, or hypertension, which is often a predisposing factor. Patients can also have unequal pulses in their extremities, if the dissection affects one of the major arteries branching off the aortic arch. Mitral regurgitation (choice C) is a result of mitral valve insufficiency, in which there is a regurgitant flow of blood across the mitral valve, from the left ventricle, into the left atrium, during systole. It is often due to rheumatic heart disease, but can also result from mitral valve prolapse, or papillary muscle rupture. Physical examination should reveal a holosystolic murmur, heard best at the apex. Wolff-Parkinson-White (WPW) syndrome (choice E) is a ventricular preexcitation syndrome associated with an atrioventricular bypass track. Patients often have paroxysmal tachycardias, and an electrocardiogram will often reveal a shortened PR interval, a delta wave, and a wide QRS complex. Question 2 of 5

What underlying condition can explain the patient's upper respiratory as well as cardiac and joint signs and symptoms? / A. Acute rheumatic fever / B. Budd-Chiari syndrome / C. Ebstein's anomaly / D. Sjögren syndrome / E. Takayasu arteritis Explanation - Q: 2.2

Close

The correct answer is A. Acute rheumatic fever is an inflammatory disorder that affects multiple systems. There are five major criteria for rheumatic fever: carditis, migratory polyarthritis, subcutaneous nodules, Sydenham chorea, and erythema marginatum. There are also minor criteria: fever, arthralgia, elevated acute phase reactants, and a prolonged PR interval. Budd-Chiari syndrome (choice B) is an occlusion of the major hepatic veins, which leads to congestive liver disease. Patients often have abdominal pain, jaundice, and hepatomegaly. Ebstein's anomaly (choice C) is due to an anomalous attachment of the tricuspid leaflets, and results in downward displacement of the tricuspid valve into the right ventricle. This results in tricuspid regurgitation. Symptoms can vary from cyanosis to arrhythmias. Sjögren syndrome (choice D) is an autoimmune disorder characterized by inflammatory changes in glands, producing dry eyes and dry mouth.

Takayasu arteritis (choice E) is a vasculitis syndrome that affects medium to large arteries, in particular, the aortic arch and its branches. It is also known as "pulseless disease" because patients have weak or absent pulses in their upper extremities. It primarily affects young Asian females. Question 3 of 5

Which of the following test results would help confirm the most likely diagnosis? / A. EIevated antinuclear antibody / B. Low anti-deoxyribonuclease B titer / C. Low anti-hyaluronidase titer / D. Low anti-streptolysin O titer / E. Throat culture positive for group A streptococci Explanation - Q: 2.3

Close

The correct answer is E. To meet criteria for the diagnosis of rheumatic fever, patients must have either two major, or 1 major and 2 minor criteria, plus evidence of an antecedent streptococcal infection. A throat culture positive for group A streptococci would fulfill the criteria in the presence of myocarditis and migratory polyarthritis. Elevated antinuclear antibody (choice A) is not associated with rheumatic fever. In the appropriate clinical setting, it is helpful in the diagnosis of rheumatologic disorders such as systemic lupus erythematous. Anti-streptolysin O (choice D), anti-deoxyribonuclease B (choice B), and anti-hyaluronidase (choice C) are all streptococcal antibody tests. In the setting of rheumatic fever associated with a recent group A streptococcal infection, the titers for these antibody tests would be elevated (in the absence of infection, they may actually be undetectable). A significant titer of any of these antibody tests would meet criteria for the documentation of an antecedent streptococcal infection. Question 4 of 5

A biopsy of the affected cardiac tissue would most likely show which of the following? / A. Angiosarcoma / B. Aschoff body / C. Atheromas / D. Hyperplastic arteriolosclerosis / E. Libman-Sacks lesions Close Explanation - Q: 2.4 The correct answer is B. The Aschoff body is the classic lesion of rheumatic fever. It is an area of focal interstitial myocardial inflammation. It is characterized by large cells, known as Anitschkow myocytes, and Aschoff

cells, which are multinucleated giant cells. Angiosarcoma (choice A), a rare malignant tumor affecting the vascular tissue, can occur in the skin, breast, liver, or musculoskeletal system. Atheromas (choice C) are fibrous plaques within the intima of arteries. They are a finding of atherosclerosis. Hyperplastic arteriolosclerosis (choice D) is characterized by concentric, laminated thickening of arteriolar walls. It often occurs in the kidneys, and may lead to malignant nephrosclerosis. Libman-Sacks lesions (choice E) are small vegetations that occur on valvular heart tissue. They can occur on either side of the valve, and are associated with endocarditis in systemic lupus erythematous. uest|on

0

The patient continues to deteriorate, he develops worsening heart failure, and requires transfer to the intensive care unit for use of an inotropic agent to increase his cardiac output. Which of the following agents would most likely be used? / A. Benazepril / B. Diltiazem / C. Dobutamine / D. Metoprolol / E. Phenylephrine Explanation - Q: 2.5

Close

The correct answer is C. Dobutamine is a positive inotropic agent used in severe cases of heart failure that require inotropic support. Benazepril (choice A) is an angiotensin converting enzyme inhibitor. Medications in this class can be used in heart failure to decrease afterload, but they do not have any direct affect on cardiac tissue. Diltiazem (choice B) and metoprolol (choice D) are both negative inotropic agents. When used in the setting of acute heart failure, the patient's course can worsen, although beta blockers such as metoprolol and carvedilol (mixed alpha and beta blocker) are sometimes cautiously used in some patients with CHF. Phenylephrine (choice E) is an alpha-receptor agonist. It causes vasoconstriction, and is used in severe cases of hypotension.

A 78-year-old man had been previously active, but found that his health was declining. Over a four-month period, his ability to perform even very minimal exercise, such as walking around his yard, declined precipitously. The family took him from doctor to doctor, none of whom were initially able to figure out what was wrong with him. Because of the patient's age, most of the physicians that the family consulted were unwilling to do much other than to listen to the family's story and then run a few screening tests. In some ways, he acted as if he were in congestive heart failure, but he initially had no evidence of fluid overload and his lungs were clear. The cardiac profile on chest X-ray was slightly enlarged. His ECG studies were interpreted as within the normal range for his age. Angiography studies showed no evidence of significant coronary artery occlusion. Pulmonary function studies were unrevealing. Question 1 of 6

Following a Thanksgiving meaI, the patient's condition worsened markedly over the next few hours, and he was taken to an emergency department. At that point, the patient was in obvious, severe, congestive heart failure with evidence of fluid overload and pulmonary edema. Intravenous furosemide was started, which over the next few hours markedly improved his clinical condition. Furosemide is classified as which of the following? / A. Carbonic anhydrase inhibitor / B. Loop diuretic / C. Osmotic diuretic / D. Potassium-sparing diuretic / E. Thiazide diuretic Explanation - Q: 3.1

Close

The correct answer is B. Large, salty, holiday meals are notorious for setting off (potentially fatal) exacerbations of what might have been previously mild congestive failure. There are a number of drugs with diuretic activity that can increase the amount of urine that is produced. Pharmacologists subclassify these drugs based on the mechanisms by which they act. Furosemide is a diuretic that is commonly used in the hospital setting in intravenous form to rapidly reduce the degree of fluid overload present in a patient in severe congestive heart failure. This diuretic acts by inhibiting the Na/K/2Cl cotransporter on the luminal membrane of the thick ascending portion of the loop of Henle. It is consequently classified as a loop diuretic, as is ethacrynic acid, which has a similar mechanism of action. Carbonic anhydrase inhibitors (choice A), such as acetazolamide and dorzolamide, act on the proximal convoluted tubule to reduce Na+ resorption secondary to an inhibition of CO2 formation with resulting decreased

intracellular bicarbonate and H+ levels. Osmotic diuretics (choice C), such as mannitol, inhibit water reabsorption throughout the nephron. Potassium-sparing diuretics (choice D), such as spironolactone, amiloride, and triamterene, act at the level of the collecting tubules and ducts by acting as aldosterone receptor antagonists. Thiazide diuretics (choice E), such as hydrochlorothiazide, indapamide, and metolazone, inhibit the Na/Cl cotransporter in the distal convoluted tubule. Question 2 of 6

The patient is seen the following morning by a cardiologist. The cardiologist does a very careful physical examination. He notes that the heart sounds appear distant. He then has the patient lie at an angle of 30 to 45 degrees, and does a careful examination of the right jugular pulse, which he finds very worrisome. The pulse is both very elevated and shows dramatic x and y descents. Further, he notes that the venous distention paradoxically increases during inspiration. This last finding is sometimes called which of the following? / A. Chvostek's sign / B. Corrigan's sign / C. Homans' sign / D. KussmauI's sign / E. Murphy's sign Explanation - Q: 3.2

Close

The correct answer is D. The sign described is Kussmaul's sign. The act of inflating the lungs during inspiration lowers the pressure in the chest while increasing that in the abdomen, drawing blood from the abdomen into the chest (and increased abdominal pressure helps to directly drive blood toward the chest). If the right atrium cannot fill, then the jugular venous pressure rises paradoxically (not so much from blood flow from the head as from the abdomen, because the inferior vena cava and superior vena cava are functionally connected through the right atrium). Kussmaul's sign is seen in patients who have non-compliant right ventricles. It can also be seen in patients with severe ascites (which increases the intra-abdominal pressure). This case illustrates the importance of considering the jugular venous pulse as well as the arterial pulse, since the cardiologist was able to find a number of significant findings pertaining to the jugular venous pulse, which other physicians had missed. The jugular venous pressure can be used at the bedside to estimate the right atrial filling pressure. The jugular venous pressure is estimated by measuring the height of the visible venous pulse above the sternal angle, and then adding 5 cm (corresponding to how far

below the sternum the right atrium is located). The jugular venous waveform has an A wave, which is followed by an X descent, then a V wave, and finally a Y descent. The A wave (first rise in pressure) reflects the right atrial contraction, while the X-descent reflects right atrial diastole, and then early right ventricular systole. The V wave is the second major positive wave, and reflects continued venous inflow into the right atrium in opposition to a closed mitral valve. The following Y-descent is the negative deflection that occurs when the tricuspid valve opens in early diastole. Chvostek's sign (choice A) is seen in tetany, and is a facial muscle spasm occurring when the facial nerve is tapped anterior to the external auditory meatus. Corrigan's sign (choice B), which suggests aortic regurgitation, is a full, hard arterial pulse, which is followed by a sudden collapse. Homans' sign (choice C) is pain at the back of the knee or calf when the ankle is dorsiflexed, and suggests venous thrombosis of the leg. Murphy's sign (choice E) is pain on palpation of the right subcostal area during inspiration, and is frequently seen in acute cholecystitis. Question 3 of 6

This patient most likely has which of the following? / A. Acute myocarditis / B. Congestive cardiomyopathy / C. OId left ventricle myocardial infarction / D. Recent left ventricle myocardial infarction / E. Restrictive cardiomyopathy Explanation - Q: 3.3

Close

The correct answer is E. The "distant" heart sounds and jugular venous pulse findings both suggest that this patient has restrictive cardiomyopathy that is limiting the heart's ability to fill during diastole and is also impairing ventricular contraction. Other findings that may be encountered on physical examination in patients with restrictive cardiomyopathy include S3 and/or S4 heart sounds, occasional mitral or tricuspid regurgitation murmurs, and, if the patient is in secondary congestive failure, peripheral edema and pulmonary rales. Restrictive cardiomyopathy is relatively rare and the findings on physical examination are subtle, and consequently this patient's history of missed diagnosis is unfortunately not all that uncommon. Underlying causes of restrictive cardiomyopathy include endomyocardial fibrosis, Loeffler eosinophilic endomyocardial disease, hemochromatosis, amyloidosis, sarcoidosis, scleroderma, carcinoid heart disease, and glycogen storage disease. Patients typically present at an advanced stage of the disease, and may have symptoms of angina, shortness of breath, peripheral edema, and ascites with abdominal discomfort (related to pooling of blood in the liver and

other abdominal organs). Once the diagnosis is suspected, echocardiography typically demonstrates normal to symmetrically thickened heart chamber walls with rapid early-diastolic filling and slow late-diastolic filling (the cardiac chambers are acting more or less like poorly distensible plastic bags). Cardiac catheterization will more or less repeat the observations seen in the analysis of the jugular venous pulse, typically showing elevated ventricular end-diastolic pressure, normal to slightly decreased ejection fraction, and prominent x and y descents. Acute myocarditis (choice A) can cause congestive cardiomyopathy (choice B), but the heart is usually larger and the constrictive findings seen in this case would not be present. While recent and old myocardial infarctions affecting the right ventricle may produce similar jugular venous findings to those seen in this case, left ventricular infarction (choices C and D) would not impair right ventricular filling and contraction. Question 4 of 6

An endomyocardial biopsy is performed, which demonstrates eosinophilic acellular deposits within the myocardial biopsy. When recut, histological sections are stained with Congo red and viewed under polarized light, and the deposits appear bright green. These deposits are most likely to be composed of which of the following? / A. Amyloid / B. Fibrin / C. Hemosiderin / D. Melanin / E. Uric acid Explanation - Q: 3.4

Close

The correct answer is A. Amyloid deposits are suspected when hematoxylin and eosin-stained histological sections show extracellular eosinophilic deposits. The presence of amyloidosis is confirmed when the characteristic "apple-green birefringence" on Congo red stain is demonstrated. Fibrin deposits (choice B) are also red on hematoxylin and eosin stain, but show no fluorescence with Congo red stain. Hemosiderin (choice C) causes yellow brown deposits; melanin (choice D) causes brown-black deposits; and uric acid (choice E) causes yellow crystalline deposits. Question 5 of 6

Which of the following features of proteins is most likely responsible for the bright green appearance of the Congo red-stained materiaI? / A. Beta pleated sheet configuration / B. Calcium binding / C. Iron containing heme moiety / D. Multiple alpha helices / E. Presence of multiple subunits Explanation - Q: 3.5

Close

The correct answer A. It was originally assumed by biochemists that amyloid was always composed of the same material. It came as something of a shock when antibody techniques were developed that demonstrated that the antigenicity of amyloid in different clinical settings varied markedly. The common feature these proteins shared that accounted for both the affinity for Congo red and their characteristic regular fibrillar structure on electron microscopy turned out to be that the proteins all have a beta pleated sheet tertiary (secondary according to some biochemical purists) configuration, best demonstrated by X-ray diffraction. Selective or non-selective binding to calcium (choice B) is common in proteins. Heme moieties containing iron (choice C) are a part of myoglobin and hemoglobin. Alpha helices (choice D) are a common secondary structure in proteins, but do not contribute to the protein forming amyloid. The presence of multiple subunits (choice E) is also common in proteins, but does not contribute to a protein forming amyloid. Question 6 of 6

Which of the following would most likely be found in the Congo red-stained extracellular deposits with the bright green appearance under polarized light? / A. Amyloid AA / B. Beta-2-microglobulin / C. Beta protein precursor / D. Immunoglobulin light chains / E. Transthyretin Explanation - Q: 3.6

Close

The correct answer is D. Amyloidosis occurs in a large variety of forms. Primary amyloidosis is one of the more common forms of systemic amyloidosis, and can affect a variety of organs, including the heart, kidney,

peripheral nerve, gastrointestinal tract, and respiratory tract. In primary amyloidosis, the amyloid is composed of immunoglobulin light chains, and the disease is now interpreted as a plasma cell disorder closely related to multiple myeloma. This interpretation is clinically significant, as it has led to modern treatments of primary amyloidosis (which formerly had a dismal prognosis) with the chemotherapies designed for multiple myeloma. The treatments are affective only if the disease is recognized and passed to the appropriate specialists as early as possible in the clinical course. Amyloid AA (choice A) is seen in inflammation-associated amyloidosis and familial Mediterranean fever. Beta-2-microglobulin (choice B) comprises the amyloid of dialysisassociated amyloidosis. Beta protein precursor (choice C) comprises the amyloid seen in the brains of patients with Alzheimer's disease and Down syndrome. Transthyretin (choice E) comprises the amyloid seen in familial amyloidosis and senile cardiac amyloidosis. A 40-year-old man presents to the emergency department complaining of severe shortness of breath. The breathlessness has been worsening over the past few years, and the patient reports growing tachypneic with mild exertion, and sometimes even at night. On examination, he has generalized edema, jugular venous distention, and hepatic distention. Cardiac examination shows a right ventricular heave, a right-sided S3, and S4 with a pulmonary ejection click. A chest x-ray film shows cardiomegaly and widening of the hilar vessels, including the pulmonary arteries. An electrocardiogram shows talI, peaked P waves in leads lI, III, and aVF, right axis deviation, and right ventricular hypertrophy. Question 1 of 5

Which of the following is the most likely diagnosis? / A. Cor pulmonale / B. Hypertrophic cardiomyopathy / C. Left ventricular failure / D. Myocardial infarction / E. Pulmonary embolus (acute) Explanation - Q: 4.1

Close

The correct answer is A. This patient has cor pulmonale, which is defined as enlargement of the right ventricle secondary to diseases of the lung, thorax, or pulmonary circulation. In this case, it is chronic, given the duration of the patient's symptoms and the presence of many clinical sequelae of the condition: edema, jugular venous distention, hepatic distention, and right

ventricular heave. The electrocardiogram also supports the diagnosis of enlargement of the right ventricle showing right axis deviation due to the increase in the mass of the right heart. Evidence of right atrial enlargement is also present, i.e., the tall peaked P waves in leads II, III, and aVF (P pulmonale). Hypertrophic cardiomyopathy (choice B) is an anomaly in which the myocardium hypertrophies. The fibers are erratic and conduction abnormalities and outflow obstruction may result. Typically, this disorder presents in the second decade of life, and will manifest as dysrhythmia and/or shortness of breath. In addition, a right axis deviation would be inconsistent with this cardiomyopathy because the left ventricle hypertrophies, as well as the right. Thus, this diagnosis is unlikely. Left ventricular failure (choice C) often accompanies right ventricular failure, but in this case, the right-sided symptoms, such as systemic edema, jugular venous distention, and hepatic congestion, are more pronounced. Left-sided failure shows engorgement of the entire pulmonary tree in conjunction with pulmonary edema. Myocardial infarction (choice D) is unlikely. The ECG findings are not consistent with the pattern typically seen in MI. In addition, this patient does not suffer from the symptoms of myocardial infarction, such as chest pain, pressure, jaw numbness, and diaphoresis. Pulmonary embolus (choice E) may cause acute right heart strain and failure, but this patient has a chronic condition. Chronic emboli may produce increased resistance in the pulmonary tree and a picture similar to this.

Question 2 of 5

Pulmonary hypertension is suspected in the patient, and a Swan-Ganz catheter is placed. Which of the following denotes the correct anatomic sequence of vessels that would be traversed by the catheter if it was introduced into the left subclavian vein? / A. Left subclavian vein, Ieft brachiocephalic vein, superior vena cava, right atrium, right ventricle, pulmonary artery / B. Left subclavian vein, Ieft common carotid, superior vena cava, right atrium, right ventricle, pulmonary artery / C. Left subclavian vein, Ieft jugular vein, Ieft atrium, Ieft ventricle, aorta / D. Left subclavian vein, Ieft jugular vein, superior vena cava, right atrium, right ventricle, pulmonary artery / E. Left subclavian vein, superior vena cava, right atrium, right ventricle, pulmonary artery

Explanation - Q: 4.2

Close

The correct answer is A. The correct sequence for a catheter inserted into the left subclavian vein is as follows: left subclavian vein, left brachiocephalic vein, superior vena cava, right atrium, right ventricle, pulmonary artery. With this catheter in place, a variety of cardiac parameters can be measured, including pressures in the pulmonary artery. Thus, this catheter can aid in establishing the diagnosis of pulmonary hypertension.

Calcium channel blockers can be used in this setting to decrease pulmonary vascular resistance. Which of the following is the calcium channel blocker that will have the most predominant effect on vascular smooth muscle? / A. Diltiazem / B. Hydrochlorothiazide / C. Nifedipine / D. Pseudoephedrine / E. Verapamil Explanation - Q: 4.3

Close

The correct answer is C. The calcium channel blockers vary in the propensity to affect vascular smooth muscle versus their effect on cardiac muscle. Thus, in this case, it is important to select an agent that has maximum ability to relax the smooth muscle in the pulmonary vessels. The effect on smooth muscle is as follows: nifedipine>diltiazem (choice A) >verapamil (choice E). The effect on cardiac muscle is as follows: verapamil>diltiazem>nifedipine. Thus nifedipine is the agent of choice. Hydrochlorothiazide (choice B) is a diuretic, and thus would have no effect on the vascular smooth muscle. Pseudoephedrine (choice D) is an alpha agonist, and therefore would cause vasoconstriction. Question 4 of 5

Which of the following physiologic stimuli will result in decreased pulmonary vascular resistance? / A. Decreased cardiac output / B. Increased cardiac output / C. Low O2 tension / D. Lung volumes near residual volume (RV) / E. Lung volumes near total lung capacity (TLC)

Explanation - Q: 4.4

Close

The correct answer is B. A unique feature of the pulmonary circulation is that it maintains itself as a low-pressure system. Many of the mechanisms that control pulmonary vasculature differ from those of the systemic circulation. One of these features is that pulmonary vasculature resistance is decreased in response to increased cardiac output. This is accomplished through distention of open capillaries and the recruitment of collapsed capillaries. Thus, the resistance in the pulmonary tree decreases in response to increased right ventricular output. In the pathologic state of pulmonary hypertension, in which the resistance is elevated and the ventricle fails, this decreased cardiac output (choice A) may compound the problem and trigger increased resistance in spite of the primary elevation. Low O2 tension (choice C) in the pulmonary vessels initiates vasoconstriction. In the systemic circulation, low O2 tension initiates vasodilation. Lung volume also affects pulmonary vascular resistance. The curve of lung volume versus pulmonary vascular resistance is U-shaped. This effect is due to the fact alveolar and extra-alveolar vessels act as resistors in series (additive), and these vessels have little intrinsic support. Thus, resistance in these vessels is affected by pleural pressures. At low lung volumes (choice D), the alveolar vessels are open, but extra-alveolar vessels are compressed. At high lung volumes (choice E), the alveolar vessels are compressed by distended alveoli, but the extra-alveolar vessels become distended due to the increase in transmural pressure. Thus a U-shaped curve describes this relationship. Question 5 of 5

Some of the examination findings indicate hepatic congestion. Which of the following terms is commonly used to identify the macroscopic pattern of red, depressed hepatic nodules with pale periphery that accompanies the chronic hepatic congestion seen in this condition? / A. Centrilobular hemorrhage / B. Cirrhosis / C. Fatty change / D. Nutmeg liver / E. Piecemeal necrosis Explanation - Q: 4.5

Close

The correct answer is D. Chronic passive congestion of the liver leads to a macroscopic pattern known as nutmeg liver. This is due to the congestion of blood in the centrilobular region (dark) with hypoxia and fatty change in the

more peripheral hepatocytes. When viewed macroscopically, this pattern resembles that seen in a cross section of a nutmeg, hence the name. In this condition, centrilobular hemorrhage (choice A) usually only occurs in severe acute ischemia. This patient has a chronic condition, and thus most likely will have nutmeg liver instead. Cirrhosis (choice B) of the liver may result from chronic damage caused by chronic congestion. It however produces a scarred, whitish, shrunken liver, and not the pattern seen here. Fatty liver (choice C) would produce a large, smooth yellow liver and would not resemble the pattern seen here. Piecemeal necrosis (choice E) is a microscopic finding of scattered hepatocellular necrosis. This diagnosis cannot be made macroscopically.

A 45-year-old man presents to the emergency department complaining of chest pain, which began twenty minutes before while he was filling up his car with gasoline. He describes the pain as substernaI, intense, dulI, and squeezing. It does not change with respiration. He also complains that he is nauseated. He has never experienced anything like this before. His temperature is 37.5 C (99.5 F), blood pressure is 124/76 mm Hg, pulse is 80/min, respiratory rate is 22/min, and oxygen saturation is 95% on room air. On physical examination, he is diaphoretic. His lungs are clear, his heart rate is regular, and he has a normaI S1 and S2 without murmur, rub, or gallop. The examiner estimates that his jugular venous pressure is elevated to the angle of the jaw. His abdomen is nontender, with normal bowel sounds. An electrocardiogram is performed, which reveals sinus rhythm, normal axis, normal intervals, and ST elevation in leads lI, III, and aVF. A chest xray film reveals no apparent cardiac or pulmonary abnormalities. Question 1 of 7

Which of the following is the most likely diagnosis? / A. Acute myocardial infarction / B. Aortic dissection / C. Gastroesophageal reflux / D. Pericarditis / E. Pulmonary embolus Explanation - Q: 1.1

Close

The correct answer is A. The differential diagnosis of chest pain is broad and includes all the answers on this list, all of which can present, as in this patient, with a relatively normal examination of the heart and lungs. Acute myocardial infarction is the only one of these findings that is associated with ST segment elevation in an anatomical distribution (in this case, the "inferior" leads). His elevated jugular venous pressure is also a clue to abnormal cardiac function; however, this might be present in severe constrictive pericarditis or pulmonary embolism as well. Aortic dissection (choice B) often presents with chest pain or pain radiating to the back. It is not, however, typically associated with ST segment changes on the EKG, unless the dissection extends proximally into the ostia of the coronary arteries, obstructing flow, and resulting in secondary acute myocardial infarction, in which case a patient could present like this. However, this presentation would be an uncommon presentation of a relatively uncommon disease. The risk for aortic dissection is increased with long-standing essential hypertension, other peripheral vascular disease, hyperlipidemia, and advanced age, as well as connective tissue disorders

such as Marfan syndrome or Ehlers-Danlos syndrome. Patients with gastroesophageal reflux (choice C) may complain of intense substernal chest pain that is difficult to distinguish from the pain of myocardial infarction. However, the ST elevations on the EKG suggest transmural ischemia of the myocardium and do not occur with isolated gastroesophageal reflux. Do not let relatively normal vital signs and gastrointestinal symptoms such as nausea fool you! Pericarditis (choice D), an inflammatory disease of the pericardium, presents with chest pain and is often associated with ST elevation on the EKG, as well as PR interval depression. However, the ST elevation usually involves multiple leads of the EKG, and is not in a strictly anatomic distribution. Pericarditis is often associated with a friction rub. Pulmonary thromboembolism (choice E) can also present with chest pain. A patient who has had a large PE, however, typically will not have normal vital signs, and is likely to have tachycardia, tachypnea, and possibly hypoxemia. Hemodynamically significant pulmonary embolism can present with elevated JVP due to right heart strain. The patient might have ST depression in the inferior leads, but will not have ST elevation. The pain associated with pulmonary embolus is generally pleuritic in nature, that is, lateralizing, and changing with inspiration. Risk factors for pulmonary embolism include hypercoagulable states, immobilization, and vascular injury (Virchow's triad). Question 2 of 7

What is the pathophysiologic process most likely to be responsible for this patient's presentation? / A. Atherosclerotic plaque rupture resulting in thrombus formation / B. Buildup of atherosclerotic stenosis to produce high-grade obstruction of the artery / C. Dissection of the vessel / D. Embolization of blood clot, air, or foreign material / E. Myocardial hypertrophy resulting in vessel narrowing Explanation - Q: 1.2

Close

The correct answer is A. Acute coronary syndromes are thought to be the result of rupture of a pre-existing atherosclerotic plaque, often one that is not producing high-grade stenosis. When the endothelial surface covering the lipid core of an unstable plaque ruptures, a platelet plug forms and the clotting cascade is activated, rapidly propagating thrombus formation and suddenly occluding the vessel. This results in transmural ischemia, which becomes infarction, should the clot fail to recanalize quickly. Atherosclerotic stenosis (choice B) is an important pathologic component of

coronary artery disease. However, high-grade coronary stenoses most frequently cause stable angina, that is, chest pain and myocardial ischemia induced when an increase in myocardial oxygen demand exceeds the fixed supply that can be obtained through a severely stenotic vessel. As vessels progressively narrow over time, they produce ischemia, but not sudden infarction. Often distal to a narrowed vessel, collaterals will form from less diseased vessels, compensating for the reduced flow. Coronary artery dissection (choice C) is a rare phenomenon that can produce transmural ischemia and infarction, but most commonly occurs in the setting of instrumentation of the vessel. Embolization (choice D) is a relatively rare phenomenon in the coronary circulation under normal circumstances. Foreign material can only enter the left side of the heart via right-to-left intracardiac shunt, pulmonary vein interruption, or surgical opening of the left heart. Thrombus that forms in the left atrium (usually the result of low-flow states such as mitral stenosis or atrial fibrillation) can also embolize; patients with atrial fibrillation are anticoagulated to reduce the risk of cerebral embolization of left atrial clot. Myocardial hypertrophy (choice E) can restrict subendocardial oxygen supply by creating high capillary pressure relative to arteriolar pressure. This does not occur in large coronary vessels, but can produce subendocardial ischemia in disease states that are associated with hypertrophic myocardium, such as aortic stenosis, long-standing essential hypertension, and idiopathic hypertrophic subaortic stenosis (IHSS). Question 3 of 7

Which of the following vessels is most likely to be diseased in this patient? / A. Coronary sinus / B. Left anterior descending coronary artery / C. Left circumflex coronary artery / D. Left main coronary artery / E. Right coronary artery Explanation - Q: 1.3

Close

The correct answer is E. Acute myocardial infarction is usually due to obstruction of one coronary vessel. The right coronary artery exits the right sinus of Valsalva of the aorta and gives off branches to the right ventricle, the SA nodal artery (in 70% of patients), the AV nodal artery, and, in the 85% of patients whose circulations are said to be "right-dominant," the posterior descending artery, which supplies the inferior wall of the RV and the LV as well as the posterior 1/3 of the interventricular septum. This patient has EKG signs of an inferior myocardial infarction, with ST elevation in the inferior leads, II, III, and aVF. He also has a physical sign of right ventricular

dysfunction (elevated jugular venous pressure). The coronary sinus (choice A) is the principal vein draining the left ventricle and runs alongside the circumflex artery in the posterior AV groove. It is not a common site for atherosclerotic disease or for obstruction. The left anterior descending artery (choice B) supplies the anterior and anteroseptal portions of the left ventricle. Obstruction would produce ST elevation in the anterior (V2-V6) and occasionally the lateral (I, aVF) leads of the EKG, with possible "reciprocal" ST depression in the inferior leads. The left circumflex artery (choice C) supplies the lateral wall of the left ventricle. 85% of patients have a "right-dominant" coronary anatomy. That is, the right coronary artery gives off the posterior descending artery (PDA). In the "left-dominant" remaining 15%, the PDA comes off the circumflex. Isolated inferior EKG lead changes are, therefore, most likely to be due to RCA obstruction; circumflex obstruction typically produces EKG lead changes in the lateral (I, aVL, V5, V6) leads. The left main coronary artery (choice D) exits the aorta at the left sinus of Valsalva and divides into the left anterior descending and left circumflex arteries. Obstruction of the left main makes the entire left ventricle ischemic, often resulting in cardiogenic shock. This would produce ST segment elevation in leads I, aVL, and V2-V6. Question 4 of 7

This patient is given aspirin in the emergency department. Decreased production of which of the following mediators is responsible for the beneficial effects of aspirin in this disorder? / A. cAMP / B. PIatelet glycoprotein lIB/IIIA / C. Prostacyclin / D. Thromboxane A2 / E. Ubiquinone (coenzyme Q) Explanation - Q: 1.4

Close

The correct answer is D. Aspirin irreversibly inhibits the enzyme cyclooxygenase, which produces all the prostaglandin mediators from arachidonic acid. Cyclooxygenase in platelets produces thromboxane A2, which is a potent promoter of platelet aggregation and vasoconstriction. By blocking this, aspirin irreversibly inhibits platelet function, preventing aggregation at the site of plaque rupture. Platelets, having no nuclei, are unable to synthesize more cyclooxygenase, and therefore thromboxane

production is inhibited for the life of the platelet, approximately 10 days. Cyclic AMP (choice A) is an intracellular small molecule responsible for multiple signal transduction pathways. In cardiac myocytes, it activates protein kinases responsible for the phosphorylation of calcium channels, promoting entry of calcium into the cell. cAMP is broken down by phosphodiesterase, the inhibition of which is responsible for the beneficial effects of inotropic phosphodiesterase inhibitors such as milrinone. Platelet surface glycoprotein IIb/IIIA (choice B) binds fibrinogen and von Willebrand factor, promoting aggregation and clot formation. It is inhibited by drugs like eptifibatide and tirofiban, which are used in acute coronary syndromes to further inhibit platelet aggregation and thrombus formation. Prostacyclin (choice C) is produced by cyclooxygenase in endothelial cells, where it promotes vasodilation and inhibits platelet aggregation. Prostacyclin should therefore be a beneficial mediator. Aspirin inhibits prostacyclin formation, however, endothelial cells can produce more cyclooxygenase and are able to continue to synthesize adequate amounts of prostacyclin. Ubiquinone (choice E), or coenzyme Q, is a naturally occurring coenzyme that plays a vital role in the mitochondrial electron transport chain. Studies have shown an association between decreased levels of coenzyme Q and heart disease, so inhibition of ubiquinone production would not likely be beneficial. Question 5 of 7

EIevation of which of the following serum proteins is the most specific biochemical marker for this patient's condition? / A. AIanine aminotransferase / B. Creatine phosphokinase / C. Lactate dehydrogenase / D. Transferrin / E. Troponin Explanation - Q: 1.5

Close

The correct answer is E. Troponins (in isoforms troponin C, troponin I, and troponin T) are required for actin-myosin cross linking in cardiac muscle. Small elevations in serum troponin levels are currently the most sensitive clinical serum markers for myocardial injury, elevating within 3-12 hours of infarction. Levels remain elevated for 5-14 days. Alanine aminotransferase (choice A) occurs in both cardiac muscle and in the liver, and has been used in the past as a marker of cardiac injury.

However, currently, its more common clinical use is as a marker of hepatocyte injury. Its time course of elevation in MI is intermediate between CPK and LDH (see below). Creatine phosphokinase (choice B) has been the mainstay of diagnosis of myocardial injury for many years. CPK has several isoforms, of which the MB isoform is specific for cardiac muscle. The fraction of the total CPK that is the MB isoform has been used to differentiate myocardial injury from other injury processes elevating CPK. CPK is also elevated with skeletal muscle and with brain injury, but neither of these tissues contains significant amounts of MB isoform. In MI, CPK levels usually rise within 8 to 24 hours and return to normal after 48 to 72 hours. Lactate dehydrogenase (choice C) is, like alanine aminotransferase, an enzyme that is released both with cardiac injury and with hepatocellular injury. It can also be elevated in hemolysis and with some neoplasms. In MI, it generally rises within 12 hours and peaks after 24-48 hours, remaining elevated for 10-14 days. These properties made LDH, prior to the introduction of troponin assays, the test used to detect MI occurring more than a day previously. Transferrin (choice D) is a plasma protein responsible for the uptake of iron after absorption in the small intestine, and is responsible for iron-binding capacity in the blood. It is measured (usually as "total iron-binding capacity") in the differential diagnosis of the anemias. Question 6 of 7

Three days after hospital admission, the patient suddenly develops shortness of breath and becomes hypotensive. His heart rate is 100/min, with a normaI PR and QRS intervaI. His blood pressure is 75/50 mm Hg. His respiratory rate is 38/min and his oxygen saturation on 2 Iiters via nasal cannula drops to 60%. A chest x-ray reveals bilateral fluffy infiltrates in the lung fields. Which of the following complications of his condition has most likely occurred? / A. Dilation of the left ventricle / B. Dressler syndrome / C. Rupture of the left ventricular free wall / D. Rupture of the posteromedial papillary muscle / E. Ventricular tachycardia Explanation - Q: 1.6

Close

The correct answer is D. This patient is suddenly in cardiogenic shock with severe pulmonary edema. This could be the result of arrhythmia, cardiac tamponade, or left ventricular valvular dysfunction. He had an inferior MI, which is most likely due to thrombosis of the right coronary artery. The posteromedial papillary muscle is supplied by the RCA alone in most patients

and is prone to rupture in inferoposterior MI. Rupture leads to acute and severe mitral valve dysfunction, resulting in pulmonary edema and poor forward ejection. Dilation of the left ventricle (choice A) typically occurs after extensive damage occurs, which would have appeared on this patient's acute EKG as ST elevation in the anterior leads. Dilation can result in mitral regurgitation, but typically of an insidious onset. Dressler syndrome (choice B) is a late complication of MI that may occur weeks to months later, characterized by symptoms of pericarditis including pleuritic chest pain, fever, friction rub, and elevated white blood cell count. Patients can also develop early postinfarction pericarditis in the days to weeks following MI with friction rub and pericardial effusion. This is seldom associated with cardiac tamponade. Rupture of the LV free wall (choice C) is a complication more likely to occur with more extensive damage to the LV than is produced in an inferior MI such as this patient had. However, LV free wall rupture would produce cardiac tamponade, which could produce this clinical picture. Ventricular tachycardia (VT) (choice E) is a complication of MI (though risk is highest early in the course of infarction) and could also produce this clinical picture. However, it is excluded by the EKG, which reveals an atrialventricular conducted rhythm (VT displays no P waves) and a narrowcomplex QRS (VT typically has a QRS much greater than 0.12 s). Question 7 of 7

The patient is taken emergently to the operating room. During surgery, a sample of affected myocardial tissue is sent to the pathology Iaboratory for examination. Which of the following would be the likely pathologic finding(s)? / A. Acellular fibrosis / B. Monocyte infiltration, absent nuclei and striations / C. Myocyte disarray / D. Myocyte edema, hemorrhage, and dense neutrophil infiltration / E. Wavy myofibers with eosinophilic contraction bands Explanation - Q: 1.7

Close

The correct answer is B. Three days after infarction, coagulation necrosis is complete, with complete loss of cellular structure (hence the high risk of mechanical complications such as rupture) and infiltration of monocytes to phagocytize debris. Acellular fibrosis (choice A) replaces necrosis after many weeks when debris is removed and fibroblasts have invaded the dead tissue and replaced

it with collagen. Myocyte disarray (choice C) is associated with hypertrophic subaortic stenosis (IHSS), rather than myocardial infarction. Myocyte edema, with hemorrhage and neutrophil infiltration (choice D) occurs within 4-12 hours after infarction. Wavy myofibers and contraction bands (choice E) are the first light microscopic pathologic changes to occur after MI, and appear within 1-3 hours after infarction.

A 52-year-old man presents to the emergency department because of severe chest pain. The excruciating pain began abruptly, 30 minutes previously, and feels to the patient as if something were "ripping." When asked to point to where the pain is worst, the patient points to the precordium. The man additionally reports that the pain seems to be changing in position slowly. Question 1 of 6

Which of the following is most likely causing the patient's severe pain? / A. Aortic dissection / B. Atherosclerotic aortic aneurysm / C. Esophageal reflux / D. Myocardial infarction / E. Peptic ulcer Explanation - Q: 2.1

Close

The correct answer is A. This patient has a classic presentation of aortic dissection. Any time a patient in excruciating chest pain describes the pain as "tearing" or "ripping," you should strongly consider the diagnosis of aortic dissection. The pain may move with time as the dissection progresses. Aortic dissection is a highly lethal condition that may lead to aortic rupture, most often into the pericardial cavity or left pleural space. The two most common sites of origin of the dissection are in the proximal aorta within 5 cm of the aortic valve and in the descending thoracic aorta just distal to the origin of the left subclavian artery. CT scan with contrast is often used to confirm the diagnosis suspected clinically. Therapy is promptly initiated with medications that lower the blood pressure to try to prevent extension of the dissection. Surgery is usually then performed in patients in which the dissection begins in the proximal aorta near the aortic root; medical therapy alone can sometimes be used for those with distal aortic dissection that has not compromised blood flow to limbs or organs. Atherosclerotic aortic aneurysm (choice B) more commonly involves the

abdominal aorta, and, when rupturing, may produce excruciating pain that is usually referred to the lower abdomen and back. The pain of esophageal reflux (choice C) is rarely excruciating, and usually does not produce a ripping or tearing sensation. Myocardial infarction (choice D) can produce severe precordial chest pain, but the pain usually does not move with time and does not have a tearing or ripping character. Peptic ulcer pain (choice E) may be severe and referred to the chest, but patients are more likely to use terms like "burning" than ripping or tearing, and the pain does not slowly change position. Question 2 of 6

Extension of this patient's disease process would be most likely to produce which of the following? / A. Aortic insufficiency / B. Aortic stenosis / C. Mitral insufficiency / D. Mitral stenosis / E. Tricuspid stenosis Explanation - Q: 2.2

Close

The correct answer is A. Dissecting aneurysms tend to start near the root of the aorta, and aortic insufficiency is a common complication. This can be very helpful in the initial evaluation of the patient, since up to 2/3 of the patients with proximal aortic dissection demonstrate, on auscultation, the characteristic murmur of aortic insufficiency, which is a pandiastolic decrescendo murmur that is loudest over the sternum and left lower sternal border. Aortic stenosis (choice B) usually does not occur. Involvement of the mitral (choices C and D) and tricuspid valves (choice E) would be very rare, and probably only seen if the aortic dissection interrupted the orifices of the coronary arteries and induced a secondary myocardial infarction.

Question 3 of 6

If enzyme chemistries were sent, which would be the most likely results? / A. Decreased AST, elevated CK, decreased LDH / B. EIevated AST, elevated CK, normal to decreased LDH

/ C. EIevated AST, normaI CK, normaI LDH / D. NormaI AST, elevated CK, elevated LDH / E. NormaI AST, normaI CK, normal to elevated LDH

Explanation - Q: 2.3

Close

The correct answer is E. Unless aortic dissection secondarily causes a myocardial infarction secondary to occlusion of the coronary arteries, aspartate aminotransferase (AST) and creatine kinase (CK) levels should be normal. Lactic dehydrogenase (LDH) levels may be normal, or elevated if some hemolysis is occurring within the area of dissection. In general, AST can be elevated (choices B and C) in a variety of cardiac diseases (e.g., myocardial infarction, heart failure, myocarditis, pericarditis), muscle damage (e.g., myositis, muscular dystrophy, trauma), and damage to liver, pancreas, kidney, or brain. AST is decreased (choice A) in pyridoxine (vitamin B6) deficiency and in the terminal stages of liver disease. In general, CK can be elevated (choices A, B, and D) in disease or damage involving heart, muscle, or brain. Decreased CK has no medical significance. Lactic dehydrogenase (LDH) can be elevated (choice D) in myocardial infarction, pulmonary infarct, hemolytic and pernicious anemia, hematologic malignancies, and disease of liver, kidney, or brain. Decreases in LDH (choices A and B) are not medically significant. Question 4 of 6

If surgery is necessary to repair this problem, the surgeon will be required to understand the anatomic relationship of the aorta to the surrounding structures. Which of the following most accurately describes the descending portion of the thoracic aorta? / A. It descends on the right side of the thoracic vertebrae / B. It flattens the posterior aspect of the trachea / C. It is to the left of the esophagus at the hiatus / D. It is to the left of the thoracic duct at the T10 Ievel / E. It is to the right of the inferior vena cava Explanation - Q: 2.4

Close

The correct answer is D. The thoracic duct is the main lymphatic duct and it lies on the bodies of the inferior seven thoracic vertebrae. It conveys most of the lymph of the body to the venous system. It passes superiorly from the cisterna chyli (the expanded inferior end of the thoracic duct) through the aortic hiatus in the diaphragm. The thoracic duct ascends in the posterior mediastinum, on the right of the thoracic aorta and to the left of the azygos vein. At the level of T4, T5, or T6, the thoracic duct crosses to the left, posterior to the esophagus and ascends to the superior mediastinum. The thoracic duct empties into the venous system near the union of the left

internal jugular and subclavian veins. As a continuation of the aortic arch, the descending aorta begins on the left side of the inferior border of the body of the T4 vertebra and descends in the posterior mediastinum on the left sides of T5 to T12 (choice A). The trachea travels in the superior mediastinum and does not have direct contact with the descending thoracic aorta. The trachea is kept patent by a series of C-shaped tracheal cartilages. The posterior aspect is flat where it is applied to the esophagus, not the aorta (choice B). At the level of the esophageal hiatus (choice C), the esophagus lies anterior to the descending thoracic aorta. The inferior vena cava (choice E) is located to the right of the abdominal aorta, not the thoracic aorta. The IVC returns blood from the lower limbs, most of the abdominal wall, and the abdominopelvic viscera. This vessel begins anterior to L5 vertebra by union of the common iliac veins. It then ascends on the right psoas major muscle to the right of the median plane and aorta. It passes through the vena caval foramen in the diaphragm at the level of T8 to enter the right atrium. Question 5 of 6

Which of the following would be most likely to be seen on pathological examination of a specimen removed from this patient at surgery? / A. Bacterial vegetations / B. Cystic medial degeneration / C. Multiple small granulomas / D. Parasitic organisms / E. Polyarteritis nodosa Explanation - Q: 2.5

Close

The correct answer is B. Cystic medial degeneration is a disruption and fragmentation of the elastic tissue in aortic media, with formation of areas devoid of elastin. These changes weaken the aortic wall, predispose for dissection, and are seen in the majority of cases of aortic dissection. Bacterial vegetations (choice A) are a feature of endocarditis. Multiple small granulomas (choice C) can be seen in temporal arteritis and Takayasu arteritis. Parasitic organisms (choice D) do not usually affect the aorta; the organisms of trichinosis and Chagas disease can affect the heart.

Polyarteritis nodosa (choice E) is a focal inflammation that usually involves smaller blood vessels than the aorta. Question 6 of 6

Which of the following conditions has been associated with this patient's disease? / A. Cushing syndrome / B. Dandy-Walker syndrome / C. Kawasaki syndrome / D. Marfan syndrome / E. Tourette syndrome Explanation - Q: 2.6

Close

The correct answer is D. Marfan syndrome is an autosomal dominant connective tissue disease characterized by skeletal changes (e.g., tall stature, long limbs, long fingers, lax joints), a tendency to develop dislocations of the lens of the eye, and a tendency to develop aortic dissection secondary to prominent cystic medial degeneration in the aortic media. A similar condition, Ehlers-Danlos syndrome, also predisposes for dissecting aneurysm. Other predisposing factors include congenital cardiovascular abnormalities (e.g., coarctation of the aorta, patent ductus arteriosus, bicuspid aortic valve) that increase the turbulence of blood flow in the aorta, atherosclerosis, and trauma (including iatrogenic trauma during arterial catheterization and cardiovascular surgical procedures). The other conditions listed in the choices are unrelated to aortic dissection. Cushing syndrome (choice A) is a characteristic pattern of physical changes (truncal obesity, moon face, buffalo hump), biochemical/hormonal changes (hypertension, altered carbohydrate and protein metabolism, amenorrhea), and sometimes psychiatric disturbances that are seen in patients with increased levels of adrenocortical hormones. Dandy-Walker syndrome (choice B) is a congenital abnormality of the cerebellum and fourth ventricle. Kawasaki syndrome (choice C) is a febrile childhood disease that predisposes for the formation of tiny aneurysms of the coronary arteries. Tourette syndrome (choice E) is a motor and vocal tic disorder.

A 35-year-old man with no significant past medical history presents to clinic with a six week history of worsening chest discomfort and pain.

He describes the pain as a substernal burning sensation that occasionally wakes him up at night and is often worse after he eats. He sometimes notices a sour taste in his mouth when he wakes up in the morning. He has no dysphagia or odynophagia. The pain is unrelated to exertion, and he jogs 3 miles every other day without difficulty or chest pain. His vital signs and physical examination are normaI. Question 1 of 6

Which of the following is the most likely diagnosis? / A. Acute viral pericarditis / B. Aortic dissection / C. Candida esophagitis / D. Gastroesophageal reflux disease (GERD) / E. Stable angina Explanation - Q: 3.1

Close

The correct answer is D. A patient who presents with chest discomfort that is burning in nature, and worsened after eating without symptoms of dysphagia or odynophagia, most likely has gastroesophageal reflux disease (GERD). GERD occurs when there is reflux of gastric contents into the esophagus. This may occur with or without inflammation. It is often caused by inappropriate relaxation of the lower esophageal sphincter. Certain foods such as peppermint, caffeine, and high-fat and spicy foods are often associated with GERD. Acute viral pericarditis (choice A) would present with more severe and sudden onset of chest pain that is relieved with leaning forward or sitting up. Acute viral pericarditis is often associated with a prodrome and usually presents with a fever. Occasionally, a pericardial friction rub can be heard on exam. Aortic dissection (choice B) would also present as sudden onset of severe chest pain, which often radiates to the back. Patients can have hypotension, depending on the severity of the dissection. Patients can also have unequal pulses in their extremities if the dissection affects one of the major arteries branching off the aortic arch. Candida esophagitis (choice C) would present with dysphagia and odynophagia. Patients also have oral thrush, and generally are immunocompromised. These patients usually have a fever. Stable angina (choice E) should present with typical chest pain that is worsened after exertion. The fact that this patient can jog 3 miles without difficulty goes against stable angina. Furthermore, he is young and does not have any risk factors for cardiac disease such as hypertension, diabetes, or hypercholesterolemia.

Question 2 of 6

Which of the following tests would be most likely to confirm the probable diagnosis? / A. 24-hour ambulatory esophageal luminal pH monitoring / B. Cardiac angiogram / C. Chest radiograph / D. Exercise treadmill test / E. Serologic blood tests for H. Pylori infection Explanation - Q: 3.2

Close

The correct answer is A. Twenty-four hour ambulatory esophageal luminal pH monitoring is one of the most sensitive tests for GERD. In most cases, the disease is diagnosed clinically by history, but pH monitoring would help confirm the diagnosis. Cardiac angiograms (choice B) are used to evaluate the coronary arteries for signs of blockage, to evaluate heart function, or to evaluate cardiac valve function. A chest radiograph (choice C) can be used to evaluate the structures in the thorax, but will not help confirm the diagnosis because GERD patients generally have normal chest radiographs. An exercise treadmill test (choice D) is used to evaluate patients who are believed to have underlying coronary heart disease or to rule out heart disease. Serologic blood testing for H. pylori infection (choice E) only documents the presence of a current infection or the history of an H. pylori infection. A past or present infection does not confirm a diagnosis, because GERD can occur in the setting with or without H. pylori. Furthermore, the role of H. pylori in GERD is still unclear. Question 3 of 6

The patient is treated with cimetidine, which completely relieves his symptoms. Which of the following is the mechanism of action of this medication? / A. Beta-1 adrenergic blockade / B. Histamine H2 receptor blockade / C. Inhibition of cell wall synthesis / D. Inhibition of cyclooxygenase / E. Smooth muscle relaxation Close Explanation - Q: 3.3 The correct answer is B. Cimetidine and other histamine H2 receptor blockers such as ranitidine block the action of histamine on H2 receptors,

resulting in a decrease in gastric acid production, thus decreasing the symptoms of GERD. Beta-1 adrenergic blockade (choice A) (e.g., atenolol, metoprolol) is used to lower blood pressure, which is not related to GERD. Inhibition of cell wall synthesis (choice C) (e.g., amoxicillin) is a mechanism that is used by many antibiotics. GERD can be associated with the presence of H. pylori, but the treatment of H. pylori with antibiotics in GERD patients remains controversial, and its benefit remains questionable. Inhibition of cyclooxygenase (choice D) (e.g., ibuprofen, naproxen) does not play a role in the treatment of GERD. GERD may or may not be associated with inflammation of the esophagus, but anti-inflammatory agents may actually worsen symptoms. Smooth muscle relaxation (choice E) (e.g., nitroglycerin) does play a role in the relief of esophageal spasm, but this patient does not complain of dysphagia or odynophagia. Question 4 of 6

The physician cautions the patient about cimetidine because of which of the following potential side effects? / A. CNS depression / B. Hypertensive crisis / C. Inhibition of hepatic metabolism / D. Masking symptoms of hypoglycemia / E. Ototoxicity Explanation - Q: 3.4

Close

The correct answer is C. Many drugs can lead to clinically significant drug interactions via inhibition of the hepatic drug-metabolizing enzymes, particularly the cytochrome P450 isozymes. This can lead to unwanted elevations of plasma drug levels. Cimetidine is a classic example of one of these drugs. Other examples include erythromycin, ketoconazole, sulfonamides, quinidine, and disulfiram. Benzodiazepines and barbiturates are examples of drugs that can cause CNS depression (choice A). MAO inhibitors prior to the ingestion of tyramine-containing foods can cause a hypertensive crisis (choice B). Beta blockers can mask the symptoms of hypoglycemia (choice D).

Aminoglycosides can produce ototoxicity (choice E). The risk of ototoxicity may be further increased if the patient is also taking loop diuretics. Question 5 of 6

The patient's symptoms are initially controlled on cimetidine. After 10 years, he develops refractory symptoms, and the physician places him on a proton pump inhibitor. Which of the following medications was most likely prescribed? / A. Lansoprazole / B. Loperamide / C. Metoclopramide / D. Ondansetron / E. Ranitidine Explanation - Q: 3.5

Close

The correct answer is A. Lansoprazole is a proton pump inhibitor and acts directly to inhibit the gastric parietal cell hydrogen-potassium ATPase. It can be used as the initial treatment for GERD, or for refractory cases. Loperamide (choice B) is an anti-diarrheal agent, which inhibits peristalsis. Using it in this setting may worsen the symptoms of GERD. Metoclopramide (choice C) stimulates upper gastrointestinal motility. It can be used in refractory cases of GERD, but it is not a proton pump inhibitor. Ondansetron (choice D) is an antiemetic and acts by selectively antagonizing serotonin 5-HT3 receptors. It is primarily used is severe cases of nausea, such as in patients receiving chemotherapy for cancer treatment. Ranitidine (choice E) is also an histamine H2 receptor blocker, like cimetidine. Some patients who do not respond to one histamine H2 receptor blocker, may respond to another, but ranitidine blocks the action of histamine on H2 receptors, resulting in a decrease in gastric acid production. It is not a proton pump inhibitor. Question 6 of 6

Histologic examination of the affected tissue shows Barrett's esophagus. This is most correctly described as which of the following? / A. Adenocarcinoma / B. Esophageal stricture / C. H.Pylori infection / D. Localized outpouching of the esophageal wall / E. Metaplasia of the squamous epithelium

Explanation - Q: 3.6

Close

The correct answer is E. Patients who have long-standing GERD are at risk for development of Barrett's esophagus, which is the replacement of the normal esophageal squamous epithelium with columnar epithelium (metaplasia). This is a premalignant lesion that needs to be monitored regularly for the development of adenocarcinoma. Adenocarcinoma (choice A) is a malignant lesion that can result from cellular metaplasia, but Barrett's esophagus is the premalignant lesion that occurs before the development of adenocarcinoma of the esophagus. Esophageal strictures (choice B) can occur in patients with long-standing GERD, but the presence of a stricture does not mean that there is cellular dysplasia, or Barrett's esophagus. H. pylori infection (choice C) can occur in the setting of GERD, but it is not synonymous with Barrett's esophagus. An esophageal diverticulum is a localized outpouching of the esophageal wall (choice D). This is unrelated to Barrett's esophagus.

A 45-year-old man presents with a 3-day history of persistent, severe chest pain. Prior to this, he had flu-Iike symptoms for 2 weeks, including fever, cough, myalgias, and arthralgias. His pain is worse when he takes a deep breath and is improved when he sits up. On physical examination, he is febrile, and his pulse is 110/min. His oxygen saturation is normaI, and his breath sounds are equal and clear to auscultation over all lung fields. There is a scratching and scraping, high-pitched sound on auscultation of the heart over the left third intercostal space, which is increased when the patient is sitting forward.

Question 1 of 4

Which of the following is the most likely diagnosis? / A. Acute pericarditis / B. Aortic dissection / C. Pneumonia / D. Pulmonary embolus / E. Tension pneumothorax

Explanation - Q: 4.1

Close

The correct answer is A. This patient has symptoms that are typical of inflammation of the pericardial sac. In addition, the sound that is heard over his heart is a pericardial friction rub. Acute pericarditis is often associated with viral syndromes, connective tissue diseases, renal failure, myocardial infarction, and tumor invasion of the pericardium. Aortic dissection (choice B) will also present with severe chest pain, but it is tearing in quality, and is not positional, nor pleuritic in nature. Pneumonia (choice C) may present similarly, but auscultation of the lung fields should reveal abnormal breath sounds, and a pericardial friction rub should not be heard. Pulmonary embolus (choice D) can present with chest pain that is worse with deep breaths. However, it is not positional in nature, and it is not associated with a pericardial friction rub. Depending on the size of the embolus, the oxygen saturation may be abnormal. Tension pneumothorax (choice E) is not associated with flu-like symptoms nor a pericardial friction rub. Furthermore, there should be an absence of breath sounds over the affected part of the lung. Question 2 of 4

Which of the following would help confirm the diagnosis? / A. Angiogram showing a clot in one of the coronary arteries / B. Chest radiograph showing multiple emphysematous bullae / C. CT scan of the chest showing a widened mediastinum / D. EIectrocardiogram showing diffuse ST elevation / E. Endoscopy revealing esophageal varices Explanation - Q: 4.2

Close

The correct answer is D. Acute pericarditis often presents with diffuse ST elevation on an electrocardiogram. An angiogram showing a clot in one of the coronary arteries (choice A) would be seen in a patient having an acute myocardial infarction. A chest radiograph showing multiple emphysematous bullae (choice B) would be seen in a chronic smoker, and would be a possible cause of spontaneous pneumothorax. CT scan of the chest showing a widened mediastinum (choice C) would be

seen in a patient with an acute aortic dissection. Endoscopy revealing esophageal varices (choice E) is seen in patients with chronic liver disease or portal hypertension. Question 3 of 4

Other than an antecedent viral syndrome, which of the following conditions can predispose a patient to this problem? / A. AIcohol abuse / B. Liver failure / C. Peptic ulcer disease / D. Recent total hip replacement / E. Renal failure Explanation - Q: 4.3

Close

The correct answer is E. Patients with renal failure or uremia can often present with a fibrinous or serofibrinous pericarditis. Alcohol abuse (choice A) and liver failure (choice B) do not necessarily predispose a patient to pericarditis, nor are they associated with pericarditis. Peptic ulcer disease (choice C) may cause epigastric pain that can be confused with chest pain, and patients may have recurrent bleeding, but this disorder is not associated with pericarditis. A recent total hip replacement (choice D) predisposes patients to the development of a deep venous thrombosis, which can embolize and cause a pulmonary embolus. This is not associated with pericarditis. Question 4 of 4

The patient is treated with a nonsteroidal anti-inflammatory agent. Which of the following was prescribed? / A. AIIopurinol / B. Gemfibrozil / C. Indomethacin / D. Labetalol / E. Methocarbamol Explanation - Q: 4.4

Close

The correct answer is C. Indomethacin or any of the other nonsteroidal anti-inflammatory agents can be used in the treatment of acute pericarditis. Allopurinol (choice A) is used for the treatment of gout. It inhibits xanthine

oxidase, which decreases the production of uric acid. Gemfibrozil (choice B) is a lipid lowering agent used in patients with hypercholesterolemia. Labetalol (choice D) is a beta blocker and is used in hypertensive emergencies. Methocarbamol (choice E) is a muscle relaxant, which helps to relieve pain associated with muscle spasms.

A 62-year-old white man is brought to the emergency department after experiencing twenty minutes of sudden onset, crushing, substernal chest pain radiating to his neck and left shoulder. The pain started when the patient was lifting a heavy trolley. He denies shortness of breath, back pain, or loss of consciousness. In the emergency department, he is given sublingual nitroglycerin tablets, which relieve the pain almost immediately. The patient has a history of hypertension, drinks beer several times a week, smokes one pack of cigarettes per day, and does not exercise. He is not on any medications. On physical examination, he is a diaphoretic patient with a temperature of 36.6 C (98 F), blood pressure of 150/80 mm Hg, pulse of 84/minute, and respirations of 16/minute. His heart and breath sounds are normaI, his abdomen is soft and non-tender, and no bruits were heard. A chest radiograph is normaI. An electrocardiogram confirms T-wave inversion in leads V2 and V3, with an otherwise normal sinus rhythm. The patient is started on atenoloI, a nitroglycerin paste patch, and aspirin. He is admitted into the hospitaI, where he remains pain free. Troponin levels taken every eight hours for the next 24 hours are all within normal limits. Question 1 of 5

Which of the following is the most likely diagnosis? / A. Angina / B. Gastroesophageal reflux disease / C. Myocardial infarction / D. Pulmonary embolism / E. Thoracic aortic aneurysm rupture Explanation - Q: 5.1

Close

The correct answer is A. The patient has four independent cardiac risk factors (male sex, smoker, hypertensive, inactivity). He presented with typical cardiac chest pain on exertion, accompanied by diaphoresis, which was relieved with nitroglycerin. This is typical of angina. T-wave inversion is also

consistent with ischemia. However, this is unlikely to be a myocardial infarction (choice C) because there were no raised ST segments on the electrocardiogram and his cardiac enzymes (e.g., troponin) were not elevated. He would probably require a cardiac stress test to see if the EKG changes could be reproduced in a controlled stressful environment (e.g., treadmill). On this visit, his symptoms are better described as being anginal with ischemic EKG findings on the anterior cardiac wall. Gastroesophageal reflux disease (choice B) would present with a longitudinal burning sensation in the epigastric area, usually after the consumption of large meals, caffeinated products, exercising, or lying flat after eating. It is treated with H2-blockers and proton pump inhibitors, along with eating smaller meals and the avoidance of caffeine. It is an important part of the differential diagnosis of chest pain. The patient has risks for thromboembolism, including smoking and inactivity, which could lead to pulmonary embolism. Pulmonary embolism (choice D) would be suggested if the patient had a sudden onset of shortness of breath and chest pain. In addition, on examination, one would look for tachycardia, tachypnea, and diaphoresis. The work-up includes D-dimers, a lower leg Doppler, prothrombin time (PT), partial thromboplastin time (PTT), and a ventilation/perfusion scan. Treatment would be anticoagulation with heparin, and then eventually with Coumadin. Rupture of a thoracic aortic aneurysm (choice E) would present as severe chest pain radiating to the back. The patient would typically be elderly. The blood pressure may drop and there will sometimes be a discrepancy in armto-arm blood pressure readings. Question 2 of 5

Which of the following vessels is most likely affected in this patient? / A. Circumflex coronary artery / B. Left anterior descending artery / C. Marginal artery / D. Posterior descending artery / E. Right coronary artery Explanation - Q: 5.2

Close

The correct answer is B. The electrocardiogram revealed inverted T-waves (suggestive of ischemia) in leads V2 and V3. These leads correspond mainly to the anterior wall of the left ventricle, where the ischemia is likely to have taken place. The left anterior descending artery is a terminal branch of the left coronary artery (the other branch being the circumflex artery). It passes posterior to the pulmonary artery, then anteriorly between this vessel and the left auricle to reach the anterior longitudinal sulcus. Since it supplies blood to the anterior walls of both ventricles, ischemia in this location would manifest

as T-wave inversion in leads V1 to V4. The circumflex artery (choice A) supplies the lateral wall of the heart (atrium and ventricle). Ischemia with this artery would manifest as T-wave inversion of leads I, aVL, and leads V5 and V6. The circumflex artery branches off the left coronary artery and follows the left part of the coronary sulcus giving branches to the left atrium and ventricle. The marginal artery (choice C) is a branch of the right coronary artery, and it supplies branches to both surfaces of the right ventricle. Disease in this vessel is not associated with specific EKG changes. Inverted T-waves in leads II, III, and aVF would correspond mainly to ischemia of the inferior wall of the left ventricle. The main artery supply to this area is the right coronary artery (choice E). It arises from the anterior aortic sinus, then passes between the conus arteriosus and the right auricle, and then runs in the right portion of the coronary sulcus. From there it continues on the diaphragmatic surface of the heart from the right to left, as far as the posterior longitudinal sulcus. It eventually arrives at the apex of the heart in the form of the posterior descending artery (choice D).

Question 3 of 5

Which of the following is most likely responsible for this patient's symptoms? / A. Atheromatous plaque rupture / B. Hyaline arteriosclerosis / C. Hyperplastic arteriosclerosis / D. Thrombus embolization / E. Vasospasm of the coronary vessels. Explanation - Q: 5.3

Close

The correct answer is A. Several hypotheses have been formulated about atheromatous plaque formation. One hypothesis involves endothelial wall damage, resulting in monocyte and platelet adhesion and migration of monocytes into the intima from the lumen and smooth muscle media. Another hypothesis involves hyperlipidemia causing an increased rate of LDL (low density lipoprotein) penetration into the artery wall, causing endothelial injury and the promotion of foam cells. Once the atheromatous plaque is formed, any rupture (e.g., with vasospasm) will expose subendothelial collagen, promoting platelet adherence, and the eventual aggregation of platelets. This may cause a total occlusion of the coronary vessel (as in the case of an infarction) or a transient subocclusion (that leads to angina due to

ischemia). Arteriosclerosis typically produces less acute effects than the occlusion seen with atheromatous plaque formation. Hyaline arteriosclerosis (choice B) is microangiopathy seen in hypertensive and diabetic patients. It is due to leakage of plasma components across the vascular endothelium and increasing extracellular matrix production by smooth muscle walls. This eventually narrows the arteriolar lumina causing reduced blood flow for that organ e.g., the kidney. Hyperplastic arteriosclerosis (choice C) is usually seen in "malignant" or severe hypertension and is characterized by laminated thickenings of the wall of the lumina. The most commonly affected sites are the kidney, gallbladder, pancreas, and intestines. An embolus is a detached intravascular solid, liquid, or gaseous mass carried by the blood to a site distal to the point of origin. Thrombus embolization (choice D) represents 99% of all emboli. An example is a pulmonary (thrombo)embolism. Although atherosclerosis may provide a site for thrombus formation and embolization, this is not the most common mechanism for angina. Vasospasm (choice E) may cause angina more rarely; this is called Prinzmetal's angina. There may be essentially no major atheromatous changes in the coronary vessels. Prinzmetal's angina is often treated with calcium channel blockers. Question 4 of 5

The cardiac enzyme marker troponin is normally found predominantly in which of the following sites? / A. Cell membranes of myocardial cells / B. Endoplasmic reticulum / C. Mitochondria / D. Myofibrils / E. Nuclei / F. Ribosomes Explanation - Q: 5.4

Close

The correct answer is D. Troponins are tightly bound structural proteins that regulate the calcium-mediated interaction of actin and myosin in striated muscle (myofibrils). Troponin release would indicate cell death, such as that seen in infarction. The rise of this marker after cardiac injury parallels that of creatine kinase (CK) (another cardiac enzyme marker). However, in contrast to CK, baseline levels of troponin are undetectable in normal volunteers.

Neither troponin nor creatine kinase are found in cell membranes (choice A) of the myocardium. Creatine kinase is a cardiac marker that resides in the cytoplasm and mitochondria (choice C). During cardiac injury, destruction of the cell walls and sarcolemma of the myocardial cells causes rapid release of the CK into the bloodstream. It is also released in noninfarction cardiac injuries (e.g., blunt chest trauma). The nucleus (choice E), ribosomes (choice F), and endoplasmic reticulum (choice B) are sites of transcription, translation and post-translational modification of troponin. Troponin is then incorporated into the myofibril structure via covalent attachment. Question 5 of 5

Which of the following is the mechanism of action of nitroglycerin? / A. Increases stroke-work index of the heart / B. Maintains coronary perfusion despite fluctuations in blood pressure / C. Relaxes bronchial smooth muscle / D. Relaxes vascular smooth muscle / E. Unblocks coronary vessels by fibrinolysis Explanation - Q: 5.5

Close

The correct answer is D. Relaxation of vascular smooth muscle is the principal pharmacologic action of nitroglycerin. Nitroglycerin (glyceryl trinitrate) is a therapeutically active member of the nitrate group of drugs. Nitrates improve oxygen supply to the myocardium (dilation of the coronary artery, reduction of the end-diastolic pressure in the left ventricle) and reduces myocardial oxygen requirements (dilation of the peripheral arteries, lowering of the systemic blood pressure). Reduction of venous return is a particular benefit (reduction of preload) in the event of heart failure. In smooth muscle cells, nitrates increase the intracellular concentration of nitric oxide (NO). NO stimulates guanylate cyclase, forming cGMP, which probably dephosphorylates the phosphorylated form of myosin light chain, leading to smooth muscle relaxation. Myocardial oxygen consumption or demand (as measured by the pressurerate product, tension-time index, and stroke-work index, choice A) is decreased by both the arterial and venous effects of nitroglycerin, and a more favorable supply-demand ratio can be achieved. Therapeutic doses of intravenous nitroglycerin reduce systolic, diastolic, and mean arterial blood pressures. Effective coronary perfusion pressure (choice B) is usually maintained, but

can be compromised if blood pressure falls excessively or increased heart rate decreases diastolic filling time. The bronchial smooth muscle is unaffected by nitrates. Bronchodilation (choice C) can be achieved by stimulation of beta-2 adrenergic receptors. Inhalation of beta-agonists, such as albuterol, can help reduce wheezing during asthmatic episodes. Reperfusion of coronary vessels through fibrinolysis (choice E) is the action of fibrinolytic agents like alteplase or streptokinase. They are used primarily in myocardial infarctions. If the agent is administered done quickly enough, the infarct size is reduced and left ventricular function is preserved, thereby reducing mortality.

A 19-year-old college student is referred to a family medicine physician for an abnormal heart sound heard on a routine school physical examination. On questioning, he denies chest pain or shortness of breath, but says he does occasionally experience palpitations. He is adopted and does not know his family history. On physical examination, he is 201 cm (79 inches) talI, weighs 73 kg (160 Ib), and has long arms and legs. On examination, there is a mid-systolic click, heard best at the apex. The click also occurs earlier when the patient stands or performs a Valsalva maneuver. Question 1 of 6

Which of the following cardiac abnormalities does this patient most likely have? / A. Aortic regurgitation / B. Aortic stenosis / C. Mitral stenosis / D. Mitral valve prolapse / E. Tricuspid regurgitation Explanation - Q: 6.1

Close

The correct answer is D. Mitral valve prolapse is a common valvular abnormality that can be benign and asymptomatic, but it can also cause palpitations, chest pain, and shortness of breath. It is due to stretching of the posterior mitral valve leaflets, resulting in prolapse of the valve. This change produces the classic midsystolic click, that is sometimes is followed by a murmur. Aortic regurgitation (choice A) would result in a blowing, diastolic murmur heard best at the left second intercostal space. Aortic stenosis (choice B) would result in a systolic ejection murmur heard

best at the right second intercostal space. Mitral stenosis (choice C) produces a low-pitched diastolic murmur heard best over the apex. It is often preceded by an opening snap. Tricuspid regurgitation (choice E) produces a blowing systolic murmur heard best at the left lower sternal border. Question 2 of 6

An echocardiogram is performed. Which of the following would likely be observed during the study? / A. Ballooning of the aortic valve into the ventricle during diastole / B. Ballooning of the mitral valve into the atrium during diastole / C. Rupture of the aortic valve / D. Rupture of the tricuspid valve / E. Stenotic mitral valve Explanation - Q: 6.2

Close

The correct answer is B. Ballooning of the mitral valve into the atrium during diastole is diagnostic of mitral valve prolapse. Ballooning of the aortic valve into the ventricle during diastole (choice A) does not occur in mitral valve prolapse. In addition, this would not result in a mid-systolic click. Rupture of the aortic valve (choice C) could result in severe hemodynamic compromise, especially if it were acute, due to a large regurgitant flow of blood. A harsh diastolic murmur would be heard on examination. Rupture of the tricuspid valve (choice D) would cause a blowing systolic murmur heard over the left lower sternal border. A stenotic mitral valve (choice E) is not associated with mitral valve prolapse. In mitral stenosis, the valve is thick and stiff. There is a low-pitched diastolic murmur, which is often preceded by an opening snap. Question 3 of 6

Which of the following genetic disorders can be associated with this abnormality? / A. KIinefelter syndrome / B. Marfan syndrome / C. Osler-Weber-Rendu syndrome / D. Tay-Sachs disease / E. Wilson disease Explanation - Q: 6.3

Close

The correct answer is B. Marfan syndrome is a connective tissue defect due to a deficiency in fibrillin. This results in defects in skeletal, visual, and cardiovascular structures, such as mitral valve prolapse and/or aortic aneurysms. Klinefelter syndrome (choice A) is chromosomal disorder, most often characterized by the karyotype 47,XXY. It results in hypogonadism, and in some cases, mild mental retardation. Osler-Weber-Rendu syndrome (choice C) is also known as hereditary hemorrhagic telangiectasia. It is characterized by telangiectasias of the skin and mucous membranes and hemorrhage at these sites. Tay-Sachs disease (choice D) is a lysosomal storage disease caused by a deficiency of hexosaminidase A. It results in central nervous system degeneration, mental retardation, motor deterioration, and blindness. Wilson disease (choice E) is a disorder of copper metabolism. It is characterized by decreased serum ceruloplasmin and results in accumulation of copper in the liver, kidney, brain, and cornea. Question 4 of 6

This patient is diagnosed with a genetic disorder associated with the valvular abnormality. Which of the following modes of inheritance does this disease follow? / A. Autosomal dominant inheritance / B. Autosomal recessive inheritance / C. Mitochondrial inheritance / D. X-Iinked dominant inheritance / E. X-Iinked recessive inheritance Explanation - Q: 6.4

Close

The correct answer is A. Marfan syndrome is inherited in an autosomal dominant manner. Usually, in diseases with this pattern of inheritance, one heterozygous parent carries a gene with the phenotypic expression of the disorder, and the other parent is normal. Autosomal recessive inheritance (choice B) usually occurs when both parents are heterozygotes and do not phenotypically manifest the disorder. Cystic fibrosis is an example of an autosomal recessive disease. Mitochondrial inheritance (choice C) is due to the maternal transmission of mitochondrial genes. Mitochondrial myopathies are inherited in this pattern. X-linked dominant inheritance (choice D) is a rarity. The heterozygous

female and the hemizygous male will manifest the phenotype. X-linked recessive inheritance (choice E) is most often the result of a heterozygous female parent and a genotypically and phenotypically normal male parent. Male children who inherit the affected X chromosome will manifest the disorder, but the female children will only be carriers. Question 5 of 6

Over the next year the patient develops chest pain, shortness of breath, and progressive fatigue. On auscultation, he has a midsystolic click, which is now followed by a high-pitched, blowing systolic murmur. Which of the following is the most likely cause of this new development? / A. Aortic aneurysm / B. Aortic stenosis / C. Mitral regurgitation / D. Mitral stenosis / E. Myocardial infarction Explanation - Q: 6.5

Close

The correct answer is C. In severe cases of mitral valve prolapse, patients can develop mitral regurgitation, which may require surgical treatment if the symptoms are severe. Aortic aneurysm (choice A) can occur in Marfan disease, but it is not associated with mitral prolapse or a systolic murmur. Aortic stenosis (choice B) causes a systolic ejection murmur heard best at the right second intercostal space, which often radiates to the carotid arteries. It is not associated with extra heart sounds. Mitral stenosis (choice D) is not a complication of mitral valve prolapse. Furthermore, it causes a diastolic murmur, which can be preceded by an opening snap. Myocardial infarction (choice E) is not associated with mitral valve prolapse. Question 6 of 6

The patient is treated with propanoloI. What is the mechanism of action of this medication? / A. AIpha-1 selective adrenergic blockade / B. Beta-1 selective adrenergic blockade / C. Calcium channel blockade / D. Nonselective beta adrenergic blockade / E. Nonselective alpha adrenergic blockade Explanation - Q: 6.6

Close

The correct answer is D. Propanolol, as well as timolol, pindolol, nadolol, and labetalol, all exhibit nonselective beta-adrenergic blockade. They are generally used in the treatment of hypertension, angina, and arrhythmias. Alpha-1 selective adrenergic blockade (choice A) is a mechanism used by prazosin, terazosin, and doxazosin. These agents can be used to treat hypertension or urinary retention due to benign prostatic hypertrophy. Beta-1 selective adrenergic blockade (choice B) is a mechanism used by metoprolol, atenolol, and esmolol. Unlike the nonselective beta-adrenergic blockers, these medications selectively block beta 1 receptors over beta 2 receptors. Calcium channel blockade (choice C) is a mechanism used by nifedipine, diltiazem, and verapamil. They can also be used to treat hypertension and angina. Verapamil and diltiazem can also be effective in the treatment of arrhythmias. Nonselective alpha-adrenergic blockade (choice E) is a mechanism used by phenoxybenzamine and phentolamine. They can be used in the treatment of pheochromocytoma.

A 21-year-old woman presents to an urgent care clinic with complaints of sharp, knife-Iike chest pain and shortness of breath for the past 4 days. She states that the pain is worse with inspiration, coughing, and movement. However, it is not related to position. She also complains of fever, chills, and a dry cough. She has a history of some type of rheumatologic disease, but she does not know which one. She was previously seen by a rheumatologist, but was lost to follow up, and stopped taking her medications 6 months ago. Her physical examination is notable for a temperature of 37.8 C (100 F), tachycardia, decreased breath sounds at both lung bases, and a friction rub. There is no pinpoint chest wall tenderness. Question 1 of 6

Which of the following is the most likely cause of this patient's chest pain? / A. Angina pectoris / B. Anxiety / C. Esophageal spasm / D. PIeuritis / E. Rib fracture Explanation - Q: 7.1

Close

The correct answer is D. Pleuritis is inflammation of the parietal pleura of the lung, which can often result in a friction rub heard on auscultation.

Patients can also have pleural effusions. Pleuritis is associated with many different disease processes, including collagen vascular diseases and infections. Angina pectoris (choice A) is chest pain related to underlying coronary artery disease. The pain is usually substernal, worse with exertion, and relieved by rest. It is not associated with fever or chills, nor is it pleuritic in nature. Anxiety (choice B) can often be a cause of chest pain. It is a clinical diagnosis and should be associated with other symptoms of anxiety. It is not pleuritic in nature, and is not associated with a cough, fever, or chills. Esophageal spasm (choice C) is due to uncoordinated contractions of the esophagus. It is generally associated with dysphagia. Rib fracture (choice E) should result in well-localized chest pain. There is pinpoint chest wall tenderness, and bone crepitus can be noted on palpation. Question 2 of 6

On a chest radiograph, the patient is noted to have bilateral pleural effusions. Evaluation of the fluid would most likely reveal which of the following? / A. Exudative effusion / B. Low pleural fluid LDH / C. PIeural fluid to serum lactate dehydrogenase (LDH) ratio 38 C (100.4 F), or leukocytosis (>10,000/mm3), or laboratory documentation of chlamydial or gonorrheal infection. Treatment is with antibiotics. Ectopic pregnancy (choice A) is ruled out with the negative urine pregnancy test. An ectopic pregnancy is a pregnancy that is implanted abnormally-- most often in the fallopian tubes. As a pregnancy, it secretes human chorionic gonadotropin (hCG), which can be found in the blood or urine. When this is not present, ectopic pregnancy is ruled out. Gonococcal cervicitis (choice B) presents with findings localized to the cervix. This patient has findings that go beyond a cervicitis. Given her abdominal tenderness with rebound, cervical motion tenderness, and adnexal tenderness, she is manifesting involvement of the fallopian tubes and peritoneum. While the gonococcus may be the offending organism in this case, this patient has more than a gonococcal cervicitis. A patient with a spontaneous abortion (choice D) can present in a variety of ways, but most commonly she will present with complaints of vaginal bleeding or the passage of tissue from the vagina. A spontaneous abortion represents a failed pregnancy and this patient has no evidence of a failed pregnancy, and an abundance of evidence for PID. A patient with a urinary tract infection (UTI) (choice E) will usually present complaining of frequency, urgency, or dysuria. Also, the urinalysis will typically show white blood cells. This patient has no urinary complaints and a negative urinalysis.

One of the antibiotics that this patient is started on is doxycycline. This antibiotic works via which of the following mechanisms? / A. Inhibition of bacterial cell wall synthesis / B. Inhibition of bacteriaI DNA gyrase / C. Inhibition of bacterial protein synthesis / D. Inhibition of ergosterol synthesis / E. Inhibition of viraI DNA synthesis after conversion by thymidine kinase

The correct answer is C. Doxycycline is a member of the tetracycline family of antibiotics. These bacteriostatic antibiotics are broad-spectrum in nature and are used in a variety of conditions. The mechanism of action of the tetracycline family of antibiotics is that they prevent the attachment of transfer RNA to the 50S ribosomal subunit, thus inhibiting bacterial protein synthesis. The tetracycline family of medications is used in the treatment of syphilis, Chlamydia, gonorrhea, amebiasis, urinary tract infections, as well as pelvic inflammatory disease. Along with its use in pelvic inflammatory disease, doxycycline is also well-known for its use in the treatment of Lyme disease. Penicillins and cephalosporins are two families of antibiotics that work via the mechanism of inhibition of bacterial cell wall synthesis (choice A). Cefotetan (a third-generation cephalosporin) or cefoxitin (a second generation cephalosporin) are often used with doxycycline in the treatment of PID. Fluoroquinolones are antibiotics that function through the inhibition of bacterial DNA gyrase (choice B). They are used to treat a variety of organisms including Escherichia coli, Proteus mirabilis, and Neisseria gonorrhoeae. They are often used in the treatment of urinary tract infections. Miconazole is the drug that works through the inhibition of ergosterol synthesis (choice D). By doing so, it disrupts fungal cell wall synthesis. It is often used in the treatment of topical fungal infections, but can also be used intravenously in the treatment of systemic fungal infections. Acyclovir is the drug that functions through the inhibition of viral DNA synthesis after conversion by thymidine kinase (choice E). It is used predominantly in the treatment of herpes and varicella-zoster infections.

Doxycycline is not used during pregnancy because of possible adverse effects on which of the following fetal structures? / A. Heart / B. Kidneys / C. Limbs D. Teeth / E. Tendons

The correct answer is D. Tetracyclines are known to chelate with calcium ion. This characteristic results in these drugs becoming incorporated into dental enamel and bone. When used by pregnant women, the tetracyclines have been shown to cause staining of teeth and suppression of skeletal development. These drugs, therefore, should not be used by pregnant women or children younger than 8 years of age. Also, because of the interaction of the tetracyclines with calcium, calcium-containing supplements and antacids should not be used at the same time as tetracycline antibiotics. At least 2 hours separation between the ingestion of the 2 substances should be given. Cardiovascular birth defects are among the most common birth defects. Lithium, which has been used to treat manic depressive disorder and other psychiatric problems, has been associated with Ebstein's anomaly, a defect of the fetal heart (choice A). Angiotensin converting enzyme (ACE) inhibitors are medications commonly used in patients with hypertension. This family of medications includes drugs such as captopril, enalapril, lisinopril, quinapril, and several others. These medications should not be used during pregnancy because of the adverse effects they have on fetal kidneys (choice B). If used in pregnancy, they can cause severe fetal renal damage and death. Thalidomide is a medication that was first used in Europe in the 1950s (and later the United States) for the treatment of insomnia and nausea in pregnant women. Its use resulted in the birth of thousands of babies with stunted growth of the limbs (choice C). Even one dose of the medication can be associated with birth defects. Fluoroquinolones should not be used during pregnancy because of possible effects on fetal tendons (choice E). Studies in dogs have shown that use of these drugs in pregnancy can cause arthropathies. This patient's condition can lead to adhesions in the pelvis and around what other structure? / A. Brain / B. Heart / C. Kidneys / D. Liver / E. Lungs The correct answer is D. The Fitz-Hugh-Curtis syndrome is a syndrome of perihepatitis that approximately 5% of women with pelvic inflammatory disease will develop. The features of the Fitz-Hugh-Curtis syndrome are right upper quadrant pain and tenderness, sometimes with mildly abnormal liver function test results. When laparoscopy is performed on a patient with Fitz-Hugh-Curtis

syndrome, fibrous, "violin string" adhesions can be seen extending from the dome of the liver to the diaphragm. The syndrome is most commonly associated with gonococcal or chlamydial PID. It is believed that the intraperitoneal spread of the infection (and inflammation) is the cause of the condition. Because there is a direct anatomic connection between the liver and pelvic structures, direct intraperitoneal spread of infection and inflammation can occur. The brain (choice A) is not an intraperitoneal structure. Infection of the brain from pelvic inflammatory disease would be exceedingly rare and would require a mechanism other than direct intraperitoneal spread. The heart (choice B) is also not an intraperitoneal structure. In order to develop a myocarditis (infection of the myocardium) or an endocarditis (infection of the endocardium), would require a mechanism other than direct intra-abdominal spread. The kidneys (choice C) are a retroperitoneal structure. Direct intraperitoneal extension in a patient with pelvic inflammatory disease could not cause nephritis. The lungs (choice E) are also not an intraperitoneal structure. Pelvic inflammatory disease leading to pneumonitis would also be rare.

A 58-year-old woman comes to the physician because of vaginal bleeding. She states that she had her last menstrual period 11 years ago and since that time, she has not had any vaginal bleeding, until now. She has no other complaints. Her past medical history is significant for diabetes and hypertension. She has never had surgery. She currently takes glyburide and captopriI. She was taking estrogen replacement therapy for hot flashes untiI 5 years ago. She has no known drug allergies. On physical examination, she is a 5 foot, 2 inch female who weighs 220 pounds. Speculum examination reveals some old blood in the vaginal vault. Pelvic examination is limited by the patient's body habitus. The remainder of the physical examination is within normal limits. An endometrial biopsy is performed, that shows crowded endometrial glands with severe cytologic and architectural abnormalities, and greater than 50% solid areas.

Which of the following is the most likely diagnosis? / A. Ectopic pregnancy / B. Endometrial atrophy / C. Endometrial carcinoma / D. Pelvic inflammatory disease

/ E. Polycystic ovary syndrome

The correct answer is C. This patient's presentation and findings are most consistent with endometrial carcinoma. Endometrial carcinoma is the most common gynecologic malignancy. It is approximately two times more than ovarian cancer and roughly three times more common than invasive cervical cancer. The major risk factors for the development of endometrial cancer are obesity and the use of unopposed estrogen replacement therapy. Hypertension and diabetes also appears to be significant risk factors. Other risk factors include infertility, nulliparity, early menarche, and late menopause. This patient is at significant risk given her hypertension, diabetes, obesity, and history of exposure to unopposed estrogen. Diagnosis is made by endometrial biopsy. Treatment is surgical: total abdominal hysterectomy with bilateral salpingo-oophorectomy and staging. In patients who are poor candidates for surgery, radiation alone can be used, but the cure rate is low. Ectopic pregnancy (choice A) can cause abnormal vaginal bleeding. However, for an ectopic pregnancy to occur, ovulation must take place in the woman. A woman, such as this patient, who is postmenopausal, is extremely unlikely to ovulate and, therefore, ectopic pregnancy is not the most likely diagnosis. Furthermore, her symptoms, findings and pathology results establish the diagnosis of endometrial cancer. Endometrial atrophy (choice B) can cause vaginal bleeding in postmenopausal women. However, in these cases, the biopsy demonstrates atrophy, and not endometrial cancer as in this patient. Pelvic inflammatory disease (choice D) is a disease predominantly of young, sexually active women. Its peak incidence is in the 15 to 24-year-old group. It is characterized by abdominal tenderness, cervical motion tenderness, and adnexal tenderness with confirmatory laboratory findings. This patient's presentation is not consistent with pelvic inflammatory disease. Polycystic ovary syndrome (choice E) appears to place women at increased risk for the eventual development of endometrial hyperplasia and cancer. However, this patient is no longer "at risk." She, in fact, has endometrial cancer.

Administration of which of the following hormones might have prevented this patient's condition? / A. Follicle stimulating hormone / B. Human chorionic gonadotropin / C. Insulin

/ D. Progesterone / E. Testosterone

The correct answer is D. One hypothesis for the development of endometrial cancer is that unopposed estrogen leads to the proliferation and eventual cancerous change of the endometrial lining. It has been well-demonstrated that postmenopausal women who have a uterus (i.e., postmenopausal women who have not had a hysterectomy) have an increased probability of developing endometrial cancer that correlates directly with the dose and duration of unopposed estrogen use. The increased risk has also been shown to persist even after the estrogen has been stopped. "Unopposed" estrogen refers to estrogen that is not opposed by progesterone. This patient has no past surgical history, that is, she has never had a hysterectomy. She was taking unopposed estrogen during her 50s. She should have also been on a progestin at that time to prevent the development of endometrial cancer. Follicle stimulating hormone (choice A) is produced by the anterior pituitary and acts upon the ovary. It is not given with estrogen to postmenopausal women to prevent the development of endometrial hyperplasia and cancer. Human chronic gonadotropin (choice B) is produced by the trophoblast cells of the developing embryo to support the corpus luteum so that the corpus luteum will continue to secrete hormones to support the pregnancy during its first several weeks. It is not given to postmenopausal women to oppose the effects of estrogen. Insulin (choice C) is a peptide hormone secreted by the pancreas and is essential for glucose metabolism and regulation. Testosterone (choice E) is used by some physicians in postmenopausal women to enhance sexual libido and well-being. It is not used like progesterone to oppose the effects of estrogen on the endometrium. One of the risks of the endometrial biopsy that was performed on this patient is perforation of the uterus. The endometrial biopsy device is placed through the cervix and into the endometrial cavity. If complete perforation occurs, what is the sequence of layers that the biopsy device would penetrate prior to entering the peritoneal cavity? / A. Endometrium, myometrium, serosa / B. Ovary, fallopian tube, broad ligament / C. Round ligament, cardinal ligament, uterosacral ligament / D. Serosa, myometrium, endometrium / E. Uterine body, fundus, cervix

The correct answer is A. The uterus is a hollow, muscular organ that lies between the bladder and the rectum in the true pelvis. The uterus can be divided into three major segments: the uterine fundus, the uterine corpus (or body), and the cervix. The fallopian tubes enter the uterine fundus laterally in the region called the cornua. Moving from the inside (or hollow) portion of the uterus to the peritoneal cavity, the layers that would be penetrated are the endometrium, the myometrium, and the serosa. The endometrium is the portion of the uterus that proliferates during the menstrual cycle and sheds (the menses) if pregnancy does not occur, or supports the pregnancy if conception and implantation take place. The myometrium is comprised of three layers: an inner layer of longitudinal smooth muscle, a middle layer of circular smooth muscle (the stratum vasculare), and an outer layer of longitudinal and circular smooth muscle (the stratum supervascularae). The final layer is the uterine serosa. The ovary, fallopian tube, and broad ligament (choice B) are found posterolateral to the uterus. A biopsy device that perforated the uterus could also injure these structures, but the direct path of perforation is through the endometrium, myometrium, and serosa. The round ligament, cardinal ligament, and uterosacral ligament (choice C) are supporting structures attached to the uterus. Again, while a perorating biopsy instrument could possibly involve these structures, this would not be the direct path of perforation. The serosa, myometrium, and endometrium (choice D) are the correct structures that would be perforated, but they are in the wrong order. Starting within the uterus, the first layer is the endometrium, next is the myometrium and finally the serosa. The uterine body, fundus, and cervix (choice E) describe the three general parts of the uterus.

Estrogen is believed to play a crucial role in the development of this disease. Which of the following substances is the precursor of this hormone? / A. Arachidonic acid / B. Argininosuccinate / C. Cholesterol / D. Oxaloacetate / E. Succinate

The correct answer is C. There are 5 principal types of steroid hormones: glucocorticoids, mineralocorticoids, estrogens, progestins, and androgens. These steroids hormones are synthesized from cholesterol in the cytosol and the mitochondria. The main organs of steroid biosynthesis are the adrenal cortex, ovaries, testes, and the placenta. The first step in the synthesis of the steroid hormones, which is also the rate limiting step, is the conversion of cholesterol to pregnenolone. This reaction is catalyzed by the desmolase complex. Several other intermediate steps must occur for the eventual formation of the steroid hormones. Defects in these pathways can result in congenital adrenal hyperplasia. Arachidonic acid (choice A) is a precursor in the formation of prostaglandins, thromboxanes, and leukotrienes. These substances cause a multitude of physiologic responses. Argininosuccinate (choice B) is an intermediate compound in the urea cycle. Urea is the major excretory product of nitrogen metabolism in human beings. Argininosuccinate is formed when aspartate, ATP, and citrulline react in a reaction catalyzed by argininosuccinate synthetase. Oxaloacetate (choice D) and succinate (choice E) are compounds found in the Krebs cycle (also called the citric acid cycle or the tricarboxylic acid cycle). This cycle represents the final common pathway for the degradation of fatty acids, amino acids, and carbohydrates.

A 57-year-old woman presents to a physician with chronic pelvic pain of many years duration. On further questioning, the woman also reports urinary frequency, constipation, pain with intercourse, and bloating. On physical examination, a large mass is felt in the pelvic area. Follow-up ultrasound examination demonstrates that the mass involves the right adnexa and is composed of multiloculated cystic spaces. Prior to sending the patient to surgery, the primary care physician wants to order a serum tumor marker to screen for ovarian cancer. Which of the following would be the best choice? / A. Adrenocorticotropic hormone / B. CA-125 / C. Galactosyltransferase / D. Ribonuclease / E. S-100

The correct answer is B. The diagnosis of ovarian cancer is often delayed, because symptoms often do not occur until late in the disease when the mass is large and applies pressure to other abdominal organs. The symptoms illustrated in the case are typical. Patients may also develop ascites with shortness of breath, a variety of gastrointestinal symptoms related to impaired motility secondary to pressure, and, if the ovarian cancer is a type that secretes hormones, menstrual irregularities or abnormal hair growth. The most widely studied tumor marker in ovarian cancer is CA-125. This marker is not of particular use in screening of the general population for two reasons: 1) small cancers often do not cause serum elevations of the marker; and 2) serum levels of the marker can be elevated by a variety of cancers (including those of ovary, breast, pancreas, colon, and lung) and benign conditions (including endometriosis, pregnancy, liver disease, and congestive heart failure). However, if there is already a strong suspicion or a known history of ovarian cancer, CA125 can be helpful in establishing if the cancer burden (if present) is large, and whether following the CA-125 levels following surgery can be used to monitor for recurrent disease. While small tumors are not always picked up (50% of women with small ovarian cancers have normal CA-125), more than 80% of women with advanced ovarian cancer have CA-125 elevations. Adrenocorticotropic hormone (choice A) can be elevated in ectopic hormoneproducing lung cancers. Galactosyltransferase (choice C) can be elevated in a wide variety of cancers, including those from lung, breast, esophagus, stomach, pancreas, and colon. It is not used as a marker for ovarian cancer. Ribonuclease (choice D) is a marker for pancreatic cancer. S-100 (choice E) is a marker for melanoma and neuroendocrine tumors.

In this particular case, serum levels of the chosen tumor marker are within the normal range, which is interpreted to mean that the woman either does not have ovarian cancer or has a smaller amount of cancer, which is probably confined to the ovary. The woman is taken to surgery and a 20-cm diameter adnexal mass is removed and sent for intraoperative examination. The specimen submitted to pathology is roughly the size and shape of a large melon, has a smooth external surface, shows a pale color on cross-section, and is nearly completely replaced by a large number of cystic spaces of varying sizes from nearly microscopic to over 6 cm diameter. These cysts contain cloudy white gelatinous material that slowly oozes fluid and the cyst walls are generally thin. Which of the following is the most likely diagnosis at this point in the evaluation? / A. Benign or malignant Brenner tumor / B. Benign or malignant granulosa celI-theca cell tumor

/ C. Dermoid cyst or immature teratoma / D. Mucinous cystadenoma or mucinous cystadenocarcinoma / E. Serous cystadenoma or serous cystadenocarcinoma

The correct answer is D. In real life, the gross examination of a specimen tends to be most helpful if it provides enough clues to tentatively place the lesion into a broad category. Usually, unless clear-cut invasion is seen grossly, the gross examination cannot "prove" whether the lesion is benign or malignant. In this case, a large ovarian mass virtually completely replaced by cystic spaces filled with gelatinous material, and whose stroma appears pale is most likely a cystic mucinous lesion, such as mucinous cystadenoma or mucinous cystadenocarcinoma. Brenner tumors (choice A), which are predominately benign, are often solid (but may be somewhat cystic with less complete replacement of the tumor than is illustrated in this case) and the stroma tends to be firm and white, resembling the normal stroma of the ovary. The various granulosa cell and theca cell tumors (choice B) are usually benign (rarely malignant, most commonly as a granulosa cell tumor) and tend to form solid or partially cystic tumors that have a yellow hue if the tumor is endocrinologically active. Dermoid cysts (choice C, also called mature cystic teratomas) are typically filled with cheesy white material (derived from shedding of the surface of skin) and hair; while the related (and malignant) immature teratomas (choice C) are uncommon in the ovary and tend to be mostly or completely solid. Serous cystadenomas and cystadenocarcinomas (choice E) closely resemble the mucinous variants, but the cystic spaces are filled with clear fluid rather than gelatinous material.

The cells lining the cystic spaces in this tumor are most closely related to which of the following normal cell types? / A. Decidual cell in the ovarian stroma / B. Fibroblastic ovarian stroma cell / C. Oocyte in preantral follicle / D. Ovarian surface epithelial cell / E. Smooth muscle cell in ovarian stroma

The correct answer is D. The ovary is vulnerable to a surprisingly large variety of primary tumors. Ovarian tumors are broadly classified into epithelial tumors (resembling the epithelial cells of the ovary and including the serous tumors, mucinous tumors, endometrioid tumors, clear cell tumors, and transitional cell tumors); the sex cord-stromal tumors (related to choices B, D, and E, and including the granulosa cell tumors, the thecoma-fibroma tumors, the Sertoli cell tumors, the androblastomas, and the steroid cell tumors); and the germ cell tumors (related to choice C, and including the teratomas, dysgerminomas, yolk sac tumors, and mixed germ cell tumors). In this patient's large cystic mucinous tumor, the cells lining the cystic spaces are epithelial in nature, and thought to be most closely related to the epithelial cells on the surface of the ovary. Incidentally, peritoneum in other sites can undergo metaplasia to an epithelium resembling that on the surface of the ovary, and rarely, tumors that histologically resemble mucinous and serous cystic tumors of the ovary can arise in other sites involving peritoneum.

Extensive sampling reveals an area of frank cancer within the tumor, and a metastasis is also found on the surface of the uterus. Following surgery, the decision is made to add carboplatin. This drug acts by which of the following mechanisms? / A. BIocking microtubule assembly / B. Cross-Iinking DNA / C. Inhibiting topoisomerase ll / D. Interrupting folate metabolism / E. Substituting for a nucleotide in DNA or RNA

The correct answer is B. Most cases of ovarian mucinous cystadenocarcinoma of the ovary that are not of a very low stage receive chemotherapy in addition to surgery. The most common agent chosen is carboplatin, which is a drug similar to cisplatin and acts by a similar mechanism. Both of these drugs cross-link DNA. Vinblastine is an example of a chemotherapeutic agent that acts by blocking microtubule assembly (choice A). Etoposide is an example of a chemotherapeutic agent that acts by inhibiting topoisomerase II (choice C). Methotrexate is an example of a cancer chemotherapeutic agent that acts by interrupting folate metabolism (choice D).

Examples of cancer chemotherapeutic agents that act by substituting for a nucleotide (choice E) include 5-fluorouracil, cytarabine, and 6-mercaptopurine.

Which of the following genes has been most closely associated with familial cases of ovarian cancer? / A. BRCA1 / B. NF1 / C. NF2 / D. VHL / E. WT1

The correct answer is A. Roughly 5-10% of cases of ovarian cancer occur in individuals who are a member of a cancer family. In familial cases of ovarian cancer, the cancers tend to appear at a younger age, but are thought to possibly have, on average, a slightly better prognosis than do isolated cases. Families with multiple cases of breast cancer due to genetic mutations in BRCA1 or BRCA2 also have an increased incidence of ovarian cancers. The BRCA1 mutation is particularly virulent, and up to 30-40% of women with this mutation develop ovarian cancer. (There are also families in which the women have a BRCA1 mutation and develop ovarian, but not breast cancer at an increased rate.) The BRCA1 and BRCA2 mutations have a particularly high incidence in Ashkenazi Jews. Another cancer syndrome, the Lynch II syndrome (also known as the hereditary nonpolyposis colorectal cancer syndrome), is also associated with ovarian cancer (and cancers of the uterus, breast, stomach, and pancreas). In both the breast cancer families and the Lynch II families, the genetic pattern seen is usually autosomal dominant with variable penetrance. Prophylactic oophorectomy is often offered women in cancer families who have reached the age of 35 and have had their children. NF1 (choice B) is associated with neurofibromatosis type I, neuroblastoma, melanoma, and colon cancer. NF2 (choice C) is associated with neurofibromatosis type II, acoustic neuromas, and meningiomas. VHL (choice D) is associated with von Hippel-Lindau disease, renal cell carcinoma, and pheochromocytoma. WT1 (choice E) is associated with Wilms tumor.

Which of the following tends to decrease the risk of ovarian cancer? / A. Age greater than 50 years / B. Caucasian race / C. Combined oral contraceptive pills / D. History of breast cancer / E. No history of pregnancy

The correct answer is C. Decreased risk of developing ovarian cancer is associated with processes that seem to interrupt ovulation, including the use of the combined oral contraceptive pill, increasing parity, and breast feeding. An increased risk of ovarian cancer is seen with age greater than 50 years (choice A), no history of pregnancy (choice E), breast cancer (choice D), and Caucasian race (choice B). Less well established and less common risk factors include exposure of the genitalia to asbestos and talc, mumps virus, and estrogen therapy in postmenopausal women. Several months later, the patient returns to the hospital and is found to have widespread metastases. The patient is currently arousable and oriented. The patient has designated a family member as her health care power of attorney (HCPOA). The most appropriate first step is to determine which of the following? / A. If the patient is interested in a hospice referral / B. If the patient understands what is likely to happen / C. The patient's main coping style / D. Who is the HCPOA designee / E. Why the patient chose her HCPOA

The correct answer is B. The Health Care Power of Attorney designates someone to make the patient's medical decisions in the event that the person becomes incompetent or unable to make decisions. If this patient is awake and understands her situation, the HCPOA is not yet activated. Hospice (choice A) can be chosen by the patient when she desires palliative, rather than attempted curative therapy. The immediate concern in this case is to address the patient wishes in the very near future. Coping styles (choice C) can impede diagnosis and treatment of disease and even add to the disease state. The immediate concern in this case is to find out what the patient wishes should she code.

It is important to know the designee (choice D) because he or she becomes the decision maker when the patient is no longer able. However, determining if this patient is still able to make her own decisions is the immediate concern. Why the patient chose her HCPOA (choice E) is irrelevant.

The patient is successfully stabilized and returns home. She is admitted to a nursing home three months later. One day, her physician finds her somnolent and difficult to arouse. The Health Care Power of Attorney (HCPOA) designee has told the physician the he wants nothing done except supportive care. Which of the following is the most appropriate next step? / A. Order an EEG / B. Discuss intravenous fluids / C. Order a head CT scan / D. PIace a central line / E. Transfer to the emergency room

The correct answer is B. Fluids and pain control are consistent with supportive care. Additional tests (e.g., EEG, choice A) are not consistent with supportive care. A head CT scan (choice C) implies diagnosis and treatment. It is not consistent with supportive care. Placing a central line (choice D) implies aggressive treatment and is not consistent with supportive care. Transferring to the emergency room (choice E) implies diagnosis and treatment. It is not consistent with supportive care.

As the physician is speaking with the designated HCPOA, a frantic staff member enters the room and reports an estranged sibling has called on the telephone, requesting that "everything be done" until he can arrive. He has stated he wilI "sue the pants off that doctor if they give up on his sister." The designated HCPOA reiterates the plan for supportive care. The most appropriate next step is which of the following?

/ A. Begin medical treatment to buy time for the family to all agree to the same

plan / B. Call the EMS to take the patient to the hospital / C. Discuss the plan for supportive care and honor the designated HCPOA's request / D. Tell the family you will turn the brother into adult protective services / E. Refuse to speak to the designated HCPOA because he does not know what he is doing

The correct answer is C. The designated HCPOA's decision is the final one and the legal agreement has been set up to prevent the patient from any measures she did not want to be subjected to, despite the wishes of any other family members, doctors, etc. Beginning medical treatment (choice A) is inappropriate and illegal given the clear wishes of the HCPOA. Calling the EMS (choice B) is inappropriate and illegal given the clear wishes of the HCPOA. Telling the family you will turn the brother into adult protective services (choice D) is inappropriate and not indicated. It is within the rights of the patient's HCPOA to proceed with supportive care as planned. Refusing to speak to the designated HCPOA because he does not know what he is doing (choice E) is not acceptable for a competent physician. It is acceptable to explain your recommendations or concerns, but the final decision rests with the HCPOA.

A 17-year-old girl is evaluated by a gynecologist because she has never had a menstrual period. On physical examination, the girl is noted to be 5'2" talI, have slight webbing of her neck, and a broad chest with widely spaced nipples. The girI's breasts show papilla elevation only. In her pubic area, only villus hair is seen. No mature axillary hair is seen. Question 1 of 5

This girI's body is at which of the following Tanner stages of development? / A. Stage 1 / B. Stage 2 / C. Stage 3 / D. Stage 4 / E. Stage 5 Explanation - Q: 1.1

Close

The correct answer is A. Tanner stages are used to define the degree of sexual maturation of a girl's body. Stage 1 is the prepubertal stage, and is characterized by elevation of only the papilla of the breast, and pubic hair consisting only of fine villus hair. The child's height usually increases at a basal rate of 5-6 cm per year. Stage 2 (choice B) corresponds to the beginning of puberty and is characterized by palpable breast buds, enlargement of the areola, minimal coarse, pigmented hair mainly on the labia, and accelerated rate of height increase of typically 7-8 cm per year. Stage 3 (choice C) is characterized by elevation of breast contour with areolar enlargement, dark curly hair over mons pubis, axillary hair development, and peak height increase of about 8 cm/year. Acne vulgaris may develop in this stage. Stage 4 (choice D) is characterized by formation, by the areola, of a secondary mound on the breast, adult quality pubic hair with no spread to the junction of the medial thigh with the perineum, and height increase of about 7 cm/year. Stage 5 (choice E) is characterized by adult breast contour with recession of the areola to the general contour of the breast, adult distribution of pubic hair with spread to medial thigh, and no further increases in height.

Question 2 of 5

Most girls who are this patient's age are at which of the following Tanner stages? / A. Stage 1 / B. Stage 2 / C. Stage 3

/ D. Stage 4 / E. Stage 5

Explanation - Q: 1.2

Close

The correct answer is E. This patient is markedly behind her peers in sexual development, since Tanner stage 5 is usually reached by age 16. Stage 1 (choice A) is the prepubertal stage that begins at birth. Stage 2 (choice B) typically becomes noticeable at around 11 years (9-13 years for normal range), with breast bud enlargement often slightly preceding early pubic hair development. Stage 3 (choice C) typically occurs at about 12 years (9 1/2 to 14 years) and often includes the onset of menstruation. Stage 4 (choice D) typically occurs at about 13 years (10 1/2 to 15 1/2 years).

Question 3 of 5

Which of the following is the most likely diagnosis? / A. Down syndrome / B. Edwards syndrome / C. KIinefelter syndrome / D. Triple X syndrome / E. Turner syndrome Explanation - Q: 1.3

Close

The correct answer is E. This patient has a number of the stigmata of Turner syndrome, including primary amenorrhea (no menstrual periods ever), failure of onset of puberty, short stature, webbed neck, and widely spaced nipples. Other features that may be a part of Turner syndrome include multiple pigmented nevi, short 4th metacarpals and metatarsals, prominent finger pads, nail hypoplasia, and some diminution of perceptual ability, usually without frank mental retardation. Patients with Turner syndrome are now treated with oral sex hormone replacement, and will go through puberty and begin menstruation. They are usually infertile (exceptions being rare mosaic patients with some residual ovarian function), but recent work suggests that they may be able to carry a baby produced by in vitro fertilization, if appropriate hormonal support is given throughout pregnancy. Growth hormone supplementation begun by about age 9 if possible will

increase height. Down syndrome (choice A) is characterized mental retardation, characteristic facies (slanted eyes, small head, flattened occiput, flattened nose), single palmar (simian) crease, and short fingers. Edwards syndrome (choice B) is characterized by severe mental retardation and usually death in infancy. Klinefelter syndrome (choice C) is characterized by male phenotype, tall stature, small testes, and a predisposition for learning difficulties. Triple X syndrome (choice D) is characterized by a phenotypically apparently normal female who may or may not have sterility and menstrual irregularities

Question 4 of 5

Which of the following genotypes would be most likely present in this patient? / A. 45,XO / B. 47,XXX / C. 47,XXY / D. Trisomy 18 / E. Trisomy 21 Explanation - Q: 1.4

Close

The correct answer is A. Turner's syndrome is a sex chromosome abnormality, which in about half of live birth cases, is due to 45,XO karyotype. The remainder are usually mosaics, with 45,XO/46,XX or 45,XO/47,XXX karyotypes. It is thought that approximately 98% of Turner's conceptions die in utero early in pregnancy. 47,XXX (choice B) is called triple X syndrome. 47,XXY (choice C) is the karyotype for Klinefelter syndrome. Trisomy 18 (choice D) is also known as Edwards syndrome. Trisomy 21 (choice E) is also known as Down syndrome.

Question 5 of 5

This patient's condition is most strongly associated with which of the following congenital anomalies? / A. Atrial septal defect / B. Coarctation of the aorta / C. Patent ductus arteriosus / D. Tetralogy of Fallot / E. Ventricular septal defect Explanation - Q: 1.5

Close

The correct answer is B. 35% of patients with Turner's syndrome have coarctation of the aorta, which may be asymptomatic. An easy screening test is to check the pulses at both ankles and both wrists to note any discrepancy in pulse strength that might suggest the presence of a segment of aortic narrowing. Coarctation of the aorta is also associated with bicuspid aortic valve, intracranial berry aneurysms, ventricular septal defect, and acquired intercostal aneurysms. Atrial septal defect (choice A) can occur as part of the autosomal dominant condition Holt Oram syndrome, which also is associated with upper extremity bony abnormalities. Patent ductus arteriosus (choice C) is associated with a very large number of infectious and genetic congenital conditions, with or without other congenital heart disease. Tetralogy of Fallot (choice D) is associated with Down syndrome. Ventricular septal defect (choice E) is a component of tetralogy of Fallot, and can also be associated with Down syndrome, Patau syndrome, Edwards syndrome, and Holt-Oram syndrome.

A 34-year-old woman consults a physician because her menstrual periods have been irregular for the last 5 to 10 years. Her cycles are often greater than six weeks in length, with eight or fewer periods in a year. The periods often vary in character, with lengthy bleeding episodes, scant or heavy periods, or frequent spotting. Question 1 of 4

One of the findings noted on this woman's physical examination is the presence of poorly defined large patches of darkened skin on the back of her neck and in skin creases under her arms and breasts. These darkened skin patches have a very slightly rough, velvety texture. The Iesions are not itchy or irritated in feeling, and the patient had been only vaguely aware of them. Which of the following skin diseases would be

most likely to produce this type of skin manifestation? / A. Acanthosis nigricans / B. Lichen planus / C. Malignant melanoma / D. Psoriasis vulgaris / E. Sézary syndrome Explanation - Q: 2.1

Close

The correct answer is A. The lesions are those of acanthosis nigricans. While they appear hyperpigmented, there is actually little or no increase in melanin in these areas, and the dark coloring is instead the result of papillomatous epidermal and superficial dermal hyperplasia. Lichen planus (choice B) is an inflammatory skin condition characterized by itchy, purple papules or plaques. Malignant melanoma (choice C) is a malignant skin tumor (loosely a malignant mole) that can produce darkly pigmented skin, but it would be very unusual to have multiple lesions or lesions under the breasts or arms. Also, while the edges of a melanoma may be feathery, the lesion's circumference is usually relatively well-defined. Psoriasis vulgaris (choice D) is a proliferative disease of the skin and produces clearly defined plaques with a deep red color ('salmon-colored") and silvery scale and often involves the limbs. Sézary syndrome (choice E) is an intensely erythematosus, itchy, skin reaction that involves the whole body and is a reaction to a T-cell lymphoma

Question 2 of 4

In addition to being associated with the condition causing this patient's menstrual irregularities, the patient's skin condition can be associated with which of the following? / A. Gastric carcinoma / B. Gout / C. Myocardial infarction / D. Rheumatoid arthritis / E. UIcerative colitis Explanation - Q: 2.2

Close

The correct answer is A. Acanthosis nigricans can be subdivided into "benign" (as this patient has) and "malignant" variants. The "benign" form typically has milder lesions with localized involvement of the back of the

neck, arm pits, areas below the breast, groin, vulva, between the thighs, and sometimes on the hands, elbows, and knees. One or several patches of involved skin may be seen. This form is associated with endocrine abnormalities, including insulin-resistance, obesity, hyperandrogen states, and other endocrine abnormalities. The "malignant" form of acanthosis nigricans is not actually a malignant skin disease, but it is so-called because it is associated with underlying carcinomas, often of the gastrointestinal tract (frequently stomach). This form tends to produce much more severe skin lesions, with a much broader distribution (sometimes involving almost all of the skin), and patients with "malignant" acanthosis nigricans often die within 2 years of their underlying cancer. Gout (choice B) patients can have tophi, composed of deposits of urate with the accompanying skin reaction; the tophi are most often recognized on the external ear, although they can occur in other locations. Myocardial infarction (choice C) does not usually produce a skin manifestation unless congestive heart failure develops, in which case, edema of the ankles may develop. Rheumatoid arthritis (choice D) is associated with subcutaneous rheumatoid nodules, vasculitis, and leg ulcers. Inflammatory bowel disease, e.g., ulcerative colitis (choice E), is associated with oral mucosal ulcerations (aphthous ulcers) and pyoderma granulosum (causes a severe ulcer of the lower leg).

Question 3 of 4

Physical examination reveals hirsutism. Pelvic examination demonstrates bilateral enlargement of the adnexa to approximately 3 times the size of normaI. Follow-up ultrasound examination demonstrates a "string of pearls" appearance in both ovaries. Given these findings, and the patient's menstrual irregularities, hirsutism, and skin changes, which of the following is the most likely diagnosis? / A. Bilateral dermoid cysts / B. Bilateral hydrosalpinx / C. Bilateral mucinous cystadenocarcinoma / D. Multiple leiomyomas / E. Polycystic ovary disease Explanation - Q: 2.3

Close

The correct answer is E. The diagnosis that best accounts for the patient's

complex findings is polycystic ovary disease (also known as Stein-Leventhal syndrome). The clinical presentation of polycystic ovary disease can include menstrual problems (amenorrhea, infrequent menses, oligomenorrhea, irregularity), infertility (secondary to infrequent or absent ovulation), symptoms of androgen excess (hirsutism, alopecia, acne), chronic pelvic pain or palpable mass, obesity, and acanthosis nigricans. The polycystic ovaries are typically enlarged 1.5 to 3 times, and the finding of the "string-ofpearls" (e.g., small cystic spaces lined up in a row under the ovarian surface) on ultrasound examination is also typical. Dermoid cysts (benign ovarian teratomas that may produce hair and skin, choice A), hydrosalpinx (pathologically dilated fallopian tube, choice B), and cyst adenocarcinomas (a form of malignant ovarian tumor that does not usually secrete hormones, choice C) can cause adnexal masses, but would not produce the hirsutism or acanthosis nigricans seen in this case. Leiomyomas (choice D) are solid uterine tumors.

Question 4 of 4

Which of the following most accurately describes the pathophysiology of the adnexal masses present in this patient's condition? / / / / /

A. Areas of focal abscess formation B. Areas of focal necrosis C. Follicles in various stages of maturation D. Small cysts within a benign tumor E. Small cysts within a malignant tumor Explanation - Q: 2.4

Close

The correct answer is C. The cysts that give polycystic ovary disease its name are actually ovarian follicles in varying states of maturation. One of the underlying mechanisms leading to the formation of the polycystic ovaries appears to be a benign, nontumorous, proliferation of ovarian stromal tissue under the influence of a high androgen state, which then makes it physically much harder for a follicle to rupture the ovarian surface and release its egg. Any menstrual cycle in which the egg fails to leave the ovary is anovulatory, and has no chance of conception on that cycle. Further, the corpus luteum fails to develop, and the progesterone levels remain low rather than rising. The cysts are not related to true tumor (choices D and E), necrosis (choice B), or abscess formation (choice A).

A 38-year-old woman is seen by a gynecologist on a routine visit. A complete history is taken and is non-contributory except for the fact that the patient's menstrual irregularities apparently developed insidiously over the preceding decade. Screening physical examination is performed. Routine serum chemistries and complete blood count are sent, as are Pap smear studies. Pelvic ultrasound studies are also performed. Pregnancy test is negative. Endometrial biopsy shows proliferative endometrium. AII other results are within normal limits, except for evidence of a mild iron deficiency anemia.

Question 1 of 3

Which of the following is the most likely diagnosis? / A. Cervical carcinoma / B. Cervical dysplasia / C. Dysfunctional uterine bleeding / D. Dysfunctional uterine bleeding / E. Turner syndrome Explanation - Q: 3.1

Close

The correct answer is C. This patient most likely has dysfunctional uterine bleeding. Other, potentially more serious, diseases have been mostly excluded by the patient's extensive evaluation (more than many patients receive in this setting). Cervical dysplasia or carcinoma (choices A and B) usually produce abnormal Pap smears, and sometimes produce an abnormal vaginal examination. Endometrial carcinoma (choice D) would usually be evident in either the endometrial biopsy or on the pelvic ultrasound examination. Patients with the genetic Turner syndrome (choice E) do not menstruate. Question 2 of 3

In most patients with this patient's disease, the condition is etiologically related to which of the following? / A. Anovulatory cycles / B. High androgen levels / C. High progesterone levels / D. Persistent corpus luteum / E. Persistent secretory endometrium

Explanation - Q: 3.2

Close

The correct answer is A. Approximately 90% of patients with dysfunctional bleeding have anovulatory menstrual cycles. In these patients, the failure of ovulation leads to a failure of the corpus luteum to form (compare with choice D), and consequently a failure of normal progesterone secretion (compare with choice C) and a failure to develop secretory endometrium (compare with choice E). Estrogen levels remain high but androgen levels (choice B) do not usually play a significant role. The unopposed estrogen secretion stimulates endometrial proliferation, and the thickened endometrium eventually outgrows its blood supplies and begins to die, producing the prolonged menstrual blood flow. In the remaining 10% of patients who have dysfunctional uterine bleeding and ovulatory cycles, the prolonged progesterone secretion is associated with irregular endometrial shedding. Some of these latter patients also have polycystic ovary disease and may have abnormal androgen metabolism.

Question 3 of 3

The physician decides to treat this patient with an oral drug that will stop endometrial growth and support and organize the endometrium to allow organized sloughing on the next menstrual cycle after withdrawal of the drug. Which of the following is the most appropriate pharmacotherapy? / A. Estrogens, conjugated / B. GIyburide / C. Hydralazine hydrochloride / D. Levothyroxine / E. Medroxyprogesterone acetate Explanation - Q: 3.3

Close

The correct answer is E. Medroxyprogesterone acetate (Provera) is the drug of choice for most patients with anovulatory dysfunctional uterine bleeding. It does not control the acute episode of bleeding, but it does allow the establishment of a more stable endometrium, and the bleeding following the withdrawal of the Provera usually has a more normal character. Conjugated estrogens (choice A) are also used to treat dysfunctional uterine bleeding, and in pharmacologic doses, cause the rapid growth of endometrial tissue over a denuded epithelial surface. Glyburide (choice B) is a sulfonylurea drug that is used as an anti-diabetic agent.

Hydralazine (choice C) is an antihypertensive. Levothyroxine (choice D) is used to replace thyroid hormone.

A 7-year-old Caucasian girl is evaluated by a pediatrician because she has just begun to menstruate. The mother states that her daughter started developing breasts at age five. Question 1 of 6

In the normal physiology of puberty, which of the following is the earliest change seen, indicating that puberty is beginning physiologically? / A. Appearance of pulsatile LH release during sleep / B. Circulating FSH Ievels increase progressively / C. Difference between sleeping and waking LH secretory patterns disappears / D. Increase in circulating estradiol / E. Serum prolactin concentrations increase modestly Explanation - Q: 4.1

Close

The correct answer is A. While the physical changes of puberty are most obvious to the observer, puberty progresses under the control of a variety of hormones. The earliest measurable change to herald the onset of puberty is a pulsatile release of LH (luteinizing hormone, from the pituitary) during sleep. It has been speculated that this is linked to a pulsatile release of gonadotropin releasing hormone (GnRH,) from the hypothalamus. As puberty continues, these LH peaks increase in frequency and amplitude, and also begin to be found in the waking period, until by the end of puberty, the waking and sleeping patterns of LH release are very similar (choice C). The response of LH to GnRH (gonadotropin-releasing hormone, from the hypothalamus) also increases through puberty. FSH (follicle stimulating hormone) levels increase less markedly than do LH levels, but do progressively increase (choice B). The rising levels of LH and FSH also trigger an increase in circulating estradiol (choice D), which is responsible for the development of secondary sexual characteristics in a female. Serum prolactin concentrations also increase modestly during puberty (choice E). Question 2 of 6

On physical examination, the girl is noted to have large breast mounds for her age with an approximately 4 inch diameter and 1 1/2 inch high palpable breast mound. No secondary mound is seen. This child's breasts should be classified as which of the following Tanner breast stages? / A. Stage 1

/ / / /

B. Stage 2 C. Stage 3 D. Stage 4 E. Stage 5 Explanation - Q: 4.2

Close

The correct answer is C. Sexual development is usually classified in terms of the Tanner stages. This child has tanner stage 3 breasts. Tanner stage 1 breasts (choice A) are preadolescent breasts in which only the papillae are elevated. Tanner stage 2 breasts (choice B) would be smaller than this child's breast and have an elevated bud and papilla with a small mound and increased areolar diameter. Tanner stage 3 breasts (choice C), such as this child has, are larger than stage 2 breasts and have increased palpable glandular tissue. Tanner stage 4 breasts (choice D) differ from stage 3 breasts in that the areola and papilla are elevated to form a second mound above the level of the rest of the breast. Tanner stage 5 breasts (choice E) are mature adult breasts (and much larger than stage 3 breasts), in which the areola recesses to the mound of breast tissue, with projection of only the papilla being evident.

Question 3 of 6

The child's pubic area shows a small area of moderately dark, slightly coarse, curly hair along the labia to the pubic junction. The hair is more mature than vellous hair, but does not have a completely adult texture. This child's pubic hair should be classified as which of the following Tanner pubic hair stages? / A. Stage 1 / B. Stage 2 / C. Stage 3 / D. Stage 4 / E. Stage 5

Explanation - Q: 4.3

Close

The correct answer is C. This child's pubic hair is also in Tanner stage 3. Stage 1 (choice A, preadolescent) is vellus hair only (similar to that on the anterior abdominal wall). Stage 2 (choice B) shows sparse growth of long, slightly pigmented, downy hair. These hairs develop along the labia. Stage 3 (choice C), this child's stage, is characterized by darker, coarser, a little curled, and is found up to the pubic junction. Stage 4 (choice D) is adult-type hair, but with a smaller distribution than in most adults, and with no spread to the medial surface of the thighs. Stage 5 (choice E) is characterized by adult-type hair distributed as an inverse triangle with spread to the medial surface of the thighs.

Question 4 of 6

Which of the following is the most likely diagnosis? / A. Polycystic ovary syndrome / B. Precocious puberty / C. Primary amenorrhea / D. Secondary amenorrhea / E. Turner syndrome Explanation - Q: 4.4

Close

The correct answer is B. Caucasian girls usually begin to develop breasts, and then pubic hair between the ages of 8 and 13 years. Most girls beginning menstruating at 12 to 13 years of age. This child has an early pubarche (beginning of genital hair changes), an early thelarche (beginning of breast development), and an early menarche (beginning of menstruation), consistent with precocious puberty. Precocious puberty is the term used for the appearance of secondary sexual maturation in Caucasian girls younger than 7 years, African-American girls younger than 6 years, or boys younger than 9 years. Precocious puberty is much more common in girls than in boys. Approximately 80% of cases of precocious puberty in the United States are due to a premature activation of the hypothalamic-pituitary-gonadal axis. The term "central precocious puberty" is sometimes used for these cases, which are contrasted with cases of "precocious pseudopuberty" [also known as "gonadotropin-independent precocious puberty" and still classified as "precocious puberty" by many authors] in which the sexual development

occurs as a consequence of other disease. Polycystic ovary syndrome (choice A) can be associated with virilism in adult women, but is not associated with precocious puberty. Amenorrhea refers to a failure to menstruate, either never (primary amenorrhea, choice C) or after having previously started (secondary amenorrhea, choice D). Turner syndrome (choice E) has XO genetics and is associated with primary amenorrhea.

Question 5 of 6

Which of the following would be the most appropriate pharmacotherapy? / A. Epoetin / B. Dapsone / C. GIycopyrrolate / D. Leuprolide / E. Pindolol Explanation - Q: 4.5

Close

The correct answer is D. Precocious puberty is usually medically treated, primarily to prevent social ostracization of the child and also to allow time for the patient's height to reach near normal to normal adult height (growth in stature often ends when puberty does). The mainstay of therapy is now the gonadotropin-releasing hormone agonists, including leuprolide and nafarelin. These medications successfully suppress LH and FSH levels (and thus ovarian and testicular steroidogenesis) because the pituitary responds to pulsatile GnRH cues, rather than constant ones. Epoetin (choice A) is the pharmacologic name for erythropoietin, used to stimulate red cell growth. Dapsone (choice B) is an antimicrobial agent used in leprosy. Glycopyrrolate (choice C) is an anticholinergic medication used to suppress GI secretions. Pindolol (choice D) is an antihypertensive agent.

Question 6 of 6

In some children, a similar condition can be part of a syndrome that is also characterized by polyostotic fibrous dysplasia and café-au-Iait skin pigmentation. Which of the following is the most likely diagnosis in these individuals? / A. CREST syndrome / B. Crigler-Najjar syndrome / C. Cushing syndrome / D. Dandy-Walker syndrome / E. McCune-AIbright syndrome Explanation - Q: 4.6

Close

The correct answer is E. McCune-Albright syndrome is a rare syndrome characterized by fibrous dysplasia (a type of bony defect), café-au-lait skin pigmentation (also seen in neurofibromatosis), and autonomous endocrine hyperfunction. The endocrine abnormalities most commonly take the form of gonadotropin-independent precocious puberty, but may also include hyperthyroidism, hypercortisolism, and pituitary gigantism. These patients are also vulnerable to chronic liver disease and sudden death (possibly related to cardiac arrhythmias). CREST syndrome (choice A) is a variant of scleroderma with calcinosis, esophageal motility problems, telangiectasias, Raynaud's phenomenon, and sclerodactyly. Crigler-Najjar syndrome (choice B) is a hereditary defect in bilirubin metabolism. Cushing syndrome (choice C) refers to the physical problems seen as a result of hypercortisolism. Dandy-Walker syndrome (choice D) is a congenital malformation of the brain.

A 47-year-old man presents to a psychiatrist with signs and symptoms of depression. His son, who brought him in, said that he has become absentminded, and has difficulty concentrating or remembering things. During the session, the patient makes sudden jerky movements with his hands and grimaces frequently. The psychiatrist also notes that the patient has problems with coordination and balance, and observes stuttering and slightly slurred speech during the interview. The man's mother is a healthy, 75-year-old retired saleswoman; his father committed suicide at the age of 60. What are the major characteristics of this patient's disease? / A. Anterograde amnesia, ataxia, ophthalmoplegia / B. Autosomal dominant, choreiform movements, and behavioral changes / C. Autosomal recessive, progressive dementia, hyperoral behavior / D. Progressive memory loss, progressive aphasia, impaired executive function / E. Tremor, rigidity, bradykinesia Explanation - Q: 1.1

Close

The correct answer is B. This patient suffers from Huntington disease (chorea). Huntington disease (HD) is a progressive neurodegenerative disease primarily affecting neurons in the basal ganglia. HD affects about 1 in 10,000 individuals, and is transmitted in an autosomal dominant fashion. Its average onset is from 30-40 years of age, and the duration of the disease is typically about 15 years, but shows a wide range. Early onset (juvenile HD) is associated with more rapid progression, rigidity, and ataxia. The main motor features of HD are irregular, sudden limb and facial jerks. The disease may begin with "piano-playing" movements of the fingers or slight facial twitching, but symptoms are slowly progressive and become uncontrollable. The speech slowly becomes incomprehensible and swallowing is difficult. The gait is poorly coordinated, with a dancing-like (choreiform) pattern. Although patients appear to be off-balance, the ability to balance is actually well-preserved. Depression, apathy, social withdrawal, and irritability are common. Memory is typically affected at later stages of the disease. Since HD is incurable, the ultimate result is death. The family history is relevant, since with autosomal dominant inheritance, the chance of inheriting the Huntington gene is 50%. The patient's mother does not have the disease, so the patient's father must have had it. A history of family members who commit suicide in middle-age is often associated with HD. Wernicke-Korsakoff syndrome is due to profound thiamine deficiency. Initially, confusion, ataxia, and ophthalmoplegia dominate the clinical picture, but if the thiamine deficiency is not corrected, an amnestic syndrome with predominantly anterograde amnesia supervenes (choice A). Autosomal recessive inheritance, progressive dementia, hyperoral behavior, emotional disinhibition, and language disturbances are clinical features of

Pick disease (choice C). Alzheimer disease is the most common cause of dementia, in which cognitive and behavioral impairment interfere with social and occupational functioning. Although, other neurological systems can be affected (e.g., impaired visuospatial skills and spastic paraparesis), as the disease progresses, the most prominent feature is progressive memory loss (choice D). Parkinson disease is characterized by a combination of tremor, rigidity, bradykinesia, and a distinctive disturbance of gait and posture (choice E).

Where are the primary sites of cell degeneration in this patient's disease? / A. Basal ganglia and thalamus / B. Frontal and temporal cortical regions / C. Neostriatum and cerebral cortex / D. Substantia nigra pars compacta and locus coeruleus / E. Upper and lower motor neurons Explanation - Q: 1.2

Close

The correct answer is C. Cell death in the caudate and putamen (neostriatum) produces the chorea. Release of the globus pallidus from striatal inhibition results in suppression of subthalamic activity, which may also contribute to the choreiform movements. Impaired cognitive function and ultimate dementia is caused by the loss of cerebral cortical cells, and possibly, by disruption of the cognitive portions of the basal ganglia. Atrophy of the caudate nuclei can be seen using neuroimaging studies in the midand late stages of the disease, while more diffuse cortical atrophy is visible late in the disease. Microscopically, there are no significant pathologic features other than some gliosis. Histochemically, there is a marked decrease of GABA and its synthetic enzyme glutamic acid decarboxylase throughout the basal ganglia. The levels of other transmitters, including substance P and enkephalins, are also decreased. Pathologic changes of cavitary degeneration in the basal ganglia with extensive gliosis and neuronal loss, in association with a marked decrease of the copper content in this region of the brain are characteristic of Wilson disease. Similar changes may be seen in the thalamus (choice A). Pick disease is defined pathologically by severe atrophy, neuronal loss, and gliosis. Swollen neurons and argentophilic neuronal inclusions (Pick bodies) affect the frontal and temporal cortical areas (choice B). In Parkinson disease, loss of substantia nigra pars compacta neurons leads

to the depletion of dopamine in the striatum, which ultimately decreases thalamic excitation of the motor cortex. Pigmented neurons of the locus coeruleus also degenerate (choice D). Amyotrophic lateral sclerosis is ultimately a diffuse disease, but the onset is often focal and asymmetric. At the onset, bulbar motor neurons can be involved, or spinal anterior horn cells can be affected. Later, spread to other motor areas produces the classic combination of upper and lower motor neuron degeneration (choice E).

Which of the following is the most appropriate pharmacotherapy for this patient? / A. Acetylcholinesterase inhibitors / B. Anticholinergics / C. Dopamine antagonists / D. Dopamine precursors / E. Thiamine Explanation - Q: 1.3

Close

The correct answer is C. Although, no therapy is currently available to delay the onset of symptoms or to prevent the progression of the disease, symptomatic treatment of HD patients may improve the quality of life and prevent complications. Dopamine antagonists (antipsychotic agents) are used for the psychosis, paranoia, and delusional states that occur in HD, although they are used in lower doses than is often required for primary psychiatric disorders. Antipsychotic agents may also improve choreic movements in patients with HD. HD patients on antipsychotics have to be monitored, because the drugs can worsen the rigidity seen in these patients. Another approach is to decrease levels of dopamine via reserpine, although patients need to be monitored for hypotension and depression. Tetrabenazine, another dopamine-depleting agent, seems to be the most effective suppressant of choreiform movements, but this drug is not available in the U.S. and is categorized as investigational. Other possibly useful drug treatments in HD are SSRIs and carbamazepine for depression. Centrally acting acetylcholinesterase inhibitors (e.g., rivastigmine, galantamine, donepezil, tacrine) (choice A) are used in the treatment of Alzheimer disease. Anticholinergics (choice B), such as benztropine or trihexyphenidyl, are used in Parkinson disease. L-dopa, a dopamine precursor (choice D), is used in Parkinson disease. Thiamine (choice E) is used in Wernicke-Korsakoff syndrome.

Which of the following genes is involved in the pathogenesis of this disease? / A. ATP7B, Iocated on chromosome 13 / B. D23116, Iocated on chromosome 2q33-q34 / C. IT15, Iocated on chromosome 4p / D. N-acetyI-transferase 2, Iocated on chromosome 8 / E. 6p21, Iocated on chromosome 6 Explanation - Q: 1.4

Close

The correct answer is C. Huntington disease is a genetic disorder of the central nervous system, and is inherited as an autosomal dominant condition. In affected individuals, one gene of the gene pair is not functioning properly and "dominates" the other working gene. The HD gene, called IT15, is located on chromosome 4p. The gene responsible for Wilson disease is located on chromosome 13, and it is called ATP7B (choice A). Many familial cases of amyotrophic lateral sclerosis are linked to the mutations in the gene region on chromosome 2 (choice B) that codes for superoxide dismutase. The gene that has the strongest connection with Parkinson disease is Nacetyl-transferase 2 located on chromosome 8 (choice D), which codes for the enzyme responsible for degradation of toxins in the body. Region 6p21 on chromosome 6 (choice E) has been strongly linked to multiple sclerosis.

DNA samples from the patient and his unaffected mother are amplified by the polymerase chain reaction (PCR) using primers specific for the 5' coding region of the gene involved in his disease. Gel electrophoresis of their PCR products is shown on the left in the diagram.

If DNA from his deceased father had been available for testing, which option represents the most likely PCR pattern of the father? / A. A / B. B / C. C / D. D / E. E Explanation - Q: 1.5

Close

The correct answer is A. Huntington disease is caused by the rapid expansion of a triplet repeat (CAG) in the 5' coding region of the IT15 gene for huntingtin. The normal huntingtin allele has 5 adjacent CAG codons encoding 5 adjacent glutamine residues in the protein. In families with Huntington disease, the CAG sequence has expanded somewhat with each generation, and the polyglutamine tract in the huntingtin protein is longer. One would most likely expect the expanded CAG sequence to be shorter in the father than in his son, and the corresponding PCR-amplified fragment to migrate faster (lower on the gel) during electrophoresis. Choices B, C, D, and E might represent results of a paternity test to determine whether the man was his father. In this case, the PCR products would be generated to amplify a region with a known polymorphic marker, often a short tandem repeat (microsatellite). The patterns (B and C) would be consistent with the man as father, while patterns (D and E) would be inconsistent with the man being his father.

The class of mutation causing this disease is often associated with a characteristic feature in a family pedigree. Which of the following is most likely to be a characteristic of pedigrees caused by this class of mutation? / A. AIIelic heterogeneity / B. Anticipation / C. Consanguinity / D. Imprinting

/ E. Multifactorial inheritance

Explanation - Q: 1.6

Close

The correct answer is B. In diseases caused by triplet repeat expansions, the symptoms worsen and the age of onset decreases in successive generations, a characteristic known as anticipation. Important triplet repeat expansion diseases include Huntington disease, Fragile X syndrome, and myotonic dystrophy. In each, the pedigree typically shows anticipation. Allelic heterogeneity (choice A) indicates that different types of mutations in a gene cause a particular disease. For example, there are over 400 mutations in the glucose 6-phosphate dehydrogenase gene known to cause some form of hemolytic anemia. Each family would generally have only one of these mutations. All cases of Huntington disease are caused by a triplet repeat expansion in the same 5' locus. Consanguinity (choice C) is a more common feature of autosomal recessive diseases, in which two copies of an affected gene must be inherited to produce the disease phenotype. This is more likely when there are incestuous or consanguineous (between first or second cousins) matings. Genetic imprinting (choice D) occurs when a gene is normally expressed only from either the paternally inherited, or from the maternally inherited chromosome. In such a case, if the normally active gene is deleted from the chromosome, a disease such as Prader-Willi (deletion in paternal 15) or Angelman syndrome (deletion in maternal 15) may occur. In Huntington disease, for unknown reasons, the triplet usually expands more rapidly when the affected chromosome is passed through a male, but this is not imprinting. In a family with Huntington disease, and in normal families, both IT15 alleles on the maternal and the paternal chromosomes are active (expressed). The preferential expansion of the triplet in Fragile X is through the mother, and in the more severe forms of myotonic dystrophy, through the mother. Multifactorial inheritance (choice E) refers to a disease in which there are many factors contributing to phenotypic disease. Multiple genes may be involved, as well as nongenetic environmental factors. Coronary artery disease and cancer would be examples of multifactorial diseases. Huntington disease is solely attributed to the triplet repeat expansion in the IT15 gene.

The patient is sent for imaging studies. Which of the following will most likely be found? / A. Decrease in 18F-dopa uptake / B. Diffuse corticaI/cerebral atrophy / C. Enlarged lateral ventricles

/ D. Hypodensities in the putamen / E. T2 hyperintensities

Explanation - Q: 1.7

Close

The correct answer is C. Enlarged, dilated lateral ventricles are visible on head CT scan. CT scan or MRI may reveal loss of the normally convex prominence of the caudate nucleus into the lateral ventricles. Although no single imaging technique is necessary for diagnosis, in patients with typical symptoms, a CT scan strongly supports the diagnosis of Huntington disease. A reliable imaging marker of this disease is measurement of the bicaudate diameter by MRI or CT scan. Other imaging studies e.g., positron emission tomography (PET) scanning and proton magnetic resonance spectroscopy may show abnormalities, but they are used very rarely as a part of the workup for HD. PET decrease in 18F-dopa uptake (choice A) in the contralateral putamen is characteristic for Parkinson disease. Diffuse cortical/cerebral atrophy (choice B) is seen on brain MRI in patients with Alzheimer disease. CT of the head showing hypodensities in the putamen (choice D) is seen in patients with Wilson disease. Typical multiple sclerosis lesions appear as T2 hyperdensities (choice E) in the periventricular regions.

A 63-year-old man experiences the sudden onset of a right facial droop and an inability to speak, while talking to his niece. His niece, who is a physician, rushes him to the local emergency department. On arrivaI, his vital signs are within normal Iimits. On examination, he cannot speak, and he is noted to have a marked right facial droop and right arm weakness. An emergent head CT scan does not reveal any abnormalities. This patient's disease most likely involves which of the following arteries? A. Left anterior cerebral artery B. Left middle cerebral artery C. Left posterior cerebral artery D. Right posterior inferior cerebellar artery E. Right superior cerebellar artery

/ / / / /

Explanation - Q: 2.1

Close

The correct answer is B. The middle cerebral artery supplies the lateral convexity of the cerebral hemisphere and the underlying insula. It supplies both Broca's and Wernicke's speech areas in the dominant hemisphere. It also supplies the face, arm, and trunk areas of the motor and sensory homunculus. Occlusion can result in right-sided face and arm weakness and aphasia, as seen in this patient. An occlusion of the left anterior cerebral artery (choice A) would result in right lower extremity weakness. The anterior cerebral artery supplies the medial surfaces of the frontal and parietal lobes, as well as the corpus callosum. An occlusion of the left posterior cerebral artery (choice C) would result in a contralateral hemiparesis and a right homonymous hemianopsia. The posterior cerebral artery supplies the midbrain, the posterior half of the thalamus, the occipital lobe, and the inferior surface of the temporal lobe. An occlusion of the right posterior inferior cerebellar artery (choice D) would result in a lesion in the lateral tegmentum of the medulla, called a Wallenberg syndrome (lateral medullary syndrome). This is characterized by ipsilateral palsies of cranial nerves V (ipsilateral hemifacial numbness), IX (ipsilateral decreased gag and taste), X (dysphagia and hoarseness), and XI, as well as an ipsilateral Horner syndrome, cerebellar ataxia, and nystagmus. There would also be a loss of pain and temperature sense over the contralateral body. Occlusion of the right superior cerebellar artery (choice E) results in a lateral superior pontine syndrome. This is characterized by ipsilateral ataxia (due to lesions of the superior and middle cerebellar peduncles), dysmetria (due to lesion of the dentate nucleus), contralateral loss of pain and temperature (due to lesions in the spinothalamic and trigeminothalamic tracts), ipsilateral Horner syndrome, and contralateral loss of proprioception and vibration sense in the trunk and leg.

Which of the following would the patient most likely be able to do on further evaluation? / A. Name objects / B. Raise his eyebrows symmetrically / C. Repeat words / D. Stick out his tongue without deviation / E. Write words Explanation - Q: 2.2

Close

The correct answer is B. Corticobulbar fibers have bilateral input to the upper face division of the facial nucleus, while the lower face division of the facial nucleus has contralateral input. Therefore, an upper motor neuron lesion involving these corticobulbar fibers, such as a cortical stroke, would result in weakness of the lower half of the contralateral face, while sparing the upper half of the contralateral face due to this bilateral input. A lower motor neuron lesion involving the facial nucleus or the facial nerve (i.e., a Bell's palsy) would result in weakness of the entire ipsilateral half of the face. The ability to name objects (choice A) requires an intact Broca's area, which seems to be involved in the patient's stroke, since he is unable to speak. The ability to repeat words (choice C) requires an intact Wernicke's area, Broca's area, and arcuate fasciculus, any one or all of which would be involved in a left-sided middle cerebral artery distribution stroke. The patient described above clearly has difficulty speaking, and would most likely be unable to repeat words. The ability to stick out his tongue without deviation (choice D) would most likely not be preserved in this patient with a stroke resulting in contralateral face and arm weakness. The tongue area is adjacent to the face area on the motor homunculus, and a lesion affecting one would most likely affect both. This would result in an upper motor lesion, and would cause the tongue to deviate away from the side of the lesion. The ability to write words (choice E) requires an intact Broca's area, which seems to be involved in the patient's stroke, since he is unable to speak.

In the emergency department, it is decided that the patient is a candidate for emergent pharmacologic thrombolysis. Which of the following is the mechanism of action of this treatment? / A. Activates antithrombin lll / B. Facilitates the conversion of plasminogen to plasmin / C. Impairs the synthesis of vitamin K-dependent clotting factors / D. Irreversibly inhibits cyclooxygenase / E. Irreversibly inhibits the ADP pathway involved in fibrinogen binding Explanation - Q: 2.3

Close

The correct answer is B. Thrombolytics, such as t-PA and urokinase, aid in the conversion of plasminogen to plasmin, which then goes on to cleave thrombin and fibrin clots and actively lyse clots. It has been shown that giving tPA within 3 hours of the onset of a stroke can improve outcome and is approved by the US Food and Drug Administration for treating acute

ischemic stroke within 3 hours of onset of symptoms. Activation of antithrombin III (choice A) is the mechanism of action of heparin, which is an anticoagulant, not a thrombolytic. It prevents the formation of new blood clots, but does not actively lyse formed clots. It is given parenterally, and has a rapid onset of action. Impairing the synthesis of vitamin K-dependent clotting factors (choice C) is the mechanism of action of warfarin. Warfarin is an anticoagulant, not a thrombolytic. It prevents the formation of new blood clots, but does not actively lyse formed clots. Unlike heparin, warfarin is given orally, and has a slow onset of action. Irreversibly inhibiting cyclooxygenase (choice D) is the mechanism of action of aspirin. This inhibits platelet aggregation and prevents clot formation. It has a slow onset of action, and does not actively lyse formed clots. Irreversibly inhibiting the ADP pathway involved in fibrinogen binding (choice E) is the mechanism of action of ticlopidine. It inhibits platelet aggregation and prevents clot formation. It has a slow onset of action and does not actively lyse formed clots.

After emergent pharmacologic thrombolysis, the patient's symptoms quickly resolve. However, he begins complaining of a headache and nausea. A head CT reveals a 2 cm x 2 cm blood clot in the patient's right cerebellar hemisphere. What would be the best therapy to reverse the patient's coagulopathy? / A. Aminocaproic acid / B. Dialysis / C. PIatelets / D. Protamine / E. Vitamin K Explanation - Q: 2.4

Close

The correct answer is A. Aminocaproic acid is useful in enhancing hemostasis when fibrinolysis contributes to bleeding. The antifibrinolytic effects of aminocaproic acid appear to be exerted principally via inhibition of plasminogen activators, and to a lesser degree through antiplasmin activity. Dialysis (choice B) would not correct the thrombolytic effect on fibrinolysis. Platelets (choice C) would not correct the thrombolytic effect on fibrinolysis. Protamine (choice D) is used to reverse the anticoagulant action of heparin, which is not a thrombolytic. Protamine combines ionically with heparin to

form a stable complex devoid of anticoagulant activity. Vitamin K (choice E) is the antidote for warfarin toxicity. What would be the most likely pronounced neurologic deficits that would be observed in this patient with a right cerebellar hemorrhage? / A. Contralateral dyscoordination / B. Contralateral hemiparesis / C. Ipsilateral dyscoordination / D. Ipsilateral hemiparesis / E. Truncal ataxia Explanation - Q: 2.5

Close

The correct answer is C. An ipsilateral cerebellar lesion causes ipsilateral dyscoordination because it sends efferents from the dentate nucleus to the contralateral cerebral cortex by way of crossed fibers. Contralateral dyscoordination (choice A) would not result from a hemispheric cerebellar lesion. Contralateral hemiparesis (choice B) would not result from a hemispheric cerebellar lesion. It could result from an upper motor neuron lesion proximal to the medullary decussation. Ipsilateral hemiparesis (choice D) would not result from a hemispheric cerebellar lesion. It could result from an upper motor neuron lesion distal to the medullary decussation. Truncal ataxia (choice E) would be seen with a vermian lesion.

A 15-year-old boy is taken to a psychiatrist because he has had a recent sudden deterioration in his schoolwork and a recent marked increase in aggressive behavior. The psychiatrist performs a physical examination that demonstrates a mild hand tremor, partial loss of the gag reflex, and impaired coordination. He additionally demonstrates hepatomegaly and mild jaundice. Which of the following diseases would be most likely to cause both hepatic and neurologic dysfunction? / A. AIzheimer disease / B. Huntington disease / C. Parkinson disease / D. Sydenham chorea / E. Wilson disease

Explanation - Q: 3.1

Close

The correct answer is E. All of the diseases listed have prominent neurologic manifestations, but only Wilson disease also usually affects the liver. Wilson disease is an autosomal recessive disease with an incidence of 1 in 30,000 that commonly presents in adolescence. The patient may come to medical attention because of psychiatric symptoms of all degrees (school performance deterioration, suicidal or homicidal impulses, depression, and rarely, illness mimicking schizophrenia or manic-depressive illness), neurologic dysfunction (tremors, difficulty walking, difficulty talking, or difficulty swallowing), or hepatic problems (jaundice, hepatomegaly, or abdominal pain). The metabolic basis of Wilson disease is an abnormality of copper metabolism, which leads to accumulation of toxic amounts of copper. One of the sites where copper accumulates is the liver, and biopsy of liver followed by chemical analysis may show copper concentrations greater than 250 micrograms/gram dry weight. Alzheimer disease (choice A) characteristically causes mental deterioration in the elderly. Huntington disease (choice B) is an autosomal dominant disease with intellectual deterioration and movement disorder that characteristically manifests in middle age. Parkinson disease (choice C) is an idiopathic progressive disorder with decreased ability to move and tremor, which usually manifests in late middleaged or older individuals. Sydenham chorea (choice D) is a motor disorder that can complicate rheumatic fever.

Measurements of the serum level of which of the following would be most likely to contribute to establishing the patient's diagnosis? / A. Carcinoembryonic antigen / B. Ceruloplasmin / C. Ferritin / D. Lead / E. Myelin basic protein Explanation - Q: 3.2

Close

The correct answer is B. Ceruloplasmin is a copper-containing serum enzyme that is present in diminished levels in both patients with, and carriers

of Wilson disease. Since the enzyme can be deficient in both heterozygotes (with a carrier frequency of 1 in 90 and no progression to clinical disease) and homozygotes, the establishment of deficient ceruloplasmin does not, in itself, establish the diagnosis. However, the diagnosis can be established if this information is taken together with other evidence of abnormal copper metabolism, such as elevated liver copper, elevated urinary excretion of copper, or the presence of Kayser-Fleischer rings on ocular examination. Carcinoembryonic antigen (choice A) is a tumor marker that is most often clinically useful in following patients with colon or lung cancer. Ferritin (choice C) is an iron-protein complex used to transport iron from the gastrointestinal tract to the tissues, and may be markedly elevated in hemochromatosis. Lead (choice D) poisoning can cause cognitive defects in children, and encephalopathy and progressive renal disease in adults. Tremor, such as this patient has, it not usually a feature. Myelin basic protein (choice E) is characteristically elevated in CSF fluid in patients with multiple sclerosis. Which of the following findings on ocular examination would be considered most specific for this patient's probable disease? / A. Corneal ulcers / B. Kayser-FIeischer rings / C. Macular degeneration / D. Papilledema / E. Retinal detachment Explanation - Q: 3.3

Close

The correct answer is B. Kayser-Fleischer rings are deposits of copper in Descemet's membrane of the cornea, which can be seen as gold or greenish-gold rings and crescents on slit-lamp ophthalmologic examination. They are considered pathognomic for Wilson disease, and the disease is sometimes picked up initially by an ophthalmologist who notes the presence of the characteristic rings. Corneal ulcers (choice A) may be due to trauma, autoimmune disease, or infection of the eye. Macular degeneration (choice C) is a common aging problem of the eye.

Papilledema (choice D) suggests increased intracranial pressure. Retinal detachment (choice E) can be due to trauma or hemorrhage behind the retina.

Which of the following pharmacologic agents would be useful to this patient by decreasing intestinal absorption of the substance causing this patient's disease? / A. Deferoxamine / B. Dimercaprol / C. Penicillamine / D. Trientine / E. Zinc acetate Explanation - Q: 3.4

Close

The correct answer is E. Zinc acetate has been widely used elsewhere in the world, and recently the FDA approved it for use in the United States. It induces an intestinal metallothionein, which binds copper, and prevents its absorption into the blood stream. It appears to be an important new addition, with fewer side effects than chelating agents, for the chemotherapy of Wilson disease. Deferoxamine (choice A) is used to chelate iron. Trientine (choice D), penicillamine (choice C), and dimercaprol (choice B) are chelating agents that increase copper excretion.

A 58-year-old left-handed man is referred to a neurologist for "involuntary left hand twitches." Between six months and a year ago, he first noticed that when his left hand was resting, it would shake. He can stop the shaking by looking at his hand and concentrating. The shaking does not impair his activities in any way. He has no trouble holding a glass of water. There is no tremor in his right hand and his lower extremities are not affected. He has had no trouble walking and there have been no falls. There have been no behavioral or language changes. On examination, a left hand tremor is evident when he is distracted. Handwriting appears small and cramped. He has bilateral cogwheel rigidity with contralateral activation, which is worse on the left. Rapid alternating movements are bradykinetic on the left. Which of the following is the most likely diagnosis? / A. Epilepsy

/ / / /

B. Guillain-Barr syndrome C. Multiple sclerosis D. Parkinson disease E. Stroke Explanation - Q: 4.1

Close

The correct answer is D. The tremor is of a Parkinsonian type. He also has the classic findings of Parkinson disease (PD), which are asymmetric tremor, rigidity, and bradykinesia. Mask-like facies, drooling, tremors, pill-rolling motion, cogwheel rigidity, and a shuffling gait all may be present. This disease results from the degeneration of the dopaminergic neurons in the substantia nigra pars compacta of the midbrain. Epilepsy (choice A) is characterized by repeated unprovoked seizures. Hand shaking can be a focal motor seizure, but the presentation overall makes epilepsy an unlikely diagnosis. Guillain-Barré syndrome (choice B) is a peripheral demyelinating disease, which usually presents as an ascending motor deficit. Multiple sclerosis (choice C) is a central nervous system (CNS) demyelinating disease. It presents with individual episodes of CNS deficits, which usually recover to some extent. Stroke (choice E) is characterized by the acute onset of a neurological deficit and is due to infarction of neural tissue. Tremor would be an exceedingly rare presentation for stroke, and it would not evolve over 6-12 months.

Which of the following would be the most appropriate pharmacotherapy for this patient? / A. AIteplase / B. Carbamazepine / C. Carbidopa-Ievodopa / D. GIatiramer / E. Interferon beta-1A / F. Sertraline Explanation - Q: 4.2

Close

The correct answer is C. Parkinson disease symptoms are due in large part to dopamine depletion. Carbidopa-levodopa can replete dopamine and alleviate symptoms. Levodopa is a precursor to dopamine that is

decarboxylated by dopa-decarboxylase. Inhibition of the peripheral form of this enzyme by carbidopa allows greater amounts of the precursor to reach the brain, where it is converted to dopamine by the uninhibited brain decarboxylase. Alteplase (recombinant tPA; choice A) is used to dissolve blood clots during acute strokes or heart attacks. Carbamazepine (choice B) is an anticonvulsant. Glatiramer (choice D) and interferon beta-1A (choice E) are used to treat multiple sclerosis. They have been shown to decrease attacks. Both are thought to work through immunomodulation. Sertraline (choice F) is a selective serotonin reuptake inhibitor (SSRI). By increasing serotonin concentrations, it is effective for the treatment of depression.

Another drug prescribed for this patient is tolcapone. Which of the following is the mechanism of action of this agent? / A. COMT inhibitor / B. Dopamine agonist / C. Dopamine uptake blocker / D. MAO inhibitor / E. Muscarinic antagonist / F. Selective serotonin reuptake inhibitor Explanation - Q: 4.3

Close

The correct answer is A. Tolcapone and entacapone are COMT inhibitors, which prevent the peripheral conversion of L-Dopa to 3-O-methyldopa, enhancing the availability of L-Dopa for transport across the blood-brain barrier. These drugs are used as adjuncts to levodopa-carbidopa and can prolong "on-time" and allow lower doses of L-Dopa to be used. Dopamine agonists (choice A), such as bromocriptine, pergolide, and pramipexole, directly stimulate postsynaptic dopamine D2 receptors to relieve parkinsonian symptoms. These agents can be used as monotherapy in early PD, or as adjuncts to levodopa-carbidopa in later stages of the disease. Amantadine works possibly by a variety of mechanisms, including dopamine uptake blockade (choice C), enhancing dopamine synthesis or release, stimulating postsynaptic dopamine receptors, or increasing dopamine

receptor sensitivity. It is used as an adjunctive therapy. Selegiline is an irreversible inhibitor of MAO (choice D). At recommended doses, it is relatively specific for MAO-B. It blocks the breakdown of dopamine and therefore extends the duration of L-Dopa. Muscarinic antagonists (choice E), such as trihexyphenidyl and benztropine, are used adjunctively to alleviate the tremor and rigidity in parkinsonism; they do not typically improve the bradykinesia. Selective serotonin reuptake inhibitors (SSRIs, choice F) such as fluoxetine, sertraline, citalopram, and paroxetine are antidepressants that act by blocking serotonin reuptake. Which of the following drugs would be contraindicated in this patient? A. Aspirin B. Chlorpromazine C. Pergolide D. Pramipexole E. Ropinirole

/ / / / /

Explanation - Q: 4.4

Close

The correct answer is B. Parkinson disease symptoms are based on dopamine depletion. Chlorpromazine is a D2 dopamine receptor antagonist and will worsen parkinsonian symptoms. Aspirin (choice A) is commonly used as a pain reliever and for its antiplatelet effect. It does not worsen parkinsonian symptoms. Pergolide, pramipexole, and ropinirole (choices C, D, and E) are all dopamine receptor agonists and are effective treatments for Parkinson disease. Which of the following enzymes is normally present in the neurons whose degeneration leads to this patient's movement disorder? / A. Catecholamine- methyltransferase / B. Dopamine -hydroxylase / C. Phenylethanolamine-N-methyl transferase / D. Tryptophan hydroxylase / E. Tyrosine hydroxylase Explanation - Q: 4.5

Close

The correct answer is E. Tyrosine hydroxylase is the rate-limiting enzyme in

catecholamine synthesis, which proceeds as follows:

The dopaminergic cells that degenerate in Parkinson disease contain tyrosine hydroxylase and dopa decarboxylase. Catecholamine-O-methyltransferase (choice A) is an extraneuronal enzyme involved in the degradation of catecholamines. Dopamine β-hydroxylase (choice B) converts dopamine to norepinephrine, and therefore would not be present in dopaminergic neurons. Instead, it is present in noradrenergic neurons, such as those in the locus ceruleus. The locus ceruleus, another pigmented nucleus, does degenerate in Parkinson disease, however, this does not lead to any movement disorders. Phenylethanolamine-N-methyl transferase (PNMT, choice C) converts norepinephrine to epinephrine, and would be present in cells that synthesize epinephrine, such as cells in the adrenal medulla and select neurons in the brain. Tryptophan hydroxylase (choice D) is the rate-limiting enzyme in the synthesis of serotonin.

The brain structure whose degeneration leads to this patient's condition is derived embryologically from which of the following structures? / A. Diencephalon / B. Mesencephalon / C. Metencephalon / D. Myelencephalon / E. Telencephalon Explanation - Q: 4.6

Close

The correct answer is B. The substantia nigra (pars compacta) degenerates in Parkinson disease, leading to the motor disorders of these patients. The substantia nigra is located in the midbrain, i.e., the mesencephalon. The mesencephalon exists in both the three- and five-

vesicle stages of neural tube development. The diencephalon (choice A) first appears during the 5-vesicle stage of neural tube development. The thalamus, subthalamus, hypothalamus, and epithalamus are all parts of the diencephalon. The metencephalon (choice C) first appears during the 5-vesicle stage of neural tube development. It later becomes the pons and cerebellum. The myelencephalon (choice D) first appears during the 5-vesicle stage of neural tube development. It later becomes the medulla. The telencephalon (choice E) first appears during the 5-vesicle stage of neural tube development. It later becomes the cerebral hemispheres.

Which of the following structures is most likely lesioned in this patient? / A. A / B. B / C. C / D. D / E. E / F. F / G. G Explanation - Q: 4.7

Close

The correct answer is A. The pigmented substantia nigra pars compacta degenerates in this disease. This structure is located anterior to the tegmentum within each cerebral peduncle of the midbrain (between the crus cerebri and the tegmentum). The substantia nigra can be divided into two

parts: the more dorsally located cell-rich pars compacta, and the more ventrally located pars reticulata. Neurons of the pars compacta contain neuromelanin, which is a polymer derived from dopamine, giving the substantia nigra its black appearance. The cytology, function, and connections of the substantia nigra pars reticulata are similar to those of the internal segment of the globus pallidus, and so these two nuclei are often considered a single functional unit. Choice B labels the medial lemniscus. These are ascending sensory tracts that carry discriminative touch, vibration, and proprioception information. Choice C labels the periaqueductal gray. It is thought to be involved in the modulation of pain perception. Choice D labels the superior colliculi, which are located in the tectum of the midbrain. They play an important role in eye movements. Choice E labels the medial geniculate body. The MGN are auditory relay centers and are actually considered to be part of the thalamus, rather than the midbrain. Choice F labels the red nucleus. The red nuclei are obvious masses in the midbrain tegmentum and are important in the outflow of information from the cerebellum. Choice G labels the corticospinal tract, located in the crus cerebri of the midbrain. This carries descending motor tract fibers from the cortex to the spinal cord.

Which of the following would most likely be seen if microscopic examination of the lesioned area were performed? / A. Hirano bodies / B. Lewy bodies / C. Neuritic plaques / D. Neurofibrillary tangles / E. Pick bodies Explanation - Q: 4.8

Close

The correct answer is B. In patients with this condition whose brains are examined after death, the substantia nigra is found to be pale. Microscopic sections taken through this region typically demonstrate intracytoplasmic inclusion in the remaining neurons. These inclusions are called Lewy bodies

and are typically round to elongated, eosinophilic, and have a dense core surrounded by a paler rim. Lewy bodies are characteristic of idiopathic Parkinson disease. When viewed by electron microscopy, they are composed of fine filaments that are densely packed at the core and loose at the periphery. They stain by immunohistochemistry for ubiquitin, and appear to be related to neuron degeneration. Hirano bodies (choice A) are associated with Alzheimer disease, and consist of actin-containing paracrystalline arrays of beaded fibers that, by light microscope, appear as elongated, glassy, eosinophilic bodies. Neuritic plaques (choice C) are associated with Alzheimer disease, and are focal collections of distorted neurites around a central amyloid core. Neurofibrillary tangles (choice D) are associated with Alzheimer disease, and consist of bundles of filaments within the cytoplasm of neurons that often form a "flame-shaped" inclusion. Pick bodies (choice E) are associated with Pick disease, and are round to oval cytoplasmic inclusions composed of filaments that stain densely with silver stains.

Which of the following agents could produce a syndrome similar to the one seen in this patient? / A. Cocaine / B. Carbon monoxide / C. Methylenedioxymethamphetamine (MDMA) / D. Methylphenyl tetrahydropyridine (MPTP) / E. Sulfonamides Explanation - Q: 4.9

Close

The correct answer is D. An illicit lab attempting to synthesize a meperidine-like drug in the 1980s, accidently produced MPTP (1-methyl 4phenyl-1,2,3,6-tetrahydropyridine), a substance that caused a rapidly developing and irreversible form of parkinsonism. This extremely neurotoxic substance can produce a syndrome virtually identical to Parkinson disease in just 5-15 days, as compared to the slowly developing idiopathic Parkinson disease, which develops over a period of years. MPTP is converted to MPP+ by MAO-B in astrocytes. MPP+ then enters dopaminergic nerve terminals and eventually causes the destruction of dopaminergic neurons in the substantia nigra. Although the microscopic findings are not exactly identical (typical Lewy bodies are not produced), MPTP produces the best model of

Parkinson disease known to date. Carbon monoxide (choice A) can cause low-density lesions of the basal ganglia as shown on CT, however it does not affect the substantia nigra. Cocaine (choice B) acts in part by blocking dopamine uptake. It can cause a variety of problems (e.g., cardiovascular, psychiatric), however, it does not directly damage the substantia nigra. MDMA (choice C), also known as methylenedioxymethamphetamine or Ecstasy, is a drug that is taken recreationally. It is a potent releaser of serotonin in the brain. Animal studies reveal damage to brain serotonergic nerve terminals, however the substantia nigra remains unaffected. Neonatal administration of sulfonamides (choice E) and some penicillins, can displace bilirubin from its albumin-binding site, effectively increasing the serum concentration of free bilirubin available to cross the BBB. The basal ganglia can be affected by this (along with other structures such as the hippocampus, geniculate bodies, cranial nerve nuclei, and cerebellum), but not the substantia nigra.

Vignette 5 of 8

A 58-year-old man presents to his neurologist with a one-year history of muscle weakness and muscle twitching, particularly in his hands and legs. The patient developed foot drop that resulted in a fall and a sprained ankle. His family states that the man has lost weight loss, especially over the last six months. His son, home from college, noticed slurring of his words and choking during a recent dinner. The patient reports no impairment in bladder or bowel controI, or sexual function. On examination, the Babinski sign is present and fasciculations are noted. Deep tendon and jaw jerk reflexes are hyperreflexic. Sensory, cognitive, and oculomotor functions are normaI. Which of the following is the correct diagnosis? / A. Amyotrophic lateral sclerosis / B. Kennedy disease / C. Lambert-Eaton myasthenic syndrome / D. Multifocal motor neuropathy / E. Myasthenia gravis Explanation - Q: 5.1

Close

The correct answer is A. Amyotrophic lateral sclerosis (ALS) is a specific, rapidly progressive form of degenerative motor neuron disease in which there are both upper and lower motor neuron signs. Amyotrophic refers to the muscle atrophy accompanying the lower motor neuron (LMN) degeneration. Other LMN signs include fasciculations and weakness. Lateral sclerosis refers to the hardness to palpation of the lateral columns of the spinal cord in autopsy specimens, where gliosis follows degeneration of the corticospinal tracts. The clinical aspects include upper motor neuron (UMN) signs: overactive tendon reflexes, clonus, Babinski and Hoffmann signs. Symptoms begin with limb involvement in approximately 75-80% of patients; the rest may present with bulbar signs. Upper and lower limb involvement occurs at approximately equal frequency at presentation. Patients with upper limb onset may complain of difficulty with buttons, turning keys, and picking up small objects; patients with lower limb onset may complain of tripping or difficulty running. Patients with bulbar onset complain of slurred speech, hoarseness, and decreased volume of speech; later, dysphagia and drooling occurs. Extraocular muscles and sphincters are typically spared. No changes on sensory examination are found. Respiratory insufficiency is usually a late event. ALS is usually fatal within 3-5 years. Riluzole, which reduces the presynaptic release of glutamate, may slow progression of the disease. ALS is also known as Lou Gehrig's disease, because it caused the death of this baseball legend. Kennedy disease (choice B) is an inherited disorder characterized by degeneration of both motor and sensory neurons. Extraocular muscles are spared. The genetic basis is an expanded repeat of the CAG trinucleotide in the androgen receptor gene on the proximal portion of the X chromosome. Lambert-Eaton myasthenic syndrome (choice C) is an autoimmune disorder of neuromuscular transmission caused by antibodies directed against voltage-gated calcium channels on the presynaptic motor nerve terminal. This leads to the decreased release of acetylcholine in response to a nerve impulse, and to weakness, especially of the proximal muscles of the limbs. Multifocal motor neuropathy (choice D) is dominated by lower motor neuron signs, and characterized by multiple motor conduction blocks on electrical testing. These patients, unlike those with ALS, respond to treatment with cyclophosphamide or IV immune globulin. Myasthenia gravis (choice E) is an autoimmune disorder characterized by fluctuating weakness and fatigability on exertion. Extraocular muscles are commonly affected. The pathophysiologic substrate is a variable degree of neuromuscular transmission block caused by autoantibodies against acetylcholine receptors at the neuromuscular junction.

Which of the following neurologic signs would most likely represent both upper motor neuron (UMN) and lower motor neuron (LMN) involvement? / A. Babinski sign / B. Decreased muscle tone / C. Fasciculations / D. Hoffmann sign / E. Weakness Explanation - Q: 5.2

Close

The correct answer is E. Weakness is a sign of both LMN and UMN syndromes. UMN weakness particularly involves the extensors of the upper limb and the flexors of the lower limbs. Patients with LMN involvement usually have marked weakness and associated muscle wasting (weight loss), which is the consequence primarily of the loss of neuronal axonal trophic factors, and partly of disuse. The Babinski sign (reflex) (choice A) is present when stimulation of the sole of the foot causes dorsiflexion of great toe with fanning of the remaining toes. It is normal in young children, but is abnormal after about 2 years of age and indicates damage to the UMNs. Decreased muscle tone (choice B), muscle atrophy, and hyporeflexia are characteristic of LMN disease, while UMN involvement shows hypertonia and hyperreflexia. LMN syndrome is characterized by muscle fasciculations (choice C), which are small and local involuntary contractions of muscle that are visible under the skin. Fasciculations in ALS are observed with the muscle at rest. The Hoffmann sign (choice D), or heightened finger flexor reflex (thumb flexes and adducts), suggests the presence of an UMN lesion affecting the hands.

Degeneration of which of the following is most likely associated with this patient's Babinski sign? / A. Dorsal funiculus / B. Dorsal horn / C. Lateral funiculus / D. Lateral horn / E. Ventral funiculus / F. Ventral horn

Explanation - Q: 5.3

Close

The correct answer is C. The lateral funiculus is spinal cord white matter located between the dorsal and ventral horns. Important tracts in this region include the lateral corticospinal tracts, the spinothalamic tracts, and dorsal and ventral spinocerebellar tracts. Upper motor neurons descend in the lateral corticospinal tracts, and degeneration of the neurons that contribute to these tracts can lead to a variety of UMN signs, including the Babinski sign. The dorsal funiculus (choice A) is spinal cord white matter located between the dorsal median septum and dorsal gray horn. The dorsal columns are located here. The dorsal horn (choice B) is part of the gray matter of the spinal cord and contains cell bodies with sensory functions. The lateral horn (choice D) is part of the gray matter of the spinal cord and contains cell bodies with autonomic functions (preganglionic neurons). The ventral funiculus (choice E) is spinal cord white matter located between the ventral gray horn and anterior median fissure. It contains tracts such as the ventral corticospinal tracts and vestibulospinal tracts. The ventral horn (choice F) is part of the gray matter of the spinal cord and contains cell bodies with motor functions. Cell bodies of LMNs are located here. These neurons do degenerate in ALS, but this does not produce a Babinski sign

Degeneration of which of the following is most likely associated with this patient's fasciculations? / A. Dorsal columns / B. Dorsal horn / C. Lateral corticospinal tract / D. Lateral horn / E. Ventral horn / F. Ventral white commissure Explanation - Q: 5.4

Close

The correct answer is E. The ventral horn is part of the gray matter of the spinal cord and contains cell bodies of the LMNs. Degeneration of these neurons lead to a variety of LMN signs, including fasciculations, atrophy,

hypotonia, decreased strength, and hyporeflexia. The dorsal columns (choice A) contain the fasciculus gracilis (legs) and fasciculus cuneatus (arms), which carry sensory information regarding discriminative touch, vibration sensation, and conscious proprioception. The dorsal horn (choice B) is part of the gray matter of the spinal cord and contains cell bodies with sensory functions. The lateral corticospinal tract (choice C) contains descending motor fibers (UMNs). The lateral horn (choice D) is part of the gray matter of the spinal cord and contains cell bodies with autonomic functions (preganglionic neurons). The ventral white commissure (choice F) carries pain and temperature information. The cell bodies originate in the dorsal horn, cross in the ventral white commissure, and ascend in the lateral spinothalamic tract to the ventral posterior lateral (VPL) nucleus of the thalamus

Neurons damaged in this patient's condition decussate in which of the following locations? / A. Cerebral cortex / B. Internal capsule / C. Medulla / D. Midbrain / E. Pons / F. Spinal cord Explanation - Q: 5.5

Close

The correct answer is C. Spasticity is the result of UMN dysfunction. UMNs originate in the cerebral cortex (choice A), descend through the posterior limb of the internal capsule (choice B), and continue down through the crus cerebri of the midbrain (choice D), the basilar part of the pons (choice E), the pyramids of the medulla, and then cross in the caudal medulla (pyramidal decussation) (choice C). The 85% of the neurons that decussate descend in the lateral corticospinal tract of the spinal cord, synapsing in the ventral horn of the spinal cord. LMNs, which are also lesioned in this disease, do not decussate.

The spasticity that this patient develops is both troubling and painfuI. Which of the following medications would most directly alleviate this symptom? / A. Amitriptyline / B. Baclofen / C. Pyridostigmine / D. Riluzole / E. Succinylcholine Close Explanation - Q: 5.6 The correct answer is B. Baclofen is a GABA-B agonist and is used to relieve spasticity. Alpha-adrenergic agonists, such as tizanidine and clonidine, may also help, but they produce hypotension, which may limit their use. Dantrolene and benzodiazepines may also be used. If depression, excessive salivation, or chronic pain occurs in these patients, amitriptyline (choice A) may be prescribed. Amitriptyline inhibits the reuptake of noradrenaline and serotonin at the nerve endings in the CNS. The drug also has a strong anticholinergic effect. Pyridostigmine (choice C) is an anticholinesterase inhibitor. It is used to enhance muscle strength in myasthenia gravis. Riluzole (choice D), a glutamate antagonist, is the only drug approved by the FDA for the treatment of ALS. In therapeutic trials, riluzole prolonged survival by 3-6 months. Succinylcholine (choice E) is a depolarizing skeletal muscle relaxant used to relax skeletal muscles during surgery.

Vignette 6 of 8

A 17-year-old man is brought in to the emergency department after being stabbed in a fight. The knife wound is located in the center of the patient's back at about the T5 IeveI. After assuring that the patient is hemodynamically stable, a neurologic examination is preformed. The patient is alert, oriented, and fluent. His cranial nerves are intact and the motor and sensory examinations of his upper extremities are within normal limits. However, the patient is unable to move his right leg. Sensory examination of his lower extremities reveals a complete loss of tactile discrimination, proprioception, and vibratory sensation in his right leg and a complete loss of pain and temperature sensation in his left leg. Which of the following is the most likely diagnosis?

/ / / / / /

A. Anterior spinal artery disruption B. Complete transection of the spinal cord C. Hemisection of the left half of the spinal cord D. Hemisection of the right half of the spinal cord E. Syringomyelia F. Tabes dorsalis Explanation - Q: 6.1

Close

The correct answer is D. Hemisection of the cord (Brown-Sequard syndrome) results in an ipsilateral spastic paresis, an ipsilateral loss of vibratory and position sense, and a contralateral loss of pain and temperature below the level of the lesion. This patient's symptoms are the result of a right-sided thoracic spinal cord hemisection. Anterior spinal artery disruption (choice A) would result in an infarction of the anterior two-thirds of the spinal cord. While the posterior columns would be spared, the infarction of the corticospinal tracts and the spinothalamic tracts would result in bilateral lower extremity spastic paralysis and sensory loss. Complete transection of the spinal cord (choice B) would result in bilateral lower extremity spastic paralysis and sensory loss. Hemisection of the left half of the spinal cord (choice C) would result in left leg spastic paresis and loss of pain and temperature sensation in the right leg. Syringomyelia (choice E) is a cyst that forms around the central canal of the spinal cord. It usually occurs in the cervical spine and 90% occur with ArnoldChiari I malformations. Syringomyelias also occur with trauma, tumors, and infection. They can present with pain, lower motor neuron findings in the upper extremities (as the disease process extends into the anterior horns of the spinal cord), upper motor neuron findings in the lower extremities (as the disease process extends into the lateral funiculus), and a cape-like loss of pain and temperature sensation (from the disruption of the decussating spinothalamic fibers through the ventral white commissure). Tabes dorsalis (choice F) results from damage to proprioceptive and other dorsal root fibers. It is classically caused by syphilis. Symptoms include paresthesias, pain, and abnormalities of gait. Vibration sense is most affected.

A Iesion in which of the following locations is most likely responsible for the patient's loss of pain and temperature sensation?

/ / / / / / /

A. Left corticospinal tract B. Left gracile fasciculus C. Left lateral spinothalamic tract D. Right corticospinal tract E. Right gracile fasciculus F. Right lateral spinothalamic tract G. Ventral white commissure Explanation - Q: 6.2

Close

The correct answer is F. A lesion of the right lateral spinothalamic tract would result in a loss of pain and temperature sensation in the left leg, beginning one segment below the lesion and extending caudally. A lesion of the left corticospinal tract (choice A) could lead to a left spastic paresis. A lesion of the left gracile fasciculus (choice B) could lead to loss of tactile discrimination, and position and vibration sensation from the left leg. A lesion involving the left lateral spinothalamic tract of the spinal cord (choice C) would result in a loss of pain and temperature sensation in the right leg, beginning one segment below the lesion, and extending caudally. A lesion of the right corticospinal tract (choice D) would lead to a right spastic paresis. A lesion involving the ventral white commissure of the spinal cord (choice F) would cause a bilateral loss of pain and temperature sensation within the dermatomes of the segments affected. The ventral white commissure contains the crossing axons of second-order sensory neurons of the lateral spinothalamic tract. These axons originate from cell bodies within the dorsal horn and travel through the ventral white commissure to ascend in the ventral half of the lateral funiculus. These neurons carry input from first-order neurons regarding pain and temperature sensation.

A Iesion in which of the following locations is most likely responsible for the patient's loss of proprioception? / A. Corticospinal tract of the spinal cord / B. Lateral spinothalamic tract of the spinal cord / C. Posterior columns of the spinal cord / D. Spinocerebellar tract of the spinal cord / E. Ventral white commissure of the spinal cord

Explanation - Q: 6.3

Close

The correct answer is C. A lesion in the posterior columns would result in an ipsilateral loss of tactile discrimination and position and vibration sensation below the lesion. In this case, the gracile fasciculus is implicated because the symptoms involve only the leg. A lesion involving the corticospinal tracts of the spinal cord (choice A) would result in an ipsilateral spastic paresis below the lesion, with upper motor neuron signs. A lesion involving the lateral spinothalamic tracts of the spinal cord (choice B) would result in a contralateral loss of pain and temperature sensation, beginning one segment below the lesion, and extending caudally. A lesion involving the spinocerebellar tract of the spinal cord (choice D) would result in ipsilateral leg dystaxia (dorsal spinocerebellar tract) and contralateral leg dystaxia (ventral spinocerebellar tract). A lesion involving the ventral white commissure of the spinal cord (choice E) would cause a bilateral loss of pain and temperature sensation within the dermatomes of the segments affected.

A Iesion in which of the following locations is most likely responsible for the patient's right leg weakness? / A. Corticospinal tract of the spinal cord / B. Lateral spinothalamic tract of the spinal cord / C. Posterior column of the spinal cord / D. Spinocerebellar tract of the spinal cord / E. Ventral white commissure of the spinal cord Explanation - Q: 6.4

Close

The correct answer is A. A right-sided spinal cord hemisection, which includes damage to the corticospinal tract, would result in a right-sided spastic paresis below the lesion with upper motor neuron signs. A lesion involving the lateral spinothalamic tracts of the spinal cord (choice B) would result in a contralateral loss of pain and temperature sensation, beginning one segment below the lesion and extending caudally.

A lesion involving the posterior columns of the spinal cord (choice C) would result in an ipsilateral loss of tactile discrimination, and position and vibration sensation below the lesion. Bilateral posterior column degeneration is seen with tertiary syphilis (tabes dorsalis). A lesion involving the spinocerebellar tract of the spinal cord (choice D) would result in ipsilateral leg dystaxia (dorsal spinocerebellar tract), and contralateral leg dystaxia (ventral spinocerebellar tract). A lesion involving the ventral white commissure of the spinal cord (choice E) would cause a bilateral loss of pain and temperature sensation within the dermatomes of the segments affected. The neurons that transmit discriminative touch and vibration information decussate at which of the following levels? / A. Medulla / B. Midbrain / C. Pons / D. Thalamus / E. Within one or two levels after entering the spinal cord Explanation - Q: 6.5

Close

The correct answer is A. After the primary sensory fiber enters the spinal cord, the ascending branch enters the dorsal columns and travels to the medulla. The fibers from the legs and trunk travel medially in the fasciculus gracilis, while those from the arm and neck travel laterally in the fasciculus cuneatus. The fasciculus gracilis and fasciculus cuneatus make up the posterior columns. These first order neurons synapse in the medulla (on the nucleus gracilis and nucleus cuneatus) and then the second order neurons decussate as the internal arcuate fibers (in the medulla) and ascend in the medial lemniscus. The second order fibers synapse in the ventroposterolateral (VPL) nucleus of the thalamus. Tertiary neurons then synapse upon the somatosensory cortex.

Upper motor neurons decussate at which of the following levels? / A. At the junction of the medulla and the spinal cord / B. At the junction of the midbrain and the medulla / C. At the junction of the pons and the medulla / D. At the thalamus / E. Within one or two levels after entering the spinal cord

Explanation - Q: 6.6

Close

The correct answer is A. The lateral corticospinal tract originates from layer V of the cerebral cortex (primary motor cortex, premotor cortex, and primary sensory cortex). These axons then travel in the posterior limb of the internal capsule and then the middle three-fifths of the crus cerebri of the midbrain. They continue through the basal pons and the pyramids in the medulla. 8590% of these fibers will decussate in the pyramidal decussation, and will continue to descend in the spinal cord as the lateral corticospinal tract; the remaining 10-15% will descend as the anterior corticospinal tract. The pyramidal decussation occurs in the caudal medulla near the junction of the medulla and spinal cord.

Vignette 7 of 8

A 41-year-old man feels a slight tingling in his right leg while getting out of his car. At the time, he ascribes it to the flu he had a week ago. He visits his friends, and then several hours later, he starts feeling increasingly weak in his arms and legs, and has difficulty turning the key in the ignition of his car when he leaves. The next morning, when trying to stand up, the man collapses and is able only to roll on the floor. His mouth feels strange, with tingling around his lips. He manges to call paramedics, and is taken to the hospitaI. On admission, complete flaccid paralysis of his arms and legs is found. He states that he feels "trapped in a body that l have no control of." The next day, Ieg and back pain begin to develop. By his fourth hospital day, the man is completely paralyzed and is put on a ventilator. Which of the following is the most likely diagnosis? A. Botulism B. Devic disease C. Guillain-Barr syndrome D. Multiple sclerosis E. Myasthenia gravis

/ / / / /

Explanation - Q: 7.1

Close

The correct answer is C. Guillain-Barré syndrome (GBS) is an acute inflammatory demyelinating polyradiculopathy, predominantly affecting motor function. It is thought to be a postinfectious immune-mediated disease directed against the peripheral nervous system myelin, axon, or both. The pathologic features are distinct, and involve multifocal areas of inflammation and demyelination with cellular infiltration of macrophages and lymphocytes.

Focal demyelination causes slowing in impulse conduction. The damage can also be axonal (denervation), which will block nerve conduction. GBS usually follows a minor gastrointestinal or respiratory infection. It commonly presents with symmetric ascending weakness, bilateral changes in reflexes, involvement of cranial nerves, mild sensory changes, gait disturbance, pain, autonomic abnormalities, and respiratory insufficiency that may require assisted ventilation. Botulism (choice A) is caused by Clostridium botulinum neurotoxin. Devic disease (choice B) is a rare, chronic, inflammatory and demyelinating disorder characterized by optic neuritis and myelitis. Multiple sclerosis (MS) (choice D) is a demyelinating, inflammatory disease of the central nervous system especially affecting individuals living in a temperate climate. Myasthenia gravis (choice E) is an autoimmune disorder of neuromuscular transmission. A Iumbar puncture is performed. Which of the following CSF findings would most likely be in this patient? / A. High protein level and normal cell count / B. Large number of cells / C. Normal protein level / D. OIigoclonal bands / E. Tau protein Explanation - Q: 7.2

Close

The correct answer is A. Typical CSF findings in GBS include elevated protein, especially IgG (80 to 200 mg/dL) and the presence of a few mononuclear leukocytes. The CSF in Devic patients has a large number of WBC (choice B), no oligoclonal bands, and normal intrathecal IgG synthesis. Normal protein CSF level (choice C) is seen in patients with botulism. Oligoclonal bands (choice D), myelin basic protein, normal glucose and protein levels, and a slightly elevated WBC count are typical CSF finding in multiple sclerosis. Increased level of tau protein (choice E) is a typical CSF feature in Alzheimer disease.

The patient develops problems with swallowing. Which of the following cranial nerves is most likely involved? / A. Abducens (VI) / B. Accessory (XI) / C. TrigeminaI (V) / D. Trochlear (IV) / E. Vagus (X)

Explanation - Q: 7.3

Close

The correct answer is E. Swallowing is a complex reflex that involves coordinated action of several cranial nerves that innervate skeletal muscles of tongue, palate, pharynx, larynx and upper third of the esophagus, and smooth muscle of the lower two-thirds of the esophagus. The vagus nerve plays an important role in swallowing. The abducens nerve (VI; choice A) innervates skeletal muscles that move eyeball laterally (m. rectus lateralis), and afferents from the receptors in those muscles. The accessory nerve (XI; choice B) nerve innervates neck skeletal muscles (m. trapezius and m. sternocleidomastoideus). The trigeminal cranial nerve (V; choice C) innervates chewing muscles (masticatory muscles). Afferents transmit sensory information from skin, skeletal muscles of the face, nose and mouth, and teeth sockets. The trochlear nerve (IV; choice D) innervates skeletal muscles that move the eyeball downward and laterally (superior oblique).

What would be the most appropriate treatment in this case? / A. Botulinus antitoxin / B. Corticosteroids / C. Interferons / D. IV immunoglobulins / E. Methotrexate Explanation - Q: 7.4

Close

The correct answer is D. Infusion of IV immunoglobulins (IVIG), 0.4 mg/kg/day for 5 days is the therapy of choice at many hospitals. Immune globulin has numerous immunomodulatory and anti-inflammatory activities. Therapeutic plasma exchange (TPE, plasmapheresis) is beneficial in patients with rapidly progressive disease. It is contraindicated in patients with cardiovascular diseases, active sepsis, and dysautonomias. The combination of IV immunoglobulin and plasma exchange offers no significant additional advantage. Botulinus antitoxin (equine, choice A), trivalent, types A, B, and E is the only treatment available for botulism and should be administered immediately. Treatment of GBS patients with corticosteroids (choice B) is ineffective and may actually prolong recovery time. Beta-1 interferons (choice C) slow the progression of MS and reduce the frequency of relapses. Methotrexate (choice E) and corticosteroids showed promising results as treatment for patients with Devic syndrome.

Which of the following infective agents is most likely associated with this disease?

Explanation - Q: 7.5

Close

The correct answer is B. C. jejuni is a gram-negative rod and is microaerophilic (requires reduced levels of oxygen). It is not commonly carried by man, although it is often isolated from healthy cattle, chickens, birds, etc. This organism is considered the most common cause of foodborne illness in the US. The illness is called Campylobacteriosis or Campylobacter enteritis. GBS is usually preceded by infections, in particular Campylobacter jejuni. Other agents less frequently identified include Cytomegalovirus, Mycoplasma pneumoniae, and Epstein-Barr virus. The virulence of C. jejuni is based on the presence of specific antigens in its capsule that may crossreact with ganglioside GM1, which is the component of normal myelin. This

process is known as molecular mimicry and it could be a possible immune pathogenetic mechanism in developing GBS. Lyme disease is a multisystemic inflammatory disease, with cranial neuropathy and radiculoneuritis as neurologic manifestations. It is caused by the spirochete Borrelia burgdorferi(choice A), which is transmitted to humans by different type of ticks (family Ixodes). Clostridium botulinum(choice C) is an anaerobic, gram-positive, sporeforming rod that produces a potent neurotoxin and causes botulism in humans and animals. Clostridium tetani(choice D) is a bacterium that causes tetanus in humans. Salmonella enteritidis(choice E) is a rod-shaped, motile, nonspore-forming and gram-negative bacterium, which causes gastroenteritis and is not associated with GBS.

Vignette 8 of 8

A 37-year-old woman presents to her doctor complaining of weakness and clumsiness in her upper and lower extremities. On examination, she is alert and oriented, and her speech is fluent. Her cranial nerves are intact. She is noted to have muscle atrophy in her upper extremities, and sensory examination reveals a cape-Iike loss of pain and temperature sensation in her shoulders and upper arms. She also has decreased tone in her arms and increased tone in her legs. Reflex testing reveals her to be hyperreflexic in her lower extremities and hyporeflexic in her upper extremities. Which of the following is the most likely diagnosis? / A. Amyotrophic lateral sclerosis / B. Parasagittal meningioma / C. Poliomyelitis / D. Syringomyelia / E. Tabes dorsalis Explanation - Q: 8.1

Close

The correct answer is D. Syringomyelia is a cyst that forms around the central canal of the spinal cord. It usually occurs in the cervical spine and 90% occur with Arnold Chiari I malformations. Syringomyelias also occur with trauma, tumors, and infection. They can present with pain, lower motor neuron findings in the upper extremities (as it extends into the anterior horns of the spinal cord), upper motor neuron findings in the lower extremities (as it extends into the lateral funiculus), and a cape-like loss of pain and temperature sensation (from the disruption of the decussating spinothalamic fibers through the ventral white commissure). Amyotrophic lateral sclerosis (choice A) is the most frequent adult-onset progressive motor neuron disease. It results in degeneration of motor neurons and the corticospinal tracts, which results in the expression of both upper and lower motor neuron signs. There are no sensory changes that occur with amyotrophic lateral sclerosis. A parasagittal meningioma (choice B) would result in bilateral lower extremity weakness if it compressed the motor cortex of both hemispheres beneath it. The area of cortex that represents the legs is in the parasagittal region of the motor homunculus. Poliomyelitis (choice C) is an acute inflammatory viral inflammation caused by the poliovirus, which is an enterovirus. It is transmitted by the fecal-oral route and infection leads to destruction of the anterior horn cells (lower motor neurons) in the spinal cord, leading to flaccid paralysis. In tabes dorsalis (choice E) there is demyelination of the posterior columns caused by tertiary syphilis. This results in impaired proprioception and locomotor ataxia. Other associated symptoms are usually present such as lightning-like pains, Argyll-Robertson pupils, and Charcot joints.

A Iesion of which of the following is the most likely cause of this patient's sensory symptoms? / A. Apical portions of the precentral and postcentral gyri, bilaterally / B. Lateral corticospinal tracts of the spinal cord / C. Lateral spinothalamic tracts of the spinal cord / D. Posterior columns of the spinal cord / E. Ventral white commissure of the spinal cord Explanation - Q: 8.2

Close

The correct answer is E. The ventral white commissure contains the crossing axons of second-order sensory neurons of the lateral spinothalamic tract. Primary sensory neurons carrying pain and temperature information

have cell bodies in the dorsal root ganglia and project axons to the spinal cord via the dorsolateral tract of Lissauer. They synapse in the dorsal horn on secondary neurons, which travel through the ventral white commissure to ascend in the ventral half of the lateral funiculus (lateral spinothalamic tract). A lesion involving the ventral white commissure (as would occur with a cervical syringomyelia) would cause a bilateral loss of pain and temperature sensation within the dermatomes of the affected segments. Bilateral deficits occur because a lesion of this area affects tracts decussating from both sides of the spinal cord. However, the posterior columns are spared, thus preserving discriminative touch, proprioception, and vibration sensation. This is called a dissociated sensory loss. A lesion involving the apical portions of the precentral and postcentral gyri bilaterally (choice A), such as a parasagittal meningioma, would result in bilateral lower extremity spastic paresis with upper motor neuron signs. A lesion involving the lateral spinothalamic tracts of the spinal cord (choice B) would result in a contralateral loss of pain and temperature sensation, beginning one segment below the lesion and extending caudally. A lesion involving the lateral corticospinal tracts of the spinal cord (choice C) would result in an ipsilateral spastic paresis below the lesion, with upper motor neuron signs. A lesion involving the posterior columns of the spinal cord (choice D) would result in an ipsilateral loss of tactile discrimination and position and vibration sensation below the lesion. Bilateral posterior column degeneration is seen with tertiary syphilis (tabes dorsalis). A Iesion of which of the following is the most likely cause of this patient's upper extremity weakness? / A. Anterior horns of the spinal cord / B. Apical portions of the precentral and postcentral gyri bilaterally / C. Brachial plexus bilaterally / D. Lateral corticospinal tracts of the spinal cord / E. Lateral spinothalamic tracts of the spinal cord Explanation - Q: 8.3

Close

The correct answer is A. This lower motor neuron lesion would result in bilateral upper extremity weakness, with muscle wasting and hyporeflexia. Such a lesion is seen in amyotrophic lateral sclerosis and, in the case of this patient, from expansion of a syrinx into the anterior horns of the cervical spinal cord.

A lesion involving the apical portions of the precentral and postcentral gyri bilaterally (choice B), such as a parasagittal meningioma, would result in bilateral lower extremity spastic paresis with upper motor neuron signs. A lesion involving the brachial plexus bilaterally (choice C), such as bilateral brachial plexus schwannomas or bilateral brachial plexitis, could result in bilateral upper extremity lower motor neuron weakness, sensory disturbances, and, in the case of brachial plexitis, pain radiating down both arms. However, these lesions rarely occur bilaterally and would not result in the lower extremity weakness exhibited by the patient. A lesion involving the lateral corticospinal tracts of the spinal cord (choice D) would result in an ipsilateral spastic paresis below the lesion with upper motor neuron signs, such as hyperreflexia and increased tone. A lesion involving the lateral spinothalamic tracts of the spinal cord (choice E) would result in a contralateral loss of pain and temperature sensation, beginning one segment below the lesion and extending caudally.

A Iesion of which of the following is the most likely cause of this patient's lower extremity weakness? / A. Anterior horns of the spinal cord / B. Apical portions of the precentral and postcentral gyri bilaterally / C. Lateral corticospinal tracts of the spinal cord / D. Lateral spinothalamic tracts of the spinal cord / E. Posterior columns of the spinal cord Explanation - Q: 8.4

Close

The correct answer is C. Extension of a syringomyelia into the lateral funiculus of the cervical spinal cord bilaterally, affecting the lateral corticospinal tracts, would result in a spastic paresis below the lesion, with upper motor neuron signs as described in this patient. A lesion involving the anterior horns of the spinal cord (choice A) would result in a lower motor neuron lesion, causing weakness with muscle wasting and hyporeflexia. However, this would occur at the level of the syrinx. A lesion involving the apical portions of the precentral and postcentral gyri bilaterally (choice B), such as a parasagittal meningioma, would result in bilateral lower extremity spastic paresis with upper motor neuron signs. However, given that the patient also has lower motor neuron signs in her upper extremities, and a dissociated sensory loss, this makes the spinal cord

a much more likely candidate. A lesion involving the lateral spinothalamic tracts of the spinal cord (choice D) would result in a contralateral loss of pain and temperature sensation, beginning one segment below the lesion and extending caudally. A lesion involving the posterior columns of the spinal cord (choice E) would result in an ipsilateral loss of tactile discrimination, and position and vibration sensation below the lesion.

Vignette 1 of 5

A 3-month-old boy is brought to the emergency department by his parents because he has been vomiting for 4 days. The baby appears very ilI, with dry mucous membranes and poor skin turgor. He is poorly responsive during the examination. According to his mother, he has not been able to hold anything down for several days, and his diapers have been dry for almost 24 hours. The mother describes the vomiting as having been very forcefuI, and accompanied with gagging and retching. She did not notice any red-brown, black, or green discoloration to the emesis, and states that it just looked like formula. No other family members have been ilI, and when the mother called the child's daycare, she was told that no other children were ill with similar symptoms. Despite the vomiting, until today, the child has behaved as if he were very hungry, and would seek the bottle again shortly after vomiting. Which of the following would be most likely to account for the patient's problems? / A. Bacterial gastroenteritis / B. Esophageal reflux / C. Gastric ulcer / D. Pyloric stenosis / E. Viral gastroenteritis Explanation - Q: 1.1

Close

The correct answer is D. This child most likely has hypertrophic pyloric stenosis, which causes obstruction of the pyloric channel secondary to muscular hypertrophy in the pyloric region. This condition may run in families, and most often does not present until the child is several weeks to months old. Hypertrophic pyloric stenosis has an incidence of about 1 of every 500 births, and a male predominance of 4-5 to 1. The vomiting may be so severe that the term "projectile" is appropriate. This child's presentation is typical. The absence of discoloration to the emesis indicates that it does not contain bile (and so the problem is above the ampulla of Vater in the duodenum) or blood. Both bacterial (choice A) and viral (choice E) gastroenteritis would usually be accompanied by significant nausea, and the child would not wish to eat after vomiting. Esophageal reflux (choice B) can cause a gentle regurgitation of formula, but does not cause severe vomiting with gagging and emesis. Gastric ulcer (choice C) would be very unusual in a child of this age.

Physical examination also shows visible gastric peristaltic waves. Which additional finding should be sought on physical examination? / A. Multiple petechiae / B. OIive shaped mass / C. Pain on palpation in the upper left quadrant below the liver / D. Spider angiomas / E. Tenderness at McBurney's point Explanation - Q: 1.2

Close

The correct answer is B. The most characteristic physical finding on abdominal examination in children with hypertrophic pyloric stenosis is the presence of a mass about the size and shape of a small olive, which is usually felt to the right of the umbilicus. This mass is most easily felt if the child is calm (or better still asleep) and gentle palpation is used to prevent guarding. The mass is produced by hypertrophy of the pyloric muscle. Multiple petechiae (choice A) would suggest a blood clotting problem. Pain below the liver (choice C) suggests gallstone disease (usually in adults). Spider angiomas (choice D) suggest alcoholic cirrhosis (usually in adults). Tenderness at McBurney's point (choice E) in the right lower quadrant suggests appendicitis.

The patient's history of vomiting would be most likely to produce which of the following? / A. Metabolic acidosis with increased anion gap / B. Metabolic acidosis with normal anion gap / C. Metabolic alkalosis / D. Respiratory acidosis / E. Respiratory alkalosis Explanation - Q: 1.3

Close

The correct answer is C. These children lose gastric hydrochloric acid in the vomitus, and consequently develop a metabolic alkalosis with low chloride levels. Other laboratory findings that may be seen are related to dehydration and include: high specific gravity in the urine, high BUN and

creatinine, and hemoconcentration. Metabolic acidosis with increased anion gap (choice A) can be seen in diabetes mellitus, lactic acidosis, renal failure, and intoxication. Metabolic acidosis with normal anion gap (choice B) can be seen with diarrhea, renal tubular acidosis, and other renal disease. Respiratory acidosis (choice D) can be seen with depression of the respiratory center, severe lung disease, and diseases that impair lung expansion. Respiratory alkalosis (choice E) can be seen in voluntary or drug-induced hyperventilation or resulting from hyperventilation at high altitudes.

Which of the following is the best choice for confirmation of the diagnosis? A. Computed tomography B. Magnetic resonance imaging C. PIain x-ray film D. Scintillation scan E. Uitrasound

/ / / / /

Explanation - Q: 1.4

Close

The correct answer is E. Most cases of hypertrophic pyloric stenosis are now being confirmed with abdominal ultrasound, which, in these cases, demonstrates a typical hypoechoic mass between the stomach and duodenum. An upper gastrointestinal series may also be helpful, if ultrasound studies are ambiguous. Computed tomography (choice A) and magnetic resonance imaging (choice B) are overly expensive and not required for diagnosis in this condition. Plain x-ray films (choice C) may show a stomach filled with air and fluid, but this is not considered as reliable a finding as the abdominal ultrasound studies. Scintillation scans (choice D) are usually used to evaluate the extent of bony disease.

Which of the following treatments would best correct this child's disease? / A. IV fluids with 3 day restriction of oral intake / B. Gastric resection

/ C. Medical management with acetaminophen / D. Medical management with antacids / E. Pyloromyotomy

Explanation - Q: 1.5

Close

The correct answer is E. After inguinal hernia, the most common cause of gastrointestinal surgery in very young children is hypertrophic pyloric stenosis. The surgery that is done is called pyloromyotomy, and involves partially cutting through the pyloric muscle, to allow it to be spread apart with resultant dilation of the lumen of the pyloric channel. This operation only takes about 30 minutes to perform after anesthesia is induced, and most of the babies are allowed to go home within 24-48 hours. Gastric resection (choice B) is not required, and various forms of medical management (choices A, C, and D) will not correct the underlying problem (although medical stabilization before surgery is appropriate).

Vignette 2 of 5

Over 50 children at an elementary school became violently ill several hours after eating lunch. AII of the affected children developed severe nausea and vomiting, and some additionally developed abdominal cramps and non-bloody diarrhea. AII of the students felt much better by the following morning. Which of the following items from the lunch would be most likely to be the cause of the food poisoning? / A. Carrot sticks / B. Chicken salad / C. Fresh pineapple / D. Lemonade / E. Whole wheat rolls Explanation - Q: 2.1

Close

The correct answer is B. In most cases of food poisoning causing violent gastrointestinal symptoms, the culprit contains meat, cheese, milk or milk products, eggs, or salad dressing. In this case, the chicken salad is the most likely culprit. Fresh fruits and vegetables, such as the carrots (choice A) and pineapple (choice C), are less likely choices unless a salad dressing has been used.

Lemonade (choice D) is too acidic to grow most bacteria, and most commercial soft drinks have enough preservatives in them to not be a problem, even if they have been badly stored for a few hours. Bread products (choice E) are also not usually implicated, unless they are moist due to being incorporated into a dessert or casserole. Gram's stain of a smear taken from the presumed source of the food poisoning shows abundant gram-positive cocci in grape-Iike clusters. Which of the following is the most likely causative organism?

Explanation - Q: 2.2

Close

The correct answer is C. "Gram-positive cocci in grape-like clusters" is the classic description used in test questions about Staphylococcus. Bacillus cereus(choice A) can cause food poisoning, but is a gram-positive rod. Escherichia coli(choice B) can cause diarrheal illness related to infection, but is a gram-negative rod. Streptococcus pneumoniae(choice D) and S. pyogenes(choice E) are grampositive cocci that usually occur in chains and are not a usual cause of food poisoning or diarrheal illness

Which of the following is the most likely source of the bacteria? / A. Food preparer's hands / B. Infected animal supplying milk / C. Infected animal used for meat / D. Poorly cleaned bowl / E. Water used in food preparation Explanation - Q: 2.3

Close

The correct answer is A. While in theory, staphylococci can enter a food specimen by any of the routes listed, in practice the most common source is

bacteria from the food handler's skin. Staphylococcus aureus is present in small numbers on the skin of many individuals, and may be present in large numbers in persons with true staphylococcal skin infections. All commercial food handlers should use gloves, and people with impetigo (skin infection due to staphylococci) should not prepare food. The presentation of the case in these questions was based in part on a real staphylococcal food poisoning outbreak that occurred in Texas and has been reported by the US Food and Drug Administration. 16 elementary schools were served by the same central kitchen, and 1,364 of 5,825 children became poisoned. The culprit was the chicken salad. The previous day, frozen chickens had been boiled for 3 hours, deboned, and then cooled to room temperature with a fan. They were then ground into small pieces and placed in 12 inch deep aluminum pans (which may not have cooled quickly) where they were stored overnight in a walk-in refrigerator at a temperature thought to be 42-45 F. The next morning, the salad was prepared. The food was then transported in thermal containers to the various schools, where it was kept at room temperature for several hours before being served to the children. The investigators thought that the contamination probably occurred while the chickens were being deboned, and then the subsequent inadequate refrigeration for much of 24 hours allowed proliferation of the staphylococci, which were easily demonstrated in the chicken salad later.

Which of the following is the most appropriate therapy to offer most patients who develop this type of food poisoning? / A. Bed rest only / B. IV cephalosporins / C. Oral ampicillin / D. Oral cephalosporins / E. Penicillin injection Explanation - Q: 2.4

Close

The correct answer is A. Staphylococcal food poisoning is usually selflimited, and can be treated with bed rest alone. Antibiotics (choices B to E) are not indicated. Death occurs rarely, and when it does, the patient is almost always in a vulnerable group, such as the very young, the elderly, or the already seriously ill. Patients in these populations may require hospitalization with intravenous fluid support through the illness.

Vignette 3 of 5

A 13-year-old girl is taken to the emergency department by her mother. The girl had awoken feeling ilI, and stayed home alone. When her mother returned from work, she found that her daughter had been vomiting more or less continuously all day. The mother was particularly concerned because her daughter seemed to be acting "weird" with an unusual degree of drowsiness and listlessness. When the physician attempts to interview the girI, she is very irritable, combative, and appears to be confused. Physical examination fails to demonstrate focal neurologic findings, but hepatomegaly is noted. No jaundice, fever, rash, or lymphadenopathy are noted. Drug screens are negative; screening serum chemistries demonstrate moderate elevations (4 times upper limit of normaI) of serum liver transaminases. By the time the laboratory results return, the girl has developed coma with progressive unresponsiveness. Emergency CT scan of the head demonstrates nonspecific findings including cerebral edema, gyral flattening, swollen white matter, and ventricular compression. Which of the following is the most likely diagnosis? / A. Kawasaki syndrome / B. Meningococcal meningitis / C. Reye syndrome / D. Rocky Mountain spotted fever / E. Wilson disease Explanation - Q: 3.1

Close

The correct answer is C. Reye syndrome is a rare, but potentially devastating disease primarily affecting individuals less than 18 years old. Because of its rarity, it is likely to be misdiagnosed as meningitis, encephalitis, diabetes, drug overdose, sudden infant death syndrome, head trauma, renal or hepatic failure, poisoning, or a psychiatric disturbance. In infants, it should be suspected when diarrhea (but not necessarily vomiting), respiratory disturbances (hyperventilation, apneic episodes), seizures, or hypoglycemia are accompanied by elevated liver transaminases (AST, ALT) in the absence of jaundice. In children, adolescents, and the rare adult patient, suspect Reye when you see a cluster of severe vomiting, elevated serum transaminases without jaundice, and signs of neurologic dysfunction. This patient's CT findings are typical, and nonspecifically reflect marked brain edema with compression of fluid-filled spaces such as the ventricles and meningeal spaces. Treatment of Reye syndrome is supportive, as no specific therapy is available. Patients may recover completely, be left with mild to severe residual neurologic defects, or die. Kawasaki syndrome (choice A) is a sometimes very severe febrile illness of children that may be associated with lymphadenopathy, rash, and late development of coronary artery aneurysms secondary to polyarteritis. Meningococcal meningitis (choice B) can present fulminantly, as in this

case, but nuchal rigidity would be noted in the case presentation and a CT scan would probably show meningeal thickening. Rocky Mountain spotted fever (choice D) can cause a fulminant illness with a prominent petechial rash. Wilson disease (choice E) can cause both psychiatric and hepatic dysfunction, but tends to become symptomatic over a period of decades.

Marked elevation of which of the following would be most helpful in confirming the suspected diagnosis? / A. BIood free erythrocyte protoporphyrin / B. BIood methemoglobin / C. Serum ammonia / D. Serum somatomedin C / E. Urine vanillylmandelic acid Close Explanation - Q: 3.2 The correct answer is C. Patients with Reye syndrome often have markedly elevated serum NH4+ levels as a consequence of the liver dysfunction. These high ammonia levels probably substantially contribute to the altered mental status seen commonly in Reye patients. Erythrocyte protoporphyrin (choice A) is increased in some forms of porphyria. Methemoglobin (choice B) is an oxidized form of hemoglobin that can be seen in hemolytic anemias. Somatomedin C (choice D) elevation can be seen in gigantism and acromegaly. Vanillylmandelic acid (choice E) can be elevated in pheochromocytoma.

This patient's disease was most likely preceded by which of the following illnesses? / A. Crohn disease / B. Diabetes mellitus / C. Hypothyroidism / D. Influenza / E. Juvenile rheumatoid arthritis Explanation - Q: 3.3

Close

The correct answer is D. Many patients who develop Reye syndrome have a recent history of a febrile viral illness, most notably influenza and varicella (chicken pox). How this predisposes for Reye syndrome is unclear, but the link appears compelling. Note that patients who have preceding chickenpox may still have the rash, as Reye syndrome typically develops on the 5th or 6th day of the illness in these patients. Reye syndrome does not appear to be an autoimmune disease, and has no links to other diseases with a strong autoimmune component, including Crohn disease (choice A), diabetes mellitus type I (choice B), and juvenile rheumatoid arthritis (choice E). Reye syndrome is also unrelated to hormonal disorders such as hypothyroidism (choice C).

Ingestion of which of the following is most likely to have contributed to the patient's illness? / A. Acetaminophen / B. Aspirin / C. Coffee / D. Pseudoephedrine / E. Tea Explanation - Q: 3.4

Close

The correct answer is B. Aspirin and salicylate use is thought to increase the risk of developing (still very rare) Reye syndrome by 35-fold. While few parents now give young children aspirin, many teenagers or their parents may consider the teens to be "adults" and use aspirin for fever and discomfort control. Also, many over-the-counter anti-nausea medications may contain salicylates, and patients may thus be exposed unknowingly to them. The other agents listed in the choices have no known link to Reye syndrome.

If this patient's liver were biopsied and a sample sent for electron microscopy, which of the following would most likely be seen? / A. Abnormally formed cilia / B. Enlarged mitochondrial with disrupted cristae / C. Linear inclusions in macrophages / D. Membrane-bound vacuoles with complex crystalline structures / E. Prominent, Iong microvilli

Explanation - Q: 3.5

Close

The correct answer is B. While the pathophysiology of Reye syndrome is still poorly understood, severe mitochondrial dysfunction (most prominent in brain and liver) is a striking feature and is confirmed by often marked morphological changes in mitochondria observed by electron microscopy. Abnormal cilia (choice A) in the respiratory epithelium is a feature of Kartagener syndrome, characterized by bronchiectasis and sometimes, situs inversus. Linear inclusions in macrophages (choice C) in the brain are a feature of the hereditary condition, Krabbe disease. Membrane-bound vacuoles with a complex crystalline structure (choice D) can be seen in the brain of patients with the hereditary condition, metachromatic leukodystrophy. Prominent, long microvilli (choice E) are a feature of mesotheliomas. Vignette 4 of 5

A 65-year-old man is struck by a car as he is staggering across the street after a night of binge drinking. When the ambulance arrives, he is noted to be belligerent and combative. A Iarge left temporal laceration is observed. On arrival to the emergency department, his blood pressure is 148/78 mm Hg, his pulse is 89/min, and his oxygen saturation is 96%. He continues to be uncooperative, so only a cursory physical examination is performed, which reveals a 6 cm left temporal Iaceration and no other gross abnormalities. The laceration is sewn closed at the bedside, and the patient left to rest. An hour later the patient's speech becomes more slurred; he begins vomiting, and is given prochlorperazine. An hour later, he is found to be unresponsive. His blood pressure is 194/100 mm Hg and his pulse is 55/min. His left pupil is 6 mm and non-reactive and his right pupil is 3 mm and reactive to light. He localizes to pain in his left upper and lower extremities, but exhibits no movement in his right upper and lower extremities. Which of the following is the most likely diagnosis? / A. AIcohol toxicity / B. AIcohol withdrawal / C. Horner syndrome / D. Subfalcine (cingulate) herniation / E. Uncal herniation

Explanation - Q: 4.1

Close

The correct answer is E. Uncal herniation occurs when a mass lesion, such as a subdural hematoma, causes increased intracranial pressure and displaces the brain downward. The uncus and parahippocampal gyrus are pushed over the tentorial edge, where they compress the brain stem, causing contralateral hemiparesis. The uncus also compresses the ipsilateral third cranial nerve, as it enters the interpeduncular cistern at the midbrain/pons junction, resulting in an ipsilateral third nerve palsy. Signs and symptoms of increased intracranial pressure often are present prior to a herniation syndrome. These include headache, nausea/vomiting, agitation, lethargy, and eventually, coma. There may be a decrease in heart rate and an increase in blood pressure as well. Alcohol toxicity (choice A) is characterized by disinhibition, emotional lability, slurred speech, and ataxia, followed by loss of consciousness and then coma. It would be unusual for a conscious intoxicated patient to progress to coma without consuming more alcohol. Alcohol withdrawal (choice B) usually occurs 2-5 days after the last drink. Autonomic hyperactivity, characterized by tachycardia, hypertension, tremors, and anxiety, occurs first. This is followed by delirium tremens, hallucinations, and confusion. Horner syndrome (choice C) results from a disruption of the sympathetic fibers that originate from the hypothalamus. These fibers travel inferiorly to synapse on second-order neurons in the intermediolateral column of the spinal cord, then synapse on third-order neurons in the superior cervical sympathetic ganglion, to finally innervate the smooth muscles of the eyelids, pupil, and sweat glands of the face and forehead. Interruption of these fibers, which can occur with a Pancoast tumor, results in ipsilateral ptosis, miosis, and anhidrosis. Subfalcine (cingulate) herniation (choice D) occurs when a mass lesion causes the cingulate gyrus to move under the free edge of the falx, and the ipsilateral foramen of Monro becomes trapped. This causes an ipsilateral large lateral ventricle and a contralateral small lateral ventricle.

What is the mechanism most likely responsible for the patient's unilateral pupillary dilation? / A. Autonomic hyperactivity due to alcohol withdrawal / B. Prochlorperazine (Compazine) / C. Unilateral compression of preganglionic parasympathetic fibers originating

from the Edinger-Westphal nucleus

/ D. Unilateral compression of postganglionic sympathetic fibers originating from

the superior cervical ganglion / E. Unilateral compression of the frontal eye fields Explanation - Q: 4.2

Close

The correct answer is C. The Edinger-Westphal nucleus gives rise to preganglionic parasympathetic fibers. These fibers leave the midbrain and travel on the dorsal superficial aspect of the oculomotor nerve. They then synapse on the ciliary ganglion. The ciliary ganglion then gives rise to postganglionic parasympathetic fibers, which terminate in the ciliary body and the iris to cause miosis. Interruption of this pathway, such as when the uncus compresses these fibers in uncal herniation, results in unopposed sympathetic input to the ipsilateral eye, which results in ipsilateral pupillary dilation. Autonomic hyperactivity due to alcohol withdrawal (choice A) usually occurs 2-5 days after the last drink. It is manifested as tachycardia, hypertension, anxiety, and tremors. Prochlorperazine (Compazine) (choice B) is a phenothiazine antiemetic, which selectively antagonizes dopamine D2 receptors. It can have atropinelike side effects, as well as cause photosensitivity and even oculogyric crisis. It would not cause a unilateral papillary dilation. Unilateral compression of postganglionic sympathetic fibers originating from the superior cervical ganglion (choice D) interrupts the sympathetic input to the ipsilateral eye and half of the face, resulting in a Horner's syndrome (miosis, ptosis, anhidrosis). Unilateral compression of the frontal eye fields (choice E) results in ipsilateral eye deviation. The frontal eye fields (Area 8) are located in the caudal middle frontal gyrus and are responsible for the initiation of saccades (rapid eye movements to a target of behavioral importance). Stimulation of area 8 results in conjugate eye deviation to the contralateral side (away from the stimulation). A lesion involving area 8 causes a transient conjugate eye deviation to the ipsilateral side (toward the lesion).

Which of the following is a common side effect of prochlorperazine? / A. Diarrhea / B. Disulfiram-Iike reaction / C. Drowsiness / D. Miosis / E. Urinary urgency

Explanation - Q: 4.3

Close

The correct answer is C. Prochlorperazine is a phenothiazine antiemetic, which commonly causes drowsiness, as well as dizziness, and can produce extrapyramidal side effects. For these reasons, it should not be used in patients with an altered mental status, as it further depresses the patient's sensorium and makes it more difficult for the treating physician to accurately follow the patient's neurologic examination. It also has atropine-like side effects, resulting in dry mouth, constipation, and urinary retention. None of the other choices are side effects of prochlorperazine.

Which of the following is the most likely cause for the patient's unresponsiveness? / A. Injury to both of the mamillary bodies / B. Injury to the left parietal lobe / C. Injury to the left subthalamic nucleus / D. Injury to the reticular activating system / E. Injury to the right parietal lobe Explanation - Q: 4.4

Close

The correct answer is D. Tonic input from the reticular activating system (RAS) keeps the brain turned "on." An injury to the RAS, such as during uncal herniation, causes the patient to fall into a coma. The RAS is located in the middle and lateral pons and midbrain. It sends signals to the subcortical structures (especially the thalamus), which then send diffuse inputs to the cortex. The cortex, in turn, sends positive feedback inputs back to the RAS. The RAS also sends signals to the spinal cord to maintain tone and activate the spinal reflexes. Injury to the mamillary bodies (choice A) is seen in Wernicke-Korsakoff encephalopathy. This disorder is seen in alcoholics with B1 (thiamine) deficiency. Wernicke encephalopathy is characterized by psychosis, ophthalmoplegia, and confusion. Korsakoff syndrome is characterized by memory loss, confabulation, and confusion. Injury to the left parietal lobe (choice B) can result in a Gerstmann syndrome. Gerstmann syndrome is characterized by right/left confusion, finger agnosia, acalculia, and agraphia. Injury to the left subthalamic nucleus (choice C) results in contralateral sudden limb flailing (hemiballismus). Injury to the right parietal lobe (choice E) can result in a left hemineglect.

On further examination, the patient exhibits decerebrate (extensor) posturing in his upper and lower extremities. What is the most likely mechanism for this? / A. An injury below the level of the vestibular nuclei / B. An injury between the red nucleus and the vestibular nuclei / C. An injury to the midbrain above the red nucleus / D. An injury to the posterior limb of the left internal capsule / E. An injury to the vermis Explanation - Q: 4.5

Close

The correct answer is B. Decerebrate (extensor) posturing is characterized by extension, adduction, and pronation of the arms, extension of the legs, and plantar flexion of the feet. It results from a lesion of the rubrospinal tract, originating from the red nucleus, which maintains flexor tone in the arms and legs. The vestibulospinal tract, which maintains extensor tone, is then unopposed, resulting in the extensor posture. An injury below the level of the vestibular nuclei (choice A) results in contralateral hemiparesis (or quadriparesis if the lesion involves both corticospinal tracts), but not in posturing. An injury to the midbrain above the red nucleus (choice C) results in contralateral hemiparesis (or quadriparesis if the lesion involves both corticospinal tracts) and decorticate (flexor) posturing due to dominance of the rubrospinal tract, which maintains flexor tone in the arms and legs. An injury to the posterior limb of the left internal capsule (choice D) results in a contralateral hemiparesis and decorticate (flexor) posturing due to dominance of the rubrospinal tract, which maintains flexor tone in the arms and legs. An injury to the vermis (choice E) results in truncal ataxia, scanning speech, and hypotonia.

Vignette 5 of 5

A 33-year-old woman presents to the emergency department complaining of nausea and vomiting. She states that she has been having significant nausea that has been worsening over the past 2 weeks. Over the past 2 days, she has had 2 episodes of vomiting. She also notes increased fatigue. She has no

abdominal pain or vaginal bleeding. She has no other complaints. Her past medical history is significant for occasional migraine headaches. She has never had surgery. She takes acetaminophen as needed for headache, and has no known drug allergies. She works as a lawyer at a local firm and lives with her husband of three years. She has no family history of cancer or heart disease. Her vital signs are stable. Examination is significant for a bluish-appearing cervix on speculum examination. The remainder of the examination, including the abdominal examination, is benign. Laboratory evaluation shows: Urine hCG: positive Leukocytes: 9,000/mm3 Hematocrit: 41% PIatelets: 250,000/mm3 Pelvic ultrasound demonstrates a gestational sac with yolk sac and fetal pole surrounded by myometrium. There is a heart rate of 154 beats per minute. Which of the following is the most likely diagnosis? / A. Appendicitis / B. Complete hydatidiform mole / C. Ectopic pregnancy / D. Intrauterine pregnancy / E. Spontaneous abortion Explanation - Q: 5.1

Close

The correct answer is D. This patient's presentation, exam findings, and studies are all consistent with a diagnosis of intrauterine pregnancy. First, her presenting complaints of nausea, vomiting, and fatigue are consistent with a first trimester pregnancy. Approximately 80% of pregnant women experience some nausea during the pregnancy, especially in the first trimester. This nausea is most commonly called "morning sickness" but it can occur at any time during the day. Fatigue is also a common first trimester complaint. On examination she has a bluish-appearing cervix. This is called "Chadwick's sign" and it is another clue that she is pregnant. Most definitively, though, she has a positive urine pregnancy test. With this positive test, the diagnosis is narrowed to intrauterine pregnancy, ectopic pregnancy, or spontaneous abortion, with mole also being a consideration. The ultrasound that demonstrates an intrauterine pregnancy fully establishes the diagnosis. Appendicitis (choice A) represents an infection of the appendix. Patients with appendicitis most commonly present with symptoms and signs of infection, including abdominal pain, fever and chills, abdominal tenderness, elevated temperature, and leukocytosis. A patient with a complete hydatidiform mole (choice B) will not have an

intrauterine pregnancy with a fetal heart rate visualized on ultrasound examination. A complete mole often appears as a "snowstorm" pattern on pelvic ultrasound. It is essential to "think ectopic!" whenever a woman of childbearing age presents for medical attention. In fact, this sign ("think ectopic!") and other such similar signs can be seen in many emergency rooms. However, patients with ectopic pregnancy (choice C) usually complain of abdominal pain or vaginal bleeding. On examination, they will often have abdominal and adnexal tenderness. Pelvic ultrasound will show no intrauterine pregnancy. This patient, with a gestational sac, yolk sac, and fetus seen surrounded by myometrium (that is, within the uterus) can be diagnosed with an intrauterine pregnancy. Most patients with spontaneous abortions (choice E) present with vaginal bleeding a passage of tissue (products of conception.) On ultrasound, no viable pregnancy (i.e., an intrauterine pregnancy with a normal heart rate) is seen. This patient could possibly go on to have a spontaneous abortion, but, at this point, she can be diagnosed with an intrauterine pregnancy.

During early pregnancy, the trophoblastic cells secrete which of the following hormones in order to maintain the corpus Iuteum? / A. Estrogen / B. Human chorionic gonadotropin (hCG) / C. Luteinizing hormone (LH) / D. Progesterone / E. Testosterone Explanation - Q: 5.2

Close

The correct answer is B. At approximately the midpoint of the menstrual cycle, the ovum is expelled. Once this occurs, the surrounding granulosa and theca cells undergo luteinization and begin to secrete a large amount of estrogen and progesterone. This mass of hormone secreting cells is known as the corpus luteum (because of its yellow color). If pregnancy does not occur, the corpus luteum will persist for 14 days, after which point it degenerates. With the degeneration of the corpus luteum, hormonal levels drop, the endometrium sloughs, and menstruation occurs. If a pregnancy does occur, the trophoblastic cells of the pregnancy will secrete human chorionic gonadotropin (hCG). This hormone goes to the corpus luteum and prevents its degeneration, signaling it to continue its excretion of estrogen and progesterone to maintain an environment that is favorable for pregnancy. The presence of human chorionic gonadotropin forms the basis for the urine and serum pregnancy test. hCG steadily increases over the first several weeks of the pregnancy, reaching a peak at approximately 10 weeks.

After that, hCG levels decrease gradually during the remainder of the pregnancy. During early pregnancy, most women experience some nausea. One of the leading theories for the cause of this nausea is the hormone hCG, because the nausea of early pregnancy appears to mirror the rise and fall of the hCG levels--peaking at roughly 10 weeks and improving after that. Estrogen (choice A) and progesterone (choice D) are produced by the corpus luteum during early pregnancy. The corpus luteum does not degenerate in a pregnant woman, as it does in the nonpregnant woman, because the trophoblastic cells secrete human chorionic gonadotropin (hCG). After the first months of pregnancy, the placenta takes over the role of secreting the large amounts of estrogen and progesterone that are needed to continue a healthy pregnancy. Once the placenta has taken over this role, the corpus luteum degenerates. Luteinizing hormone (LH) (choice C) is produced by the anterior pituitary gland. Its rate of secretion is controlled mainly by the hypothalamus, through hypothalamic luteinizing hormone-releasing hormone (LHRH, also called GnRH) secretion. The LH surge, which occurs roughly at the midpoint of the menstrual cycle, appears to cause ovulation, which occurs approximately 24 hours after the surge. LH also appears to induce the theca and granulosa cells to become the corpus luteum. Testosterone (choice E) is formed by the interstitial cells of the testicles. It is not known to be secreted by trophoblastic cells to maintain the corpus luteum.

Which of the following is a derivative of the ectoderm of the trilaminar embryo? / A. Bone / B. Germ cells / C. Muscle / D. Spinal cord / E. Spleen Explanation - Q: 5.3

Close

The correct answer is D. The neural plate, a thickened area of ectoderm, gives rise to the nervous system. The first step is the development of a groove in the neural plate. On either side of this groove are the raised neural folds. These neural folds then begin to fuse in the midline at approximately the midpoint of the neural plate. Fusion proceeds in a rostral and caudal direction; once completed, there still remains an open area of the "tube" at both the rostral and caudal ends. These openings are referred to as the rostral and the caudal neuropore. The rostral neuropore closes around day

26 and the caudal neuropore closes on approximately day 28. With the closure of the caudal neuropore, closure of the neural tube is completed. It has been convincingly shown that folic acid plays an important role in the closure of the neural tube. By giving women of childbearing age folic acid supplements, the rate of neural tube defects can be reduced dramatically. Yet, based on the embryology of the neural tube (with closure of the tube roughly by day 28), it is obvious that in order for folic acid supplementation to work, it should be started pregestationally. Unfortunately, most women do not seek prenatal care until after the neural tube has undergone its period of closure. Bone (choice A), muscle (choice C), and spleen (choice E) are all derivatives of the mesoderm. This mesoderm is formed at the end of the second week post-fertilization, when some cells of the primitive streak migrate laterally between the endodermal and the ectodermal layer. Germ cells (choice B) are derived from the endodermal layer of the trilaminar embryo. They migrate as primordial sex cells from the endoderm of the yolk sac to the urogenital ridge.

Assays for serum human chorionic gonadotropin (hCG) are commonly used as a pregnancy test. However, there have been several reported cases of false positive results, that is, women who test positive and are told that they are pregnant but later turn out not to be. If the false positive rate of serum hCG for detecting pregnancy is < 0.5%, then what is the specificity of the test? / A. < 0.5% / B. > 0.5% / C. < 99.5% / D. > 99.5% / E. It cannot be determined from the information given Explanation - Q: 5.4

Close

The correct answer is D. Serum and urine hCG tests are widely used to determine if a patient is pregnant. The development and use of these tests has been absolutely essential for the diagnosis and management of a variety of conditions including pregnancy, ectopic pregnancy, spontaneous abortion, gestational trophoblastic neoplasia, and some malignancies. However, it is important to always remember that no test is 100% perfect. Numerous case reports attest to the fact that using hCG to test for pregnancy will sometimes yield incorrect results. A false positive result refers to the situation when the test states that the patient does have the disease or condition (in this case pregnancy) when, in fact, the patient does not have the condition. A false positive rate of 0.5%

means that out of every 100 people who are not pregnant and take the pregnancy test, 0.5 will be told that they are pregnant. Multiplying this result by 2 (to get rid of the 0.5 person concept), it means that of every 200 people who are not pregnant, 1 will be falsely told that they are. A false-positive rate of < 0.5%, therefore, means that 6 years of age. Bitolterol (choice A) is a beta2 agonist, and is able to relieve reversible bronchospasm by relaxing the smooth muscles of the bronchioles. It is

indicated for the treatment for an acute asthma attack. Cromolyn sodium (choice B) acts as an antiasthmatic and an antiallergic mast cell stabilizer. By inhibiting the degranulation of mast cells, this agent prevents the release of histamine and SRS-A (composed of leukotrienes). Asthma induced by inhalation of antigens can be inhibited by varying degrees with cromolyn pretreatments. This agent has no bronchodilator, antihistaminic, anticholinergic, or anti-inflammatory activity. It is indicated for prophylactic management of severe bronchial asthma, prevention of exercise induced bronchospasm, and prevention of allergic rhinitis. Ipratropium (choice C) is an antimuscarinic agent that is structurally related to atropine. This agent is a quaternary amine (therefore, positively charged) and there is little systemic absorption. It is indicated for bronchospasm associated with COPD and rhinorrhea. Theophylline (choice E) is a xanthine derivative medication that relaxes smooth muscle. The mechanism of action may be related to its ability to block adenosine receptors or to inhibit phosphodiesterase. Theophylline is indicated for the symptomatic relief/prevention of bronchial asthma (acute, childhood, nocturnal) as well as reversible bronchospasm associated with chronic bronchitis or emphysema.

Later in life, the patient develops hypertension. Assuming that his respiratory condition is still present, which of the following agents would be the most appropriate pharmacotherapy? / / / / /

A. Atenolol B. Nadolol C. Propranolol D. Sotalol E. Timolol Explanation - Q: 2.8

Close

The correct answer is A. Patients with disease of the small airways are generally not prescribed beta-receptor blocking agents since these agents can block the bronchodilation produced by endogenous and exogenous catecholamine stimulation of the beta2 receptors. However, relatively low doses of selective beta1 receptor blocking agents, such as atenolol and metoprolol, are relatively well tolerated. All of the other answer choices are non-selective beta-receptor blocking

agents and should not be used in this patient since they are likely to exacerbate the patient's condition.

Vignette 3 of 5

A 3-year-old boy is brought to the emergency department with shortness of breath. His parents report that he has had several episodes in which he breathes heavily and turns blue. During these episodes, he is often found squatting, which appears to relieve his symptoms. His parents brought him in today, because the boy lost consciousness. On examination, he is a poorly developed, thin boy who is in acute distress. His skin appears blue, and he has labored breathing with chest retractions A systolic ejection murmur is auscultated at the left third intercostal space. A chest x-ray film shows a smalI, "boot shaped" cardiac silhouette. The boy is admitted to the hospitaI. Which of the following is the most likely diagnosis? / / / / /

A. Coarctation of the aorta B. Ebstein's anomaly C. Patent ductus arteriosus D. Tetralogy of Fallot E. Transposition of the great vessels Explanation - Q: 3.1

Close

The correct answer is D. Tetralogy of Fallot is a congenital heart anomaly that presents as respiratory distress, cyanosis, clubbing, syncope and sudden death. Historical findings often include reports of "Tet spells," in which the patient is found squatting to relieve respiratory symptoms. Examination findings include a systolic ejection murmur, clubbing, and cyanosis. A characteristic small "boot-shaped" cardiac silhouette is seen on chest x-ray. The anatomic defects of this tetralogy are pulmonary stenosis, overriding aorta, right ventricular hypertrophy, and ventricular septal defect. This anatomic tetralogy results in a right-to-left shunt, and thus a marked

decrease in pulmonary blood flow. The severity of symptoms depends on the degree of decrease in pulmonary blood flow. The squatting increases systemic vascular resistance, and decreases the right-left shunt. Coarctation of the aorta (choice A) is a narrowing of the thoracic aorta. Patients may have headache, epistaxis, and lower extremity claudication. On examination, patients have diminished lower extremity pulses, and a systolic or continuous murmur. This defect may be associated with CHF, aortic dissection, intracranial aneurysmal rupture, and bacterial endocarditis. Ebstein's anomaly (choice B) is an abnormal tricuspid valve placement that creates an abnormally large right atrium and small right ventricle. The disorder may present with cyanosis, but the clinical presentation may vary. You should suspect Ebstein's anomaly when there is a history of maternal lithium ingestion. Patient ductus arteriosus (choice C) involves a right to left shunt that may present as acyanotic respiratory distress. It will have a continuous "machinery" murmur. Transposition of the great vessels (choice E) typically presents as cyanosis and CHF in the neonatal period. An "egg shaped" cardiac contour is seen on chest x-ray films.

A complete blood count is performed, and the hemoglobin is markedly increased. Which of the following substances triggers this erythrocytosis? / / / / /

A. AIdosterone B. Angiotensin ll C. Erythropoietin D. Interleukin 1 E. Renin Explanation - Q: 3.2

Close

The correct answer is C. Erythropoietin is released from the kidney in response to renal hypoxia. This circulates to the red marrow and stimulates erythropoiesis, a process by which erythropoietic stem cells differentiate into red blood cells. Aldosterone (choice A) is a hormone released from the adrenal cortex in response to angiotensin II. It triggers sodium retention in the renal collecting

ducts. Angiotensin II (choice B) is a hormone created by a series of enzymatic steps. These steps are carried out when the kidney releases renin in response to decreased flow in the area of the renal tubule known as the macula densa. Interleukin 1 (choice D) is an inflammatory mediator. It has no effect on erythropoiesis. Renin (choice E) is an enzyme released by the kidney in response to decreased flow in the renal tubule. It initiates a cascade of enzymatic steps to create angiotensin II, which has several vascular, renal, and endocrine effects.

While this patient is in the hospital he begins to have fever and headache. On examination, he has numbness of the right side of his face, but no nuchal rigidity. A head CT shows a focal lesion with a hypodense center surrounded by a ring of enhancement. Which of the following is the most likely diagnosis? / / / / /

A. Brain abscess B. Guillain-Barr syndrome C. Meningitis D. Stroke E. Tuberculosis Explanation - Q: 3.3

Close

The correct answer is A. Patients with cyanotic heart disease (most commonly tetralogy of Fallot ) may develop a brain abscess. In chronically hypoxic patients, polycythemia with increased blood viscosity leads to poor cerebral capillary flow and reduced tissue oxygenation. This poorly perfused segment acts as a nidus for infection, and as a result, patients with tetralogy of Fallot suffer from brain abscesses. This is a typical presentation for a brain abscess, which may include fever, headache, seizures, nuchal rigidity, papilledema, and focal neurologic defects. The CT findings are also classic for brain abscess. Guillain-Barre syndrome (choice B) is an acute inflammatory demyelinating polyneuropathy. It typically presents as ascending weakness usually beginning in the legs. It does not present as focal CNS findings. Meningitis (choice C) would present as headache and fever, but focal

findings on neurologic examination and on CT would be unusual. Nuchal rigidity is often present. Stroke (choice D) would be unusual in this age group. In addition, the focal lesion seen on CT does not describe the findings of stroke. CT findings for stroke vary with the etiology of the stroke, but are typically less well circumscribed and do not display ring enhancement. Tuberculosis (choice E) could produce a meningitis, which afflicts the very old and very young. It has a long, protracted course as symptoms of fever, headache and nuchal rigidity tend to develop over a two-week course. A focal lesion would not be seen on CT scan.

A brain biopsy from this patient grows gram-positive cocci in chains. Which of the following is the most appropriate treatment?

/ / / / /

A. Cefazolin B. Ceftriaxone C. FIuconazole D. Metronidazole E. Penicillin Explanation - Q: 3.4

Close

The correct answer is B. The finding of gram-positive cocci in chains indicates the abscess contains streptococci. An antibiotic must not only be effective against these organisms, it must also penetrate the blood-brain barrier. Ceftriaxone is a third generation cephalosporin that penetrates the BBB. While the effectiveness against gram-positive organisms decreases as one moves from first to second to third generation cephalosporins, unless the medication arrives at the proper site, it cannot be effective. Cefazolin (choice A) is a first generation cephalosporin. It would kill streptococci, but it would not access the CNS. Fluconazole (choice C) is an antifungal agent. The culture indicates streptococci. Fungal brain abscess would be unusual in an immunocompetent patient. Metronidazole (choice D) is an antiparasitic that also has activity against gram-negative organisms. It would be appropriate to use in conjunction with ceftriaxone in an empiric setting. Once the definitive cause is known,

discontinuing extraneous antibiotics is appropriate. Penicillin (choice E) would kill streptococci, but it could not access the CNS in the presence of an intact blood-brain barrier.

Which of the following sets of changes depict the oxygen partial pressures in the aorta, Ieft ventricle, right atrium, and vena cava of this patient during resting conditions, compared to a healthy individuaI?

Explanation - Q: 3.5

Close

The correct answer is C. In Tetralogy of Fallot, the aorta originates from the right ventricle or overrides the septum, and therefore receives blood from both ventricles. This decreases the oxygen tension of aortic blood. Blood flowing through the lungs is still oxygenated normally, causing the oxygen tension of blood in the left ventricle to be normal. Because the oxygen content of the arterial blood is lower than normal, the oxygen tension of the venous blood (and therefore blood in the right atrium) is lower than normal.

Vignette 4 of 5

A 55-year-old man presents to the emergency department complaining of the acute onset of severe shortness of breath. His only associated symptom is a dry cough productive of scant frothy sputum. He has a 70 pack-year smoking history and has had two similar episodes in the past two years. On examination, he is afebrile, tachypneic, and distressed, but without cyanosis. He is thin, and his accessory muscles contract with each breath. He exhales through pursed lips. His chest examination reveals diminished breath sounds with hyperresonance to percussion. Which of the following is the most likely diagnosis?

/ / / / /

A. Chronic bronchitis B. Cystic fibrosis C. Emphysema D. Myocardial infarction E. Pneumonia Explanation - Q: 4.1

Close

The correct answer is C. Patients with COPD may be clinically classified as "pink puffers" or "blue bloaters" based on several characteristics. This patient is exhibiting the classic presentation for an emphysematous "pink puffer"; the patient has dyspnea but is not cyanotic. "Pink puffers" maintain their oxygenation until they decompensate precipitously. In contrast, the "blue bloater" counterparts with chronic bronchitis (choice A) have a chronic productive cough with cyanosis. Cystic fibrosis (choice B) is an inherited disease involving a defective chloride channel, and tends to affect patients at a younger age. The defective chloride channel leads to viscous mucus that is difficult to clear. As a result, patients develop chronic lung infections and fibrotic lung disease. Myocardial infarction (choice D) classically presents as crushing substernal chest pain, shortness of breath, and diaphoresis. Pain may radiate to the jaw and be accompanied by nausea. If congestive heart failure is present, patients may have dry cough, scant sputum, and rales on exam. Pulmonary infections may precipitate COPD exacerbations, but this patient's presentation is not consistent with pneumonia (choice E) . Pneumonia presents as dyspnea, fever, and productive cough.

In this disorder, the mechanism for decreased FEV1 (forced expiratory volume in 1 second) is which of the following? / / / / /

A. Airway collapse due to loss of elastic recoil B. Airway constriction due to bronchospasm C. BIockage of airways by increased mucus production D. Decreased lung compliance due to pulmonary fibrosis E. Lung collapse due to air in the pleural space Explanation - Q: 4.2 The correct answer is A. Patients with emphysema, a form of chronic

Close

obstructive lung disease, suffer from a loss of elastic recoil in the lung. As a patient exhales, the airways collapse, trapping air distal to the bronchiole. As a result of airway collapse, patients with emphysema have increased resistance to outflow, and a decreased FEV1. Asthma, another form of COPD, results from the obstruction of small airways due to bronchoconstriction (choice B). Blockage of airways due to mucus production (choice C) characterizes chronic bronchitis. While this obstruction produces COPD, it is not consistent with emphysema. Pulmonary fibrosis (choice D) would increase bronchiolar elasticity and therefore would increase FEV1. Lung collapse due to air in the pleural space (choice E) describes pneumothorax. This does not explain the decrease in FEV1 in emphysema.

Which of the following drugs might prove helpful in treating this patient? / / / / /

A. Acetazolamide B. AIbuterol C. Metoprolol D. Metronidazole E. Propranolol Explanation - Q: 4.3

Close

The correct answer is B. Albuterol is a beta-adrenergic agonist and, as such, promotes bronchial smooth muscle relaxation, and thus bronchodilation. Bronchodilation may relieve symptoms in COPD patients with acute exacerbation. Methylxanthines and/or anti-cholinergics may provide an adjunct to beta agonists. Acetazolamide (choice A) is a carbonic anhydrase inhibitor used as a diuretic. There is nothing in this patient to suggest that he is volume overloaded, and diuretics would thus be inappropriate. Propanolol and metoprolol (choices C and E) are beta blockers and may actually exacerbate this patient's symptoms through bronchoconstriction. Metronidazole (choice D) is an antimicrobial agent. While antibiotics are

often used to treat infections that may exacerbate COPD, metronidazole is not an appropriate agent to treat the pathogens that typically infect people with COPD: Haemophilus influenzae, Strep. pneumoniae, and Moraxella catarrhalis.

Which of the following sets of changes depict the mean arterial pressure (MAP), Ieft ventricular peak systolic pressure (LVPSP), pulmonary wedge pressure (PWP), and left atrial pressure (LAP) in this patient, compared to a healthy individuaI?

Explanation - Q: 4.4

Close

The correct answer is A. The elastic recoil of the lungs is decreased in emphysema. This causes the chest wall to expand sufficiently to create a new balance between the elastic recoil of the lungs and chest wall, which increases the functional residual capacity. Because the lungs are expanded to higher than normal levels, both the total lung capacity and residual volume are chronically increased. The vital capacity of the lungs (which is the difference in volume between the total lung capacity and residual volume) is decreased in emphysema because the patient cannot expel normal amounts of air from the lungs.

An inherited form of this disease can result from which of the following? / A. A deficiency of alveolar dipalmitoyl phosphatidylcholine / B. A deficiency of serum alpha-1-antitrypsin / C. A deficiency of serum angiotensin converting enzyme. / D. An elevated alveolar dipalmitoyl phosphatidylcholine / E. An elevated serum alpha-1-antitrypsin / F. An elevated serum angiotensin converting enzyme

Explanation - Q: 4.5

Close

The correct answer is B. Alpha 1-antitrypsin deficiency is associated with familial emphysema. Patients may have decreased (heterozygote) or absent (homozygote) amounts of the protease inhibitor alpha 1-antitrypsin. The precise mechanism in producing emphysema is unclear, but patients with this deficiency typically have severe disease, often with an early age of onset. Choice E therefore cannot be correct. Dipalmitoyl phosphatidylcholine (choices A and D) is a component of alveolar surfactant, which reduces alveolar surface tension. Not only does this surfactant reduce the surface tension, it changes surface tension with changing diameter. This prevents atelectasis by allowing interconnected alveoli with different diameters to remain open at the same alveolar pressures. (Without surfactant, the Law of LaPlace dictates that it takes greater alveolar pressures to open a smaller alveolus.) Elevated or decreased angiotensin converting enzyme (ACE; choices C and F) would not produce emphysema. ACE converts angiotensin I to angiotensin II, which has many effects on vascular and renal physiology, but is not associated with emphysema.

Which of the following nerves provides innervation to the diaphragm? / / / / /

A. Accessory nerve B. Intercostal nerve C. Phrenic nerve D. Splanchnic nerve E. Vagus nerve Explanation - Q: 4.6

Close

The correct answer is C. The phrenic nerve originates from cervical roots 3, 4, and 5 to provide innervation to the diaphragm. This is significant because spinal cord injury above this level renders the patient unable to breathe. The accessory nerve (choice A), provides motor innervation to the trapezius and the sternocleidomastoid muscles. The intercostal nerves (choice B) innervate the intercostal muscles responsible for chest wall expansion and retraction. The splanchnic nerve (choice D) provides motor and autonomic fibers to the

gut. The vagus nerve (choice E) provides motor innervation to the vocal cords, heart, bronchus and GI tract. In addition it provides sensory innervation to the bronchus, heart, GI tract and larynx.

Vignette 5 of 5

A 20-year-old woman is brought to the emergency department after a severe traffic collision. Initial assessment reveals a tachypneic, tachycardic, hypotensive woman in acute distress. On examination, she has multiple contusions on the left Iateral chest wall and jugular venous distention (JVD). Her chest is hyperresonant to percussion and she has diminished breath sounds on the left. Her trachea deviates to the right. A chest x-ray film reveals diminished vascular markings on the Ieft. An ECG shows sinus tachycardia. Which of the following is the most likely diagnosis? / / / / /

A. Cardiac contusion B. Hypovolemic shock C. Pericardial tamponade D. Pulmonary contusion E. Tension pneumothorax Explanation - Q: 5.1

Close

The correct answer is E. This patient has the classic findings for tension pneumothorax. She has a history of severe trauma, and the findings of dyspnea, tachypnea, tachycardia, tracheal deviation, unilaterally decreased breath sounds, and hyperresonance to percussion. This is a life-threatening condition that must be remedied immediately. Cardiac contusion (choice A) would not present with these findings. Patients will be hemodynamically unstable, often showing various arrhythmias on ECG. They will not have tracheal deviation, JVD, or hyperresonance to percussion. Hypovolemic shock (choice B) could not account for the patient's jugular venous distention, hyperresonance to percussion, diminished breath sounds,

tracheal, or chest x-ray findings. Pericardial tamponade (choice C) is a life-threatening condition often seen in the trauma setting. Beck's triad characterizes pericardial tamponade: decreased heart sounds, jugular venous distention, and hypotension. Patients may also be tachycardic with pulsus paradoxus. Pulmonary contusion (choice D) may produce respiratory distress immediately after trauma occurs but usually complicates the chronic course. It would not produce tracheal deviation, or hyperresonance to percussion. On x-ray films it would appear as an ill-defined opacity.

Which of the following is the most likely mechanism of this patient's shock? / / / / /

A. FIuid in the pericardial space prevents diastolic filling B. Hypoxia from rapid loss of pulmonary capacity impedes cardiac function C. Malpositioning of the great vessels has impeded venous return D. Massive bleeding into the pleural space has led to hypovolemia E. Myocardial dyskinesia has led to cardiac insufficiency Explanation - Q: 5.2

Close

The correct answer is C. As discussed above, this patient is suffering from tension pneumothorax, and the mechanism of shock in this condition is mechanical. As air becomes trapped in the pathologic side of the chest, the ipsilateral chest cavity expands. The enlarging cavity begins to encroach upon the mediastinal and contralateral chest contents, and as the contents herniate contralaterally, the great vessels become kinked and compressed. This inhibits venous return and thus diastolic filling. This results in shock. Immediate decompression of the chest will restore venous return. As discussed above, this patient does not have pericardial tamponade (choice A). In pericardial tamponade, fluid in the pericardial sac surrounding the heart prevents diastolic filling, resulting in shock. Immediate decompression of the pericardial space is indicated. While hypoxia (choice B) may result from pneumothorax, this is not the primary reason for the patient's shock. Massive bleeding into the pleural space (hemothorax; choice D) may cause shock in a fashion similar to pneumothorax. It is unlikely that this patient has a hemothorax, however, as evidenced by the hyperresonance to percussion.

Choice E is incorrect. Severely contused myocardium would likely show ECG abnormalities. A normal ECG makes this diagnosis less likely, and thus not a likely reason for this patient's shock.

Which of the following is the most appropriate immediate management? / / / / /

A. FIuid bolus B. Needle thoracostomy C. Open thoracotomy D. Pacemaker placement E. Pericardiocentesis

Explanation - Q: 5.3

Close

The correct answer is B. Since the tension within the chest cavity creates the danger, relieving it is the mainstay of treatment. Continued tension will continue to push the chest contents to the side contralateral to injury. This will "kink" the venous return to the heart leading to diminished end-diastolic volume with imminent cardiac collapse. In addition, the uninjured lung will have limited inspiratory capacity. Thus to accomplish immediate decompression, a needle is inserted in the second intercostal space at the mid-clavicular line. This is followed by placement of a chest tube to manage the pneumothorax more chronically. This patient may have hypovolemia, as bleeding may be present, but her hemodynamic instability may be entirely due to the tension pneumothorax. Thus, fluid (choice A) may be given later, but treating the pneumothorax is a higher priority. Open thoracotomy (choice C) would treat the pneumothorax, but would take longer than needle decompression, and may be more than is needed. This case has given no justification for pacing the heart (choice D). Pericardiocentesis (choice E) is used to treat pericardial tamponade, not pneumothorax.

When instrumenting the chest, instruments are inserted into the chest along the superior surface of the rib to avoid which of the following structures?

/ / / / /

A. External oblique muscle B. Intercostal artery C. Parietal pleura D. Phrenic nerve E. Visceral pleura Explanation - Q: 5.4

Close

The correct answer is B. The intercostal vein, artery, and nerve traverse the inferior surface of the ribs in the intercostal groove. When placing instruments into the chest, these structures must be avoided. Damaging these structures can result in bleeding into the chest and may cause significant hemothorax. Thus, instruments are inserted along the superior surface of the rib. The external oblique muscle (choice A) runs between the ribs from superior to inferior and laterally to medially. (Like the fingers of a hand placed in pants pockets.) This structure in invariably pierced when instrumenting the chest. The parietal pleura (choice C) is the layer of pleura that adheres to the chest wall. It is also pierced when instrumenting the chest. The phrenic nerve (choice D) innervates the diaphragm from spinal segments C3-5. It travels in the mediastinum and would not be at risk here. The visceral pleura (choice E) is the thin layer of pleura that envelops the lung. It would be at risk for damage if the instruments were inserted into the chest too far. Placing the instrument in the proper orientation to the rib is of no consequence

During a routine check up of a two-year-old boy, the patient's mother mentions she's noticed intermittent and recurrent rashes on her son in the past year. The frequency of the rash increases with cold, dry weather. The rash does not appear to be tender, but is associated with moderateto-severe pruritus. The patient has no significant birth history or past medical history. Physical examination reveals a healthy, welI-nourished two-year-old boy, in no acute distress. Cutaneous examination shows lichenified, dried plaques with scaling covering the plaque surface, Iocated on the antecubital and popliteal fossae, eyelids, face and the neck. There are multiple excoriated papules scattered within the plaques. Question 1 of 5

Which of the following history is important in ascertaining the correct diagnosis? / A. Family cat with fungal infection / B. Family history of asthma, hay fever, or eczema / C. Family history of psoriasis / D. Perinatal history of cradle cap / E. Recent travel history Explanation - Q: 1.1

Close

The correct answer is B. This patient has the hallmark presentation of childhood atopic dermatitis, also loosely termed eczema. These lesions typically have a lichenified, papular, slightly scaly appearance. The classic locations are the antecubital and popliteal fossa, flexor wrist, eyelids, face, and around the folds of the neck. Often there are intermingled excoriated papules, suggesting the pruritic nature of these lesions. Patients with atopic dermatitis often also have asthma or hay fever, and a family history of asthma, hay fever, and eczema. A family cat with a fungal infection (choice A) would suggest a diagnosis of tinea, which usually presents on both cheeks of children after rubbing their face against pets. Psoriasis (choice C), though it tends to run in families, does not result in an atopic tendency leading to dry, sensitive skin. A perinatal history of cradle cap (choice D) is referring to seborrheic dermatitis (dandruff) of the scalp. Again, this has not been shown to be associated with atopic dermatitis in later childhood. A recent travel history (choice E) would be consistent with an infectious origin. Because of the intermittent occurrence of the lesions, in addition to lack of other systemic signs of infection (such as fever, night sweats, malaise), a systemic infectious process is unlikely.

Question 2 of 5

After further questioning, the patient's mother reports similar lesions in the patient's father during his childhood. Which of the following is the most likely diagnosis? / A. Bacterial infection / B. Eczema / C. Fungal infection / D. Psoriasis / E. Spider bites Explanation - Q: 1.2

Close

The correct answer is B. Patients with atopic dermatitis tend to have a positive family history of asthma, hay fever, and eczema. Bacterial infection (choice A), when widespread, usually is associated with systemic symptoms as well as cutaneous findings, such as fever, increased irritability, and malaise. Also, the lesions tend to be impetiginized (yellowish exudate and crust overlying an erythematous base). Fungal infection (choice C) usually appears on faces and scalps of children, especially in daycare settings. The morphology of tinea tend to be annular, erythematous plaques with central clearing and a rim of scale on the peripheral border, thus also known as "ring worm." Psoriasis (choice D) tends to occur over extensor surfaces of the extremities, scalp, and intergluteal fold. In children, lesions can also be seen in diaper areas, resulting in misdiagnosis of diaper rash. Typically, lesions tend to be covered by thick, silvery scales. Spider bites (choice E), as well as other insect bites, tend to occur on the lower extremities. With widespread involvement, one should see bites in different stages ranging from erythematous and urticarial to bulla formation and necrotic centers

Question 3 of 5

Which of the following tests would most likely confirm the probable diagnosis? / A. Bacterial culture / B. Erythrocyte sedimentation rate (ESR) / C. Fungal culture / D. HLA-B27 / E. Immunoglobulin E (IgE) Ievel Explanation - Q: 1.3

Close

The correct answer is E. Patients with atopy can often have an elevated IgE level, correlating with increased sensitivity, as well as with the pruritic nature of the lesions. However, a normal level of IgE does not rule out eczema, which is mostly a clinical diagnosis. Bacterial culture (choice A) in eczematous lesions will occasionally grow out gram-positive staphylococcal organisms, but does not confirm a diagnosis of atopic dermatitis. Elevated erythrocyte sedimentation rate (choice B) is commonly seen in inflammatory and infectious processes. Fungal culture (choice C) would be the appropriate test to confirm tinea lesions. Human leukocyte antigen (HLA) B27 (choice D) is seen in different syndromes and diseases including psoriatic arthritis and Reiter syndrome.

Question 4 of 5

Which of the following is the most appropriate first-Iine therapy for this patient? / A. Oral antibiotics / B. Oral antifungals / C. Topical antifungals / D. Topical steroids / E. UItraviolet light Explanation - Q: 1.4

Close

The correct answer is D. Topical steroids are effective in the treatment of atopic dermatitis. Oral antibiotics (choice A) can occasionally be effective in atopic dermatitis patients, especially those superinfected with gram-positive organisms, but are not considered first-line therapy. Oral antifungals (choice B) are used in kids with tinea infection, particularly tinea capitis (fungal infection of the scalp). Topical antifungals (choice C) are used in Candida and other fungal infections. Ultraviolet light (choice E) treatment has been found to be effective in the treatment of atopic dermatitis by modulating the immune response in the skin. However, the risk of future skin cancer secondary to ultraviolet

exposure makes this therapy undesirable in children. However, the risk of future skin cancer secondary to ultraviolet exposure makes this therapy undesirable in children.

Question 5 of 5

On his one month follow up visit, the patient's mother complains that despite frequent bathing and prescribed therapy, her son is not improved. What other factors should be addressed prior to changing therapy? / A. Add prophylactic oral antibiotic therapy / B. Avoid excessive external irritation / C. Check bacterial culture for sensitivity and specificity / D. Refer patient for arthritis workup / E. Take the family cat to the veterinarian for proper treatment Explanation - Q: 1.5

Close

The correct answer is B. Any external irritation may precipitate an attack of eczema. These include excessive bathing, vigorous rubbing, or chafing. Local infection, irritating secretions, medicated baby oils, and soap and water may aggravate the disease. Prophylactic oral antibiotic therapy (choice A) is not necessary unless the patient has had repeated episodes of superinfection in the past. Bacterial culture for sensitivity and specificity (choice B) is applicable if patient has impetiginized lesions that appear to be infected and has failed a course of oral antibiotics. Referral for arthritis workup (choice D) should be considered in patients with severe psoriasis and joint complaints. Taking the family cat to the veterinarian (choice E) is important in treating young children with tinea (fungal infection).

A 35-year-old woman consults a dermatologist because of a persistent facial rash. Physical examination demonstrates an erythematous rash, without blistering or ulceration, involving both cheeks and the nose. The nasolabial folds are relatively spared. The dermatologist also notes scattered erythematous, firm, maculopapular lesions elsewhere on the woman's face and on exposed areas of the neck, upper chest, and elbows. Careful examination of the scalp demonstrates a few areas of focal alopecia. On questioning, the woman reports that she has had the

rash more or less continuously for about six months, and that it gets much worse when she is exposed to sunlight. Question 1 of 5

Which of the following autoimmune diseases would most likely produce this patient's skin problems? / A. Dermatomyositis / B. Progressive systemic sclerosis / C. Rheumatoid arthritis / D. Sjögren syndrome / E. Systemic lupus erythematosus Explanation - Q: 2.1

Close

The correct answer is E. Systemic lupus erythematosus (SLE) is a systemic autoimmune disorder in which tissues and cells are damaged by pathogenic autoantibodies (especially antinuclear antibodies) and immune complexes. Approximately 90% of systemic lupus erythematosus occurs in women. SLE is associated with a wide variety of manifestations, including characteristic skin rashes, arthritis, anemia, seizures, psychiatric illness, and may affect the kidneys, lungs, and heart. The rash involving the nose and cheeks is the characteristic malar "butterfly" rash of SLE; significant features include the rash distribution, fixed erythema, sparing of the nasolabial folds, and exacerbation with sunlight exposure. The rash is not the only cutaneous manifestation of lupus; alopecia and the maculopapular rash described in the case may also be seen. The rash of dermatomyositis (choice A) is characteristically described as a lilac or heliotrope discoloration of the upper eyelids accompanied by periorbital edema. Progressive systemic sclerosis (choice B), or scleroderma, produces thick, dense skin secondary to dermal fibrosis. Rheumatoid arthritis (choice C) characteristically produces subcutaneous rheumatoid nodules. Sjögren syndrome (choice D) does not have a characteristic skin manifestation.

Question 2 of 5

The dermatologist biopsies the rash and sends the sample for pathologic examination. The biopsy shows liquefactive degeneration of the basal layer of the epidermis accompanied by edema at the dermal junction. Immunofluorescence microscopy would most likely demonstrate which of the following?

/ A. Granular deposition of immunoglobulin and complement along the

dermoepidermal junction / B. Granular deposits of lgA selectively localized to the tips of dermal papillae / C. Lacy network of lgG deposits in the intercellular spaces lining the keratinocytes / D. Linear basement membrane depositions of immunoglobulin and complement / E. No deposits of immunoglobulin or complement Explanation - Q: 2.2

Close

The correct answer is A. While the etiology of systemic lupus erythematosus is still unproven, it is suspected to involve alterations in immunologic control that predispose for the formation of a wide variety of autoantibodies. Both systemic lupus erythematosus and discoid lupus erythematosus characteristically show granular deposition of immunoglobulin and complement along the dermal epidermal junction; in discoid lupus these deposits are confined to the area of rash, while in systemic lupus they can also be seen in clinically "normal" skin. Choice B is seen in dermatitis herpetiformis. Choice C is seen in pemphigus vulgaris. Choice D is seen in bullous pemphigoid.

*** Commercial version is infinite. Order at http://www.structurise.com/kleptomania *** Question 3 of 5

The patient is referred to an internal medicine specialist for further evaluation. The initial immunologic work-up demonstrates a high titer positive ANA test, which is followed by the demonstration of other more specific antibodies. High titers of which of the following are considered to be highly specific for this patient's disease? / A. Anti dsDNA / B. Anti Jo-1 / C. Anti SS-A / D. Anti SS-B / E. Anti-U1RNP Explanation - Q: 2.3

Close

The correct answer is A. A wide variety of autoantibodies can be seen in systemic lupus erythematosus, including all of those listed. However, high titers of anti-double-stranded DNA and anti-Sm (not listed above) are

considered to be the most specific for systemic lupus erythematosus. Anti-Jo-1 (choice B) is associated with inflammatory myopathies. Anti SS-A (choice C) and anti-SS-B (choice D) are associated with Sjögren syndrome. Anti-U1RNP (choice E) is associated with SLE, but is not very specific.

Question 4 of 5

Urinalysis in this patient demonstrates severe proteinuria and microscopic hematuria. Renal biopsy shows marked glomerular changes with a marked increase in cellularity primarily related to the proliferation of endothelial and mesangial cells. Fibrinoid necrosis and hyaline thrombi are seen in the glomerular capillary beds. Some of the glomeruli have epithelial crescents that filI Bowman's space. These changes are most consistent with which of the following? / A. Diffuse proliferative glomerulonephritis / B. Focal proliferative glomerulonephritis / C. Membranous glomerulonephritis / D. Mesangial lupus glomerulonephritis / E. Tubulointerstitial nephritis Explanation - Q: 2.4

Close

The correct answer is A. Systemic lupus erythematosus can present in a wide variety of ways, and can have an even wider variety of complications, making diagnosis difficult. SLE affects many organs in the body, but the kidneys are a particular target. Choices A to D above can all be seen in different lupus patients, and sometimes in the same patient at different times or different biopsy sites. The specific set of lesions described in the question are those of diffuse proliferative glomerulonephritis, which is the most serious of the renal lesions of lupus, and occurs in 35-40% of patients who are biopsied. Tubulointerstitial nephritis (choice E) is not usually described in lupus patients.

Question 5 of 5

With modern therapeutic management, the 10-year survival for patients with this woman's disease in most developed countries is now approximately which of the following? / / / / /

A. Less than 5% B. 20% C. 50% D. 80% E. More than 95%

Explanation - Q: 2.5

Close

The correct answer is E. Modern management of systemic lupus erythematosus has markedly improved the prognosis of systemic lupus erythematosus, and the key appears to be good control of the initial acute phase. Treatment for SLE depends on which clinical problems are present and on whether the disease is active at that time. NSAIDs are helpful for mild inflammation; corticosteroids are the mainstay of treatment for SLE, but should be used judiciously. If used, bone protection is important. Hydroxychloroquine and other anti-malarials reduce SLE activity and help with symptoms involving the joints and skin. Severe SLE requires immunosuppressive agents such as cyclophosphamide and azathioprine.

A 55-year-old man presents to the emergency department with a 2-week history of worsening "sores and blisters." He states that the lesions initially started as sores in his mouth. Over the course of two weeks, he's noted progression, with new blister formation on his face and scalp. One day prior to presentation, he developed similar lesions on his genital area as well as his axillae. He is concerned about a possible correlation with his recent unprotected sexual activity with an acquaintance. On examination, he appears welI-nourished and is afebrile. Cutaneous examination reveals lesions in various stages of healing. Over the intertriginous areas, there are several flaccid, easily ruptured, bullae that appear on erythematous bases. Some of the bullae have ruptured, forming superficial erosions. OIder lesions are covered with crusts that have little or no tendency to heaI. Some of the erosions appear to enlarge by coalescence. Similar lesions are found on the scalp, face, neck, axillae, and the genital area, as well as the oral mucosa. These lesions are moderately tender to palpation. Question 1 of 6

Which of the following is the most likely diagnosis?

/ / / / /

A. Bullous pemphigoid B. Gonococcal dermatitis C. Herpes simplex infection D. Pemphigus vulgaris E. Syphilis Explanation - Q: 3.1

Close

The correct answer is D. Pemphigus vulgaris is an autoimmune blistering disorder characterized by formation of thin-walled, relatively flaccid, easily ruptured, bullae that appear on either apparently normal skin and mucous membranes or on erythematous bases. The bullae usually rupture to form erosions with raw surfaces that ooze and bleed easily. The denuded areas soon become partially covered with crusts that enlarge by confluence. The mouth lesions appear first in 60% of cases. The short-lived bullae quickly rupture to involve most of the mucosa with painful erosions. Cutaneous lesions are commonly seen in the groin, scalp, face, neck, and axillae. Bullous pemphigoid (choice A) is an autoimmune disease characterized by large, tense, subepidermal bullae with a predilection for the groin, axillae, and flexor surfaces. Involvement of the pharynx, mucosa and eye is rare. After the bullae rupture, large denuded areas are seen, but these do not materially increase in size as they do in pemphigus vulgaris. Instead, the denuded areas show a tendency to heal spontaneously. Gonococcal dermatitis (choice B) in gonococcemia is characterized by a hemorrhagic, vesiculopustular eruption, bouts of fever, and arthralgia of one or several joints. The skin lesions begin as tiny erythematous macules that evolve into vesicopustules on a deeply erythematous base. Herpes simplex virus (HSV, choice C) is one of the most common infections worldwide. HSV 1 infection is the cause of most cases of orolabial herpes simplex. Infected patients with HSV-2 are either completely asymptomatic, or have recurrent genital herpes that presents as painful ulcers and erosions. Syphilis (choice E) is a sexually transmitted disease. Primary syphilis generally presents with a nontender, clean-based ulcer on the genitals.

Question 2 of 6

During the examination, the physician notes that slight pressure on the intact bullae causes peripheral spreading of the lesion. Which of the following most accurately describes this phenomenon? / A. Cicatricial pemphigoid / B. Epidermolysis bullosa acquisita

/ C. Nikolsky sign / D. Split papule / E. Target sign

Explanation - Q: 3.2

Close

The correct answer is C. The Nikolsky sign is a result of lack of cohesion in the epidermis, so the upper layers can easily be made to slip laterally by slight pressure or rubbing. Cicatricial pemphigoid (choice A) is actually a variant of bullous pemphigoid resulting in scarring, primarily of the mucosa. Epidermolysis bullosa acquisita (choice B) is another bullous disorder. It is not hereditary, and is characterized by mechanobullous eruption with noninflammatory scarring that occurs in the elderly population. Split papule (choice D) is one of the cutaneous manifestations of secondary syphilis. Target sign (choice E) refers to the targetoid lesions seen in erythema multiforme.

Question 3 of 6

The etiology of this disorder is best characterized by which of the following? / A. Autoimmune blistering / B. Bullous impetigo / C. Exfoliative toxin-induced / D. Mechanobullous / E. Superantigen phenomenon Explanation - Q: 3.3

Close

The correct answer is A. Pemphigus vulgaris is an autoimmune blistering disease mediated by intercellular antibodies. These are demonstrable throughout the epidermis and the oral epithelium, and circulating intercellular antibodies are present. Direct immunofluorescence is of great value in the early diagnosis of pemphigus vulgaris. Bullous impetigo (choice B) is characterized by large, fragile bullae, suggestive of pemphigus. When these rupture, they leave circinate, weepy, or crusted lesions. The majority of these are caused by phage 71 coagulasepositive Staph. aureus.

Exfoliative toxin-induced (choice C) would apply to skin erythema and desquamation caused by phage type 71. A mechanobullous (choice D) mechanism is seen in epidermolysis bullosa acquisita. The superantigen phenomenon (choice E) refers to (usually) bacterial antigens that activate all T-lymphocytes that have a T-cell receptor with a particular V β sequence. As a consequence, large numbers of T-cells are activated and systemic reactions such as shock are induced.

Question 4 of 6

Which of the following tests would be most useful for confirmation of the likely diagnosis? / A. Bacterial culture / B. Complete blood count / C. Fungal culture / D. Rapid plasma reagin (RPR) / E. Tissue biopsy with direct immunofluorescence Explanation - Q: 3.4

Close

The correct answer is E. Direct immunofluorescence (DIF) is of great value in the early diagnosis of pemphigus vulgaris. DIF shows intercellular IgG throughout the epidermis or the oral epithelium. IgG is found in both involved and clinically normal skin in nearly all patients with pemphigus. In acantholytic areas, C3 deposition is also reliably found. DIF results remain positive for a long time and may still be positive many years after clinical remission. Bacterial culture (choice A) is helpful in superinfected lesions, but does not help in confirming a diagnosis of pemphigus. Complete blood count (choice B) is helpful in infectious processes. Fungal culture (choice C) is used in tinea lesions. The rapid plasma reagin (choice D) is a serum test used in diagnosing syphilis.

Question 5 of 6

A biopsy of affected skin would show which of the following? / A. Acantholysis / B. Balloon degeneration / C. Reticular changes / D. Spongiosis / E. Subcorneal pustulosis Explanation - Q: 3.5

Close

The correct answer is A. The pathologic changes in pemphigus vulgaris are acantholysis, clefts, and blister formation in the intraepidermal areas. Acantholysis is the separation of keratinocytes from one another. The loss of cohesion or contact between cells begins with the detachment of tonofilaments from desmosomes. Evidence indicates that an IgG autoantibody actually induces these changes. Balloon degeneration (choice B) and reticular changes (choice C) are both histologic markers of herpes simplex viral-induced lesions. Spongiosis (choice D) is a general term referring to serous exudates between cells of the epidermis, with an inflammatory infiltrate in the dermis. This is frequently seen in dermatitic lesions. Subcorneal pustulosis (choice E) describes accumulation of neutrophils underneath the stratum corneum. This can be seen in a number of different disorders, including fungal infection, staphylococcal scalded-skin syndrome, and psoriasis.

Question 6 of 6

Which of the following would be the most appropriate pharmacotherapy for this disorder? / A. Antifungal agent / B. Gold salts / C. Oral antibiotic / D. Penicillamine / E. Systemic corticosteroid Explanation - Q: 3.6

Close

The correct answer is E. Corticosteroids are the standard therapy for pemphigus vulgaris. Generally, prednisone is given alone, or in combination with one or several immunosuppressants. In addition, pemphigus antibody titers are performed every 4 weeks, watching for a fall in titer. Medication is continued until clinical disease is suppressed and pemphigus antibody

disappears from the serum. Antifungal therapy (choice A) has no role in the treatment of pemphigus vulgaris. Oral antifungals are often employed in treatment of onychomycosis. Gold therapy (choice B) has been reported to be effective in the treatment of pemphigus, but is infrequently used. Bone marrow suppression and nephrotoxicity may occur. Oral antibiotics (choice C), such as tetracycline, in combination with nicotinamide, have had some success in controlling pemphigus, but are not considered the gold standard. Penicillamine (choice D) is used in Wilson syndrome; it has no role in pemphigus therapy.

A 37-year-old white man presents with a chief complaint of rash for 2-3 years. He says the rash initially started on the intergluteal fold, but progressed to involve areas on his extremities and trunk. He admits to occasionaI, moderate-to-severe pruritus with the rash. On cutaneous examination, there are large erythematous plaques with thick micaceous, silvery scales located on extensor surface of his elbows and knees bilaterally. Similar lesions are seen on his lower back and intergluteal fold. In addition, there are thick, welI-defined scaly plaques on his scalp, extending beyond the hairline.

Question 1 of 5

Which of the following is the most likely diagnosis? / A. Drug eruption / B. Eczema / C. Psoriasis / D. Reiter syndrome / E. Zinc deficiency Explanation - Q: 4.1

Close

The correct answer is C. Psoriasis is a common skin disorder characterized by round, circumscribed, erythematous, dry, scaling plaques of various sizes, covered by grayish white, imbricated, and silvery scales. The lesions have a predilection for the scalp, extensor surfaces of the limbs, elbows, knees, and sacral region. The eruption is usually symmetrical and may vary from solitary

to more than 100 lesions. The eruption usually develops slowly. Subjective symptoms, such as itching or burning, may be present and may cause extreme discomfort. Drug eruptions (choice A) usually present as morbilliform erythematous lesions, initially involving the trunk and extending to the extremities. The lesions tend to be papular, coalescing into plaques without well-defined borders. Eczema (choice B) is a common inflammatory skin disorder; patients often have a strong family history for the disorder. However, eczematous lesions are lichenified and do not have thick, micaceous scales overlying the lesions. Also, the typical location for eczema in young adult would be flexural surface areas, such as the popliteal and antecubital fossas. Reiter syndrome (choice D) is a characteristic clinical triad consisting of urethritis, conjunctivitis, and arthritis. It may also involve the skin, mucous membranes, gastrointestinal tract, and cardiovascular system. Zinc deficiency (choice E) presents most commonly in infants or patients on long term total parenteral nutritional supplements. The dermatitis found in all forms of zinc deficiency is pustular and bullous.

Question 2 of 5

A biopsy from one of the lesions would most likely show which of the following? / A. Acanthosis with intracorneal pustule formation / B. Basal cell liquefaction with band-Iike chronic inflammatory infiltrate in the papillary dermis / C. Hyperkeratosis with intra-epidermal clefts containing "corps ronds" / D. Large intraepidermal cleft in the suprabasal region / E. Large subepidermal blister with sparse inflammatory infiltrate Explanation - Q: 4.2

Close

The correct answer is A. Microscopically, psoriasis characteristically shows epidermal hyperplasia (acanthosis), often with fusion of rete ridges. Another distinctive feature in psoriasis is the presence of subcorneal and intracorneal pustules (sometimes called microabscesses of Munro) containing aggregated neutrophils. Choice B suggests lichen planus and other lichenoid lesions. Choice C suggests Darier's disease.

Choice D suggests pemphigus vulgaris. Choice E suggests bullous pemphigoid.

Question 3 of 5

Which of the following is the most appropriate next step in therapy? / A. Antibiotic / B. Antifungal / C. Corticosteroid / D. Stopping all of his oral medications / E. Vitamin supplement Explanation - Q: 4.3

Close

The correct answer is C. Topical application of corticosteroids in creams, ointments, lotions, and sprays is the mainstay of therapy for psoriasis. Antibiotics (choice A) are used in patients with eczema to augment superinfection with gram-positive organisms. Antifungals (choice B) are used in tinea infection or seborrheic dermatitis, but has no role in psoriatic therapy. Certain classes of medications can exacerbate psoriatic lesions, such as beta-blockers and lithium, but stopping all of his medications (choice D) would be a drastic step. Vitamin supplementation (choice E) does not play a role in psoriasis therapy.

Question 4 of 5

Therapy is initiated and the patient returns to the clinic 4 months later with a complaint of nail changes. What other extracutaneous manifestation is commonly associated with this new symptom? / A. Arthritis / B. Asthma / C. Diarrhea / D. Hepatitis / E. Renal failure Explanation - Q: 4.4

Close

The correct answer is A. Psoriatic arthritis is commonly seen in patient with

cutaneous and/or nail findings. It usually presents as asymmetrical distal interphalangeal joint involvement with nail damage. Asthma (choice B) is often associated with eczema or atopic dermatitis and hay fever, the triad of atopy. Diarrhea (choice C) is associated with zinc deficiency. Hepatitis (choice D) and renal failure (choice E) can be complications of therapy for psoriasis

Question 5 of 5

The most likely prognosis for this patient is which of the following? / A. 100% of patients are cured with proper management / B. Once disease free for one year, the likelihood of recurrence is very low / C. Only 10% cure rate with available therapy / D. Only recurs with upper respiratory infections / E. Persistent and recurrent disease Explanation - Q: 4.5

Close

The correct answer is E. The course of psoriasis is unpredictable. It usually begins on the scalp or elbows and may remain localized in the original region for an indefinite period, or completely disappear, recur, or spread to other parts. Two of the chief features of psoriasis are its tendency to recur and its persistence. Occasionally, patients remain disease free for years only to have lesions recur (choice B). The mechanism of psoriatic lesions has not been clarified, but underlying immune and HLA-linkage have been implicated. At this time, cure of the disease has not been achieved with therapy available (choices A and C). A variant of psoriasis, known as guttate psoriasis, can be exacerbated by upper respiratory infections (choice D).

An anxious mother brings in her 14-month-old daughter because of a rash for the past 3 days. She states that approximately 5 days prior to the onset of rash, her daughter had fever, cough, sneezing, and nasal congestion associated with malaise. At that time, she was seen by her

regular pediatrician with a diagnosis of viral upper respiratory infection. After treatment with acetaminophen and decongestants, the fever continued and rash appeared initially on her face. By the third day, the rash extended down to the trunk and extremities. On examination, the patient seems irritable. Cutaneous examination reveals erythematous, morbilliform papules coalescing into plaques involving the face, trunk, and extremities. 1 mm white papules on an erythematous base are seen on the buccal mucosa. Neurologic examination is normaI. Question 1 of 6

Which of the following is the most likely diagnosis? / A. Chicken pox / B. German measles / C. Measles / D. Molluscum contagiosum / E. Roseola Explanation - Q: 5.1

Close

The correct answer is C. Measles, also known as rubeola and morbilli, is a worldwide disease that most commonly affects children under 15 months of age. It is spread by respiratory droplets and has an incubation period of 9-12 days. The prodrome consists of fever, malaise, conjunctivitis, and prominent upper respiratory symptoms (nasal congestion, sneezing, coryza, and cough). After 1-7 days, the exanthem appears, usually as macular or maculopapular lesions on the scalp line and behind the ears. The rash spreads quickly over the face, then by the second or third day (unlike the more rapid spread of rubella) extends down the trunk to the extremities. Chicken pox (choice A) also known as varicella, represents primary infection with the varicella zoster virus. Transmission is by direct contact with the lesions and by the respiratory route. There is an initial viremia between days 4 and 6, seeding the liver, spleen, lung, and perhaps other organs. A secondary viremia occurs from day 11 to 20, resulting in infection of the epidermis. Varicella is characterized by a vesicular eruption consisting of delicate "teardrop" vesicles on an erythematous base. German measles (choice B) is also known as rubella. It has a prodrome of 1-5 days, consisting of fever, malaise, sore throat, eye pain, headache, and red eyes. Pain on lateral and upward eye movement is characteristic. The exanthem also begins on the face, spreads to cover the entire body within 24 hours, and resolves by the third day. Molluscum contagiosum (choice D) results in smooth-surfaced, firm, domeshaped, pearly papules, averaging 3-5 mm in diameter. A central

umbilication is characteristic. Irritated lesions may become crusted and even pustular. Lesions tend to be on the face, trunk, and extremities. Roseola (choice E) is a common cause of sudden, unexplained high fever in young children between 6 and 36 months of age. Prodromal fever is usually high. Suddenly on about the 4th day, the fever drops. Coincident with the drop in temperature, a morbilliform erythema consisting of rose-colored discrete macules appears on the neck, trunk, and buttocks. Complete resolution of the eruption occurs in 1 to 2 days.

Question 2 of 6

The white spots on the buccal mucosa are also known as which of the following? / A. Forscheimer's sign / B. Hand-foot-mouth disease / C. Herpangina / D. Koplik's spots / E. Target sign Explanation - Q: 5.2

Close

The correct answer is D. Koplik's spots are pathognomonic for measles and appear during the prodrome. They initially appear on the buccal mucosa nearest to the lower molars as 1 mm white papules on an erythematous base. After 6 to 7 days, the exanthem clears, with simultaneous subsidence of the fever. Forscheimer's sign (choice A) is an enanthem of pinhead-sized red macules or petechiae on the soft palate and uvuvla in patients with rubella. Hand-foot-mouth disease (choice B) is most frequently caused by coxsackieviruses. Small, rapidly ulcerating vesicles surrounded by a red areola are found on the buccal mucosa, tongue, soft palate, and gingiva. Herpangina (choice C) is caused by multiple types of coxsackieviruses and echoviruses. The characteristic finding is one or more yellowish white, slightly raised 2-mm vesicles in the throat, usually surrounded by an intense areola. The lesions are most frequently found on the anterior faucial pillars, tonsils, uvula, or soft palate. The target sign (choice E) is the typical cutaneous lesion seen in erythema multiforme. These begin as sharply marginated, erythematous macules, which become raised, edematous papules over 24 to 48 hours. Typically, a ring of erythema forms around the periphery, and centrally, the lesions become flatter, more purpuric, and dusky.

Question 3 of 6

Which of the following treatments has been shown to reduce the morbidity or mortality of hospitalized children younger than 24 months of age with this disease? / A. Antibiotics / B. Antiviral medication / C. High doses of vitamin A / D. Immune serum globulin / E. Supportive care Explanation - Q: 5.3

Close

The correct answer is C. Administration of high doses of vitamin A will reduce the morbidity and mortality of hospitalized children with measles. Two doses of retinyl palmitate 200,000 IU, 24 hours apart, are recommended for all children from 6 months to 24 months of age, for immunodeficient children, children with malnutrition or evidence of vitamin A deficiency, and in recent immigrants from areas of high measles mortality. Antibiotics (choice A) are generally used only if the patient is thought to have a bacterial superinfection. Antiviral medication (choice B) is not available at this time for measles. A live virus vaccine is recommended at 15 months with a booster at 5 years. When given up to 5 days after exposure, vaccination may prevent infection. Immune serum globulin (choice D) is given to children less than one year of age who are exposed. Symptomatic treatment with supportive care (choice E), bed rest, analgesics, and antipyretics is the treatment of choice in non-complicated, non-hospitalized cases.

Question 4 of 6

Which of the following is a possible complication of this disorder? / A. Acute renal failure / B. Arthralgia and arthritis / C. Dysphagia / D. Photophobia / E. Pneumonia and encephalitis

Explanation - Q: 5.4

Close

The correct answer is E. Complications of measles include otitis media, pneumonia, encephalitis, and thrombocytopenic purpura. These complications are more common in the malnourished and in children with T cell deficiencies. Acute renal failure (choice A) has not been reported in children with measles. Arthritis and arthralgia (choice B) are common complications, especially in adult women with rubella (German measles). Dysphagia (choice C) is one of the presenting signs for herpangina along with fever, headache, sore throat, and stiff neck. Photophobia (choice D), painful urination, and extensive respiratory and alimentary tract involvement, can be complications of Stevens-Johnson syndrome.

Question 5 of 6

Which of the following is a rare long-term neurologic complication of this disease? / A. AIzheimer's disease / B. Creutzfeldt-Jacob disease / C. Friedreich's ataxia / D. Pick's disease / E. Subacute sclerosing panencephalitis Explanation - Q: 5.5

Close

The correct answer is E. Subacute sclerosing panencephalitis is a fortunately rare but devastating brain disorder that is thought to represent a persistent measles virus. It occurs months to years after the initial measles infection and is characterized by progressive intellectual deterioration that is accompanied by seizures, loss of vision, and motor abnormalities. Death almost always occurs within 1 to 3 years, with aspiration pneumonia being a common immediate cause of death. To date, only supportive care and anticonvulsants have offered any (transient) hope. All of the other conditions listed also cause devastating progressive neurologic deterioration, but none of them is related to measles infection. Prions have been implicated in Creutzfeldt-Jacob disease (choice B), while the other diseases listed are not known to have an infectious component.

Question 6 of 6

The girl has a 3-month-old healthy sibling that lives with her. Her mother is concerned about the contagious nature of this disease. What is the most appropriate action at this time? / A. Administer immune serum globulin / B. Administer live viral vaccination / C. Admit the child to hospital for close monitoring / D. Reassurance; wait for development of symptoms / E. Tell the mother that this is not a contagious condition Explanation - Q: 5.6

Close

The correct answer is A. Children less than one year of age who are exposed to measles should be treated with immune serum globulin. Administering live viral vaccination (choice B) is recommended as a preventive measure for children older than 15 months of age. Admitting a healthy child to the hospital (choice C) is unnecessary at this time. Reassurance (choice D) without preventive measures in exposed children is not considered standard therapy. Measles is transmitted by respiratory droplets, often resulting in urban outbreaks, demonstrating its contagious nature (choice E).

A 37-year-old woman presents to a walk-in clinic with a chief complaint of an "unsightly" mole on her leg that she would like to have removed. She first noted the mole approximately one year ago while shaving. Initially, there was no associated symptom with the mole. However, in the past 6 months, she has noted the mole had doubled in size with changing color, as well as pruritus and occasional tenderness. She denies any family history of skin cancer but admits to several blistering sunburns while growing up in Los Angeles. On examination, she has blond hair, blue eyes, and light skin with numerous freckles in the sun-exposed areas. Skin examination reveals an 8 x 6 mm asymmetric brown macule with an irregular, scalloped border and variegated black, brown and red pigmentation located on the right shin. No lymphadenopathy was appreciated in the popliteal and inguinal region bilaterally. Question 1 of 6

Which of the following is the most likely diagnosis? / A. Basal cell carcinoma

/ / / /

B. Congenital nevus C. Malignant melanoma D. Seborrheic keratosis E. Solar keratosis Explanation - Q: 6.1

Close

The correct answer is C. Melanomas usually have a prolonged, noninvasive, horizontally oriented growth phase in which the lesion enlarges asymmetrically. Superficial spreading melanoma is the commonest type. It affects adults of all ages and has no sex predilection. The upper back of both sexes and the shins in women are the most common sites. There is a tendency to multicoloration, not just with different shades of tan, but also black, red, brown, and white spectrum. The ABCD criteria have been developed to educate the general public. The letters stand for asymmetry, border irregularity, color variegation, and diameter larger than 6 mm. Basal cell carcinoma (choice A) is a common skin cancer composed of one or a few small, waxy, semitranslucent nodules forming around a central depression that may or may not be ulcerated, crusted, and bleeding. The edge of larger lesions has a characteristic rolled border. Telangiectasias course through the lesion. Bleeding on slight injury is a common sign. Congenital nevus (choice B) is nevocytic lesion found in newborns. These are categorized into giant, medium and small, depending on the size. Seborrheic keratoses (choice D) are multiple, oval, slightly raised, light brown to black, sharply demarcated papules or plaques, located on the chest and back. Solar (actinic) keratoses (choice E) are found chiefly on the chronically sun exposed surfaces of the face, ears, and backs of hands and forearms. They are usually multiple, discrete, flat or elevated, verrucous or keratotic, red, pigmented or skin colored.

*** Question 2 of 6

Epidemiologic studies have shown that which of the following is the present lifetime risk of developing this patient's lesion in Americans? / A. 1 in 3 / B. 1 in 11 / C. 1 in 72 / D. 1 in 250 / E. 1 in 1200

Explanation - Q: 6.2

Close

The correct answer is C. Epidemiologic studies have shown a steady increase in incidence of melanoma over the last several decades. This increase has been attributed to several factors including thinning of the ozone layer and increased public and physician awareness leading to increased diagnosis rate. Last statistic analysis approximates one in 72 Americans will have one melanoma. The prevalence of melanoma is increased in individuals with light complexions, light eyes, blond or red hair, and tendency to tan poorly and sunburn easily. There is also a familial tendency to develop of melanoma, especially in patients with multiple congenital nevi. Sunlight also plays a role in the etiologic factors of melanoma, though a less essential and direct one than that of nonmelanoma skin cancer.

Question 3 of 6

What is the most appropriate next step? / A. Check complete blood count and liver function tests / B. Chest x-ray film / C. Reassure patient that this is a benign lesion / D. Referral to hematology and oncology for initiation of treatment / E. Tissue biopsy to confirm diagnosis Explanation - Q: 6.3

Close

The correct answer is E. Although this patient's history, clinical examination and location of the lesion suggest the diagnosis of malignant melanoma, it is most appropriate to obtain a tissue biopsy to confirm the diagnosis. Complete blood count and liver function test (choice A) as well as a chest xray film (choice B) should be part of the melanoma workup after tissue confirmation of the diagnosis. Reassuring the patient of its benign nature (choice C) is inappropriate management of most pigmented lesions. Melanoma may clinically mimic other benign lesions, but because of its aggressive nature and this patient's history of size and color changes, tissue confirmation is needed. Referral to hematology and oncology for treatment (choice D) is often needed in management of malignant melanoma patients, but is not the first step in managing a new patient.

Question 4 of 6

Which of the following would a biopsy of the lesion most likely show?

/ / / / /

A. Basaloid lobules extending from basal layer of the epidermis B. Hyperkeratosis and keratin horns C. Mitosis and atypical melanocytes throughout epidermis D. Nests of nevoid cells E. Squamous proliferation with atypia and invasion into the dermis Explanation - Q: 6.4

Close

The correct answer is C. Diagnostic criteria of melanoma include atypical melanocytes scattered singly and in irregular nests, presence of mitoses, inflammatory reaction, and the absence of dermal stroma. Basaloid lobules (choice A) extending from the basal layer, showing clefting and palisading border is characteristic of basal cell carcinoma. Hyperkeratosis and keratin horns (choice B) are characteristic of seborrheic keratosis. Nests of nevoid cells (choice D) are seen in normal melanocytic nevi. Squamous proliferation with atypia and invasion into the dermis (choice E) is typical of squamous cell carcinoma.

Question 5 of 6

Which of the following is the most common subtype of this patient's lesion? / A. Acral lentiginous / B. Amelanotic / C. Lentigo maligna / D. Nodular / E. Superficial spreading Explanation - Q: 6.5

Close

The correct answer is E. Melanoma has a number of subclassifications, the members of which vary in prognosis and clinical setting. The most common subtype is superficial spreading melanoma, which occurs in approximately 70% of patients and is most commonly found on the trunk and legs. This type of melanoma presents as a flat or nearly flat brown lesion with variable black, blue, or pink discoloration and typically a size greater than 6 mm. Acral lentiginous melanoma (choice A) is an uncommon subtype of melanoma that is often found in dark-skinned individuals and occurs on the

palms, soles, or beneath the nail plate. Amelanotic melanoma (choice B) is a rare, nonpigmented form of melanoma that appears pink or flesh-colored. Amelanotic melanoma is more frequently found in metastases than in primary lesions, and is presumed to represent a deterioration in the cancer cells' genetic structure that leads to an impaired ability to synthesize melanin. Lentigo maligna melanoma (choice C) occurs in up to 15% of cases, grows slowly over a period of years to decades, and is usually found as part of a large (often more than 3 cm) precursor lesion on sun damaged skin of the head, neck, and arms. Nodular melanoma (choice D) occurs in 15-30% of patients and, like superficial spreading melanoma, is most often seen on the legs and trunks. It tends to grow rapidly (weeks to months) and causes a darkly colored papule or nodule that may bleed with minor trauma.

Question 6 of 6

Which of the following is the most important determinant of prognosis? / A. Presence of lymphocytes / B. Presence of mitoses / C. Presence of ulceration / D. Tumor surface diameter / E. Tumor thickness Explanation - Q: 6.6

Close

The correct answer is E. The most important determinant of prognosis in melanoma is the tumor thickness. This can be measured as the Breslow depth, which is measured vertically in millimeters from the top of the granular layer (or base of superficial ulceration) to the deepest site of tumor involvement. Thicker tumors have a greater risk of metastasis, and, since chemotherapy is problematic in melanoma (less than 20% response rate), increased risk of poor outcome. Thin primary tumors less than 1 mm thick and without metastases at time of diagnosis have a 5 year survival rate of over 90%; while melanomas more than 4 mm thick without known metastases have a 5-year survival rate of 45-70%. If regional lymph node metastases are present, the 5-year survival rate is in different studies has ranged from about 15 to 70%. If distant metastases are present, the 5-year survival rate drops to less than 20%, with a median survival of only 6-9 months. The presence of ulceration (choice C) is thought to be the next most important risk factor. The other features noted in the choices are usually included in pathology reports, but have lesser prognostic value.

A 54-year-old man complains of daytime fatigue and sleepiness whenever he sits down. The man states that he gets at least 6-7 hours of sleep every night and does not have any problems falling asleep. His wife reports that he is a loud snorer and has on occasion, gasped for air. He also complains of nonspecific headaches with no particular pattern, as well as a recent weight gain of 20 Ib in the last 6 months. Past medical history is noncontributory, and the patient denies taking any medications. Social habits include 2 shots of whiskey prior to bedtime each night. Physical examination reveals a height of 173 cm (68 in) and weight of 103 kg (225 Ib). His blood pressure is 148/90 mm Hg and his pulse is 85/min. His oropharynx is within normal limits, with no deviation or swelling of the uvula. The rest of the physical examination is unremarkable. Laboratory studies, including T3, T4, TSH, complete blood count, electrolytes, BUN, creatinine, and glucose are within normal Iimits. Question 1 of 3

Which of the following studies would most likely lead to the correct diagnosis? / A. Arterial blood gases / B. Echocardiogram / C. Pulmonary function tests / D. Radioiodine uptake (RAIU) / E. SIeep study Explanation - Q: 1.1

Close

The correct answer is E. The patient is presenting with symptoms suggestive of sleep apnea (daytime somnolence, snoring, recent weight gain, and mild hypertension); the diagnostic test is a sleep study (polysomnography). Obstructive sleep apnea is characterized by repetitive cessation of airflow during sleep from the collapse of the pharyngeal airway despite continuing ventilatory effort. The pharyngeal airway is anatomically small secondary to obesity, skeletofacial abnormalities, tonsillar/uvular hypertrophy, or a posteriorly positioned jaw. During waking hours, airway patency is maintained by activating the neuromuscular reflexes with increased activity of the pharyngeal dilator muscles. During sleep, this reflex is diminished or lost, so muscle activity decreases and the airway collapses. The ensuring apnea decreases PO2 and increases PCO2, causing increased ventilatory effort and arousal from sleep with resumption of breathing. The result is repetitive cycles of apnea and hyperpnea. Arterial blood gases (choice A) are more useful to aid the diagnosis of pulmonary processes causing hypoxia, however they can be used to differentiate Pickwickian syndrome from other causes of obstructive sleep apnea.

When sleep apnea becomes severe, resulting in pulmonary hypertension or left heart failure, an echocardiogram (choice B) can be used to assess cardiac status; it is seldom useful in the initial assessment for sleep apnea. Pulmonary function tests (choice C) are useful in establishing the different causes for dyspnea; it can be used to differentiate the different types of central sleep apnea. Patients with neuromuscular disorders associated with central sleep apnea can have abnormal pulmonary function tests with a diminished FEV1. Hypothyroidism can aggravate sleep apnea; a TSH level is often used to exclude primary hypothyroidism. An RAIU (choice D) is only useful in patients with hyperthyroidism.

A 54-year-old man complains of daytime fatigue and sleepiness whenever he sits down. The man states that he gets at least 6-7 hours of sleep every night and does not have any problems falling asleep. His wife reports that he is a loud snorer and has on occasion, gasped for air. He also complains of nonspecific headaches with no particular pattern, as well as a recent weight gain of 20 Ib in the last 6 months. Past medical history is noncontributory, and the patient denies taking any medications. Social habits include 2 shots of whiskey prior to bedtime each night. Physical examination reveals a height of 173 cm (68 in) and weight of 103 kg (225 Ib). His blood pressure is 148/90 mm Hg and his pulse is 85/min. His oropharynx is within normal limits, with no deviation or swelling of the uvula. The rest of the physical examination is unremarkable. Laboratory studies, including T3, T4, TSH, complete blood count, electrolytes, BUN, creatinine, and glucose are within normal Iimits. Question 2 of 3

Which of the following best distinguishes Pickwickian syndrome from this patient's condition? / A. Daytime hypersomnolence / B. Hypoventilation while awake / C. Increased PCO2 during sleep / D. Obesity / E. Presence of apneic episodes during sleep Explanation - Q: 1.2

Close

The correct answer is B. Dr. Osler first described the Pickwickian syndrome after a character by Charles Dickens in the Pickwick Papers. A patient with Pickwickian syndrome is obese, with daytime sleepiness and obstructive sleep apnea. However, the distinguishing trait is that the patient must have

awake hypoventilation (high PCO2); it is considered a severe form of obesityhypoventilation syndrome. Daytime hypersomnolence (choice A) and increased PCO2 during sleep (choice C) from the apneic episodes (choice E) characterize Pickwickian syndrome as well as other causes of obstructive sleep apnea. Not all obstructive sleep apnea patients are obese (choice D) and not all patients who are obese have Pickwickian symptoms.

Question 3 of 3

Which of the following is the most appropriate next step in therapy for this patient? / A. Acetazolamide / B. AIcohol avoidance and weight loss / C. Nasal oxygen / D. Surgery and continuous positive airway pressure (CPAP) / E. Thyroid supplementation Explanation - Q: 1.3

Close

The correct answer is B. For mild to moderate obstructive sleep apnea such as this patient has, the patient can benefit from alcohol avoidance, weight loss, and not sleeping in the supine positive. For more severe cases, continuous positive airway pressure (CPAP) can be very useful. Patients who develop Cheyne-Stokes respiration with sleep apnea are generally in heart failure. There is evidence that nasal CPAP can reduce left ventricular afterload for these patients, improve cardiac function, and regulate ventilation during sleep for better sleep quality. Decisions to use CPAP versus noninvasive measures is based on the severity of the patient's symptoms and the apnea/hypopnea index. Acetazolamide (choice A) can be used in central sleep apnea of idiopathic origin. It causes metabolic acidosis, which stimulates a central compensatory response to increase ventilatory efforts. Nasal oxygen (choice C) can be used for central sleep apnea, but is not useful in obstructive sleep apnea. Surgery (uvulopalatopharyngoplasty; choice D) can eliminate snoring, especially in patients with an enlarged uvula or pharyngeal tissues; it only cures 50% of the obstructive sleep apnea. It can be used in severe sleep apnea to decrease the amount of CPAP required. Thyroid supplementation (choice E) can help sleep apnea if the patient is

hypothyroid, but in the absence of hypothyroidism, is not useful.

A 54-year-old man is having trouble sleeping alone after his wife dies. He discusses his problems with his friends at a card game. One of the players suggest that he try some of his "sleeping pills," and later brings over a bottle of benzodiazepines that were prescribed for him. The man begins taking the medication to help him sleep. Question 1 of 3

The man later presents to a psychiatrist and admits to self-medicating with benzodiazepines. Which of the following would be a likely side effect of this medication? / A. Decreased REM sleep / B. Night terrors / C. Nocturnal myoclonus / D. SIeep walking / E. Vivid dreams Explanation - Q: 2.1

Close

The correct answer is A. Stage 4 is the deepest non-REM sleep and occurs prior to REM sleep. Benzodiazepines shorten stage 4 and suppress REM sleep. REM sleep is also decreased by barbiturates, alcohol, phenothiazines, and MAO inhibitors. Stage 4 sleep is decreased by imipramine and benzodiazepines. Night terrors (choice B) and sleep walking (choice D) are parasomnias related to intrusion of waking behavior (speaking or walking) into non-REM sleep, during stage 4. Benzodiazepines shorten stage 4, and are used to treat night terrors and sleepwalking. Imipramine is used to treat enuresis, also associated with stage 4. Nocturnal myoclonus (choice C) is a dyssomnia characterized by brief repetitive leg jerks that occur in regular 20 to 40 second intervals. It is thought to be secondary to loss of inhibition of at the level of the spinal cord. It is normal in infants. The movements lead to transient arousal from sleep and result in sleep fragmentation, and a sense of insufficient sleep. Benzodiazepines and L-Dopa are the mainstays of treatment. Vivid dreams (choice E) and nightmares occur during REM sleep (unlike sleep terrors). Nightmares are not accompanied by movement unless the person wakens. In contrast, persons with night terrors are not awake, but are moving. Reports of nightmares occur when REM activity is increased, such as after discontinuation of REM-suppressing substances (barbiturates, alcohol, phenothiazines, and MAO Inhibitors).

Question 2 of 3

The patient stops using benzodiazepines. Six months later, he reports continued daytime somnolence, and has begun waking at 4 am with difficulty returning to sleep. He no longer goes to card games or has any interest in his gardening club. He reports "feeling blue" with poor appetite, and poor energy. This disorder is associated with which of the following sleep disturbances? / A. Decreased REM Iatency and reduced proportion of REM sleep / B. Increased sleep latency and little or no REM sleep / C. Rapid cycling through sleep stages and increased proportion of slow wave sleep / D. Stereotyped forceful teeth grinding or clenching / E. Increased proportion of REM sleep and nightmares Explanation - Q: 2.2

Close

The correct answer is A. The patient meets criteria for depression. Depressed patients rapidly enter REM upon falling asleep, but overall, have a decrease in the proportion of REM sleep. Increased REM latency and very little REM sleep (choice B) is associated with mania. Manic patients may be awake for days (greatly increased sleep latency) and may have very little REM when they do sleep. Rapid cycling through sleep stages and increased proportion of slow wave sleep (choice C) is associated with alcohol use. Alcohol also decreases the amount of REM. Stereotyped forceful teeth grinding or clenching (choice D) (bruxism) is associated with dementia, mental retardation, and Parkinson disease. Increased proportion of REM sleep and nightmares (choice E) (REM rebound) occurs after discontinuation of REM-suppressing substances (e.g., barbiturates, alcohol, phenothiazines, and MAO inhibitors). Question 3 of 3

One morning, the patient develops chest pain and is later confirmed to have had a myocardial infarction. This event likely occurred during which of the following stages of sleep? / A. Stage 1 / B. Stage 2 / C. Stage 3 / D. Stage 4 / E. REM sleep

Explanation - Q: 2.3

Close

The correct answer is E. REM sleep has been termed paradoxical sleep because many physiological parameters are similar to the awake state. During REM, brain oxygen use increases and this stage is associated with near total paralysis of the skeletal system and penile erections. Pulse, respiration, and blood pressure are high and variable, during REM sleep, much higher than during non-REM sleep, and often higher than during waking. Stages 1, 2, 3, and 4 (choices A, B, C, and D) are non-REM sleep. Stage 1 is associated with low voltage theta waves. Stage 2 is associated with sleep spindles and K complexes. Stages 3 and 4 are associated with delta waves (slow wave sleep). REM sleep is low-voltage with random, fast, sawtooth waves.

A 24-year-old graduate student is studying for examinations in the library. She realizes that she has lost her notes, and becomes very upset. Suddenly, according to library patrons, her head falls forward, and she slumps over her books, striking her forehead against the table. Within minutes, she appears to recover, sits up, and is helped to the student health service by her roommate, who was present in the library. At the clinic, the patient is confused about what happened to her, but is alert and cooperative. She has no significant past medical history, and denies any previous events similar to the one today. Review of systems is mostly noncontributory, except for excessive sleepiness, and frequently waking up and feeling that she is unable to move for a few moments in the morning. She says that she has been sleeping well lately (7-8 hours/night), despite a busy study schedule. Her roommate states that she has never seen the patient sleepwalking, and that she does not snore. Her vital signs are normaI, and physical examination, including a neurologic examination, is unrevealing. A small laceration is present on her forehead, where she hit her head against the table. Question 1 of 3

Which of the following is the most likely diagnosis? / A. Chronic sleep restriction / B. Hypoglycemia / C. Mitral valve prolapse / D. Narcolepsy / E. SIeep apnea Explanation - Q: 3.1 The correct answer is D. This patient may be developing narcolepsy. Narcolepsy is a sleep disorder characterized by excessive daytime

Close

somnolence, in addition to cataplexy, hypnagogic or hypnopompic hallucinations, and sleep paralysis. This patient had a sudden loss of muscular tone elicited by emotion (cataplexy), and reports a common, but nonspecific symptom of narcolepsy, sleep paralysis, a subjective feeling of being unable to move immediately upon awakening. Chronic sleep restriction (choice A), also known as sleep deprivation, is unlikely, given her recent history of sleeping well. Hypoglycemia (choice B) would likely have produced some degree of autonomic changes, and would likely not have resolved on its own. Mitral valve prolapse (choice C) is unlikely, given the lack of pertinent physical examination findings (e.g., midsystolic click). Sleep apnea (choice E) is unlikely in a 24-year-old woman who does not snore. Question 2 of 3

The patient's scalp laceration is treated, and she is sent home. She later returns to the clinic to have her stitches removed. She reports that during the intervening time, she had a recurrence of symptoms, falling to the floor in the laundry room of her apartment building. AIso, her daytime sleepiness has continued, and she reports she "needs to take naps a lot." If an electroencephalogram were performed during the first 30 minutes of one of her naps, which of the following would most likely be observed? / A. 3 Hz spike and wave discharge / B. 4-7 Hz rhythm, sleep spindles, and K-complexes / C. Low-amplitude, 8-13 Hz rhythm / D. Low-amplitude, high frequency, irregular pattern / E. Paroxysmal diffuse outbursts of high voltage, fast rhythm with many spikes Explanation - Q: 3.2

Close

The correct answer is D. A low-amplitude, high frequency pattern is characteristic of beta activity during alertness, or rapid eye movement (REM) sleep. In narcolepsy, the latency to REM sleep is much shortened. In some cases, affected individuals enter REM nearly instantaneously upon falling asleep. In normal individuals, REM sleep does not begin for approximately 90 minutes after falling asleep, so in many cases, a short nap will not show any REM. A 3 Hz spike and wave discharge (choice A) is characteristic of petit mal

epilepsy (absence seizures). 4-7 Hz (theta) rhythm, sleep spindles and K complexes (choice B) are characteristic of stage 2 of slow wave sleep. An 8-13 Hz, low-amplitude rhythm (choice C) is called an alpha rhythm. It is most prominent over the parieto-occipital cortex with the eyes closed and is indicative of relaxed wakefulness in most normal individuals. A characteristic fast rhythm with many spikes and paroxysmal diffuse outbursts of high voltage (choice E) is seen in generalized tonic-clonic seizures (grand mal seizures). During the clonic phase, slow waves interrupt the high amplitude spikes.

Question 3 of 3

Which of the following could be used to treat this patient's excessive daytime somnolence? / A. Amitriptyline / B. Dextroamphetamine / C. FIuoxetine / D. Imipramine / E. Triazolam Explanation - Q: 3.3

Close

The correct answer is B. Therapy of narcolepsy is directed toward reduction in daytime somnolence and suppression of REM phenomena (such as cataplexy, hypnagogic hallucinations, and sleep paralysis). Reduction in daytime somnolence is accomplished with CNS stimulants, such as dextroamphetamine and methylphenidate, although a new drug, modafinil, may promote wakefulness without the addictive potential of amphetamines. Suppression of REM phenomena can be achieved via the use of tricyclic antidepressants (e.g., protriptyline and clomipramine) and selective serotonin reuptake inhibitors (SSRIs; e.g., fluoxetine, choice C). Amitriptyline (choice A) is a tricyclic antidepressant, and suppresses REM, but is very sedating, so would not be used to treat daytime somnolence. Imipramine (choice D) is used to suppress REM phenomena, not to improve alertness. Triazolam (choice E) is a benzodiazepine, and would tend to worsen the patient's daytime somnolence.

A 9-year-old boy is brought to the clinic with "sleep problems." The patient's mother reports episodes of her son sitting up in bed, screaming and crying. He appears frightened and is breathing fast and sweating. She tries to wake him up, but without success. She says that the episodes are probably more frightening for her than for her son because he does not seem to recall anything about them. Question 1 of 3

Which of the following is the most likely diagnosis? / A. Enuresis / B. Night terror disorder / C. Nightmare disorder / D. Rapid eye movement sleep behavior disorder / E. Somnambulism Explanation - Q: 4.1

Close

The correct answer is B. Night terrors are characterized by a sudden arousal from non-REM (NREM) (usually slow-wave) sleep with behavioral and autonomic manifestations of fear. During these episodes, the patients often scream or cry out, and also have a significant amount of autonomic activity (e.g., tachycardia, tachypnea, sweating) and increased muscle tone. They usually do not respond to external stimuli, and when they awaken, they are often confused, but amnestic about the episode. Enuresis (choice A) is bed-wetting. It is not directly related to night terrors. Patients with nightmare disorder (choice C) describe frightening dreams. Arousal during the dream frequently occurs. The fact that there is a dream differentiates this disorder from night terror disorder. In rapid eye movement sleep behavior disorder (choice D), the patient actually acts out their dreams, and have been known to kick, punch, jump, and run. They can injure their bed partner. These patients can awaken rapidly and become rapidly oriented. They generally have a vivid recollection of their dreams, as this disorder occurs during REM sleep. Despite this, they rarely develop excessive daytime sleepiness. Somnambulism (choice E), or sleepwalking, can present in a variety of ways. Patients may just sit up, or walk, or exhibit complex behaviors. This generally occurs during slow-wave NREM sleep. During the episode, they often have a blank, staring face and are unresponsive to others.

Question 2 of 3

Which of the following medications could be used to treat this condition? / A. Bupropion / B. Chlorpromazine / C. Diazepam / D. Diphenhydramine / E. Sertraline Explanation - Q: 4.2

Close

The correct answer is C. Medications are frequently not necessary for this disorder, but in severe cases, the patient may be prescribed a benzodiazepine, such as diazepam. Benzodiazepines suppress stage 3 and stage 4 sleep (when night terrors occur). This should only be used shortterm, especially in children, because deep sleep is the period during which growth hormone is primarily secreted. Bupropion (choice A) and sertraline (choice E) are antidepressants and may improve sleep, but do not treat disorders of slow wave NREM sleep. Chlorpromazine (choice B) is an antipsychotic medication, and is not indicated for this disorder. Diphenhydramine (choice D) is an antihistamine sometimes used as a sleep aid. Central histamine blockade results in sedation. Diphenhydramine may help with sleep latency, but does not suppress slow wave sleep. Hence, it would not treat night terrors. Question 3 of 3

Which of the following is the mechanism of action of the drug that might be prescribed to this patient? / A. BIocks dopamine and norepinephrine reuptake / B. BIocks dopamine receptors / C. BIocks histamine receptors / D. BIocks serotonin reuptake / E. Potentiates GABA effect Explanation - Q: 4.3

Close

The correct answer is E. Benzodiazepines potentiate GABA at the GABA-A receptor, by increasing the frequency of GABA-mediated chloride channel opening. Bupropion blocks the reuptake of dopamine and norepinephrine (choice A).

Chlorpromazine blocks dopamine receptors (choice B). Diphenhydramine blocks histamine receptors (choice C). Sertraline blocks serotonin reuptake (choice D).

A 57-year-old woman presents to her family physician complaining of numbness and tingling in her right thumb, index and long finger for the past four weeks. She reports that she wakes up in the middle of the night with these symptoms and needs to shake her hands to "wake" them up. She denies numbness or tingling in her other hand or either leg. She has no neck or upper arm pain. On physical examination, her symptoms are reproduced by Tinel testing (tapping the affected area). Her symptoms are also exacerbated by hyperflexion of the wrist. There is decreased sensation over the palmar aspects of the thumb, index and middle fingers. There is no apparent motor weakness. Question 1 of 6

Which of the following is the most likely diagnosis? / A. C5 cervical nerve root compression / B. Carpal tunnel syndrome / C. Cubital tunnel syndrome / D. Radial tunnel syndrome / E. Tarsal tunnel syndrome Explanation - Q: 1.1

Close

The correct answer is B. The patient's symptoms are classic for carpal tunnel syndrome. Carpal tunnel syndrome is the most common compressive neuropathy in the upper extremity. It is caused by compression of the median nerve in the carpal tunnel. The median nerve provides sensation to the palmar side of the thumb, index finger, long finger and radial half of the ring finger. The floor of the tunnel is formed by the carpal bones; the roof is formed by the transverse carpal ligament. A patient with a C5 nerve cervical nerve root compression (choice A) would have weakness in their deltoids and biceps with sensory changes in the lateral arm. Cubital tunnel syndrome (choice C) is compression of the ulnar nerve at the elbow. Symptoms include pain and paresthesias over the medial forearm and hand as well as weakness in the ulnar nerve distribution. Radial tunnel syndrome (choice D) is compression of a branch of the radial nerve at the forearm. It is a pain-only problem without motor or sensory changes. Tarsal tunnel syndrome (choice E) is compression of the tibial nerve at the medial ankle.

Question 2 of 6

Which of the following structures is responsible for the patient's symptoms? / A. Median nerve / B. Posterior interosseous nerve / C. Radial nerve / D. Sural nerve / E. UInar nerve Explanation - Q: 1.2

Close

The correct answer is A. The median nerve is responsible for the patient's symptoms. The median nerve provides sensation to the palmar side of the thumb, index finger, long finger, and radial half of the ring finger. The posterior interosseous nerve (choice B) provides no sensory innervation to the hand. The radial nerve (choice C) provides sensation to the dorsum of the hand over the radial two and one-half digits over the proximal phalanx. The sural nerve (choice D) supplies the lateral side of the foot. The ulnar nerve (choice E) innervates the palmar small finger and ulnar onehalf of the ring finger

Question 3 of 6

Which of the following muscles is innervated by the affected nerve? / A. Abductor digiti minimi / B. Abductor pollicis brevis / C. Abductor pollicis longus / D. Adductor pollicis / E. Opponens digiti minimi Explanation - Q: 1.3

Close

The correct answer is B. The symptoms above can be attributed to carpal tunnel syndrome or compression of the median nerve. Of the muscles listed, the median nerve only innervates the abductor pollicis brevis. The abductor digiti minimi (choice A) and adductor pollicis (choice D) are innervated by the ulnar nerve. The abductor pollicis longus (choice C) is innervated by the posterior

interosseous branch of the radial nerve. The opponens digiti minimi (choice E) is innervated by the ulnar nerve.

Question 4 of 6

The patient is given an injection of a corticosteroid into the affected area. What is the mechanism of action of steroids in decreasing inflammation? / A. Decreases the movement of granulocytes to the affected area / B. Inhibits the enzyme phospholipase A2 / C. Interacts with opioid receptors / D. Irreversibly inhibits cyclooxygenase / E. Suppresses phagocytosis and lysosomal enzyme activity Explanation - Q: 1.4

Close

The correct answer is B. Glucocorticoids decrease inflammation because of their ability to lower peripheral lymphocyte levels and inhibit the enzyme phospholipase A2. Phospholipase A2 is the enzyme that releases arachidonic acid, the precursor of prostaglandins. Colchicine is used to decrease the movement of granulocytes to the affected area (choice A) in the treatment of gout. Morphine and other opioids interact with opioid receptors (choice C). Aspirin works by irreversibly acetylating and thus inhibiting cyclooxygenase (choice D), which ultimately inhibits prostaglandin synthesis. Suppressing phagocytosis and lysosomal enzyme activity (choice E) is the mechanism of action of gold salts in decreasing inflammation.

Question 5 of 6

The patient does not respond to conservative methods of treatment and decides to undergo an operative intervention. Preoperatively she is given cefazolin, a first generation cephalosporin, for antibiotic prophylaxis. Which of the following is the mechanism of action of cefazolin? / A. Inhibits bacterial cell wall synthesis / B. Inhibits bacterial protein synthesis by binding the 30s subunit / C. Inhibits bacterial protein synthesis by binding the 50s subunit / D. Inhibits bacteriaI RNA synthesis / E. Inhibits DNA gyrase Explanation - Q: 1.5

Close

The correct answer is A. Cefazolin is a first generation cephalosporin. Cephalosporins inhibit bacterial peptidoglycan synthesis and thus interfere with the last step of bacterial cell wall synthesis. Many antibiotics work by interfering with protein synthesis in many ways. Aminoglycosides bind the 30s subunit of the bacterial ribosome and interfere with protein synthesis (choice B). Erythromycin inhibits bacterial protein synthesis by binding the 50s subunit (choice C). Rifampin inhibits RNA synthesis in bacteria (choice D). Quinolones inhibit DNA gyrase (choice E).

Question 6 of 6

Hypothyroidism is part of the differential diagnosis in patients with the condition described above. Which of the following laboratory values would be consistent with the diagnosis of primary hypothyroidism? / A. Decreased free T4 and decreased TSH / B. Decreased free T4 and increased TSH / C. Increased free T4 and decreased TSH / D. Increased free T4 and increased TSH / E. Increased free T4 and normaI TSH Explanation - Q: 1.6

Close

The correct answer is B. Decreased levels of T4 and increased TSH are signs of primary hypothyroidism. The TSH is high because the pituitary is working to overcome the hypothyroid state by increasing the stimulation of the thyroid gland. Decreased free T4 and decreased TSH (choice A) are seen in patients with secondary or tertiary hypothyroidism. Increased free T4 and decreased TSH (choice C) are seen in patients with hyperthyroidism. Increased free T4 and increased TSH (choice D) may be seen in patients with a pituitary TSH-secreting tumor. Increased free T4 and normal TSH (choice E) is rare and may be seen in patients with a peripheral unresponsiveness type of thyroid disease.

A 67-year-old woman has a 3 month history of increasing right shoulder pain. She is now unable to lift her arm to brush her hair or to take a can off a shelf. She denies any numbness or tingling or radiation of the pain down her arm. On physical examination, she exhibits weakness in abduction and external rotation or her right arm. Her shoulder shows a normal passive range of motion. When asked to hold her arms abducted to 90 degrees, she is unable to do so on the right. There is no motor weakness in her forearm or hand. Her right upper extremity has normal pulses and normal sensation. Question 1 of 6

Which of the following is the most likely diagnosis? / A. Brachial plexus injury / B. GIenohumeral osteoarthritis / C. Proximal humerus fracture / D. Rotator cuff tear / E. Shoulder dislocation Explanation - Q: 2.1

Close

The correct answer is D. The above history describes a rotator cuff tear. Patients classically have difficulty doing overhead activities. On physical examination, she has weakness with abduction and external rotation of her shoulder. The supraspinatus is responsible for abduction and the infraspinatus is responsible for external rotation of the shoulder. These muscles, along with the teres minor and subscapularis, make up the rotator cuff. Brachial plexus injuries (choice A) commonly present with some type of numbness or tingling in the upper extremity. Glenohumeral arthritis (choice B) is unlikely because she has a full passive range of motion, and shoulder arthritis is commonly associated with a decreased range of motion. Also this history is relatively short term. Proximal humerus fracture (choice C) and shoulder dislocation (choice E) are unlikely because this process has been present for 3 months. These injuries need more acute treatment. Also she has a full passive range of motion, which is not common with fractures or dislocations. Question 2 of 6

Which of the following muscles make up the injured structure? / A. Supraspinatus, infraspinatus, teres major, deltoid / B. Supraspinatus, infraspinatus, teres major, subscapularis / C. Supraspinatus, infraspinatus, teres minor, deltoid / D. Supraspinatus, infraspinatus, teres minor, subscapularis / E. Supraspinatus, infraspinatus, teres minor, trapezius

Explanation - Q: 2.1

Close

The correct answer is D. The above history describes a rotator cuff tear. Patients classically have difficulty doing overhead activities. On physical examination, she has weakness with abduction and external rotation of her shoulder. The supraspinatus is responsible for abduction and the infraspinatus is responsible for external rotation of the shoulder. These muscles, along with the teres minor and subscapularis, make up the rotator cuff. Brachial plexus injuries (choice A) commonly present with some type of numbness or tingling in the upper extremity. Glenohumeral arthritis (choice B) is unlikely because she has a full passive range of motion, and shoulder arthritis is commonly associated with a decreased range of motion. Also this history is relatively short term. Proximal humerus fracture (choice C) and shoulder dislocation (choice E) are unlikely because this process has been present for 3 months. These injuries need more acute treatment. Also she has a full passive range of motion, which is not common with fractures or dislocations. Question 2 of 6

Which of the following muscles make up the injured structure? / A. Supraspinatus, infraspinatus, teres major, deltoid / B. Supraspinatus, infraspinatus, teres major, subscapularis / C. Supraspinatus, infraspinatus, teres minor, deltoid / D. Supraspinatus, infraspinatus, teres minor, subscapularis / E. Supraspinatus, infraspinatus, teres minor, trapezius Explanation - Q: 2.2

Close

The correct answer is D. The muscles that make up the rotator cuff are: supraspinatus, infraspinatus, teres minor, subscapularis. An easy way to remember these are the acronym SITS (Supraspinatus, Infraspinatus, Teres minor, Subscapularis). The deltoid (choices A and C), teres major (choices A and B), and trapezius (choice E) are not considered part of the rotator cuff.

Question 3 of 6

Which of the following nerves innervates the deltoid muscle?

/ / / / /

A. Axillary B. Dorsal scapular C. Long thoracic D. Suprascapular E. Thoracodorsal Explanation - Q: 2.3

Close

The correct answer is A. The axillary nerve innervates the deltoid muscle, as well as provides sensation over the lateral part of the arm. The dorsal scapular nerve (choice B) innervates the rhomboids and levator scapulae muscles. The long thoracic nerve (choice C) innervates the serratus anterior. Injury to this results in winging of the scapula. The suprascapular nerve (choice D) innervates the supraspinatus and infraspinatus muscles. The thoracodorsal nerve (choice E) innervates the latissimus dorsi muscle.

Question 4 of 6

Which of the following structures are found in the quadrangular space? / A. Circumflex scapular vessels and the radial nerve / B. Posterior humeral circumflex artery and axillary nerve / C. Posterior humeral circumflex artery and the radial nerve / D. Profunda brachii artery and the axillary nerve / E. Profunda brachii artery and the radial nerve Explanation - Q: 2.4

Close

The correct answer is B. The quadrangular space is bounded superiorly by the teres minor and subscapularis muscles, medially by the long head of the triceps, laterally by the surgical neck of the humerus, and inferiorly by the teres major muscle. It transmits the posterior humeral circumflex vessels and the axillary nerve. The triangular space, which is bounded by the teres minor, teres major and the long head of the triceps, transmits the circumflex scapular vessels (choice A). The posterior humeral circumflex artery and radial nerve (choice C) do not

run in a named interval or space. The profunda brachii artery and the axillary nerve (choice D) do not run in a named interval or space. The triangular interval, which is bound by the teres major, long head of the triceps and the medial head of the triceps, transmits the radial nerve and the profunda brachii artery (choice E).

Question 5 of 6

Which of the following cervical nerve roots provides sensation to the lateral arm and innervates the deltoid muscle? / A. C4 / B. C5 / C. C6 / D. C7 / E. C8 Explanation - Q: 2.5

Close

The correct answer is B. The C5 nerve root innervates the deltoid and provides sensation to the lateral arm. C4 (choice A) innervates the scapular muscles and provides sensation to the lateral neck and shoulder. C6 (choice C) innervates the wrist extensors, biceps and triceps, and provides sensation to the radial forearm. C7 (choice D) innervates the triceps and wrist flexors and provides sensation to the middle finger. C8 (choice E) innervates the finger flexors and interossei and provides sensation to the ulnar hand. Question 6 of 6

The patient has failed treatment by conservative measures for the above condition and decides to undergo operative intervention. Postoperatively, she is oversedated with morphine, which depresses her respiratory drive. Which of the following drugs may be used to reverse the morphine by acting as a competitive antagonist? / A. Acetylcysteine / B. Deferoxamine / C. FIumazenil

/ D. Naloxone / E. Physostigmine

Explanation - Q: 2.6

Close

The correct answer is D. Naloxone is a opioid antagonist. It can reverse all aspects of morphine overdose, e.g., respiratory depression, level of consciousness, bowel activity, and pupil size. Keep in mind that naloxone has a short half-life, so repeated dosing may be necessary. Acetylcysteine (choice A) is used to treat acetaminophen toxicity. Deferoxamine (choice B) is used to treat iron toxicity. Flumazenil (choice C) is used to reverse benzodiazepines. Physostigmine (choice E) is used to treat anticholinergic toxicity.

A 6-week-old boy is brought to his family physician. His parents report that he has not had significant use of his right arm since birth. Birth history is significant for a prolonged labor with difficult breech delivery. On physical examination, his arm hangs at his side and is in a medially rotated position with the forearm in pronation. He will actively use his left arm, but does not move his affected right arm or hand. Question 1 of 6

Which of the following is the most likely diagnosis? / A. CIavicle fracture / B. Erb-Duchenne paralysis / C. Intracranial hemorrhage / D. KIumpke's paralysis / E. Spinal cord injury Explanation - Q: 3.1

Close

The correct answer is B. This is a classic history for a birth related ErbDuchenne paralysis (also called Erb's Palsy). There is a history of a difficult delivery. The child's arm rests at his side, is medially rotated and the forearm is pronated. This is due to traction injury to the superior trunk of the brachial plexus. The C5 and C6 nerve roots are most commonly affected. ErbDuchenne paralysis is the classic "waiter's tip" position. Clavicle fractures (choice A) occur as a result of birth trauma, however, by 6

weeks of age the fracture usually is healed and the baby is asymptomatic. Intracranial hemorrhage (choice C) is a potential birth-related injury, but its presentation is variable and usually does not result in a isolated unilateral upper extremity injury. Klumpke's paralysis (choice D) is a birth palsy that results in a claw hand due to injury of the C8 and T1 nerve roots. Spinal cord injuries (choice E) do occur after traumatic births but are not isolated to a single upper extremity finding.

Question 2 of 6

Injury to which of the following cervical nerve roots account for this patient's posture? / A. C4 and C5 / B. C5 and C6 / C. C6 and C7 / D. C7 and C8 / E. C8 and T1 Explanation - Q: 3.2

Close

The correct answer is B. This patient has an Erb-Duchenne palsy, which is the result of an injury to the superior trunks of the C5 and C6 nerve roots. The C8 and T1 (choice E) nerve roots are injured in a Klumpke's paralysis. The other combinations (choices A, C, D) all may be injured as a result of birth palsy, but do not have distinct syndromes associated with them.

Question 3 of 6

This condition is associated with injury to the phrenic nerve in approximately 5% of cases. Which of the following cervical nerve roots form the phrenic nerve? / A. C2 through C4 / B. C3 through C5 / C. C4 through C6 / D. C5 through C7 / E. C8 through C8 Explanation - Q: 3.3

Close

The correct answer B. Phrenic nerve injury is seen in 5% of Erb-Duchenne palsies. This nerve arises from the third through fifth cervical nerves (C3-C5). The C5 through C7 (choice D) nerves give rise to the long thoracic nerve, which innervates the serratus anterior. The remainder of the choices do not make up the phrenic nerve. Question 4 of 6

Paralysis of which of the following muscles results in the pronated position of this patients forearm? / A. Biceps / B. Deltoid / C. Pronator quadratus / D. Pronator teres / E. Triceps Explanation - Q: 3.4

Close

The correct answer is A. The biceps muscle is innervated by the C6 nerve, and is a supinator of the forearm as well as a flexor of the elbow. Paralysis of the biceps will result in overpull of the unaffected pronator muscles of the forearm. The deltoid (choice B) is innervated by the C5 nerve root. It is an abductor of the arm. Paralysis will result in the patient's arm hanging at his or her side. The pronator quadratus (choice C) and teres (choice D) are pronators of the forearm. Paralysis will result in less pronation of the forearm, and they are not involved in a Erb's palsy. The triceps (choice E) extends the elbow and is not responsible for this patient's pronated position.

Question 5 of 6

This child's father then states that he had a nerve palsy in his left arm. He sustained this injury when he was 21 years old. He reports that he was out with his friends drinking alcohol and woke up the next morning unable to extend his fingers or wrist. His symptoms improved 1 year after the injury. Which of the following nerves was most likely injured? / A. Axillary nerve / B. Median nerve / C. Musculocutaneous nerve / D. Radial nerve

/ E. UInar nerve

Explanation - Q: 3.5

Close

The correct answer is D. The child's father most likely had a radial nerve palsy, sometimes called a Saturday night palsy. The radial nerve injury will result in an inability to extend the elbow and extend the wrist or fingers. Injury to the axillary nerve (choice A) is usually caused by proximal humerus fractures and dislocations. It will result in weakness of lateral rotation and abduction of the arm. Injury to the median nerve (choice B), which may be a result of distal humerus fracture, will result in loss of pronation, opposition of the thumb, and flexion of the thumb and index finger interphalangeal joints. Injury to the musculocutaneous nerve (choice C) results in weakness of elbow flexion (biceps and brachialis) and supination (biceps). Injury to the ulnar nerve (choice E) results in a claw hand (ring and little fingers are hyperextended at the metacarpophalangeal joints and flexed at the interphalangeal joints).

Question 6 of 6

The child's mother also reports that she had a nerve injury in the past. She states that she was involved in a crush injury to her left leg as a teenager. After the injury, she had a foot drop (she was unable to dorsiflex her left foot at the ankle). Which of the following nerves was most Iikely injured? / A. Deep peroneal nerve / B. Femoral nerve / C. Superficial peroneal nerve / D. Sural nerve / E. Tibial nerve

Explanation - Q: 3.6

Close

The correct answer is A. This patient most likely had an injury to her deep

peroneal nerve. It is a branch of the common peroneal nerve. The deep peroneal nerve runs along the anterior surface of the interosseous membrane and supplies the muscles of the anterior compartment of the leg (tibialis anterior, extensor hallucis longus, extensor digitorum longus, and peroneus tertius). The tibialis anterior is responsible for dorsiflexion of the foot at the ankle. Injury to this nerve would result in a foot drop. The femoral nerve (choice B) innervates the flexors of the hip (iliacus, psoas, pectineus, rectus femoris and sartorius) as well as the extensors of the knee (quadriceps muscles). The superficial peroneal nerve (choice C) innervates the lateral compartment of the leg (peroneus longus and peroneus brevis). The sural nerve (choice D) provides sensation only to the lateral aspect of the foot. The tibial nerve (choice E) innervates the muscles of the superficial posterior (gastrocnemius, soleus, plantaris) and the deep posterior (popliteus, flexor hallucis longus, flexor digitorum longus, and tibialis posterior) compartments of the leg. Injury to this nerve would result in a loss of plantar flexion. A 23-year-old man is involved in a motor vehicle accident and is brought to the emergency department with a displaced fracture of the distal third of his left humeral shaft. On his right side he has a displaced fracture of the surgical neck of his humerus as well as a fracture of the medial epicondyle of his distal humerus. He complains of pain in his both arms as well as the inability to move part of his hand. On physical examination, his arm is swollen with a deformity at the corresponding parts of his humerus. His motor examination is abnormaI. Question 1 of 6

Which of the following nerves is most commonly damaged with this type of injury? / A. Axillary / B. Median / C. Musculocutaneous / D. Radial / E. UInar Explanation - Q: 4.1

Close

The correct answer is D. The radial nerve is most commonly injured in fractures to the mid shaft or distal third of the humerus. The radial nerve is a continuation of the posterior cord of the brachial plexus. It runs along the posterior wall of the axilla and passes through the triangular space. It lies in the spiral groove on the posterior aspect of the humerus and at the middle to

distal third of the humerus, it crosses the back of the humerus and pierces the lateral intermuscular septum. The axillary nerve (choice A) is commonly injured in proximal humerus fractures and dislocations. The median nerve (choice B) is commonly injured in supracondylar fractures of the humerus. The musculocutaneous nerve (choice C) is not commonly injured in humerus fractures. This nerve runs more proximal in the arm and would not be injured in a mid shaft or distal humerus fracture. It is a branch of the lateral cord of the brachial plexus and supplies the coracobrachialis, biceps brachii, and brachialis muscle. It terminates in the lateral cutaneous nerve of the forearm. The ulnar nerve (choice E) is commonly injured in medial epicondyle fractures of the distal humerus. Question 2 of 6

Which of the following muscles will this patient most likely have trouble using secondary to the nerve injury sustained in his accident? / A. Biceps / B. Extensor carpi radialis longus / C. FIexor carpi radialis / D. FIexor carpi ulnaris / E. Pronator teres Explanation - Q: 4.2

Close

The correct answer is B. The radial nerve innervates the triceps, brachioradialis, wrist and finger extensors, and supinator. The extensor carpi radialis longus is a wrist extensor and is innervated by the radial nerve. The biceps muscles (choice A) are innervated by the musculocutaneous nerve. The flexor carpi radialis (choice C) is innervated by the median nerve. The flexor carpi ulnaris (choice D) is innervated by the ulnar nerve. The pronator teres (choice E) is innervated by the median nerve.

Question 3 of 6

Which of the following arteries may be injured with the fracture of his right humerus? / A. Axillary artery / B. Brachial artery / C. Profunda brachii artery / D. Radial artery / E. UInar artery Explanation - Q: 4.3

Close

The correct answer is C. The profunda brachii (deep brachial) artery is a branch of the brachial artery, which runs posteriorly in the arm with the radial artery and may be injured with humeral shaft fractures. The axillary artery (choice A) extends from the first rib to the inferior border of the teres major muscle where it becomes the brachial artery. This location is too proximal for the fracture described. The brachial artery (choice B) extends from the inferior border of the teres major muscle to the cubital fossa in the elbow where it divides into the radial and ulnar arteries. The radial artery (choice D) is the lateral branch of the brachial artery. It would not be injured in this injury since it originates distal to the fracture site. The ulnar artery (choice E) is the medial branch of the brachial artery. It too would not be injured in this injury since it originates distal to the fracture site.

Question 4 of 6

His left shoulder is fractured at the surgical neck of the humerus. If he had a nerve injury, which of the following muscles would most likely be affected? / A. Axillary / B. Median / C. Musculocutaneous / D. Radial / E. UInar Explanation - Q: 4.4

Close

The correct answer is A. The axillary nerve is commonly injured in proximal humerus fractures and dislocations. The axillary nerve is a branch of the posterior cord of the brachial plexus. It runs along the posterior wall of the axilla and then runs through the quadrangular space where it touches the

surgical neck of the humerus. At this point it may be injured in fractures or dislocations of the proximal humerus. The median nerve (choice B) is commonly injured in supracondylar fractures of the humerus. The musculocutaneous nerve (choice C) is not commonly injured in humerus fractures. The radial nerve (choice D) is most commonly injured in fractures to the mid shaft or distal third of the humerus The ulnar nerve (choice E) is commonly injured in medial epicondyle fractures of the distal humerus.

Question 5 of 6

If his medial epicondyle fracture resulted in a nerve injury, which of the following muscles would most likely be affected? / A. Extensor digiti minimi / B. FIexor carpi radialis / C. FIexor carpi ulnaris / D. FIexor digitorum superficialis / E. FIexor pollicis longus Explanation - Q: 4.5

Close

The correct answer is C. One must first figure out that the ulnar nerve is injured in a fracture to the medial epicondyle. The flexor carpi ulnaris is the only muscle listed above innervated by the ulnar nerve. The extensor digiti minimi (choice A) is innervated by the posterior interosseous branch of the radial nerve. The flexor carpi radialis (choice B), flexor digitorum superficialis (choice D), and flexor pollicis longus (choice E) are innervated by the median nerve. Question 6 of 6

Peripheral nerves are divided into different layers. Which of the following describes the perineurium? / A. Connective tissue gaps between Schwann cells / B. Connective tissue that covers fascicles / C. Connective tissue that covers muscle bundles / D. Connective tissue that covers a single nerve fiber / E. Connective tissue that surrounds a entire nerve

Explanation - Q: 4.6

Close

The correct answer is B. Connective tissue that covers fascicles is termed the perineurium. The perineurium actually has two layers, an outer connective tissue layer and an inner epithelial layer. This inner layer has tight junctions and forms part of the blood-nerve barrier. Gaps between Schwann cells (choice A) are the nodes of Ranvier. Connective tissue that surrounds muscle bundles (choice C) is epimysium Connective tissue that covers a single nerve fiber (choice D) is called the endoneurium. Connective tissue that surrounds a entire nerve (choice E ) is called the epineurium.

A 35-year-old woman consults a physician because of increasing discomfort in her hands over the past year. Her symptoms have developed insidiously and are often worst for the first hour after rising. She has also been experiencing vague symptoms of fatigue and malaise, particularly in the early afternoon. Physical examination of her hands is notable for tenderness and synovial thickening of most of the small hand joints of both hands, particularly the proximal interphalangeal and metacarpophalangeal joints. A number of joints elsewhere, including in the feet, elbows, and ankles, are also symmetrically involved. An autoantibody directed against lgG is detected in serum. Question 1 of 6

Which of the following is the most likely diagnosis? / A. Gout / B. Osteoarthritis / C. Osteomyelitis / D. Osteoporosis / E. Rheumatoid arthritis Explanation - Q: 5.1

Close

The correct answer is E. This woman most likely has rheumatoid arthritis (RA). This chronic syndrome is characterized by inflammation of peripheral joints, and may or may not be accompanied by systemic manifestations. The American Rheumatism Association has put out revised (1987) criteria for the diagnosis of rheumatoid arthritis that require at least 4 of the following for diagnosis: morning stiffness for one hour or longer, arthritis of three or more

joints, arthritis of hand joints, symmetric arthritis, rheumatoid nodules, serum rheumatoid factor (RF; anti-IgG) present, and characteristic radiographic changes. In gout (choice A), the hands are usually spared, with common sites of involvement including the foot (typically the great toe), knee, wrist, and elbow. Osteoarthritis (choice B) usually produces only a short period of morning stiffness and commonly involves the distal as well as proximal interphalangeal joints, vertebral column, hip, and knee. Osteomyelitis (choice C) may involve vertebrae, feet (usually in diabetics), or the metaphyses of the tibia or femur (more common in children). Osteoporosis (choice D) causes bony loss, and is usually painless unless a fracture occurs.

Question 2 of 6

Arthroscopic examination and biopsy of a joint would be most likely to show which of the following? / A. Large numbers of bacteria / B. Roughly irregular and pitted cartilage surface / C. Small clusters of irregularly shaped, weakly birefringent crystals / D. Small clusters of needle-shaped, negatively birefringent crystals / E. Thickened synovium with large numbers of lymphocytes and plasma cells Explanation - Q: 5.2

Close

The correct answer is E. The synovial membrane in rheumatoid arthritis is attacked by lymphocytes and plasma cells and characteristically becomes thickened, forming a pannus that can erode the underlying cartilage. Large numbers of bacteria (choice A) suggests infectious arthritis. Roughly irregular and pitted cartilage surface (choice B) suggests osteoarthritis. Small clusters of irregularly shaped, weakly birefringent crystals (choice C) suggests pseudogout (calcium pyrophosphate crystal deposition). Small clusters of needle-shaped, negatively birefringent crystals (choice D) suggests gout.

Question 3 of 6

The patient also has a 2 cm, firm, non-tender, oval subcutaneous nodule near her right elbow. Biopsy of this nodule would most likely show which of the following? / A. Amorphous calcified material surrounded by fibrotic subcutaneous tissue / B. Central fibrinoid necrosis surrounded by epithelioid histiocytes / C. Cystic space filled with keratin and lined by squamous epithelium / D. Interwoven neural fibers / E. Membrane bound mass composed of mature adipocytes Explanation - Q: 5.3

Close

The correct answer is B. The lesion is most likely a rheumatoid nodule, which is histologically composed of an area of central fibrinoid necrosis, surrounded by palisading epithelioid histiocytes admixed with plasma cells and lymphocytes. None of the lesions seen in the alternative choices have increased incidence in rheumatoid arthritis. Amorphous calcified material surrounded by fibrotic subcutaneous tissue (choice A) would be typical of dystrophic calcification. Cystic space filled with keratin and lined by squamous epithelium (choice C) would be typical of an epidermal inclusion cyst. Interwoven neural fibers (choice D) would be typical of a neurofibroma. Membrane bound mass composed of mature adipocytes (choice E) would be typical of a lipoma.

Question 4 of 6

The patient is prescribed naproxen for treatment of her rheumatoid arthritis. Which of the following occurs with naproxen administration? / A. Binding to opioid receptors / B. BIocking voltage-dependent sodium channels / C. Changes in platelet function / D. Production of prostaglandins / E. Stimulation of cyclooxygenase Explanation - Q: 5.4

Close

The correct answer is C. Naproxen is a nonsteroidal anti-inflammatory drug (NSAID) that exerts its action by inhibiting cyclooxygenase (compare with choice E) leading to decreased production of prostaglandins (compare with choice D). This agent is indicated for treatment of mild-moderate pain (acute

and chronic) caused by inflammation. Naproxen affects platelet function, inhibiting platelet aggregation. The most common side effects of naproxen are gastrointestinal, such as nausea, vomiting, diarrhea, abdominal distress, flatulence, anorexia, and possible bleeding or ulceration. When this medication is administered with aspirin, and alcohol is ingested, there can be a dramatic increase in gastrointestinal side effects. Since naproxen can be directly irritating to the stomach mucosa, the concomitant administration of food or antacids may be necessary. There is also a risk of renal damage with naproxen, as with many of the NSAIDs, particularly in those with preexisting renal disease. Codeine and morphine are examples of drugs that bind to opioid receptors (choice A). Local anesthetics (e.g., procaine, bupivacaine, lidocaine) prevent pain by blocking nerve conduction. They block voltage-dependent sodium channels (choice B), thereby blocking depolarization and action potential conduction.

Question 5 of 6

If this patient also has a past medical history significant for psoriasis, which of the following medications is contraindicated for the treatment of rheumatoid arthritis, since it is likely to precipitate an attack of psoriasis? / A. Codeine / B. Hydroxychloroquine / C. Ibuprofen / D. Methotrexate / E. Penicillamine Explanation - Q: 5.5

Close

The correct answer is B. Hydroxychloroquine is an antimalarial agent that is used in the treatment of moderate to severe RA that is unresponsive to conventional therapy. The clinical benefits are generally seen 4-12 weeks after therapy is initiated; up to 6 months may be required to achieve a clinical response. The drug concentrates in the liver, spleen, kidney, heart, lung and brain. Warnings and precautions include precipitation of a psoriasis attack in patients with psoriasis, ophthalmic effects (irreversible retinal damage), muscle weakness, bleaching of hair, alopecia, mucosal pigmentation, and skin eruptions. The drug should not be used during pregnancy or in infants. Codeine (choice A) is an opioid analgesic that exerts its action at mu, kappa, and sigma opioid receptors. Mu receptors mediate analgesia, euphoria, and respiratory depression; kappa receptors mediate sedation and

miosis, and sigma receptors are associated with dysphoria and hallucinations. Ibuprofen (choice C) is a phenylpropionic acid derivative with analgesic, antipyretic, and anti-inflammatory effects. The anti-inflammatory and analgesic effects are due to inhibition of prostaglandin synthetase enzyme complex. It is indicated for treatment of mild to moderate pain, fever (except in children with chickenpox or flu symptoms) and inflammatory conditions (RA, rheumatic fever and osteoarthritis). The most common adverse effects are gastrointestinal, such as nausea, vomiting, diarrhea, abdominal distress, flatulence, anorexia, and possible bleeding and ulceration. Methotrexate (choice D) is indicated for the treatment of severe, active, classical or definite RA in adults who have an insufficient response with conventional therapies. It is also used as an antineoplastic therapy. The most common adverse reactions are nausea, vomiting, stomatitis, alopecia, leukopenia, and pancytopenia. Penicillamine (choice E) acts by suppression of RA disease activity. It is indicated for severe active RA that is unresponsive to conventional therapy.

Question 6 of 6

Several years later, the patient develops dry eyes and mouth. Buccal biopsy demonstrates a lymphocytic infiltrate around fibrotic minor salivary gland tissue. This patient has most likely developed which of the following other diseases? / A. Felty syndrome / B. Lyme disease / C. Reiter syndrome / D. Sjögren syndrome / E. Still disease Explanation - Q: 5.6

Close

The correct answer is D. The patient has developed Sjögren syndrome, which is characterized by an autoimmune attack on tear gland and salivary gland tissue. Rheumatoid arthritis may have multiple extra-articular manifestations, which may include rheumatoid nodules, vasculitis, lymphadenopathy, Sjögren syndrome, Felty syndrome (rheumatoid arthritis with neutropenia and splenomegaly, choice A), and episcleritis. Lyme disease (choice B) is due to a spirochete infection and produces an arthritis that can mimic rheumatoid arthritis.

Reiter syndrome (choice C) is associated with a reactive arthritis that follows an antecedent infection such as urethritis or diarrhea. Still disease (choice E) is a form of rheumatoid arthritis that lacks rheumatoid factor.

A 68-year-old man presents to his primary care physician with increasing deformity and disability in his right hand. He reports a five-year history of symptoms, beginning with nodules over the palmar aspect of his ring and small fingers of his hand. These have become larger and more painfuI. He now comes in because he feels "cords" in these two fingers, and he is unable to fully extend the fingers. He reports that his father had a similar problem in the past. On physical examination, there are nodules over the metacarpals of his ring and small fingers. Additionally, there is a flexion contracture of 30 degrees, i.e., the range of motion at the metacarpophalangeal joint of both these fingers is from 30 to 85 degrees. Question 1 of 3

Which of the following is the most likely diagnosis? / A. Dupuytren disease / B. Ledderhose disease / C. Osgood-Schlatter disease / D. Peyronie disease / E. Sever disease Explanation - Q: 6.1

Close

The correct answer is A. This patient has classic symptoms of Dupuytren disease, which is a palmar fibromatosis. The palmar fascia develops nodules and cords. It is seen in 1-2% of the population, with 50% of patients having bilateral disease. 10% of patients have a similar condition in other locations. It has variable familial inheritance, is seen most commonly in people of Celtic origin, and males are affected twice as commonly as females. Ledderhose disease (choice B) is a similar condition affecting the plantar surfaces of the feet (plantar fibromatosis). Osgood-Schlatter disease (choice C) is a osteochondrosis that occurs at the tibial tuberosity in children 11-15 years of age. It is a traction apophysitis from the stress of the extensor mechanism in a growing child. It is usually self-limited. Peyronie disease (choice D) is a similar condition to Dupuytren disease, but

affects the penis. Sever disease (choice E) is another osteochondrosis that affects the apophysis of the calcaneus in children 9-11 years old.

Question 2 of 3

What anatomic structure is most likely involved is this disease? / A. Carpal tunnel / B. FIexor digitorum profundus / C. FIexor digitorum superficialis / D. Median nerve / E. Palmar fascia Explanation - Q: 6.2

Close

The correct answer is E. The palmar fascia is the anatomic structure involved in Dupuytren disease. The carpal tunnel (choice A) is a space through which the median nerve and flexor tendons run. It is not involved in Dupuytren disease. The flexor digitorum profundus (choice B) and superficialis (choice C) are flexors of the fingers, and are not involved in Dupuytren disease. The flexion contracture is a result of the palmar fibrosis, not the flexor tendons. The median nerve (choice D) is involved in carpal tunnel syndrome. It supplies sensation to the radial three and a half fingers, and provides motor innervation for opposition of the thumb.

*** Commercial version is infinite. Order at http://www.structurise.com/kleptomania *** Question 3 of 3

After some questioning, this man reports that he also has a painful band on the dorsal aspect of his penis, but he was too embarrassed to tell anyone about this. Which of the following is the most likely diagnosis? / A. Dupuytren disease / B. Ledderhose disease / C. Osgood-Schlatter disease / D. Peyronie disease / E. Sever disease Explanation - Q: 6.3 The correct answer is D. Peyronie disease is a similar condition to

Close

Dupuytren disease, which affects the penis. It is characterized by fibrosis of the sinusoidal spaces of the corpora cavernosa. This patient has classic symptoms of Dupuytren disease (choice A) which is a palmar fibromatosis. The palmar fascia develops nodules and cords It is seen in 1-2% of the population with 50% of patients having bilateral disease. 10% of patients have a similar condition in other locations. It has variable familial inheritance, is seen most commonly in people of Celtic origin, and males are affected twice as commonly as females. Ledderhose disease (choice B) is a similar condition affecting the plantar surfaces of the feet. Osgood-Schlatter disease (choice C) is an osteochondrosis that occurs at the tibial tuberosity in children 11-15 years of age. It is a traction apophysitis from the stress of the extensor mechanism in a growing child. It is usually self-limited. Sever disease (choice E) is another osteochondrosis that affects the apophysis of the calcaneus in children 9-11 years old.

Approximately 4 days after a romantic encounter with a new partner at a ski lodge, a 32-year-old man develops mild urethral discomfort followed, a few hours later, by painful urination accompanied by a purulent discharge. He consults a physician the following day. Physical examination demonstrates red, swollen urethral lips and a purulent, yellowishgreen urethral discharge. Gram's stain of the discharge performed in the doctor's office demonstrates neutrophils packed with gramnegative kidney bean-shaped diplococci.

Explanation - Q: 1.1

Close

The correct answer is B. The only medically important gram-negative cocci are in the genera Neisseria and Moraxella. Neisseria meningitidis is the etiologic agent of meningococcal meningitis and meningococcemia and Neisseria gonorrhoeae is the etiologic agent of gonorrhea. Moraxella catarrhalis is a cause of pharyngitis, otitis media, and sinusitis in children. Chlamydia(choice A) is an intracellular organism that would not be visible with the Gram's stain. Staphylococcus(choice C) and Streptococcus(choice D) are gram-positive cocci and would not be associated with this set of symptoms. Treponema(choice E) is a spirochete that would not be visible on Gram's stain.

Explanation - Q: 1.2

Close

The correct answer is C. This patient has a classic presentation for gonorrhea. In men, the infection typically produces urethritis with a purulent urethral discharge 2 to 14 days after sexual contact. In women, gonorrhea may be nearly or completely asymptomatic, or may cause dysuria, increased frequency of urination, and vaginal discharge. Rectal infection in men or women may be asymptomatic or cause perianal discomfort and rectal discharge. AIDS (choice A) is a viral infection that would not cause these symptoms, and would not be diagnosed with a Gram's stain. Chlamydia urethritis (choice B) would not cause a purulent discharge because Chlamydia trachomatis is an intracellular parasite of mucosal cells, and it would not be visible on a Gram's stain. Lymphogranuloma venereum (choice D) is also caused by Chlamydia trachomatis, but by different serotypes than those that typically cause chlamydial STD in the U.S. Because Chlamydia is an intracellular pathogen, it would not cause a purulent discharge. Syphilis (choice E) characteristically causes production of a chancre at the site of inoculation rather than an urethritis, and the causative organism is a narrow spirochete that can only be visualized by darkfield microscopy or direct fluorescence.

Question 3 of 5

Following culture of the causative organism, which of the following biochemical attributes would be most definitive in determining the causative species?

/ / / / /

A. Catalase production B. Nitrate reduction C. Production of acid from glucose but no other sugars D. Production of acid from maltose E. Production of cytochrome c oxidase Explanation - Q: 1.3

Close

The correct answer is C. Neisseria gonorrhoeae is distinguished from all other members of the genus by its utilization of glucose, but not maltose. The entire genus Neisseria produces catalase (choice A); it is therefore not a characteristic that distinguishes the species, but only the genus. Nitrate reduction (choice B) is not a useful criterion, as it is absent in both Neisseria meningitidis and N. gonorrhoeae. Production of acid from maltose (choice D) is an attribute of Neisseria meningitidis, but not of N. gonorrhoeae. Production of cytochrome c oxidase (choice E) is an attribute of the entire genus Neisseria, as well as several gram-negative bacilli, such as Campylobacter, Helicobacter, and Vibrio.

Question 4 of 5

Genetic coding found for resistance to beta-Iactam antibiotics resides in which of the following locations? / A. Chromosome / B. Conjugative plasmids / C. Cytoplasmic membrane / D. Non-conjugative plasmids / E. Phage genome

Explanation - Q: 1.4

Close

The correct answer is D. In Neisseria gonorrhoeae, the plasmids that code for the production of beta lactamases possess the ori T, but not the tra operon. Other plasmids in the cell mediate production of the conjugal bridge by their possession of the tra operon, and the plasmids containing the drug resistance genes are mobilized across the preformed conjugal bridge along with the fertility factor plasmid. The chromosome (choice A) is the site of coding of genes for penicillin-

binding proteins, but most enzyme-mediated drug resistances are coded for on plasmids. Conjugative plasmids (choice B) is not correct. Although in most cases of enzyme-mediated drug resistance, the coding exists on plasmids, in the specific case of Neisseria gonorrhoeae, the genes exist on plasmids that do not have the tra operon and are thus, non-conjugative plasmids. The cytoplasmic membrane (choice C) is not correct because although this is the location of penicillin-binding proteins, and mutations in these proteins are responsible for low-level penicillin resistance in some species, there is no "genetic coding" i.e., DNA in this location. Furthermore, this is not the mechanism of drug resistance in Neisseria at the present time. The phage genome (choice E) is not correct because although phage may impart other important pathogenic features to bacteria, they are not generally associated with drug resistances.

Question 5 of 5

If this man had not sought treatment and had infected his pregnant wife, his baby would be at most risk for developing which of the following unless prophylactic measures were taken? / A. Condylomata lata / B. Ecthyma gangrenosum / C. Granuloma inguinale / D. Granulomatosis infantiseptica / E. Ophthalmia neonatorum Explanation - Q: 1.5

Close

The correct answer is E. The Centers for Disease Control (CDC) recommends routine use of 1% silver nitrate, erythromycin, or tetracycline ophthalmic ointments or drops instilled in each eye after the delivery of an infant to prevent ophthalmia, which might be caused by Neisseria, Chlamydia, or Treponema. Condylomata lata (choice A) is a manifestation of the secondary stage of syphilis, when flat, wart like growths appear on the mucosa. Ecthyma gangrenosum (choice B) is the characteristic skin lesion associated with Pseudomonas infections. Granuloma inguinale (choice C) is a disease of the genitalia caused by

Calymmatobacterium (Klebsiella) granulomatis. Granulomatosis infantiseptica (choice D) is an infection of the fetus in utero with Listeria monocytogenes.

A 19-year-old man presents to his college health clinic complaining of a painless sore on his penis. The patient states that 4 weeks prior to presentation, he had unprotected sexual intercourse with a new partner. About two weeks after this encounter, he developed a red spot on the glans of his penis. It has always remained small and now has created a "crater" on the tip of his penis. He denies dysuria, fevers, chills, meatal discharge, or any similar previous episodes. Three months ago he had an HIV test, which was negative. Upon examination, the physician elicits bilateral inguinal adenopathy that is firm but not tender to palpation. There is no discharge elicited from the urethral meatus. On the right side of the glans penis is a small chancre with indurated edges. The base of the lesion is clean, and no fluid can be expressed upon applying pressure. Rectal examination shows normal sphincter tone with a firm, appropriately sized, non-tender prostate. Urine dipstick is negative for any sign of infection. Explanation - Q: 2.1

Close

The correct answer is E. This patient has primary syphilis. Syphilis, a sexually transmitted disease, may present 2-4 weeks after exposure and begins as a hyperemic or erythematous spot. This painless papule or pustule, develops on the glans, corona, foreskin, shaft, suprapubic area, or scrotum. It may break down to form an indurated, punched-out lesion. The syphilitic (hard) chancre is relatively deep, has indurated edges and a clean base, and is not tender on pressure. The lesion may be so small and transient that it is missed. Without treatment, the lesion will heal spontaneously and slowly. Inguinal adenopathy may be tender or nontender and is typically firm and "rubbery." The ulcer associated with chancroid (choice A) is painful, and is deep, with an undermined border and a friable base that bleeds easily. The adenopathy is painful and, with chronic infection, may cause lymphatic obstruction. Incubation is one to four days. Chlamydia(choice B) causes urethritis, and not genital ulcers. It typically presents 7-21 days after exposure, with dysuria and mild-to-moderate whitish or clear urethral discharge. Genital herpes (choice C) typically presents as penile lesions of grouped

vesicles on an erythematous base that do not follow a neural distribution. The lesions are tender to touch and the associated adenopathy is bilateral, mildly tender, non-fixed, and slightly firm. The primary episode is more severe than recurrent attacks and the incubation period is 2 -10 days. Granuloma inguinale (choice E) initially presents as a papule that ultimately forms a non-tender, erythematous ulcer with hemorrhagic secretions. There is inguinal swelling or pseudobubo, which is actually a subcutaneous granulomatous process rather than a true lymphadenopathy. Untreated, it enlarges by direct extension, or may erode through the skin.

Explanation - Q: 2.2

Close

The correct answer is E. Syphilis is caused by the spirochete Treponema pallidum. It gains access through intact or abraded skin or mucous membranes. Calymmatobacterium granulomatis(choice A) is the causative agent of granuloma inguinale. Chlamydia trachomatis(choice B) is the bacteria responsible for nongonococcal urethritis. Chancroid is caused by Haemophilus ducreyi(choice C). The herpes simplex virus (choice D) is associated with genital herpes. Eighty percent of the genital lesions are caused by the type II virus. These lesions are painful, and associated with systemic symptoms such as malaise, anorexia, and fever.

Question 3 of 6

The treatment of choice for this infection is an intramuscular injection of penicillin G. Which of the following antibiotics has antimicrobial activity that is similar to that of the penicillins? / A. Aminoglycosides / B. Cephalosporins / C. Erythromycin / D. FIuoroquinolones / E. Sulfonamides Explanation - Q: 2.3

Close

The correct answer is B. The penicillins are classified as beta-lactam drugs. This is because of their core structure, which contains a thiazolidine ring attached to a beta-lactam ring that carries a secondary amino group. The mechanism of action of these agents involves damage to the bacterial cell wall. The steps involved in this are (1) attachment to specific penicillinbinding proteins that serve as drug receptors on bacteria, (2) inhibition of cell wall synthesis by blocking transpeptidation of peptidoglycan, and (3) activation of autolytic enzymes in the cell wall, which results in lesions that cause bacterial death. The cephalosporins also have a similar basic structure that incorporates the beta-lactam ring and therefore, their mechanism of action is similar to the penicillins. The aminoglycosides (choice A) are bactericidal by virtue of irreversible inhibition of protein synthesis. They penetrate the bacteria's cell wall and then bind to the 30S subunit of the bacterial ribosome to inhibit ribosomal protein synthesis. Erythromycin (choice C) is both inhibitory and bactericidal. It works by binding to the 50S subunit (specifically onto the 23S rRNA) of the ribosome. Protein synthesis is inhibited as aminoacyl translocation reactions and the formation of initiation complexes are blocked. The quinolones and fluoroquinolones (choice D) are potent inhibitors of nucleic acid synthesis. They block the action of DNA gyrase (topoisomerase II), the enzyme responsible for packing and unpacking supercoiled DNA. Sulfonamides (choice E) are bacteriostatic, and work by competitive inhibition. These medications compete with p-aminobenzoic acid (PABA) for the enzyme dihydropteroate synthetase to block a step in the pathway of the formation of purines, and therefore, ultimately, nucleic acids.

Question 4 of 6

If the patient were not treated, and instead developed advanced disease, which of the following most accurately describes the likely findings on physical examination? / A. Normal pupils / B. The pupils accommodate but do not react to light / C. The pupils accommodate and react to light / D. The pupils do not accommodate and do not react to light / E. The pupils do not accommodate but react to light Explanation - Q: 2.4

Close

The correct answer is B. Tertiary syphilis can affect the central nervous system (this is termed neurosyphilis) and spirochetes may be found in the cerebrospinal fluid. This may lead to tabes dorsalis, which is a neurological deficit caused by the destruction of the dorsal columns and dorsal roots of the spinal cord. Patients may also develop general paresis, due to invasion and destruction of brain parenchyma. Frequently, those patients with neurosyphilis will exhibit Argyll-Robertson pupils. This pathological finding consists of pupils that will accommodate but have an absent pupillary reflex to light. With accommodation, the eyes move medially and the pupils constrict when focusing on a close object. However, the pupils fail to constrict in response to a bright light, i.e., the pupillary reflex is absent.

Question 5 of 6

If his disease were left untreated, which of the following regions of the spinal cord would most likely be affected in this patient? / A. Anterior white commissure and ventral horns / B. Dorsal columns and dorsal roots / C. Dorsal columns, Iateral corticospinal tracts, spinocerebellar tracts / D. Lateral corticospinal tracts and ventral horns / E. Ventral horns Explanation - Q: 2.5

Close

The correct answer is B. Tabes dorsalis, seen in patients with neurosyphilis, is a slowly progressive degenerative disease that involves the dorsal columns and dorsal roots. Demyelination of the dorsal columns leads to a loss of tactile discrimination and position and vibration sensations. Pain and paresthesias can occur with irritative involvement of the dorsal roots. These patients can present with a positive Romberg sign. Lesions of the anterior white commissure and ventral horns (choice A)

occurs in syringomyelia. Lesions of the dorsal columns, lateral corticospinal tracts, and spinocerebellar tracts (choice C) occurs with subacute combined degeneration and Friedreich's ataxia. Lesions of the lateral corticospinal tracts and ventral horns (choice D) occurs in amyotrophic lateral sclerosis (ALS). Lesions of the ventral horns (choice E) occur in poliomyelitis.

Explanation - Q: 2.6

Close

The correct answer is C. In order to evaluate for the presence of syphilis, two strategies may be employed. One is the microscopic visualization of the spirochete from the skin lesion and the other is a serologic (blood) test to identify evidence of the body's reaction to syphilis. Fluorescent treponemal antibody-absorption is a treponemal-specific serological test that will remain positive for life. With this test, the patient's serum is filtered to separate out any treponemal antibodies that are not specific to T. pallidum. The patient's serum is poured onto a slide covered with T. pallidum antigens, then the slide is washed, leaving the anti-treponemal antibody-antigen complexes on the slide. Fluorescent antibodies that react with human immunoglobulins are then added, and bind to the antibody-antigen complexes on the slide. The slide is then examined with an ultraviolet microscope, any fluorescence indicates a positive test. Darkfield microscopy (choice A) and the direct fluorescent antibody T. pallidum test, or DFA-TP (choice B) are both techniques that utilize fluid from either the chancre or the maculopapular rash (if secondary syphilis). Darkfield microscopy uses direct visualization of the organism. DFA-TP consists of mixing anti-T. pallidum antibodies conjugated to fluorescein with a sample of fluid. If the sample contains the spirochete, the antibodies will bind and allow visualization of the spirochete with an ultraviolet microscope. Once

the lesion heals, there will be no spirochetes to see. VDRL (choice E) and RPR (choice D) are non-treponemal serological tests that are used for screening and to monitor the success of treatment. They employ an antigen that is a mixture of cardiolipin, cholesterol, and lecithin. Their titers rise following infection, and parallel disease activity. As the disease improves, RPR and VDRL levels will decrease, therefore, titers are followed to monitor treatment. Nontreponemal serological tests are nonspecific, and other conditions may give a positive result.

A 27 year-old man visits his primary care physician because of testicular pain. He states that over the last 3 weeks he has been feeling a vague and heavy sensation in his right testicle. He denies any dysuria, urethral discharge, testicular trauma, prior testicular surgery, fevers, or chills. His genital examination is normal except for scrotal examination. His right testicle is enlarged with an irregular, non-tender mass that appears to be arising from and obliterating the normal testicular architecture. The mass is not reducible, does not transilluminate, and does not change with Valsalva maneuver. The spermatic cord can be palpated superiorly to the mass and is normaI. Urinalysis is normaI. Question 1 of 5

Which of the following is the most likely diagnosis? / A. Acute epididymitis / B. Hydrocele / C. Inguinal hernia / D. Testicular cancer / E. Varicocele Explanation - Q: 3.1

Close

The correct answer is D. This patient has the classic signs and symptoms of testicular cancer. The most common symptom of testicular cancer is painless enlargement of the testis. Enlargement is usually gradual, and a sensation of testicular heaviness is not unusual. The mass is typically firm and nontender, and the epididymis should be easily separable from it. Acute epididymitis (choice A) is an infection of the epididymis acquired by the retrograde spread of organisms down the vas from the urethra. Patients present with heaviness and a dull, aching discomfort in the affected hemiscrotum that can radiate up to the ipsilateral flank. On examination, the epididymis will be markedly swollen and exquisitely tender to touch, eventually becoming a warm, red, enlarged, scrotal mass. Fevers and chills may develop.

A hydrocele (choice B) is a fluid collection within the tunica vaginalis surrounding the testis. It presents as a painless swelling of the scrotum that transilluminates. This transillumination is often necessary to differentiate a hydrocele from a testicular carcinoma. An inguinal hernia (choice C) is the result of a weakness in the floor of the inguinal canal. It may be an incidental finding on physical examination or may present with pain in the groin with Valsalva maneuver. If a scrotal mass is present, it may contain a loop of bowel protruding through the weakness in the inguinal canal. A mass associated with an inguinal hernia can usually be differentiated from the testicle itself. A varicocele (choice E) is an abnormal dilatation of the veins of the pampiniform plexus and the internal spermatic vein of the spermatic cord. Left-sided varicoceles are most common, occurring in approximately 15% of normal adult males. The dilated veins are best palpated with the patient in the standing position and aided by a Valsalva maneuver. The vessels are palpated superior to the testicle and are described as feeling like "a bag of worms."

Question 2 of 5

Prior to removal of the testicle (radical orchiectomy), what two blood tests must be performed on this patient? / A. AIdosterone and beta hCG (human chorionic gonadotropin) / B. AIpha-fetoprotein (AFP) and beta hCG / C. Complete blood count (CBC) and calcium / D. Prostate-specific antigen and alpha-fetoprotein / E. Testosterone and alpha-fetoprotein Explanation - Q: 3.2

Close

The correct answer is B. Many germ cell tumors produce specific oncofetal protein markers, either AFP or hCG, that can be detected in patients' serum or tissue. Ninety percent of patients with nonseminomatous testis tumors will have elevations of one or both markers and 5-10% with pure seminomas will demonstrate elevations of hCG only. The amount of tumor burden is proportional to the degree of marker elevation. It is important to draw these levels prior to surgery. After removal, the tumor markers are monitored to determine if there is residual disease (i.e., progression to retroperitoneal lymph nodes). Aldosterone (choice A) is produced within the outer cortical zona glomerulosa of the adrenal cortex. Production is under the influence of angiotensin II and the renin-angiotensin system. Aldosterone acts at the distal tubule of the nephrons to cause sodium retention and potassium

secretion. It plays no role in the evaluation or monitoring of disease progression for testicular cancer. A complete blood count (choice C) measures the patient's white blood cells, hemoglobin, hematocrit, and platelet counts. These values are not normally altered in patients with testicular cancer. Calcium levels may be abnormal in patients with malignancy and some paraneoplastic syndromes. Testicular tumors are not generally associated with paraneoplastic syndromes. Prostate-specific antigen (PSA) (choice D) is a tumor marker for prostate cancer. It is secreted by the prostate and may be elevated in patients with prostate cancer. It has no role in the work-up of testicular cancer. Testosterone levels (choice E) are not altered with testicular cancer. There is no need to measure this substance in these patients.

Question 3 of 5

Radical orchiectomy requires an inguinal incision and removal of the testicle and spermatic cord. Which of the following nerves runs parallel to the spermatic cord within the inguinal canaI? / A. Femoral / B. IIioinguinal / C. Lateral femoral cutaneous / D. Obturator / E. Pudendal Explanation - Q: 3.3

Close

The correct answer is B. The inguinal canal is an oblique passage through the inferior part of the anterior abdominal wall. The chief protection of the inguinal canal is muscular. Its main constituent is the spermatic cord in the male and the round ligament of the uterus in females. It contains the ilioinguinal nerve in both sexes. This nerve is derived from the L1 segment, enters the abdomen posterior to the medial arcuate ligament, and passes inferolaterally, anterior to the quadratus lumborum muscle. The nerve pierces the transversus abdominis muscle near the superior iliac spine, travels within the inguinal canal, and passes through the superficial (external) inguinal ring to supply the skin of the groin and scrotum or labium majora. The femoral nerve (choice A) is the largest branch of the lumbar plexus. It forms in the abdomen within the substance of the psoas major muscle and descends posterolaterally through the pelvis to the midpoint of the inguinal ligament. It then passes lateral to the femoral vessels, outside the femoral sheath enclosing them. At no point in its course does it enter the inguinal

canal. The lateral femoral cutaneous nerve (choice C) originates from L2 and L3 and is a direct branch of the lumbar plexus. It enters the thigh deep to the lateral end of the inguinal ligament, near the anterior superior iliac spine. It supplies the skin on the anterior and lateral aspects of the thigh. The obturator nerve (choice D), originating from L2, L3, and L4 of the lumbar plexus, is the nerve of the adductor muscles of the thigh. This nerve descends through the psoas major muscle, leaving its medial border at the brim of the pelvis. It pierces the psoas fascia, crosses the sacroiliac joint, passes lateral to the internal iliac vessels and ureter, and enters the pelvis minor. It leaves the pelvis through the obturator foramen and enters the thigh. The obturator nerve supplies motor innervation to the obturator externus, which is responsible for laterally rotating the thigh. This nerve also provides a small cutaneous branch, which is responsible for sensation to the medial aspect of the thigh. The pudendal nerve (choice E) arises from the sacral plexus by separate branches of the ventral rami of S2, S3, and S4. It accompanies the internal pudendal artery and leaves the pelvis between the piriformis and coccygeus muscles. It hooks around the sacrospinous ligament to enter the perineum through the lesser sciatic foramen. Here, it supplies the muscles of the perineum, including the external anal sphincter, and ends as the dorsal nerve of the penis or clitoris. Question 4 of 5

Patients with advanced stages of this condition may be treated with bleomycin. Which organ system toxicity is unique to bleomycin? / A. Bone marrow / B. Cardiac / C. Neurologic / D. Pulmonary / E. Renal Explanation - Q: 3.4

Close

The correct answer is D. Bleomycin, a chemotherapeutic agent, works by binding to and then breaking DNA strands. Toxicities include pneumonitis and pulmonary fibrosis. Patients should have pulmonary function testing (PFT) prior to the administration of this medication. Bone marrow toxicity (choice A), i.e., myelosuppression, is seen with many different chemotherapeutic agents and is not unique to bleomycin.

Cardiac toxicity (choice B) is a rare complication of chemotherapy. It is associated with the administration of doxorubicin. Toxicity to the nervous system (choice C) is not usually encountered with bleomycin. Platinum-based chemotherapeutic regimens are most effective against testicular tumors. Cisplatin is a known nephrotoxic agent and its use is limited in patients with renal insufficiency. Carboplatin is another platinumbased medication not associated with renal toxicity (choice E).

Question 5 of 5

A biopsy taken from an infant with this condition would most likely reveal which of the following? / A. Lymphoma / B. Mixed cell type / C. Seminoma / D. Teratoma / E. Yolk sac tumor Explanation - Q: 3.5

Close

The correct answer is E. Testicular tumors are classified as either germ cell or non-germ cell tumors. Germ cell tumors comprise 95% of all testicular tumors. The two major divisions are seminomas and nonseminomas. The nonseminoma division includes yolk sac, embryonal carcinoma, teratoma, choriocarcinoma, and mixed tumors. Yolk sac tumors, also called endodermal sinus tumors or orchioblastomas, are nonseminomatous germ cell tumors that have a peak incidence in the infant and childhood age group. Grossly, these tumors appear yellow. Histologically, they contain SchillerDuval bodies that resemble 1-2 week old embryos, i.e., a cavity surrounded by syncytiotrophoblasts and cytotrophoblasts. Yolk sac tumors metastasize hematogenously, compared to other germ cell tumors, which spread via the lymphatics. Lymphoma (choice A) is the most common metastatic (secondary) tumor of the testis and the most common testis tumor in men > 50 years old. Mixed cell type (choice B) refers to tumors that contain a combination of nonseminoma and/or seminoma. These account for up to 40% of testicular germ cell tumors. If a tumor contains both seminoma and nonseminoma, it is treated as a nonseminoma. Seminoma (choice C) accounts for approximately 35% of germ cell tumors.

There are three histological subtypes, classic seminoma, anaplastic seminoma, and spermatocytic seminoma. The classic variant accounts for 85% of all seminomas and is most common in the fourth decade of life. Teratomas (choice D) contain derivatives of all three cell layers: ectoderm, endoderm, and mesoderm. Microscopically, they appear as clear or mucinous cystic areas interspersed with solid tissue including bone, muscle, or cartilage. The peak incidence is in the 25-35 year old age group.

An 18-year-old girl is involved in a traffic accident resulting in head trauma and intracerebral hemorrhage. She undergoes extensive rehabilitation and her neurological and mental status improve. At present, she presents with complaints of polydipsia, polyuria and nocturia, and frequent daytime napping. On examination, no signs or symptoms of significant dehydration are found. Her glucose tolerance test and urine analysis are normaI. Serum osmolality is 316 mOsmoI/kg, and her urine osmolality is 136 mOsmoI/kg. Question 1 of 5

Which of the following is the most likely cause of polyuria in this patient? / A. ADH deficiency / B. Compulsive water drinking (psychogenic polydipsia) / C. Iatrogenic polydipsia / D. Increased metabolism of ADH / E. Lithium therapy Explanation - Q: 1.1

Close

The correct answer is A. Polyuria is defined as urine production more than 3 L/day. There are two main causes of polyuria: water diuresis (osmolarity less than 250 mosmol/L) and osmotic (solute) diuresis (more than 300 mosmol/L). Water diuresis can be caused by diabetes insipidus (DI) or primary polydipsia with an increased water intake and consequent polyuria. Decreased secretion (central diabetes insipidus) or action (nephrogenic DI) of arginine vasopressin (AVP, ADH) is characterized by the production of abnormally large volumes of urine (more than 50 mL/kg/d) with low osmolarity (less than 200 mosmol/L). The polyuria leads to the symptoms of urinary frequency and nocturia, which may disturb sleep, causing daytime somnolence. Central DI can be primary (familial, idiopathic) or secondary (posttraumatic, infection, tumors, sarcoidosis, histiocytosis X). ADH is synthesized within supraoptic and paraventricular nuclei of the hypothalamus, but the posterior pituitary lobe is the major site of AVP storage and release. Injury may damage the axons of the pituitary stalk and impair the transport and release of AVP. Administration of ADH in normal people and those with primary polydipsia, does not increase urine osmolality more than 9%. In central DI, the rise in urine osmolality is greater than 9%. In nephrogenic DI, there is no change in urine osmolality with either dehydration or with ADH administration. Plasma ADH is low in central DI. In primary polydipsia, plasma sodium is low-normal, while in DI, it is normal to high. Emotionally disturbed patients can drink up to 6-7 L of fluid per day (psychogenic, primary polydipsia, choice B). They usually do not have nocturia. The polydipsia leads to the suppression of endogenous ADH and resultant polyuria. Patients usually remain normonatremic, despite the large

fluid intake, although plasma osmolality may be normal or slightly reduced. Psychotherapy is required for most patients with compulsive water drinking. Thioridazine and lithium should be avoided since they cause polyuria. Iatrogenic polydipsia (choice C) results from recommendations of popular media or some health professionals to increase fluid intake for its presumed preventive or therapeutic benefits for other diseases. This form can often be corrected by patient counseling. Vasopressinase, the enzyme that metabolizes AVP, increases during pregnancy from the 4th to the 38th week by 1000-fold. As a result, vasopressinase-induced DI (choice D) may be seen in the late pregnancy or puerperium. A circulating enzyme destroys native AVP, but synthetic (DDAVP) is unaffected, and therefore may be used to control the symptoms. This form is usually referred to as gestational DI. Vasopressin-resistant (nephrogenic) DI may be caused by the congenitally defective expression of V2 receptors or the aquaporin-2 (AQP2) protein, or may be acquired in association with certain diseases (pyelonephritis, myeloma, Sjögren syndrome, sickle cell anemia) and/or drug treatment (demeclocycline, lithium, methicillin). In this case, the polyuria is unresponsive to vasopressin, and the hormone secretion is normal. Lithiuminduced DI (choice E) may respond to amiloride even if lithium treatment is continued.

Question 2 of 5

Which of the following would produce polyuria via a mechanism of action most similar to the mechanism of the disease process in this patient? / A. Ethanol / B. Hyperosmolality / C. Hypotension / D. Lithium / E. Nausea Explanation - Q: 1.2

Close

The correct answer is A. Ethanol inhibits vasopressin release from the posterior pituitary, thereby inducing diuresis. Ethanol also inhibits oxytocin secretion and has tocolytic activity due to both direct action on uterine muscle and inhibition of oxytocin release. Phenytoin, haloperidol, clonidine, and alpha adrenergic agonists also inhibit vasopressin secretion. When plasma osmolality rises above the "threshold" of 280 mOsmol/kg,

specialized cells known as osmoreceptors, located in the anteromedial hypothalamus near the supraoptic nucleus, react and stimulate AVP secretion, which leads to antidiuresis that is maximal when plasma osmolality reaches 295 mOsmol/kg (choice B). When blood pressure falls (choice C), stretch receptors in the aorta and carotid arteries are stimulated. They project via the vagus and glossopharyngeal nerves to the nucleus tractus solitarius, and from there to the supraoptic and paraventricular nuclei, thereby increasing the output of ADH. Lithium (choice D) does not affect ADH secretion. The primary effect of lithium is to block ADH-mediated water reabsorption by inhibiting ADHsensitive adenylate cyclase production in the collecting duct cells, thereby inducing ADH-resistance and causing polyuria. Emetic stimuli (choice E) are very potent since they typically induce an immediate 50- to 100-fold increase in plasma vasopressin. These stimuli act via the emetic center in the medulla oblongata and can be completely blocked with antiemetics, e.g. fluphenazine.

Question 3 of 5

Which of the following vasopressin receptors most likely mediates vasopressin's antidiuretic action? / A. AQP6 / B. Oxytocin receptor / C. V1a / D. V1b / E. V2 Explanation - Q: 1.3

Close

The correct answer is E. The vasopressin receptor family is unique among all classes of peptide receptors, since its members couple to different subsets of G proteins. Vasopressin binds to four distinct types of receptor subtypes V1a, V1b, V2 and the oxytocin receptor. The V2 receptors are located on the basolateral membrane of collecting duct cells. This receptor belongs to the seven-transmembrane domain, G-protein-coupled receptor superfamily. Stimulation of the V2 receptor by vasopressin leads to the activation of a G protein (Gs), resulting in the activation of adenylyl cyclase, the production of cAMP, and the stimulation of protein kinase A (PKA). PKA stimulates aquaporin 2, a water channel protein, which undergoes vesicular trafficking, inserting into the apical membrane of the duct cells, thereby increasing the water permeability of that membrane. At maximal ADH levels,

less than 1% of the filtered water is excreted, with the minimum urine volume of 500 mL/day and osmolality of 1200 mosmol/L. Long term regulation by vasopressin involves the effects of cAMP on the cAMP-response elements in the 5'-flanking region of the AQP2 gene, which increases the abundance of aquaporin 2 protein in the target cells. The human V2 receptor gene, AVPR2, is located in chromosome region Xq28. The genetic form of nephrogenic diabetes insipidus is caused by mutations in the coding region of the V2 receptor gene. AQP6 (choice A) is a water channel protein that is localized exclusively in intracellular membranes in renal epithelia. Vasopressin has low affinity for oxytocin receptors. The oxytocin receptor (choice B) is a typical class I G protein-coupled receptor, primarily stimulating phospholipase C beta activity. The V1a receptor (choice C) mediates smooth muscle contraction (vasoconstriction in splanchnic, coronary, cerebral, peripheral, pulmonary, and intrahepatic vessels), platelet aggregation, hepatic glycogenolysis, and coagulation factor release by vasopressin. It is also proposed that brain V1a vasopressin receptors may mediate the effect of ADH on short-term memory and learning, antipyresis, cardiovascular responsivity, and brain development. V1b (or V3) receptors (choice D) are expressed in the anterior pituitary, and their activation leads to the potentiation of ACTH secretion from pituitary corticotrophs. The binding of vasopressin to V1b and V1a receptors results in activation of phospholipase C mediated by a specific pertussis toxininsensitive receptor-coupled G protein (Gq). This activation induces the production of diacylglycerol (DAG), which activates protein kinase C, and inositol trisphosphate (IP3) that mobilizes intracellular calcium.

Question 4 of 5

The principal target of vasopressin actions in the kidney is water permeability of the collecting ducts. Which of the following water channel proteins is the mediator of this vasopressin effect? / A. Aquaporin 0 (AQP0) / B. AQP1 / C. AQP2 / D. AQP3 / E. AQP5 Explanation - Q: 1.4

Close

The correct answer is C. The aquaporins (AQPs) represent a family of water-selective membrane channels that play a critical role in controlling the water content of cells. To date, ten AQPs have been identified and cloned in humans, from AQP0-AQP9. They are integral membrane proteins and belong to the major intrinsic protein family. Aquaporin 2 (AQP2) is expressed exclusively in the apical membrane and subapical vesicles in inner medullary collecting duct principal cells. It is diffusely distributed in the cytoplasm in the euhydrated state, whereas apical localization is seen in the dehydrated conditions or after vasopressin administration. The binding of vasopressin (ADH) to its receptor in the cells causes the translocation of the intracellular pool of AQP2, and its exocytic insertion into the apical membrane (the shuttle mechanism), and also stimulates transcription of the AQP2 gene. The cycle of insertion of water channels into, then removal from the luminal membrane is referred to as vesicular trafficking. AQP2 forms a tetrameric complex that spans the membrane, creating a channel, which allows rapid water movement in response to an osmolar gradient. When open, the channel allows 3 billion molecules of water to pass each second. The net rate of water flux across the cell is determined by the number of AQP2 channels in the apical membrane and the strength of the osmotic gradient between tubular fluid and the renal medulla. Tight junctions on the lateral surface serve to prevent unregulated water flow. This water channels facilitate, in some cases, the transport of other small solutes, e.g., urea and glycerol. They are, however, completely impermeable to charged particles, such as protons, which is critical for the conservation of membrane electrochemical potential. Mercurial diuretics bind to a specific site on AQP2 and block water reabsorption. AQP0 (choice A), a major intrinsic protein of the ocular lens fiber membrane, is the prototype molecule of the water channel protein family. AQP1 (choice B, also known as CHIP, channel-forming integral protein of 28 kDa) was the first protein shown to function as a molecular water channel. It is expressed in the proximal tubule, the descending thin limbs of Henle loops, and in vasa recta, contributing to the urinary concentrating mechanism. AQP3 (choice D) is localized at the basolateral membranes of collecting duct cells in the kidney. The permeability of the basolateral membrane is not regulated by vasopressin, and the presence of AQP3 (together with AQP4) means the basolateral membrane has continuous water permeability. AQP5 (choice E) is implicated in the generation of saliva, tears, and pulmonary secretions.

Question 5 of 5

Which of the following is the most appropriate pharmacotherapy? / A. Carbamazepine / B. Chlorpropamide / C. Desmopressin (DDAVP) / D. Hydrochlorothiazide / E. Indomethacin (NSAIDS) Explanation - Q: 1.5

Close

The correct answer is C. DDAVP (desmopressin acetate, 1-deamino-8-Darginine vasopressin), a synthetic analog of arginine vasopressin, has prolonged antidiuretic (but not vasopressor) activity lasting for 12 to 24h, and may be administered intranasally, sc, or IV. DDAVP produces a slight increase in total body water (1-3%) and a commensurate decrease in plasma osmolality and sodium concentration that rapidly eliminates thirst and polydipsia. DDAVP acts selectively at V2 vasopressin receptors to increase urine concentration and decrease urine flow in a dose-dependent manner. Carbamazepine (choice A), an anticonvulsant, may be beneficial because of its ADH-releasing properties, but is not the most appropriate choice. Chlorpropamide (choice B), a hypoglycemic agent, has an antidiuretic action, and also promotes a renal response to ADH. At doses similar to those used in the treatment of DM, chlorpropamide increases urine concentration, decreases urine flow, thirst and polydipsia in a manner similar to DDAVP, but with higher doses there is a risk of hypoglycemia. Hydrochlorothiazide (choice D), a diuretic, can paradoxically decrease polyuria to some extent with low salt intake, in the absence of ADH. Urine volumes may fall 25-50% during the daily administration of 15-25 mg/kg of a thiazide diuretic. Inhibition of prostaglandin synthesis reduces delivery of solute to distal tubules. This effect would reduce urine volume and increase urine osmolality. Indomethacin (choice E) combined with restriction of sodium intake and thiazide diuretics would be the most efficient therapy for nephrogenic DI, but in this case of vasopressin deficiency, antidiuretic hormone replacement is most appropriate.

A child is taken to a pediatrician because his mother notices that his eyes appear very puffy. The mother notes that the boy's eyes appeared

normal two days ago, and part of what caused her concern was that her child seemed to be rapidly becoming ilI. On physical examination, the boy is noted to have generalized edema. No hypertension or jaundice is noted. BIood urea nitrogen and serum creatinine are within normal Iimits. A urine sample is collected, and the nurse notices that the top of the urine has a small amount of foam at the top. Urinalysis is negative for glucose, red cells, white cells, casts, crystals, and bacteria. A 24-hr-urine specimen is collected, which demonstrates proteinuria of 55 mg/h/m2. Question 1 of 6

Which of the following is the most likely diagnosis? / A. Acute renal failure / B. Chronic renal failure / C. Lower urinary tract disease / D. Nephritic syndrome / E. Nephrotic syndrome Explanation - Q: 2.1

Close

The correct answer is E. Proteinuria greater than 40 mg/h/m2 in a child or 3.5 g/d/1.73 m2 in an adult produces nephrotic syndrome, which is characterized by generalized edema, often most noticeable in the face. The condition develops when large amounts of protein are spilled through the glomeruli into the duct system of the kidneys. In contrast, nephritic syndrome (choice D) is accompanied by lower levels of proteinuria, together with microscopic or macroscopic hematuria (red cells and blood in urine). Acute (choice A) and chronic (choice B) renal failure are characterized by rising serum levels of BUN and creatinine. The findings seen do not suggest lower urinary tract disease (choice C), which is typically due to a lower urinary tract infection (which would show bacteria and white cells), calculi (crystals would likely be present), or tumor (which would show abnormal bladder epithelial cells).

Question 2 of 6

The edema seen in this patient is the result of which of the following processes? / A. Lowered hydrostatic pressure in capillaries / B. Lowered hydrostatic pressure in the extracellular interstitial space / C. Lowered oncotic pressure in capillaries / D. Lowered oncotic pressure in the extracellular interstitial space / E. Occlusion of lymphatic channels Close Explanation - Q: 2.2

The correct answer is C. Edema occurs when there is a net shift of fluid into the extracellular interstitial space. In the case of the edema produced by nephrotic syndrome, the urinary protein losses lead to a decreased serum concentration of albumin, which, in turn, lowers the oncotic pressure within the capillary bed. This favors a shift of fluid from the vascular space to the extracellular interstitial space. Edema formation can also be favored by raising the hydrostatic pressure in the capillary bed (opposite of choice A), lowering the hydrostatic pressure in the extracellular interstitial space (choice B, as happens when a person wearing support hose takes them off and develops ankle edema), or occluding the lymphatic channels (choice E, as can happen in filarial elephantiasis or following axillary lymph node dissection for breast cancer). In contrast, lowering the oncotic pressure in the extracellular interstitial space (choice D), as can happen when there is resolution of a previous tissue injury with resorption of proteins released by cell injury, tends to shift fluid back into the vascular space, and reduce edema.

Question 3 of 6

Which of the following is the most common cause of this patient's symptom pattern in children? / A. Membranoproliferative glomerulonephritis / B. Membranous glomerulonephritis / C. Minimal change disease / D. Post-infectious acute glomerulonephritis / E. Proliferative glomerulonephritis as a complication of systemic lupus erythematosus Explanation - Q: 2.3

Close

The correct answer is C. Minimal change disease accounts for roughly 90% of cases of nephrotic syndrome in children. This condition also occurs less commonly in adults. Membranoproliferative glomerulonephritis (choice A) can also cause nephrotic syndrome in children, but is less common and may produce both nephrotic syndrome and microscopic hematuria. Membranous glomerulonephritis (choice B) is the most frequent cause of nephrotic syndrome in adults greater than 40 years of age. Post-infectious acute glomerulonephritis (choice D) is relatively common in children, but usually causes nephritic syndrome, rather than nephrotic

syndrome. Proliferative glomerulonephritis related to lupus (choice E) is a common secondary cause of nephrotic syndrome, but most of the patients are adults.

Question 4 of 6

The peak age at which this patient's specific disease occurs is which of the following? / A. 2 years / B. 5 years / C. 8 years / D. 12 years / E. 15 years Explanation - Q: 2.4

Close

The correct answer is A. The peak age of incidence in children with minimal change disease is 2 years, and over 80% of cases occur in children 6 years of age or less. Adults who develop minimal change disease have mean age of onset of 40 years.

Question 5 of 6

If a renal biopsy were performed, which of the following findings on electron microscopy is most characteristic of this patient's specific disease? / A. Effacement of podocyte foot processes / B. Expansion of mesangial cell population / C. "Spikes" of basement membrane material / D. Subendothelial deposits / E. SubepitheliaI "humps" deposits Explanation - Q: 2.5

Close

The correct answer is A. Minimal change disease is so named because of its very bland light microscopic appearance on renal biopsy. With electron microscopy, widespread fusion is seen of the foot processes of the podocytes that form the outer layer of the glomerular loops. You should be aware that this finding, which is classically associated with minimal change disease in test questions, is actually not completely specific for minimal change disease and is instead found as well in other glomerular diseases associated with high proteinuria. However, in these other diseases, the glomeruli will show other changes (electron dense deposits and/or cellular

proliferations) as well. "Spikes" (choice C) of basement membrane material separating epimembranous electron-dense deposits are characteristic of membranous glomerulonephritis. Subendothelial electron-dense deposits (choice D) and the expansion of the mesangial cell population (choice B) can be seen in a variety of glomerular diseases, including proliferative glomerulonephritis and membranoproliferative glomerulonephritis. Electron-dense "humps" (choice E) are characteristic of post-infectious acute glomerulonephritis.

Question 6 of 6

A drug from which of the following classes would be most likely to correct this patient's proteinuria? / A. Antibiotic / B. Corticosteroid / C. Potassium-sparing diuretic / D. Sex steroid / E. Thiazide diuretic Explanation - Q: 2.6

Close

The correct answer is B. Approximately 90% of children with minimal change disease respond within two weeks to corticosteroid therapy with a decrease in proteinuria. This rapid response, together with the high incidence of minimal change disease compared to other causes of nephrotic syndrome in children, has led to a common practice among pediatricians of reserving renal biopsy for those cases in which the response to steroids was either non-existent or much less pronounced. Of the children who respond, most will undergo a complete remission, although in roughly half of cases recurrent episodes of minimal change disease may later occur. Adults and adolescents will also often respond to steroid therapy, although the time to remission may be up to 16 weeks. Antibiotics (choice A) are used in patient with minimal change disease to control intercurrent infections that develop, in part, because of the pharmacologic immunosuppression. Potassium-sparing diuretics (choice C) and sex steroids (choice D) are not usually used in this population.

Thiazide diuretics (choice E) are occasionally used in those patients with very severe edema, particularly if accompanied by respiratory symptoms.

A child is brought to an emergency department after his mother notes browndiscolored urine in the toilet. The child has periorbital edema, although generalized edema is not seen. Urinalysis demonstrates 2+ proteinuria and hematuria. A few neutrophils and renal epithelial cells are also seen in the urinalysis sample. Question 1 of 8

AIso noted on urinalysis are numerous large, elongated, stick-Iike, translucent and colorless structures with low refractive index that are difficult to discern on microscopy. These are most likely which of the following? / A. Ammonium biurate crystals / B. Calcium oxalate crystals / C. Hyaline casts / D. Triple phosphate crystals / E. Urothelial cells Explanation - Q: 3.1

Close

The correct answer is C. The structures are hyaline casts. Ammonium biurate crystals (choice A) have an unusual "crab-like" structure and a yellow-brown color. Calcium oxalate crystals (choice B) form colorless, birefringent, octahedrons. Triple phosphate crystals (choice D) form colorless, birefringent crystal with a "coffin-lid" shape. Urothelial cells (choice E) are clear and round, with a visible clear, ovoid central nucleus.

Question 2 of 8

The significance of the elongated, colorless structures seen on urinalysis is which of the following? / A. They indicate that desquamation of the urothelial mucosa is occurring / B. They indicate that infection is present / C. They indicate that stone formation is likely / D. They indicate that the kidney is involved / E. They indicate that the person may have an increased risk of developing gout

Explanation - Q: 3.2

Close

The correct answer is D. Casts form within the renal tubular system, and specifically indicate that the disease process involves the kidneys. The presence of urothelial cells in urine would indicate that desquamation of urothelial mucosa was occurring (choice A), as happens normally to a small degree. The presence of bacteria and neutrophils would suggest infection (choice B). The presence of some types of crystals in the urine (choice C) may suggest that a predisposition for stone formation is present. While urate crystals in tissues are a feature of gout, their presence in urine is not considered a risk factor for developing gout (choice E).

Question 3 of 8

Which of the following is the most likely diagnosis? / A. Acute nephritis / B. Acute pyelonephritis / C. Chronic pyelonephritis / D. Minimal change disease / E. Transitional cell carcinoma of the bladder Explanation - Q: 3.3

Close

The correct answer is A. This child has an acute nephritis, as evidenced by the combination of hematuria, evidence of renal involvement (e.g., the epithelial cells and casts), proteinuria, and lack of bacteria or significant inflammatory cell response in the urine. Acute nephritis is clinically subdivided into cases related to previous infection and those related to other conditions (e.g., Henoch-Schönlein purpura, Berger disease, systemic lupus erythematosus, familial forms, shunt nephritis). Acute pyelonephritis (choice B) would show numerous bacteria and neutrophils in the urine. Chronic pyelonephritis (choice C) typically is associated with an unremarkable urine, although scanty casts may be found. Minimal change disease (choice D) would be characterized by severe

proteinuria without hematuria. Transitional cell carcinoma of the bladder (choice E) would be very unusual in a child, and might cause hematuria and abnormal urothelial cells in urine, but would not cause casts in the urine. Question 4 of 8

If this child's renal disease is related to a prior infection, the child most likely has a history of which of the following in the preceding 7-21 days? / A. Cystitis / B. Diarrhea / C. Gastritis / D. Pharyngitis / E. Urethritis Explanation - Q: 3.4

Close

The correct answer is D. Postinfectious glomerulonephritis most commonly follows a sore throat, but can also follow streptococcal skin infections. The peak incidence of poststreptococcal glomerulonephritis is 5-6 years, with most cases occurring between the ages of 4 and 12. Cases have been reported, however in children as young as 1 year of age and up to adults as old as 90 years. Patients may exhibit a range of symptoms from one extreme of an asymptomatic child with urinary abnormalities picked up incidentally to the opposite extreme of a child with severe oliguria, edema, hypertension, and azotemia. Statistically, most patients have disease toward the mild end of the spectrum. Poststreptococcal glomerulonephritis does not usually follow lower urinary tract infections (choices A and E) or gastrointestinal disease (choices B and C).

*** Commercial version is infinite. Order at http://www.structurise.com/kleptomania *** Question 5 of 8

Which of the following is the most common causative organism implicated in the infection preceding the child's urinary symptoms?

Explanation - Q: 3.5

Close

The correct answer is A. Nephritogenic strains of Group A beta-hemolytic streptococci (e.g., type 12, associated with pharyngitis and type 49, associated with impetigo) are most often implicated in postinfectious glomerulonephritis. Much less commonly, Group C Streptococcus(choice C), Staphylococcus aureus(choice D), and Staphylococcus epidermidis(choice E) have been implicated. Group B Streptococcus(choice B) has, to date, not yet been implicated.

Question 6 of 8

Which of the following is also commonly present in patients with this child's disease? / A. Arthritis / B. Hypertension / C. Hypothyroidism / D. Middle ear infection / E. Psoriasis Explanation - Q: 3.6

Close

The correct answer is B. Hypertension is reported in 50-90% of children who are hospitalized with acute poststreptococcal glomerulonephritis. The degree of increase in blood pressure is variable, but may reach systolic pressures greater than 200 mm Hg and diastolic pressures greater than 120 mm Hg. Children with severe hypertension may have headache, vomiting, altered mentation, visual disturbances, aphasia, coma, and convulsions. The other answers are distracters.

Question 7 of 8

While renal biopsy is no longer common in this child's condition, it has been historically important in defining the pathophysiology of the disease. Which of the following findings on electron microscopic examination is the most specific for this disease? / A. Crescent formation in Bowman's space / B. Eosinophils in interstitium / C. Fusion of podocyte foot processes / D. KimmelstieI-Wilson nodules / E. SubepitheliaI "humps" Explanation - Q: 3.7

Close

The correct answer is E. Subepithelial electron dense "humps," composed of IgG and C3, are the most characteristic finding in acute postinfectious glomerulonephritis. It is unclear whether these form in situ, or are circulating complexes that show a specific tendency to lodge in this site. Serum C3 levels are often depressed in these patients, secondary to consumption in the humps. Crescent formation in Bowman's space (choice A) is also common in postinfectious glomerulonephritis, but this finding is not specific. It does suggest that severe disease is present. Eosinophils in the interstitium (choice B) can be seen in drug- or allergyrelated tubulointerstitial disease. Fusion of podocyte foot processes (choice C) is classically associated with minimal change disease, although it may also be seen in some other renal diseases. Kimmelstiel-Wilson nodules (choice D) are a feature of diabetic renal disease.

Question 8 of 8

Which of the following is the most likely outcome for this patient? / A. Complete resolution / B. Persistence of hematuria for more than 3 years / C. Progression to complete renal failure in less than 1 year / D. Progression to complete renal failure in more than 5 years / E. Prolonged relapsing and remitting course lasting for a decade or more Explanation - Q: 3.8 The correct answer is A. Poststreptococcal glomerulonephritis has a

Close

favorable prognosis in more than 95% of cases, with most patients experiencing complete resolution. Typically, the edema resolves within 5-10 days. The blood pressure usually drops to normal within several weeks (up to 6 weeks), and serum C3 concentration returns to normal within a few months. Microscopic hematuria and proteinuria may last up to 1 or 2 years (rarely 3 or more years, still with eventual complete resolution). Medications used to treat patients with postinfectious glomerulonephritis include antihypertensives, furosemide, and antibiotics (to eradicate any residual streptococci).

A 77-year-old man in an intensive care unit because of severe gram-negative pneumonia and sepsis develops oliguria, with a urinary output of only 25 cc/hr. A urine sample sent for chemical studies demonstrates a sodium concentration of 15 mEq/L, urine creatinine of 41 mg/dL, and an osmolality of 570 mOsm/kg. A serum sample taken at the same time shows sodium of 142 mEq/L and creatinine of 2.2 mg/dL. Two days previously, the patient's serum creatinine had been within normal limits. Question 1 of 7

The major role of creatine in muscle is which of the following? / A. Bind phosphate to form a storage compound for energy / B. Bind to glucose in the pathway that forms galactose / C. Catalyze formation of water molecules in mitochondria / D. Combine with ammonia in the beginning of the urea cycle / E. Serves as a coenzyme in ketone body synthesis Explanation - Q: 4.1

Close

The correct answer is A. Phosphocreatine, which is a phosphorylated form of creatine, is a major storage form of chemical energy used by muscle. In the resting muscle state, levels of phosphocreatine are high, and then as the muscle contracts, the levels drop as the chemical energy is used. Some of the phosphocreatine will also spontaneous lose inorganic phosphate, producing creatinine. The rate at which this occurs in any individual tends to be relatively constant (although individuals with high muscle mass, e.g. athletes, will have higher production rates than those with low muscle mass). Thus, the day to day changes in serum creatinine in an individual tend to reflect day to day variations in renal function. Choices B, C, D, and E refer respectively to uridine diphosphate, cytochromes a and a3, CO2, and coenzyme A.

Question 2 of 7

What is this patient's fractional excretion of sodium [Fex Na (%)]? / A. 0.12% / B. 0.37% / C. 0.56% / D. 1.23% / E. 3.57% Explanation - Q: 4.2

Close

The correct answer is C. The fractional excretion of sodium has become popular as a means of clinically differentiating prerenal azotemia from other forms of acute renal failure, and in particular acute tubular necrosis. It is calculated with the following formula:

The fractional excretion of sodium is almost always less than 1% in prerenal failure (due to underperfusion), and it is almost always greater than 1.5% in acute tubular necrosis, which can occur in similar clinical settings but should be treated very differently medically.

Question 3 of 7

What is this patient's renal failure index (RFI)? / A. 0.5 mEq/L / B. 0.8 mEq/L / C. 1.5 mEq/L / D. 1.8 mEq/L / E. 3.2 mEq/L Explanation - Q: 4.3

Close

The correct answer is B. Like the fractional excretion of sodium, the renal failure index is a popular index used to distinguish prerenal failure from other forms of acute renal failure. It is very similar to the Fex Na, but assumes that the serum sodium is constant:

With prerenal failure, the RFI is usually less than 1 mEq/L, while in most other established forms of renal failure, it is usually greater than 3 mEq/L.

Question 4 of 7

Which of the following processes would be most likely to produce this patient's rising serum creatinine? / A. Benign prostatic hyperplasia / B. Drug toxicity causing acute tubular necrosis / C. Hypotension / D. Post-infectious glomerulonephritis / E. Stone impacted in ureter Explanation - Q: 4.4

Close

The correct answer is C. Both the fractional excretion of sodium and the renal failure index suggest that this patient has prerenal failure. Prerenal failure is due to inadequate perfusion of the kidney, and can be seen in settings in which the blood pressure drops to the point of impairing blood flow to individual glomeruli. The tubular network in this setting is still usually functional, and actively reabsorbs sodium and water, producing relatively low values of urine sodium and high osmolarity. Acute tubular necrosis (choice B) and glomerulonephritis (choice D) are causes of intrarenal failure; while stone impaction (choice E, usually bilateral unless only one kidney is functional) and severe benign prostatic hyperplasia (choice A) can cause lower urinary tract obstruction with post-renal failure.

Question 5 of 7

This patient's low urine sodium suggests that he is still actively reabsorbing sodium. Which of the following hormones acts directly on the renal tubular system to increase sodium reabsorption? / A. AIdosterone / B. Angiotensin l / C. Angiotensin ll / D. Erythropoietin / E. Renin

Explanation - Q: 4.5

Close

The correct answer is A. This question asks you to think about the specifics of the function of the renin-angiotensin-aldosterone system. In this patient's case, decreased blood pressure in the renal afferent arteriole stimulates the baroreceptors in the juxtaglomerular cells, which secrete the enzyme renin (choice E). Renin acts on circulating angiotensinogen made by the liver to produce the prohormone angiotensin I (choice B). Angiotensin I is then converted by angiotensin converting enzyme (ACE) in the lung to angiotensin II (choice C). The angiotensin II acts on the adrenal zona glomerulosa to trigger the secretion of aldosterone (choice A). It is the aldosterone that acts on the renal tubular system to increase the absorption of Na+ and water. Erythropoietin (choice D) is produced by the kidney and stimulates the growth of bone marrow erythroid elements.

Question 6 of 7

The hormone that acts directly on the renal tubular system to increase sodium reabsorption has which of the following effects? / / / / /

A. Exchanges Na+ for HCO3- in the distal collecting duct B. Exchanges Na+ for HCO3- in the loop of Henle C. Exchanges Na+ for HCO3- in the proximal tubule D. Exchanges Na+ for K+ in the collecting ducts E. Exchanges Na+ for K+ in the loop of Henle

Explanation - Q: 4.6

Close

The correct answer is D. Aldosterone promotes the activity of a Na/KATPase-dependent pump located in the epithelial cells lining the distal tubules and collecting ducts. This pump moves Na+ into the renal extracellular fluid in exchange for K+. The other choices are distracters.

Question 7 of 7

Which of the following bacterial components is the most common exogenous mediator of sepsis in gram-negative bacterial infections? / A. Double stranded RNA

/ / / /

B. Endotoxin C. FIagellin D. Lipoteichoic Acid E. Peptidoglycan Explanation - Q: 4.7

Close

The correct answer is B. Endotoxin, also known as lipopolysaccharide, is located in the outer membrane of almost all gram-negative bacteria. Endotoxin is composed of three parts: lipid A, core sugars, and long repeats of oligosaccharide side chains. The lipid A moiety is buried in the membrane. It is the component of endotoxin that elicits the release of mediators that can cause fever, shock and death. The mortality rate when septic shock occurs is approximately 45%. Most patients who die have multi-organ failure. Up to one quarter of septic shock patients develop acute renal failure. Other major complications include adult respiratory distress syndrome, central nervous system dysfunction, liver failure, and disseminated intravascular coagulation. All of the other answers are bacterial components that have been implicated in mediating septic shock. It is worthwhile to note that peptidoglycan (choice E) is thought to be the main mediator of sepsis in gram-positive bacterial infections.

A 50-year-old obese man complains of being tired all the time. He is always thirsty, and has increased frequency of urination. His blood pressure is 145/92 mm Hg, his pulse is 75/min, and his respirations are 15/min. Laboratory studies show: UrinaI sis: GIucose ++ Ketones - negative Creatinine - 40 mg/dL Urine fIow _3 mL/min BIood: GIucose _ 374 mg/dL Creatinine _1.0 mg/dL Question 1 of 6

Which of the following is the best estimate of glomerular filtration rate in this individuaI? / A. 40 mL/min / B. 60 mL/min / C. 80 mL/min / D. 100 mL/min / E. 120 mL/min Explanation - Q: 5.1

Close

The correct answer is E. Although not a perfect glomerular filtration marker, the clearance of creatinine can be used to give a good estimate of glomerular filtration rate (GFR). Creatinine has the advantage of being produced endogenously, and does not need to be infused like inulin. GFR = 40 mg/dL x 3 mL/min 1.0 mg/dL = 120 mL/min. The patient has a normal GFR, which is not uncommon in type 2 diabetes mellitus. On the other hand, approximately 50% of patients with type 1 diabetes have increased GFR (hyperfiltration) within the first year of the disease.

Question 2 of 6

Subcutaneous fat is obtained from the patient and from a normal individuaI. The tissue is homogenized and solubilized with detergent. Total protein is separated by polyacrylamide gel electrophoresis and then transferred to nitrocellulose for subsequent Western blot. BIots are probed for the insulin receptor (both and subunits), the phosphorylated form of the insulin receptor, insulin receptor substrate-1 (IRS-1), and the phosphorylated form of IRS-1. The following data are obtained:

Which of the following is the most likely primary defect in insulin signaling in the patient? / A. Decreased IRS-1 activity

/ / / /

B. Decreased tyrosine kinase activity of the insulin receptor C. Down regulation of the insulin receptor -subunit D. Down regulation of the insulin receptor -subunit E. Down regulation of IRS-1 Explanation - Q: 5.2

Close

The correct answer is B. The Western blot shows that the phosphorylated form of the insulin receptor is decreased in the patient compared to normal. When insulin binds to the α-subunit of its receptor it causes a conformational change in the β-subunit that activates its tyrosine kinase activity. This leads to autophosphorylation of tyrosine residues in the cytoplasmic domain of the β-subunit. There is actually cross-phosphorylation, where the tyrosine kinase on one β-subunit phosphorylates tyrosine residues on the other β-subunit. A diminished activity of the receptor tyrosine kinase activity could explain the decrease in the phosphorylated form of the receptor. IRS-1 activity is also likely to be decreased (choice A), given its decreased relative phosphorylation in the patient. But because phosphorylation of IRS-1 occurs after the insulin receptor is phosphorylated, its decreased activity would be a secondary rather than primary defect. The relative amount of insulin receptor protein (both α- and β-subunits) is the same in both individuals, making downregulation of the receptor (choices C and D) an unlikely cause of the insulin resistance. Likewise, the Western blot shows that the relative amount of the IRS-1 protein is the same in both individuals, making it unlikely that decreased IRS1 (choice E) is the problem in the patient.

Question 3 of 6

A reduced calorie diet and regular exercise program are prescribed. The patient is also started on glyburide (5 mg per day). Which of the following best describes the effect of glyburide? / A. Decreases glucose absorption by the small intestine / B. Hyperpolarizes the membrane potential of beta cells / C. Increases ATP formation within beta cells / D. Increases GLUT 4 expression in fat and muscle / E. Increases insulin secretion from beta cells Explanation - Q: 5.3

Close

The correct answer is E. Sulfonylureas act primarily by increasing insulin secretion from β cells. The drug apparently binds to or near ATP-sensitive K+ channels in the β cell membrane and closes them. The resultant decrease in

K+ efflux depolarizes the membrane resting potential and causes the opening of voltage-gated Ca++ channels. The resultant influx of Ca++ stimulates the exocytosis of insulin from secretory granules. Postprandial hyperglycemia can be diminished by drugs, like acarbose, that decrease glucose absorption (choice A). Acarbose is a competitive inhibitor of intestinal brush border α-glucosidases. The digestion and absorption of dietary sucrose and other carbohydrates is diminished. Sulfonylureas do not affect carbohydrate absorption. Hyperpolarization of the membrane of the β cell (choice B) would make the voltage-gated Ca++ channels less likely to open, and would decrease insulin secretion. Glucose stimulates insulin secretion by entering β cells and being metabolized. The resultant increase in intracellular ATP closes the same ATP-sensitive K+ channels that are affected by the sulfonylureas. The subsequent opening of voltage-gated Ca++ channels and influx of calcium stimulates insulin secretion. The sulfonylureas do not affect intracellular concentration of ATP (choice C). There is some in vitro evidence that sulfonylureas increase the action of insulin at its target cells. However, there is no evidence that the drug works by increasing the expression of the GLUT 4 transporter (choice D). Clinical studies show that sulfonylureas do not have any therapeutic benefit on longterm glycemic control when added to insulin therapy, suggesting that extra β cell effects of the drug have little clinical significance.

Question 4 of 6

Three years later, the patient is still overweight. Fasting plasma glucose is 300 mg/dL and HbA1c is 12% (normaI, 5-8%). He complains of impotence, that has worsened over the past year. Nocturnal erections are absent. Injection of papaverine (vasodilator) into the corpus cavernosum produces an erection that lasts for an hour. Which of the following is the most likely explanation for the erectile dysfunction? / A. Autonomic neuropathy / B. BIood vessel occlusion / C. Diabetic amyotrophy / D. Peripheral sensory neuropathy / E. Psychological Explanation - Q: 5.4

Close

The correct answer is A. Impotence is a long-term complication of diabetes.

It is due to dysfunction of the autonomic nerves that mediate the erection. An erection is produced when the parasympathetic nerves that innervate the penile arterioles release acetylcholine, vasoactive intestinal peptide (VIP), and nitric oxide (NO). These neurotransmitters produce vasodilation with increased blood flow into the erectile tissues of the penis. As the penis becomes engorged with blood, the penile veins become compressed, so that outflow is blocked. Diabetes can be complicated by autonomic neuropathy. The exact cause of the neuropathy is unknown, but may include direct toxic effects of high extracellular glucose, accumulation of sorbitol within the nerves, or poor blood supply to the nerves because of decreased NO release by blood vessel endothelial cells. Blood vessel occlusion (choice B) is a potential cause of erectile dysfunction. It is sometimes responsible for the impotence in diabetes because of vasculopathy. However, the fact that papaverine (vasodilator) injection into the penis evoked a normal erection suggests that blood supply to the penis is not compromised. Peripheral motor neuropathy is also part of the long-term complications of diabetes. Sometimes the patient with motor neuropathy experiences acute pain and weakness in the thigh muscles bilaterally, accompanied by muscle wasting. This has been termed diabetic amyotrophy (choice C). Peripheral motor neuropathy would not affect erectile function. Peripheral sensory neuropathy (choice D) is also a complication of diabetes. It is characterized by years of paresthesias and pain that progress to numbness and decreased tactile sensation. It is usually bilateral and symmetrical. The causes of the sensory neuropathy are probably the same as that for motor and autonomic neuropathy. A psychological (choice E) cause of the impotence is unlikely, given the fact that nocturnal erections do not occur.

Question 5 of 6

After another three years, Iater the patient is still struggling with poor glycemic controI. Urine output is decreasing, plasma creatinine is 3.0

mg/dL (normaI 0.7 -1.2 mg/dL), and proteinuria is greater than 4 g/day. A biopsy of renal tissue is likely to reveal which of the following abnormalities? / A. Acute tubular necrosis / B. Immune complex glomerulonephritis / C. Membranous nephropathy / D. Minimal change disease / E. Nodular glomerulosclerosis Explanation - Q: 5.5

Close

The correct answer is E. The patient is undoubtedly progressing toward diabetic renal failure. Urine output is falling. In diabetic renal failure, unlike other renal diseases, large amounts of protein continue to be excreted, even in the face of decreased GFR and urine flow. Glomerular disease is characterized by thickened capillary walls that can obliterate the vessels. There is also deposition of basement membrane matrix in the mesangium. In 50% of the cases, the glomerulosclerosis is nodular (Kimmelstiel-Wilson nodules). Acute tubular necrosis (choice A) is an intrarenal cause of acute renal failure. It is characterized by proximal tubular or thick ascending limb of the loop of Henle necrosis due to either renal ischemia (e.g., septic shock or hypotension during surgery) or toxic agents (e.g., aminoglycoside antibiotics). Given the chronic nature of the patient's renal disease, acute tubular necrosis is unlikely. Immune complex glomerulonephritis (choice B) is unlikely. The glomerulus is highly susceptible to the entrapment of immune complexes. For example, immune complexes related to systemic lupus erythematosus can form subepithelial deposits outside glomerular capillaries. The presence of the immune complexes can be visualized at the light microscope level using fluorescein-tagged antibodies against human IgG. The resulting injury causes retraction of epithelial cell foot processes. The result is the clinical picture of nephrotic syndrome. There is no inflammatory response leading to an active urine sediment (with casts or red cells), because the immune complexes are sequestered from blood-borne inflammatory cells. This sort of subepithelial cell damage is also called membranous nephropathy (choice C). Minimal change disease (choice D) is a common cause of nephrotic syndrome in children, but is less common in adults. Electron micrographs show diffuse foot process fusion. The cause is unknown, but likely involves the immune system. Glucocorticoids usually are effective in reversing the disorder.

Question 6 of 6

Several weeks later, the patient is brought to the emergency department by EMS. His wife was unable to wake him from an afternoon nap. He is comatose. One week prior to the present episode, he came down with a case of the flu. He has been extremely thirsty since. BIood is drawn and sent to the laboratory. Which of the following findings is most likely to be present in this patient? / A. Bicarbonate - 10 mEq/L (normaI, 22-28 mEq/L) / B. pH - 7.1 (normaI 7.35-7.45) / C. PIasma glucose - 45 mg/dL (normaI, 70-110 mg/dL) / D. Serum ketones - 45 mg/dL (normaI, < 3 mg/dL) / E. Serum osmolarity - 340 mosm/L (normaI, 280-295 mosm/L) Explanation - Q: 5.6

Close

The correct answer is E. The patient most likely is suffering from hyperosmolar hyperglycemic coma. Lethargy and confusion develop as serum osmolarity exceeds 300 mosm/L, and coma can occur if osmolarity exceeds 330 mosm/L. The most common precipitating factor is infection. Although serum sodium values are not given, the blood glucose concentration can be estimated using the following formula: Effective serum osmolarity = 2 x ([Na+] in mEq/L) + ([glucose] in mg/dL)/18. Assuming a normal sodium concentration of 140 mEq/L, blood glucose would be 1080 mg/dL (340 mosm/L = 2 X 140 + [glucose]/18). Serum sodium is likely to be on the low side, however, because of the extra water caused by the osmotic effect of the glucose. With a low serum sodium, the glucose would be even higher. Diabetic ketoacidosis (choices A, B, and D) is rare in patients with type 2 diabetes. The insulin deficiency must be severe to produce ketoacidosis, and type 2 diabetics have enough residual insulin effect to prevent excess ketone body production. Blood pH is usually > 7.3 in type 2 diabetics with hyperosmolar hyperglycemic coma. Because H+ production is not increased, serum bicarbonate remains within normal limits. Low serum bicarbonate would be expected if metabolic acidosis were present. Hypoglycemia (choice C) can cause coma, but blood levels need to be below 30 mg/dL. At 45 mg/dL, there is impaired cognition, confusion, blurred vision, weakness, and lethargy. In this patient, with uncontrolled type 2 diabetes, hypoglycemia is not likely.

A 32-year-old woman presents to her family doctor complaining of 3 days of lower abdominal pain and dysuria. She also complains of urinary urgency and frequency. She is married and in a monogamous relationship with her husband. She has no other significant past medical history. Her temperature is 37.2 C (99.0 F), blood pressure is 122/64 mm Hg, pulse is 88/min, and respirations are 16/min. Her cardiac and lung examinations are unremarkable. Abdominal examination is unremarkable except for mild suprapubic tenderness to palpation. She has no costovertebral angle tenderness. Her pelvic examination is also normaI. Question 1 of 5

Which of the following is the most likely diagnosis? / A. Appendicitis / B. Cystitis / C. Gastroenteritis / D. Pelvic inflammatory disease / E. Urolithiasis Explanation - Q: 6.1

Close

The correct answer is B. Suprapubic tenderness as well as dysuria, frequency, and urgency are classic signs and symptoms of cystitis, which is an infection of the urinary bladder. It is most often due to enteric organisms. Women are more prone to cystitis, due to the shorter urethra. Appendicitis (choice A) is due to bacterial proliferation and invasion of the appendiceal mucosa. It is thought to be caused by the obstruction of the appendiceal lumen, most often by a fecalith. Patients present with severe abdominal pain that may initially be periumbilical, then localized to the right lower quadrant. There are often peritoneal signs, such as involuntary guarding and rebound on physical examination. Gastroenteritis (choice C) is an inflammation of the stomach and intestinal lining due to an infectious organism, which can be viral or bacterial. It is associated with abdominal pain, and either vomiting or diarrhea. Pelvic inflammatory disease (choice D) generally presents with severe lower abdominal pain and fever, and is associated with a purulent cervical discharge and cervical motion tenderness. Urolithiasis (choice E) is due to a stone in the ureter. It often causes severe pain due to acute distension of the ureter. Patients often present with severe flank pain radiating to the groin. It is generally not associated with dysuria, unless there is a concomitant urinary tract infection.

Question 2 of 5

A urine culture reveals multiple colonies of gram-negative rods that ferment lactose. Which of the following is the most likely pathogen?

Explanation - Q: 6.2

Close

The correct answer is A. Escherichia coli is a common enteric organism that causes urinary tract infections. It ferments lactose. Other enteric organisms that ferment lactose include Enterobacter and Klebsiella. Proteus mirabilis(choice C) and Pseudomonas aeruginosa(choice D) are also enteric organisms that can cause urinary tract infections, but they do not ferment lactose. Neisseria gonorrhoeae(choice B) is a gram-negative coccus. It is associated with pelvic inflammatory disease and sexually transmitted diseases. Staphylococcus aureus(choice E) is a gram-positive coccus that is involved in many different types of infection, including scalded skin syndrome, toxic shock syndrome, abscess formation, endocarditis, and food poisoning.

Question 3 of 5

The patient is prescribed trimethoprim-sulfamethoxazole. She returns one week later complaining of severe right-sided back pain and fever in addition to her previous complaints. She did not fill her prescription for the antibiotics because she read that the medication can cause a skin reaction. To which of the following reactions is she most likely referring? / A. Eczematous dermatitis / B. Mottling of tooth enamel / C. Psoriasis / D. Stevens-Johnson syndrome / E. Verruca vulgaris Close Explanation - Q: 6.3 The correct answer is D. Stevens-Johnson syndrome is a generalized hypersensitivity reaction usually due to a drug, in this case,

sulfamethoxazole. It initially presents with skin and mucous membrane manifestations but can involve multiple organ systems. Patients often have a sudden onset of fever as well as a progressive pleomorphic rash which can include petechiae, vesicles, and bullae. Sulfonamides, penicillins, and anticonvulsants are common culprits. Trimethoprim is not generally associated with Stevens-Johnson syndrome. Eczematous dermatitis (choice A) is a pruritic inflammatory disorder. It is associated with atopic individuals as well as infection and chemical irritant contact. Mottling of tooth enamel (choice B) is a side-effect of tetracyclines, when used in children under 8 years of age or in pregnant women (effects on the fetus). Psoriasis (choice C) is a chronic inflammatory skin process characterized by erythematous papules and plaques. They have a characteristic silver scale, and the lesions are sharply demarcated. Verruca vulgaris (choice E) is also known as the common wart. It is a benign papilloma caused by certain strains of human papilloma virus.

Question 4 of 5

On examination, the patient now has costovertebral angle tenderness and has a temperature of 38.9 C (102 F). A urinalysis reveals white cell casts. Which of the following is the most likely diagnosis? / A. Diffuse cortical necrosis / B. GIomerulonephritis / C. Pelvic inflammatory disease / D. Pyelonephritis / E. Urolithiasis Explanation - Q: 6.4

Close

The correct answer is D. The patient previously had an uncomplicated cystitis. Since the infection was not treated, it migrated up her urinary system and caused an ascending pyelonephritis. The presence of white cell casts in the urine is highly suggestive of acute pyelonephritis. A descending pyelonephritis is caused by bacterial dissemination to the kidney by a hematogenous route, such as in the case of endocarditis. Diffuse cortical necrosis (choice A) is an acute generalized ischemic infarction of both kidney cortices. It is most often associated with obstetric

catastrophes, septic shock, and other causes of vascular collapse. Glomerulonephritis (choice B) would produce hematuria and red cell casts, oliguria, azotemia, and some degree of hypertension. Pelvic inflammatory disease (choice D), as noted above, generally presents with severe lower abdominal pain and fever, and is associated with a purulent cervical discharge and cervical motion tenderness. It is not associated with urinary symptoms. Urolithiasis (choice E), as noted above, often causes severe pain due to acute distension of the ureter. Patients often present with severe flank pain radiating to the groin. It is generally not associated with dysuria, fever, or costovertebral angle tenderness unless there is a concomitant urinary tract infection affecting the kidney.

Question 5 of 5

The patient is treated with a fluoroquinolone. Which of the following medications was used? / A. Cefazolin / B. Ceftriaxone / C. Levofloxacin / D. Nafcillin / E. Oxacillin Explanation - Q: 6.5

Close

The correct answer is C. Levofloxacin is a fluoroquinolone, which is a bactericidal agent that inhibits DNA gyrase and topoisomerase IV. Examples of other fluoroquinolones include ciprofloxacin and gatifloxacin. Cefazolin (choice A) and ceftriaxone (choice B) are cephalosporins. They are beta-lactam antibiotics that inhibit cell wall synthesis. They are less susceptible to penicillinases. Cefazolin is a first generation cephalosporin, and ceftriaxone is a third generation cephalosporin. Nafcillin (choice D) and oxacillin (choice E) are both penicillinase-resistant penicillins. They are bactericidal agents that bind penicillin-binding proteins and block transpeptidase cross-linking of the cell wall. They also activate autolytic enzymes within the bacteria. ☺

Vignette 1 of 6

A 26-year-old woman complains to her physician of discomfort during intercourse. Pelvic examination demonstrates a frothy, yellow-green vaginal discharge with a strong odor. SmalI, red, ulcerations of the vaginal wall are also seen. A wet mount preparation demonstrates motile, flagellated protozoa. Which of the following is the most likely causative organism?

Explanation - Q: 1.1

Close

The correct answer is E. Trichomonas vaginalis, the causative organism of trichomoniasis, is a single-celled protozoan parasite with a predilection for infecting the vagina. The organism can be identified with intra-office wet mount preparations, cultured, or recognized on Pap smears (although this should be confirmed by culture, as the Pap smear is not a particularly reliable method). Balantidium coli(choice A) is a ciliated protozoan that can affect the gut. Candida albicans(choice B) is a spore- and hyphae-forming fungus that can affect the vagina, often producing a discharge with a white, "cheesy" appearance. Entamoeba histolytica(choice C) is an amoeba that can infect the gut. Giardia lamblia(choice D) is a flagellated parasite that can infect the gut.

Most cases of infection with this organism are acquired by which of the following routes? / / / / /

A. Contaminated blood products B. Contaminated fomites C. FecaI-oral route D. Sexual transmission E. Spread from skin colonization

Explanation - Q: 1.2

Close

The correct answer is D. Trichomoniasis is commonly spread by sexual transmission, with both heterosexual spread and female homosexual spread being effective routes of transmission. Infected males are commonly asymptomatic; infected females may either be asymptomatic, or have the symptoms noted in the question stem. The other methods of spread noted in the choices do not usually occur.

Which of the following medications is most often used to treat this woman's condition? / / / / /

A. Ampicillin B. Cephalosporins C. Erythromycin D. Metronidazole E. Tetracycline Explanation - Q: 1.3

Close

The correct answer is D. Metronidazole is the only agent known to be effective against trichomoniasis. Often, symptomatic women (and their asymptomatic partners as well) can be treated with a single oral dose, but highly resistant strains are also emerging that require treatment with high dose IV or topical metronidazole. All the agents in the other choices are ineffective. If this woman had been pregnant and had not been treated, she and/or her baby would be at most significantly increased risk of which of the following? / / / / /

A. Congenital cardiac malformation B. Congenital nervous system malformation C. Hydronephrosis D. Preeclampsia E. Premature rupture of membranes Explanation - Q: 1.4 The correct answer is E. Pregnant women with trichomoniasis can be safely treated with metronidazole, and this should be done to prevent the

Close

increased risk of premature rupture of membranes and preterm delivery. Other complications of trichomoniasis not limited to pregnant women are uncommon, but may include cuff cellulitis following hysterectomy and emphysematous blebs in the vaginal wall. Trichomoniasis does not predispose for congenital malformations (choices A and B) or the maternal complications of pregnancy of hydronephrosis (choice C) and preeclampsia (choice D). A woman diagnosed with this disease should also be evaluated for which of the following? / / / / /

A. Functional bowel dysfunction B. Hypertension C. Other venereal disease D. Ovarian malfunction E. Pyelonephritis Explanation - Q: 1.5

Close

The correct answer is C. The same sexual encounters that spread trichomoniasis can also spread a wide variety of other sexually transmitted disease, including herpes, human papilloma virus, syphilis, gonorrhea, chlamydia, and AIDS. Additionally, the lesions produced by the trichomoniasis particularly facilitate the spread of the AIDS virus, since the normal mucosal barrier is breached. The woman's chance of transmitting AIDS to her partner is also increased by the presence of the trichomoniasis open sores. The other choices list distracters unrelated to trichomoniasis.

Vignette 2 of 6

A 32-year-old woman comes to the physician because of a vaginal discharge and itching and discomfort in her genital area. She states that the symptoms started about 3 days ago. Since that time, she has noted a progressive worsening. She also complains of dyspareunia and dysuria. She has no significant past medical history. Her past surgical history is significant for an appendectomy at the age of 17. She takes no medications and is allergic to penicillin. Pelvic examination demonstrates marked erythema and mild edema of the vulva with a few excoriations of the vulva. A vaginal discharge is seen, which is white, thick, and clumpy with a cottage cheese appearance. The vaginal pH is 4.5. A sample of the vaginal discharge is placed on a slide and treated with 10% potassium hydroxide. Microscopy reveals lysis of normal cellular elements with branching pseudohyphae and buds.

Which of the following is the most likely diagnosis? / / / / /

A. Bacterial vaginosis B. Candidiasis C. Genital herpes D. Lichen sclerosis E. Trichomoniasis Explanation - Q: 2.1

Close

The correct answer is B. This patient's presentation is most consistent with the diagnosis of candidiasis. Candidiasis is most commonly caused by the organism Candida albicans; although up to nine different fungi have been shown to cause mycotic vulvitis. Itching and irritation are the most common presenting symptoms of patients with candidiasis. The irritation can be worsened by urination or with intercourse, with resulting dysuria and dyspareunia. Patients with candidiasis also frequently complain of a vaginal discharge that is often described as "cottage cheesy" in its appearance. Examination typically reveals erythema of the vulva or vagina, often with some edema. Excoriations from the patient's scratching are also often seen. The discharge is usually thick, white, and clumpy, although occasionally it can be thin and watery. The clinical diagnosis can be confirmed by performing 10% potassium hydroxide (KOH) microscopy, which demonstrates the branching pseudohyphae and buds of the Candida. The KOH causes lysis of normal cellular elements, which allows the buds and pseudohyphae to be more easily visualized. Bacterial vaginosis (choice A) is characterized by a malodorous vaginal discharge. Examination shows a discharge coating the vaginal walls, clue cells on microscopy, and a vaginal pH greater than 4.5. Genital herpes (choice C) is characterized by painful lesions that begin as fluid-filled papules or vesicles and progress to coalescent, shallow-based ulcers. An initial herpes infection is associated with systemic symptoms such as fever, myalgias, and malaise. Recurrent infections usually occur without systemic symptoms. Lichen sclerosis (choice D) is a vulvar dystrophy in which the vulva develops an atrophic appearance that is often described as cigarette-paper or parchment-like. It usually occurs in postmenopausal women, though it can sometimes be seen in the pediatric age group. Trichomoniasis (choice E) is characterized by a malodorous vaginal discharge that is usually described as being greenish or yellow-green.

Microscopic evaluation demonstrates the motile organism, Trichomonas vaginalis.

Which of the following is the most likely pathogen?

Explanation - Q: 2.2

Close

The correct answer is A. Candida albicans belongs to the family of opportunistic mycoses and it is a common cause of vulvovaginitis in women. It is a normal inhabitant of the GI tract, the upper respiratory tract, and the female genital tract. However, when conditions are "right," candidal overgrowth can result. Patients that are particularly predisposed to candidal infections are those who are immunosuppressed and diabetics. Other affected groups include patients on antibiotics, and certain women who are prone to the disorder for unclear reasons. Along with vulvovaginitis, Candida albicans is also associated with thrush, intertrigo, and esophagitis. Occasionally, the organism can cause pneumonia, meningitis, and enteritis, and it is also associated with endocarditis in IV drug abusers. Gardnerella vaginalis(choice B) is a facultatively anaerobic, oxidase- and catalase-negative, gram-variable rod. It is a normal inhabitant of the human vagina and can be found in up to 70% of asymptomatic women. Overgrowth of the organism leads to the condition known as bacterial vaginosis. Herpes simplex virus 2 (choice C) is a double-stranded DNA virus that is a member of the herpesviruses (which also includes HSV-1, varicella-zoster virus, cytomegalovirus, Epstein-Barr virus, Human Herpesvirus 6 and 8.) It most commonly causes genital infection in women. This patient's symptoms and clinical exam findings are not consistent with genital herpes. The postmenopausal estrogen decrease (choice D) that occurs in women can cause a condition known as atrophic vaginitis. This condition is characterized by dryness and irritation of the vagina and is a result of the decline in estrogen that occurs after the menopause. This patient is premenopausal and has candidiasis. Trichomonas vaginalis(choice E) is a parasite that causes trichomoniasis in

women. A wet mount of vaginal discharge from a patient with this infection will show motile, flagellated trophozoites.

This patient is started on miconazole. This medication works via which of the following mechanisms? / / / / /

A. Binding to bacterial ribosomes B. Inhibition of DNA synthesis after conversion of the drug C. Inhibition of mycolic acid transfer to the cell wall D. Inhibition of the synthesis of ergosterol E. Inhibition of the synthesis of mycolic acids

Close Explanation - Q: 2.3 The correct answer is D. Miconazole is an antifungal agent that inhibits ergosterol synthesis, thus disrupting cell wall synthesis. It is used as a topical agent for infections with Candida and infections with mixed fungi. Miconazole can also be used intravenously to treat systemic fungal infections. When miconazole is used topically, the main toxicity is localized irritation and burning. When used intravenously, miconazole can cause thrombophlebitis and, rarely, cardiorespiratory arrest.

Numerous antibiotics function by binding to bacterial ribosomes (choice A). For example, the aminoglycosides (e.g., gentamicin), tetracyclines (e.g., doxycycline), and macrolide antibiotics (e.g., erythromycin) all bind to bacterial ribosomes. Miconazole does not function via this mechanism. Acyclovir brings about inhibition of DNA synthesis after conversion of the drug (choice B). Viral thymidine kinase converts acyclovir to a nucleoside analog that then inhibits DNA synthesis. Ethambutol is an antitubercular agent that functions through inhibition of mycolic acid transfer to the cell wall (choice C). Isoniazid (INH) blocks the formation of the mycobacterial cell wall through inhibition of the synthesis of mycolic acids (choice E). It is used in the treatment of tuberculosis, though resistance of the organism Mycobacterium tuberculosis to INH is increasing. The KOH wet preparation is falsely negative in 25% of cases of vulvovaginal candidiasis. Which of the following represents the sensitivity of the KOH wet preparation for identifying candidiasis? / A. 10% / B. 25%

/ C. 50% / D. 75% / E. 99%

Explanation - Q: 2.4

Close

The correct answer is D. The sensitivity of a test represents the percentage of individuals with a disease who test positive with the diagnostic test. If the potassium hydroxide (KOH) wet preparation is falsely negative in 25% of cases, this means that 25 out of every 100 patients who have candidiasis will have a negative wet preparation. This means that 75 of every 100 patients with candidiasis will test positive by the wet preparation. And this 75 per 100 or 75% represents the sensitivity of the test. It is important to realize that the sensitivity and specificity of a test are qualities of the test and are not changed by the nature of the population being tested. Positive and negative predictive values, on the other hand, depend on the prevalence of the disease in the population being tested. Vignette 3 of 6

A 28-year-old woman comes to the physician because of an increased vaginal discharge that has an unpleasant odor. She states that the symptoms started 2 days ago and have been worsening since. Other than the malodorous discharge, she has no other complaints. Past medical history is significant for occasional migraine headaches. Past surgical history is significant for tonsillectomy as a child. She takes no medications regularly and has no known drug allergies. Speculum examination demonstrates a homogeneous, grayish-white discharge, partially adherent to the vaginal walls. There is no edema or erythema of the cervix, vagina, or vulva. A drop of the vaginal discharge is tested for pH, which is 4.9. When potassium hydroxide (KOH) is added to the discharge, there is an intense amine odor. A normal saline wet preparation of the discharge reveals a predominance of clue cells. Which of the following is the most likely diagnosis? / / / / /

A. Bacterial vaginosis B. Candidiasis C. Chlamydia D. Gonorrhea E. Trichomoniasis Explanation - Q: 3.1

Close

The correct answer is A. Bacterial vaginosis has been referred to by many

other designations over the years including Gardnerella-associated vaginitis and nonspecific vaginitis. It is the number one cause of vaginitis in premenopausal women. Patients with this condition most commonly complain of an increased vaginal discharge that is malodorous. The odor will often be described as musty or fishy. However, up to 50% of patients with bacterial vaginosis will be asymptomatic. Examination of a patient with bacterial vaginosis will demonstrate a homogeneous vaginal discharge that is white or grayish-white. The vaginal pH will be greater than 4.5, which is more basic than the usual vaginal pH. Clue cells, which are epithelial cells covered by Gardnerella vaginalis, will be predominant on the normal saline wet preparation. Finally, the discharge will have a fishy odor, either before or after the application of 10% KOH to the sample. This is known as a positive "whiff test." Treatment is with metronidazole or clindamycin. Metronidazole should be avoided in the first trimester of pregnancy. Patients with candidiasis (choice B) most commonly complain of a thick, "cottage-cheese" vaginal discharge. They also note itching and redness in the vulvovaginal area and may complain of dyspareunia and dysuria. Examination demonstrates the thick, white discharge, with erythema of the vulva and vagina, and pseudohyphae on the KOH microscopic preparation. Patients with Chlamydia(choice C) are often asymptomatic or have mild symptoms that go unrecognized. A cervicitis is sometimes seen on physical examination, and diagnosis is made most often with the use of DNA probe systems. Patients with gonorrhea (choice D) also are often asymptomatic or have only mild symptoms. Endocervical gonorrhea is associated with a purulent vaginal discharge. Diagnosis is usually made by culture of the organism or with DNA probe systems. Trichomoniasis (choice E) is diagnosed when the motile organism is visualized on the normal saline wet preparation. This patient has a predominance of clue cells on the wet preparation, which is consistent with bacterial vaginosis and not trichomoniasis.

Which of the following is the most likely pathogen?

Explanation - Q: 3.2

Close

The correct answer is C. Gardnerella vaginalis is a facultatively anaerobic, oxidase- and catalase-negative, gram-variable rod; that is, it has a cell wall or a laminated cell wall typical of neither gram-positive nor gram-negative bacteria. However, there is much uncertainty surrounding the exact pathophysiology that underlies the development of bacterial vaginosis in women. Currently, it is thought that the infection results from the synergism between a number of bacteria including Gardnerella vaginalis, Mycoplasma hominis, Ureaplasma urealyticum, Peptostreptococcus species, and anaerobic gram-negative rods. When a critical concentration of certain vaginal bacteria is present, particularly the anaerobes, Gardnerella vaginalis is able to "overgrow" in the vagina, replace the native Lactobacillus species, and cause symptoms. On the normal saline wet preparation, Gardnerella vaginalis can be seen adherent to the epithelial cells. Candida albicans(choice A) is a fungus that is the most common cause of candidiasis. It normally colonizes the human gastrointestinal tract and vagina, but overgrowth can lead to significant symptoms and severe infection. Along with causing candidiasis, it can also cause endocarditis, pneumonitis, and meningitis. Chlamydia trachomatis(choice B) is an obligate intracellular organism that is most commonly associated with cervicitis in women. It is also contributory in many cases of pelvic inflammatory disease. It can also cause pharyngitis, pneumonitis, and conjunctivitis. Neisseria gonorrhoeae(choice D) is also known as the gonococcus. It appears on Gram's staining as gram-negative diplococci. It can cause cervicitis, urethritis, meningitis, pharyngitis, arthritis, endocarditis, and ophthalmia. It is not implicated in bacterial vaginosis. Trichomonas vaginalis(choice E) is a protozoan that moves via flagellae. Diagnosis is made when motile trophozoites are seen on microscopic examination of a vaginal discharge. The treatment of choice is metronidazole.

The patient is started on metronidazole. This medication works via which of the following mechanisms? / A. Binds 50s subunit of bacterial ribosome and inhibits translocation of the

peptide chain from the A site to the P site

/ B. BIocks cross-Iinking of the N-acetyl muramic acid/N-acetyl glucosamine

backbone by binding to D-ala-D-ala / C. Inhibits DNA-dependent RNA polymerase by binding to the subunit and inhibiting initiation / D. Interferes with DNA gyrase activity and prevents the winding of DNA helix into the supercoiled form / E. Is reduced to a substance that inhibits cellular DNA synthesis Explanation - Q: 3.3

Close

The correct answer is E. Metronidazole is reduced to a substance that inhibits cellular DNA synthesis. Metronidazole is commonly known as Flagyl and is used to treat infections with anaerobic bacteria other than Actinomyces. It is also used to treat anaerobic protozoa including Giardia lamblia, Trichomonas vaginalis, and Entamoeba histolytica. It can be administered both intravenously and orally, and has excellent bioavailability when given orally. Metronidazole can increase the anticoagulant effect of drugs in the coumarin class, and can cause a disulfiram-like reaction in patients ingesting alcohol. Patients started on metronidazole should be cautioned not to drink. Another common side effect is a metallic taste in the mouth. Peripheral neuropathy, seizures, and ataxia have been seen with prolonged use. Binding to the 50s subunit of bacterial ribosome and inhibiting translocation of peptide chain from A site to P site (choice A) describes the action of chloramphenicol. Blocking the cross-linking of the N-acetyl muramic acid-N-acetyl glucosamine backbone by binding to D-ala-D-ala (choice B) describes the mechanism of action of vancomycin. Inhibiting the DNA-dependent RNA polymerase by binding to the β subunit and inhibiting initiation (choice C) describes the mechanism of action of the rifamycins, such as rifampin. Interfering with DNA gyrase activity and preventing the winding of the DNA helix into the supercoiled form (choice D) describes the mechanism of action of the quinolones.

Which of the following is the most commonly reported adverse response to metronidazole? / A. Discoloration of teeth in children / B. Gastrointestinal distress

/ C. Hemolytic anemia in patients with G6PD deficiency / D. Pseudomembranous colitis / E. "Red man" syndrome

Explanation - Q: 3.4

Close

The correct answer is B. Metronidazole is associated with a large number of side effects and adverse reactions. The most commonly reported side effect is gastrointestinal distress. Other adverse reactions that can occur include convulsions, insomnia, peripheral neuropathy, and thrombophlebitis. Perhaps the most "interesting" side effect (or most frequently discussed) is the disulfiram-like reaction that alcohol consumption can lead to in patients taking the drug. Metronidazole has also been shown, in therapeutic doses, to be carcinogenic in mice and rats. Discoloration of teeth in children (choice A) is an adverse effect that is associated with tetracycline use. This drug should, therefore, not be used in pregnant women or children less than age 8. Hemolytic anemia in patients with G6PD deficiency (choice C) is an adverse effect associated with primaquine. Primaquine is a drug that is used to treat malaria. Pseudomembranous colitis (choice D) has been reported in association with amoxicillin and cephalosporins, but it is most often associated with clindamycin use. In the setting of these antibiotics, Clostridium difficile overgrows and causes the colitis. "Red man" syndrome (choice E) is associated with vancomycin and occurs when the medication is given too rapidly intravenously. Vancomycin can also cause ototoxicity. To study the efficacy of metronidazole for the treatment of this disease, studies are conducted in which the subjects are divided into 2 groups. One group receives metronidazole and the other group receives a placebo. Two weeks later the subjects are tested for bacterial vaginosis. Which of the following terms best describes these studies? / / / / /

A. Case-control studies B. Case series C. Cohort studies D. Cross-sectional studies E. Randomized controlled trials

Explanation - Q: 3.5

Close

The correct answer is E. A randomized controlled trial is an experimental clinical trial in which a group is randomly divided into two separate groups. One group is given the experimental treatment (in this case metronidazole) and the other group (the control group) is given a placebo or "standard" medication. These trials can be single-blinded so that the subjects do not know which medication they are receiving, or double-blinded so that neither the patients, nor the researchers know. These studies are effective at reducing bias, but they are often difficult to coordinate. Case-control studies (choice A) are performed by looking at a group of patients with a disease (the cases) and a group without the disease (the controls) and comparing exposures that the two groups had. These studies are usually easier to perform than experimental studies but can be confounded by numerous biases. Case series (choice B) are reports on a set of patients. For example, a case series involving metronidazole and bacterial vaginosis might report on 10 consecutive patients with HIV and bacterial vaginosis treated with metronidazole and describe the outcomes. Cohort studies (choice C) are prospective observational studies that follow a population over time to look at outcomes and exposures. Cohort studies allow a relative risk to be calculated among different populations. Cross-sectional studies (choice D) look at a group of subjects at one point in time. They are often easier to perform than prospective studies, but often provide less information. Vignette 4 of 6

A 55-year-old woman complains to a physician because she has been having chronic pelvic pressure. On further questioning, she reports chronic lower back pain, constipation, difficulty with walking, and impaired coitus. Pelvic examination demonstrates that the uterine cervix lies low within the vaginal canaI, but does not protrude through the introitus. Which of the following is the most likely diagnosis? / / / / /

A. Cystocele B. Femoral hernia C. Rectocele D. Sarcoma botyroides E. Uterine prolapse

Explanation - Q: 4.1

Close

The correct answer E. Uterine prolapse is the term used for when the uterus descends from its normal position to one lower in the body. The prolapse is graded based on the level of descent. This woman has first degree prolapse, with descent of the cervix into the vagina. Second degree prolapse is characterized by protrusion of the cervix through the introitus. Third degree prolapse is characterized by complete eversion of the vagina. Uterine prolapse, rectocele, and cystocele are usually due to problems with the strength of the muscles forming the pelvic floor, and the lesions are classified as pelvic relaxation disorders. A history of trauma (notably related to multiple pregnancies) and hormonal changes (e.g., menopause) is frequently elicited. Cystocele (choice A) refers to the descent of the bladder to a lower than normal position. It is usually due to problems with the strength of the muscles forming the pelvic floor, and the lesions are classified as pelvic relaxation disorders. A history of trauma (notably related to multiple pregnancies) and hormonal changes (e.g., menopause) is frequently elicited. A femoral hernia (choice B) would present with a mass in the upper part of the thigh, adjacent to the groin. Rectocele (choice C) refers to descent of the rectum to a lower than normal position. Sarcoma botyroides (choice D) is a form of embryonal rhabdomyosarcoma that can present with polypoid masses protruding from the vagina, usually in children under 5 years of age. Uterine prolapse, rectocele, and cystocele are all usually due to which of the following? / / / / /

A. CNS dysfunction B. Infection C. Neoplasia D. Pelvic inflammatory disease E. Pelvic relaxation disorder Explanation - Q: 4.2

Close

The correct answer is E. All three of lesions are usually due to problems with the strength of the muscles forming the pelvic floor, and the lesions are

classified as pelvic relaxation disorders. A history of trauma (notably related to multiple pregnancies) and hormonal changes (e.g., menopause) is frequently elicited. Only rarely is the change in organ position related to other disease (choices A, B, C, and D).

Which of the following is the largest and most important muscle in the pelvic floor? / / / / /

A. Coccygeus B. External anal sphincter C. Levator ani D. Obturator internus E. Piriformis muscles Explanation - Q: 4.3

Close

The correct answer is C. The pelvic floor is formed by the large levator ani (with parts including the pubococcygeus, puborectalis, and iliococcygeus) and the much smaller coccygeus (choice A). The small external anal sphincter (choice B) lies below the levator ani and is not technically part of the pelvic floor. The obturator internus (choice D) is in the lateral wall of the pelvis, and the piriformis muscles (choice E) are in the posterior wall of the pelvis. During childbirth, which of the following muscles is most often injured by a tear of the perineum? / A. Coccygeus / B. IIiococcygeus / C. Pubococcygeus / D. Puborectalis / E. Superficial transverse perineal Close Explanation - Q: 4.4 The correct answer is C. The fetal head is supported during childbirth by the pelvic floor. If injury occurs during the passage of the child through the dilated uterine cervix, this injury most frequently involves a tear of the perineum between the vagina and anus. This tear usually involves the pubococcygeus, which is the main part of the levator ani. The pubococcygeus runs from the pubic bone anteriorly to the coccyx posteriorly, and contains an anterior opening for the urethra and vagina and a posterior opening for the rectum. Tears to this muscle that heal poorly may

consequently predispose for uterine prolapse, cystocele, and rectocele. The coccygeus (choice A) is a relatively small muscle of the pelvic floor that runs from the ischial spine to the inferior end of the sacrum. The iliococcygeus (choice B) is another part of the levator ani, and runs from the iliac bone to the coccyx, forming part of the more lateral aspects of the pelvic floor. The puborectalis (choice D) is the third part of the levator ani, and runs from pubic bone to pubic bone, forming a sling-like structure around the wall of the anal canal. The superficial transverse perineal muscle (choice E) is a usually very small muscle that runs laterally to medially from the ischial tuberosity to the perineal body (between the vagina and anus).

The muscle that is most often injured by a tear of the perineum is innervated by which of the following? / / / / /

A. Inferior gluteal nerve B. Pelvic splanchnic nerve C. Posterior femoral cutaneous nerve D. Pudendal nerve E. Superior gluteal nerve Explanation - Q: 4.5

Close

The correct answer is D. The pudendal nerve (from S2-S4) is the principal nerve to innervate structures of the perineum, including sensory innervation to the genitalia, and motor innervation to muscles of the perineum, the external urethral sphincter, and the external anal sphincter. This innervation may have clinical significance, as babies can also have uterine prolapse, which can be due either to congenital weakness in the pelvic musculature or to defects in innervation. The inferior gluteal nerve (choice A) supplies the gluteus maximus. The pelvic splanchnic nerve (choice B) supplies the pelvic viscera via the inferior hypogastric and pelvic plexuses. The posterior femoral cutaneous nerve (choice C) supplies the skin of the buttock and upper portions of the medial and posterior aspects of the thigh.

The superior gluteal nerve (choice E) supplies the gluteus medius and gluteus minimus muscles.

For minor degrees of this patient's disorder, Kegel exercises are sometimes prescribed. These can be performed by asking the patient to do which of the following? / / / / /

A. CIose the glottis and increase abdominal pressure B. Contract muscles to interrupt a stream of flowing urine C. Press the knees together D. Tense the buttocks E. When in a sitting position, press the knees upward against the hands Explanation - Q: 4.6

Close

The correct answer is B. The Kegel exercises are specifically designed to contract the pubococcygeus muscle, and can be performed by interrupting the flow of urine. Patients with minor degrees of prolapse may also be treated with a pessary, which is a small object placed in the vagina that serves as a mechanical support for the uterus. Estrogen therapy is also of some help in some patients. Severe degrees of prolapse may require either hysterectomy or surgical strengthening of the pelvic floor. Choice A describes the Valsalva maneuver, which these patients should not do, because it increases intra-abdominal pressure and may worsen the prolapse. (They should also avoid lifting heavy objects for the same reason.) The other answers are distracters.

Vignette 5 of 6

A 15-year-old girl is seen by gynecologist and a pelvic examination is performed. The physician begins by doing a manual palpation of the pelvic organs with one hand while fingers of the other hand are in the vagina. During the pelvic examination, a speculum is inserted into the vagina and the cervix is observed. An irregular, roughly circular line is noted to be surrounding the cervical os. This corresponds to the histological junction between which of the following? / A. The pseudo-stratified columnar epithelium of the ectocervix and the

stratified squamous epithelium of the endocervix / B. The simple columnar epithelium of the ectocervix and the simple squamous epithelium of the endocervix

/ C. The simple squamous epithelium of the ectocervix and the pseudo-stratified

epithelium of the endocervix / D. The stratified squamous epithelium of the ectocervix and the simple columnar epithelium of the endocervix / E. The stratified squamous epithelium of the ectocervix and the simple squamous epithelium of the endocervix

Explanation - Q: 5.1

Close

The correct answer is D. The line in question is called the Z-line, and marks the boundary between the stratified squamous epithelium of the ectocervix and the simple columnar epithelium of the endocervix. This is also sometimes called the transition or transformation zone, and is clinically significant because this junctional area is a common site for development of squamous cell carcinoma. The Z line is not fixed for life, but tends to start out in teenage girls clearly visible on the ectocervix, and then migrate upward into the (sometimes deep) endocervix as the girls age. For this reason, women in their 30's and 40's may have high grade dysplasia and even cervical cancer without any lesion being apparent on vaginal examination.

The physician takes a sample for Pap smear analysis. Review of the Pap smear demonstrates the presence of koilocytes, hich are approximately the same size as an intermediate celI, with enlarged hyperchromatic nuclei and perinuclear halos. If no other significantly abnormal cells are present, this Pap smear would be classified as which of the following? / / / / /

A. Carcinoma in situ B. High-grade squamous intraepithelial lesion (CIN II-III) C. Invasive squamous cell carcinoma D. Low-grade squamous intraepithelial lesion (CIN I) E. Reactive change

Explanation - Q: 5.2 The correct answer is D. The cells described are typical for low-grade squamous intra-epithelial lesion (LSIL), which corresponds in the tissue section diagnosis of cervical intra-epithelial lesion grade I (CIN I).

Close

Carcinoma in situ (choice A) and the higher grade range of high-grade squamous intraepithelial lesions (HSIL; corresponding to CIN III) are approximately the same diagnosis. High-grade squamous intraepithelial lesions (HSIL, choice B) have smaller cells with hyperchromatic nuclei, and correspond to the tissue section diagnoses of CIN II and CIN III. Descriptions of invasive squamous cell carcinoma (choice C) should include references to both obviously malignant cells, and to the presence of a "tumor background" with blood, cellular debris, and heavy inflammation. Reactive change (choice E) is characterized by inflammatory cells and epithelial cells with large, vesicular nuclei that are not hyperchromatic.

The abnormal cells observed in the Pap smear specimen are infected by which of the following viruses? / / / / /

A. AIDS virus B. Cytomegalovirus C. Herpes simplex l D. Herpes simplex ll E. Human papilloma virus Explanation - Q: 5.3

Close

The correct answer is E. Various strains of the human papilloma virus (HPV, notably types 16, 18, 31, 33, 35, and 39) have been implicated as causing a variety of genital lesions, including genital warts, cervical dysplasia, cervical cancer, vaginal dysplasia, vaginal cancer, and penile cancer. (Strains of human papilloma virus have also been implicated in the common wart found often found on fingers and in laryngeal warts.) The koilocyte is the term used to describe the characteristic nuclear alterations and perinuclear halo seen with HPV infection in the cervical squamous epithelial cells. Infection with the AIDS virus (choice A) cannot be recognized by cytologic changes. Cytomegalovirus (choice B) infection causes nuclei to be large, eosinophilic, and "smudged"; cytomegalovirus is uncommonly seen in Pap smears. Herpes simplex I and II (choice C and D) infections cannot be distinguished cytologically, as both produce multinucleated cells whose nuclei develop a

characteristic "ground-glass" appearance.

The gynecologist decides to simply monitor the patient; however, she is soon lost to follow-up. The patient does not seek gynecological attention again until the age of 26. At that time, she goes to an emergency department because she has been feeling chronic pelvic "heaviness" for several months, and has had more or less continuous vaginal spotting. Pelvic examination demonstrates a 2.5 cm ulcerated area involving the region of the cervix near the cervical os. Pap smear taken from this area shows small neoplastic cells with enlarged hyperchromatic nuclei (that are occasionally very large and bizarre) and brightly eosinophilic cytoplasm. AIso noted on the smear are the presence of abundant blood, neutrophils, and cellular debris. The cells that are seen are most likely derived from which of the following? / / / / /

A. Endocervical adenocarcinoma B. Endometrial adenocarcinoma C. Leiomyoma D. Leiomyosarcoma E. Squamous cell carcinoma Explanation - Q: 5.4

Close

The correct answer is E. The description of the Pap smear is typical for invasive squamous cell carcinoma. We are unfortunately seeing potentially fatal cervical cancer in young women, who have typically begun having sexual intercourse at a young age. The time between the initial HPV infection and the development of cancer may be as short as 8 years, which means that HPV infection may, in extreme cases, be more rapidly fatal than HIV infection. On a related note, medical educators tend to teach students that "cancer isn't contagious," which is true in general, but in the case of the HPVrelated genital cancers, the underlying infection leading to the cancer certainly is contagious. Adenocarcinomas of either the endocervix or endometrium (choices A and B) shed cells that have clear, rather than orange, cytoplasm and resemble the cells of normal columnar epithelium. Benign leiomyomas (choice C) and malignant leiomyosarcomas (choice D) are tumors that usually form in the muscle of the uterine wall, and do not usually shed cells that can be reached with a Pap smear.

Further studies demonstrate that this young woman's cancer extends beyond the cervix but not to the pelvic wall and involves the upper but not lower portion of the vagina. This tumor has which of the following stages? / / / / /

A. Stage 0 B. Stage l C. Stage ll D. Stage lll E. Stage IV Explanation - Q: 5.5

Close

The correct answer is C. This patient's tumor is in clinical stage II. Stage 0 (choice A) is used for carcinoma in situ. Stage I (choice B) is carcinoma confined to the cervix. Stage II is carcinoma extending beyond the cervix but not to the pelvic wall; and carcinoma involving the vagina but not the lower third of the vagina. Stage III (choice D) is carcinoma extending to the pelvic wall, with cancer-free rectum, and with tumor involving the lower 1/3 of the vagina. Stage IV (choice E) is cancer extending beyond the true pelvis or involving the mucosa of the bladder or rectum. The prognosis of cervical cancer is strongly dependent on the stage, with 90% of women with Stage I cancer having an at least 5 year survival, but only 20% or fewer of women with Stage IV cancer having a 5 year survival. Treatment depends on stage, and may involve surgery, radiation, or both.

Vignette 6 of 6

A 28-year-old woman presents to her family physician complaining that she has a "growth" in her genital area. She states that she first noticed it 3 weeks ago and it seems to have grown somewhat since that time. It does not bother her except for its presence. She has hypothyroidism, for which she takes thyroid hormone replacement. She has no other medical problems and has never had surgery. She is allergic to penicillin. She has had 3 sexual partners over her lifetime and is currently involved in a monogamous relationship. She has never had a sexually transmitted disease in the past. She smokes 1 pack of cigarettes per day. Examination demonstrates two nontender 6 mm welI-circumscribed, fleshcolored, apillated, oval lesions on the labia majora. There is no ulceration, erythema, purulence, or inguinal lymphadenopathy. Which of the following is the most likely diagnosis? / A. Chancre / B. Condyloma

/ C. Herpes / D. Squamous cell carcinoma / E. Syphilitic gummas

Explanation - Q: 6.1

Close

The correct answer is B. This patient, who presents with asymptomatic, well-circumscribed, benign-appearing, flesh-colored lesions in the genital area most likely has condyloma acuminatum, or genital warts. These lesions result from contact with infected partners and represent a sexually transmitted disease. Most of the time these lesions are benign, without carcinogenic potential except if they are associated with the cervix, where they are an important etiologic agent for cervical carcinoma. Chancre (choice A) is characterized as a painless ulcer and is the primary genital lesion of syphilis. This patient did not have an ulceration. Herpes simplex lesions (choice C) are characterized by grouped vesicles on an erythematous base. These lesions are usually symptomatic with a prodrome of burning or irritation. This patient had an asymptomatic lesion without vesicles, ulcerations, erosions, or erythema. Squamous cell carcinoma (choice D) can present in this location, but usually is characterized with erythema, erosion, or ulceration. Syphilitic gummas (choice E) are manifestations of tertiary syphilis. They are nontender, pink to dusky red nodules or plaques that vary in size from millimeters to several centimeters, and favor sites of previous trauma, but may appear anywhere. They are usually firm, but can develop a "gum"-like consistency. This patient had no history of syphilis, and the clinical presentation is not consistent with these destructive lesions.

Histological examination of diagnostic tissue from this patient would most likely reveal which of the following? / / / / /

A. Architectural atypia B. Cytologic atypia C. Dysplastic cells D. Epidermal necrosis E. Koilocytes Explanation - Q: 6.2 The correct answer is E. Koilocytes are large keratinocytes with an

Close

eccentric, pyknotic nucleus surrounded by a perinuclear halo. They are characteristic of human papilloma virus (HPV) infection. The koilocytotic cells are usually abundant in condylomata. Anogenital warts also are characterized by acanthosis and parakeratosis, and they lack a granular layer. Architectural atypia (choice A) is a histologic description suggesting malignancy, which this patient does not have. Cytologic atypia (choice B) is also a histologic description suggesting malignancy, which this patient does not have. Dysplastic cells (choice C) is a term used to describe atypical pigmented cells, such as melanoma and dysplastic nevi. They usually have a large nucleus and/or large size. Epidermal necrosis (choice D) occurs in skin conditions such as toxic epidermal necrolysis (TEN). This patient, with condyloma, would have an intact epidermis without necrosis. Which of the following is the most likely pathogen? / / / / /

A. Epstein-Barr virus B. Herpes simplex virus C. Human papilloma virus D. Treponema pallidum E. Varicella zoster virus Explanation - Q: 6.3

Close

The correct answer is C. Human papilloma virus (HPV) is associated with condyloma acuminatum. HPV is a double-stranded DNA virus found in human and other species. For the most part, these viruses induce subclinical lesions that are very slow-growing and thus, may take many years before obvious lesions become apparent. There are at least 60 different types of HPV. Anogenital warts are mostly associated with types 6, 11, 16, 18, 31, 33-35, 39-40, and 51-60. Common warts and flat warts are associated with different types of HPV. Epstein Barr virus (choice A) is associated with infections mononucleosis and Burkitt's lymphoma, among other diseases. The Monospot test is used to diagnose this virus. Herpes simplex virus (choice B) is associated with oral and anogenital lesions. There are two types (I and II) that can be seen in either location. These lesions are characterized as painful clustered vesicular eruptions. This

patient does not have the characteristic eruption of genital herpes. Treponema pallidum(choice D) is the spirochete that causes syphilis. This patient does not have the characteristic presentation for primary, secondary, or tertiary syphilis. Varicella zoster virus (choice E) is associated with chicken pox and shingles. This patient does not have any signs of the pruritic vesicular rash associated with this virus. This patient is treated with imiquimod. This medication works via which of the following mechanisms? / / / / /

A. As a bacteriostatic agent B. As a chemotherapeutic agent C. As an antiviral agent D. Through physical destruction of infected cells E. Through stimulation of release of proinflammatory cytokines Explanation - Q: 6.4

Close

The correct answer is E. Imiquimod is a potent stimulator of proinflammatory cytokines. It thus functions as an immunostimulant. It is a relatively recently approved drug for the treatment of external warts. It has been demonstrated in prospective, double-blind, placebo-controlled trials to be safe and effective in the treatment of patients with external anogenital warts. It is used as 5% cream, and can be used to 16 weeks depending upon the individual patient's response to treatment. A drug that functions as a bacteriostatic agent (choice A) would not be used in the treatment of condyloma acuminatum, as this disease does not have a bacterial etiology. Drugs that function as chemotherapeutic agents (choice B) are widely used for the treatment of condyloma. However, imiquimod is not a chemotherapeutic agent. The most commonly used chemotherapeutic agents used in the treatment of anogenital warts are podophyllin and bleomycin. Antiviral agents (choice C) such as acyclovir, are not effective in eradicating the human papillomavirus that causes condyloma acuminatum. Several modalities are used to treat condyloma through physical destruction of infected cells (choice D). The most commonly used method of physical destruction is with cryotherapy (liquid nitrogen.)

Vignette 1 of 3

A 13-year-old girl is evaluated by a pediatrician because she has been having fainting and near-fainting spells at schooI, articularly during physical education classes. These episodes are typically accompanied by dizziness and shortness of breath, and are sometimes accompanied by chest pain. She is found to have a number of abnormal findings on cardiac examination. The first finding to catch the pediatrician's attention is a systolic ejection crescendo-decrescendo murmur that is heard best between the apex and the left sternal border. This murmur is most likely due to which of the following? / / / / /

A. BIood flowing out of the left atrium and into the left ventricle B. BIood flowing out of the left ventricle and into the aorta C. BIood flowing out of the right atrium and into the right ventricle D. BIood flowing out of the right ventricle and into the pulmonary artery E. BIood flowing out of the right ventricle and into the right atrium Explanation - Q: 1.1

Close

The correct answer is B. It can be helpful both clinically, and on examinations, to get to the point that you can analyze the significance of a cardiac murmur. In this case, the fact that the murmur is a systolic ejection murmur indicates that it probably involves blood flow out of one of the ventricles. This allows you to exclude choices A and C, which involve blood flow out of the atria, which would occur during diastole. The fact that the murmur is heard best between the apex and left sternal border indicates that it involves the left ventricle, thereby excluding choices D and E, which involve the right ventricle, and would be heard best along the right sternal border. The murmur described involves the left ventricular outflow tract into the aorta.

After hearing the murmur, the physician asks the patient to perform a Valsalva maneuver. This maneuver causes the intensity of the systolic ejection murmur to diminish. Which of the following describes the Valsalva maneuver? / A. Forced expiratory effort against a closed airway / B. Maintenance of the larynx briefly at the highest position in the neck by

voluntary muscular contraction / C. Mouth closed and nose held followed by swallowing / D. Pressure applied to cricoid cartilage / E. Thrusting a fist inward between the navel and costal margin

Explanation - Q: 1.2

Close

The correct answer is A. The Valsalva maneuver is a forced expiratory effort against a closed airway. This increases the intrathoracic pressure, and consequently increases the preload into the left side of the heart. This, in turn, tends to diminish the pressure gradient across the left ventricular outflow tract, and consequently diminishes the intensity of the ejection murmur. Choice B refers to the Mendelsohn maneuver, which is used as a therapeutic technique for swallowing disorders. Choice C refers to the Toynbee maneuver, which is used to open the Eustachian tube. Choice D refers to Selleck's maneuver, which is used to prevent regurgitation during tracheal intubation. Choice E refers to the Heimlich maneuver, which is used to expel a food bolus from the airways.

Another murmur heard in this patient is a holosystolic murmur heard best at the apex and in the left axilla. This would be most likely to be caused by which of the following? / / / / /

A. Combined mitral and tricuspid stenosis B. Mitral regurgitation only C. Mitral stenosis only D. Tricuspid regurgitation only E. Tricuspid stenosis only Explanation - Q: 1.3

Close

The correct answer is B. The holosystolic murmur suggests that regurgitation is occurring from a ventricle to an atrium during ventricular contraction. This lets you exclude choices A, C, and E. The locations at which the murmur is heard best favor the mitral valve over the tricuspid valve (choice D), which would have been heard best at the lower left sternal border. This murmur is due to systolic anterior motion of the mitral valve and is related to a significant left ventricular outflow gradient.

Other findings that the physician notes include a double apical impulse, a double carotid arterial pulse, and a jugular venous pulse that has a prominent "a" wave. The "a" wave change is most immediately related to which of the following? / / / / /

A. Diminished left ventricular compliance B. Diminished right ventricular compliance C. Increased compliance of both the right and left ventricles D. Increased left ventricular compliance E. Increased right ventricular compliance Explanation - Q: 1.4

Close

The correct answer is B. The jugular venous pulse is best appreciated when the patient is supine, with head slightly raised. The normal jugular venous pulse has three recognizable upward waves, "a", "c", and "v". The "a" wave is the first of these waves and is due to transmitted right atrial pressure to the jugular veins during right atrial systole. It occurs just prior to LV ejection (which can be recognized by simultaneous palpation of the carotid pulse upstroke). An increased "a" wave can be due to decreased compliance of the right ventricle, which inhibits right ventricular filling during atrial systole. The state of the left ventricle's compliance is not directly measured with the "a" wave, although in cases like this patient's, septal hypertrophy may affect the compliance of both the right and left ventricles.

Two-dimensional echocardiography reveals systolic anterior motion of the anterior mitral valve (which appears morphologically normaI) and asymmetric septal hypertrophy. Which of the following is the most likely diagnosis? / / / / /

A. Bacterial endocarditis B. Congenital heart disease C. Congestive heart failure D. Hypertrophic cardiomyopathy E. Rheumatic heart disease Explanation - Q: 1.5

Close

The correct answer is D. This patient has hypertrophic cardiomyopathy, which has been formerly known by a variety of names, including idiopathic hypertrophic subaortic stenosis, and asymmetric septal hypertrophy. These old terms have been replaced by the slightly broader term hypertrophic cardiomyopathy, since it was discovered that the hypertrophy is not always limited to the left ventricular outflow tract and septum. This patient's presentation has many typical features, and the murmurs heard are often

present in these patients. Other presenting symptoms can include sudden cardiac death, angina, palpitations, and congestive heart failure. Hypertrophic cardiomyopathy may be either obstructive, as in this case, or nonobstructive. In obstructive cases, anterior mitral valve movement toward the septum during systole contributes to the pathophysiology by increasing the level of obstruction over that present in diastole. This, in turn, causes several of the cardiac findings noted earlier in this case, including the double apical impulse and double carotid pulse (due to a "doubled" forceful ejection of blood during systole before and after the obstruction increases by mitral valve movement), the mitral regurgitation murmur, and the high flow systolic ejection murmur in the left ventricular outlet. Bacterial endocarditis (choice A) and rheumatic heart disease (choice E) are primarily valvular diseases, and marked valvular abnormalities would be seen on echocardiography. Congenital heart disease of various types (choice B) would be obvious on echocardiography, and features such as atrial or ventricular septal defects, abnormally formed valves, or aberrant major blood vessel connections would be easily apparent. Congestive heart failure (choice C) can complicate many conditions, including hypertrophic cardiomyopathy, but none of the findings illustrated in the case specifically suggest that it is present. In approximately 50% of cases, this patient's disease has a genetic basis, which is of which of the following types? / / / / /

A. Autosomal dominant B. Autosomal recessive C. Numerical abnormality of autosomes D. Numerical abnormality of sex chromosomes E. X-Iinked recessive Explanation - Q: 1.6

Close

The correct answer is A. The genetics of hypertrophic cardiomyopathy is typically autosomal dominant with variable penetrance and variable expressivity. It is thought that many of the isolated cases may be new autosomal dominant mutations. The autosomal dominant pattern of inheritance is important, because it leads to the clinical suggestion that immediate relatives of the patient be screened for hypertrophic cardiomyopathy. This is particularly important now that medical (e.g., antiarrhythmia medication) and surgical (left ventricular myomectomy, pacemaker implantation, implantable cardioverter defibrillator) methods of

treating this disease exist. The other choices are distracters. Which of the following is the leading cause of sudden cardiac death in both preadolescent and adolescent children? / / / / /

A. Bacterial endocarditis B. Congenital heart disease C. Congestive heart failure D. Hypertrophic cardiomyopathy E. Rheumatic heart disease Explanation - Q: 1.7

Close

The correct answer is D. Sudden death is death that occurs over a few minute period and was medically unexpected. Hypertrophic cardiomyopathy has a mortality rate of 4% per year overall, with a higher mortality rate of sudden death in children of 6% per year. This makes this condition the leading cause of sudden cardiac death in both preadolescent and adolescent children. The death is often unexpected and is often associated with sports or vigorous exertion. Sudden death can even be the presenting manifestation of hypertrophic cardiomyopathy. It is thought that in most of these cases, ventricular fibrillation is the arrhythmia that causes the death. Patients with hypertrophic cardiomyopathy that does not cause ventricular outflow obstruction may also die of arrhythmias. The other conditions listed in the choices may also cause death, but it is usually not sudden.

Vignette 2 of 3

A 32-year-old woman consults a physician because she has been having episodes of a sensation of motion accompanied by a ringing in her left ear. She can tell when she is going to have an episode, roughly an hour before it happens because she develops a sense of fullness in the ear. Once the symptoms of motion become severe, she also experiences nausea, vomiting, diarrhea, pallor, and sweating. The physician asks the patient to sit at the end of the examining table. She is then asked to lay back quickly while the physician supports her head and carefully hyperextends her neck. The physician then turns the head from one shoulder to the other and then back again. When the maneuver described is performed, the patient initially feels all right, but after a few seconds delay, she experiences symptoms characteristics of her vertigo spells. Which of the following is the most likely diagnosis?

/ / / / /

A. Aspirin toxicity B. Herpes zoster encephalitis C. Meniere disease D. Migraine headache E. Transient ischemic attack Explanation - Q: 2.1

Close

The correct answer is C. This patient most likely has Meniere disease. This condition, which is also sometimes known as endolymphatic hydrops, typically presents as illustrated in this case. The underlying problem is an increase in volume and pressure of the fluid in both the vestibular and cochlear apparatus. This dysfunction, in turn, causes nerve damage to both the cochlear and vestibular branches of cranial nerve VIII, producing tinnitus with hearing loss and vertigo, respectively. The other diseases listed in the question choices are in the differential diagnosis of Meniere disease. Aspirin toxicity (choice A) can cause tinnitus and hearing loss, but does not usually cause vertigo. Herpes zoster encephalitis (choice B), migraine (choice D), and transient ischemic attacks (choice E) can all cause vertigo but do not usually cause tinnitus.

Which of the following agents would be most likely to relieve her vertiginous symptoms? / / / / /

A. Apomorphine B. Meclizine C. Ondansetron D. Pilocarpine E. Promethazine Explanation - Q: 2.2 The correct answer is B. Meclizine (Antivert), an H1 antihistamine, has antivertiginous properties and can be useful to patients with this disorder. Other medications that can also help with the vertigo are scopolamine, dimenhydrinate (Dramamine), and diazepam (Valium). Apomorphine (choice A) is a dopamine agonist, which is used to induce emesis in some poisoning cases.

Close

Ondansetron (choice C) is a 5-HT3 antagonist, which is used as an antiemetic. It would not be useful for vertigo. Pilocarpine (choice D) is a muscarinic agonist and would likely exacerbate this patient's vertigo. Promethazine (choice E) is a dopamine antagonist (phenothiazine), which is used as an antiemetic. It would not be useful for vertigo.

Which of the following is the most common morbidity associated with this disease? / / / / /

A. Chronic otitis externa B. Chronic otitis media C. Permanent hearing loss D. Secondary development of a craniopharyngioma E. Secondary development of an acoustic neuroma Explanation - Q: 2.3

Close

The correct answer is C. Patients with Meniere disease often have transient hearing loss in the affected ear(s) during the vertigo episodes. With repeated episodes, the vertigo often improves, but the hearing loss can become permanent. Patients with Meniere disease can usually be treated with medication to reduce their vertigo (antihistamines such as meclizine, anticholinergics such as scopolamine, benzodiazepines such as diazepam); some cases with incapacitating vertigo are treated with surgery or ototoxic drugs (such as high dose streptomycin). Meniere disease is unrelated to middle or outer ear infection (choices A and B). Meniere disease is apparently unrelated to tumor (choices D and E, although occasionally tumors involving the middle ear can mimic Meniere disease).

Which of the following structures contains endolymph? / A. Helicotrema / B. Scala media / C. Scala tympani

/ D. Scala vestibuli / E. Spiral ganglion

Explanation - Q: 2.4

Close

The correct answer is B. The cochlea of the inner ear is a coiled structure filled with endolymph and perilymph. Within it are three longitudinal compartments. The membranous labyrinth within the cochlea is called the scala media, or cochlear duct. This compartment contains the hair receptors and is filled with endolymph that is secreted by the stria vascularis. The composition of endolymph resembles intracellular fluid (K+> Na+). The two perilymph-filled compartments, the scala tympani (choice C) and the scala vestibuli (choice D) surround the cochlear duct. This space is continuous with subarachnoid space, and therefore the perilymph closely resembles cerebrospinal fluid. The helicotrema (choice A) is the apical connection between the scala tympani and scala vestibuli. The spiral ganglion (choice E) is located in the modiolus (the cochlear part of the body labyrinth), and contains the cell bodies of the bipolar cells that comprise the cochlear division of the eighth nerve.

Vignette 3 of 3

A fifty-year-old man presents to his primary care physician complaining that he often feels as if the room is spinning when he gets up from a recumbent position or turns his head. He has not lost consciousness and has had no chest pain. He has no cardiac history and a recent treadmill test showed no abnormalities. On examination, the sensation can be produced by rapidly turning the head. It can be reproduced many times, but it eventually ceases. Nystagmus is elicited. Hearing is normaI. Which of the following is the most likely diagnosis? / / / / /

A. Benign paroxysmal positional vertigo B. Brain stem tumor C. Meniere disease D. Syncope E. Vertebrobasilar TIA

Explanation - Q: 3.1

Close

The correct is answer A. Benign paroxysmal positional vertigo (BPPV) is vertigo that is precipitated by head position. It can be precipitated by trauma, but often no precipitating factor is identified. It generally abates after weeks to months. On exam, patients display nystagmus and the symptoms can be reproduced by head movement. In addition, the symptoms show latency, fatigability and habituation. Brain stem tumor (choice B) and vertebrobasilar TIA (choice E) are causes of central causes of vertigo, but would often be associated with other neurologic findings. Meniere disease (choice C) displays the classic triad of unilateral tinnitus, unilateral deafness, and paroxysmal vertigo. Since the patient has normal hearing, this is unlikely to be Meniere syndrome, although hearing loss may not manifest until later stages of the disease. Syncope (choice D) is defined as transient loss of consciousness of cardiovascular origin. Thus since the patient has a lack of other cardiovascular symptoms and has not lost consciousness, this is not syncope.

Which of the following is the most likely mechanism for this patient's symptoms? / / / / /

A. Aberrant stimulation of hair cells B. Hair cell death in the semicircular canals C. Insufficient cardiac output D. Insufficient cerebral perfusion E. Massive infection in the middle ear Explanation - Q: 3.2

Close

The correct answer is A. In benign paroxysmal positional vertigo, calcium carbonate crystals called otoliths, which are usually fixed in a gelatinous otolithic membrane, float freely in the endolymph. Usually, movement is sensed by the movement of this heavy membrane as it stimulates hair-like projections on sensory hair cells fixed to the membrane in the inner ear. In BPPV, the loose crystals aberrantly stimulate the hair cells with certain head movements. Thus the sensation of vertigo is produced. Hair cell death (choice B) does not cause vertigo. Insufficient cardiac output (choice C) and cerebral blood flow (choice D)

lead to syncope, not vertigo. Massive infection in the middle ear (choice E) will produce ear pain and fever. Only occasionally will it produce vestibular symptoms, as the vestibular apparatus is in the inner ear.

Which of the following agents will help alleviate this patient's symptoms? / / / / /

A. Amitriptyline B. CIonidine C. Digitalis D. FIumazenil E. Scopolamine Explanation - Q: 3.3

Close

The correct answer is E. Scopolamine alleviates the symptoms of all types of vertigo. It acts by antagonizing acetylcholine receptors. Amitriptyline (choice A) is a tricyclic antidepressant and would not be used to treat vertigo. Clonidine (choice B) is an alpha 2 agonist used to treat hypertension. It would not be used for vertigo. Digitalis (choice C) is used as an inotrope or an antiarrhythmic, but the patient's symptoms are not referable to cardiac disease. Thus this would not be an appropriate agent for this patient. Flumazenil (choice D) is a benzodiazepine antagonist used in benzodiazepine overdose.

Which of the following structures is responsible for detecting rotational acceleration? / / / / /

A. Cochlea B. Fovea centralis C. Saccule D. Semicircular canals E. Utricle Explanation - Q: 3.4

Close

The correct answer is D. The semicircular canals are responsible for detecting rotational acceleration. As the head is rotated, fluid currents in the endolymph cause the deflection of the cupula and stimulate its hair cells. The magnitude of the fluid currents is proportional to the rotational acceleration, and the direction is related to the direction of the acceleration. The cochlea (choice A) is the inner ear structure in which the mechanical energy of sound waves is transduced into neural impulses. The fovea centralis (choice B) is the location on the retina responsible for most color vision. The saccule (choice C) and utricle (choice E) are inner ear structures that detect linear acceleration.

Which of the following nerves transmits impulses originating from the vestibular apparatus? / / / / /

A. Cranial nerve ll B. Cranial nerve VII C. Cranial nerve VIII D. Cranial nerve XI E. Cranial nerve XII Explanation - Q: 3.5

Close

The correct answer is C. The vestibular portion of cranial nerve VIII (vestibulocochlear nerve) carries impulses from the vestibular apparatus to the vestibular ganglion, which then conveys the impulses to the vestibular nuclei of the brainstem. Cranial nerve II (optic nerve, choice A) carries impulses generated by the retina. Cranial nerve VII (facial nerve, choice B) carries efferent motor impulses to the face and afferent sensory impulses to the anterior two thirds of the tongue. Cranial nerve XI (accessory nerve, choice C) carries motor innervation to the trapezius and sternocleidomastoid muscles. Cranial nerve XII (hypoglossal nerve, choice E) carries motor innervation to

the tongue and strap muscles.

Vignette 1 of 4

A 35-year-old woman presents with complaints of weakness and spasticity in her right leg. She also admits to difficulties with balance and malaise. The patient states that about 6 months ago, while taking a shower one morning, she noticed that she could not discriminate between hot and cold water on her right leg. She began to feel very tired, and had some difficulties passing urine. Several months ago she had noticed that the vision deteriorated in her right eye, although it recovered over a period of two months. On examination, abnormalities of the left eye and optic disc are present. Neurologic examination is significant for weakness, Iack of coordination, abnormally brisk reflexes in both legs, and difficulty walking were registered. Which of the following is the pathological hallmark of this disease? / / / / /

A. Ballooned cells and argyrophilic bodies B. Concentrated, noninflammatory demyelination within the central pons C. Demyelinated plaques with perivascular infiltration D. GIiosis, foamy macrophages, Cowdry-A bodies E. Noncaseating epithelioid granulomas Explanation - Q: 1.1

Close

The correct answer is C. Multiple sclerosis (MS) is an inflammatory, demyelinating disease of the central nervous system (CNS). MS is primarily a disease of individuals living in temperate climates. The pathological hallmark of MS is the demyelinated plaque, which consists of a welldemarcated hypocellular area with loss of myelin, relative preservations of axons, and the formation of astrocytic scars (chronic plaques). Plaques vary from 1 mm to several centimeters in diameter. The lesions have a propensity for the optic nerves, periventricular white matter, brain stem, cerebellum, and spinal white matter. The visual problems the patient had were most likely due to optic neuritis, in association with demyelination of the optic nerve. Active plaques show perivascular infiltration of lymphocytes (predominantly T cells) and macrophages, microscopically. These lesions appear as indurated areas in gross preparations, thus the term "sclerosis." At sites of inflammation, the blood-brain barrier is disrupted, but the vessel wall is preserved. In some inflammatory lesions, dissolution of the multilayered myelin sheath with reconstitution as a lattice-like network of myelin membrane fragments may be seen (vesicular demyelination). It seems that interleukin-12, a potent inflammatory substance is expressed at high levels in early lesions. Cerebrospinal fluid abnormalities include abnormally increased levels of newly synthesized IgG, oligoclonal banding, and mononuclear pleocytosis.

The argyrophilic intraneuronal inclusions (Pick bodies) and ballooned neurons (choice A) are characteristic pathological findings in Pick disease. Central pontine myelinolysis is characterized by concentrated, symmetric, noninflammatory demyelination of the central parts of the pons (choice B). Some patients may have extrapontine lesions in the thalamus, basal ganglia, and cerebellum. Subacute sclerosing encephalitis (SLE) develops following the reactivation of latent measles virus. The gross pathology includes cortical atrophy, loss of white matter, and ventricular enlargement. Intense gliosis, foamy macrophages in the white matter, and Cowdry-A inclusions (choice D) consisting of a central eosinophilic nuclear mass surrounded by a halo (also common in herpes simplex encephalitis) are found in patients with SLE. Noncaseating epithelioid granulomas (choice E) are associated with sarcoidosis.

Which of the major histocompatibility antigens is associated with an increased risk of this disease? / / / / /

A. A3 B. B27 C. DR2 D. DR3 E. DR4 Explanation - Q: 1.2

Close

The correct answer is C. Histocompatibility antigens are glycoproteins that are expressed at the surface of all vertebrate cells. Human HLA antigens are present on the surface of all nucleated cells. Major histocompatibility antigens are major antigens responsible for tissue recognition, and stimulate the acute, rapid type of graft rejection. Each person has unique HLA antigens. The presence of some HLA antigens have been correlated with the certain autoimmune diseases. Susceptibility to MS is linked to the HLA-DR2 haplotype. The severity of iron overload in hemochromatosis is determined predominantly by genetic factors, namely the expression of HLA-A3 (choice A) region of the major histocompatibility complex. 85% of patients with Reiter syndrome will have HLA-B27 (choice B) antigen

present on leukocytes. There is a strong association between type 1 diabetes mellitus and specific HLA-D phenotypes, particularly HLA-DR3 (choice D). Approximately 35% of Caucasians are HLA-DR4-positive (choice E). HLADR4-positive patients are genetically predisposed to autoimmune diseases such as rheumatoid arthritis and type 1 diabetes.

Demyelination is the major feature of this disease. Which of the following cells forms myelin in the central nervous system? / / / / /

A. Astrocytes B. Ependymal cells C. Microglia D. OIigodendrocytes E. Schwann cells Explanation - Q: 1.3

Close

The correct answer is D. The myelin sheath is formed by oligodendrocytes in CNS and by Schwann cells (choice E) in the peripheral nervous system (PNS). The gaps formed between myelin sheath cells along the axons are called nodes of Ranvier. Since the lipid structure of myelin serves as a good insulator, the myelin sheaths increase the rate of propagation and efficacy of transmission of the impulse along the axon. The electrical impulse jumps from one node to the next at the rate as fast as 120 m/s. This rapid type of conduction is called saltatory conduction. Demyelination can occur early in life as consequence of congenital metabolic disorders. Demyelination later in life can be repaired with glia, which explains the frequent exacerbations and remissions in MS. It is believed that in MS, multiple mechanisms of immune injury of myelin coexist: cytokine-mediated injury of oligodendrocytes and myelin, digestion of surface myelin antigens by macrophages, complementmediated injury, and direct injury by CD4+ and CD8+ T cells. This type of injury causes the loss of saltatory conduction in nerve fibers. The exposed axon is susceptible to further injury, resulting in irreversible axonal damage. Tumor necrosis factor-alpha and IFN gamma may contribute directly to the damage by injuring oligodendrocytes or the myelin membrane. Glial cells do not carry action potentials, but they have many important functions. There are several types of glia cells: astrocytes (choice A), which transport nutrients to neurons, hold neurons in place, digest parts of dead neurons, and regulate the composition of extracellular space, and microglia (choice C), which possess phagocytic function, clean up debris, and protect

the brain from microorganisms. Ependymal cells (choice B) form the epithelial lining of the ventricles. Which of the following interferon (IFN) classes would most likely be used in this patient? / / / / /

A. IFN alpha-2a B. IFN alpha-2b C. IFN alpha-n3 D. IFN beta E. IFN gamma Explanation - Q: 1.4

Close

The correct answer is D. Interferons (IFNs) are produced by the body in response to viral infections and tumors, and for the regulation of immunity. They belong to cytokine family of proteins. Three types of natural IFNs are produced: alpha, beta, and gamma. Alpha and beta types are classified as Class I since they bind to IFN cell surface receptors type 1. IFN gamma is class II, since it binds only to type 2 receptors. The interferons reduce the proliferation of T cells and the production of tumor necrosis factor-alpha, decrease antigen presentation, alter cytokine production to favor ones by type 2 helper T cells, increase the secretion of interleukin-10, and reduce the passage of immune cells across the blood-brain barrier. Beta interferons are made by fibroblasts. IFN beta-1a, a non-glycosylated molecule, with the same amino acid composition as natural human IFN beta, reduces the frequency of relapses of MS by 40%. It has an additional benefit of slowing and preventing the development of MS-related brain atrophy. IFN beta-1b, a glycosylated molecule, reduces the relapse rate and slows disease progression. Another agent, glatiramer acetate, is a mixture of synthetic polypeptides. It may promote proliferation of Th2 cytokines, and competes with myelin basic protein for presentation on Class II MHC molecules, thereby inhibiting antigen-specific T cell activation. The drug also alters the function of macrophages and induces antigen-specific suppressor T cells. Mitoxantrone is a FDA-approved agent that reduces neurologic disability and the frequency of relapses in patients with progressive MS. Treatment of MS may also include azathioprine, methotrexate, cyclophosphamide, methylprednisolone, and 2-chlorodeoxyadenosine, depending on indications. Leukocytes produce the interferon alpha group. Interferons alpha-2a (choice A) and alpha-2b are made by recombinant DNA technology using genetically engineered E. coli. Interferon alpha-2a is an effective antineoplastic,

immunomodulator and antiviral agent. It is used to treat hairy cell leukemia, AIDS-related Kaposi sarcoma, chronic myelogenic leukemia, and chronic hepatitis C. IFN alpha-2b (choice B) is a highly purified protein of 165 AA, and is indicated in AIDS-related Kaposi sarcoma, hairy cell leukemia, and chronic hepatitis C. It is also used in condylomata acuminata and malignant melanoma. IFN alpha-n3 (choice C) is made from human leukocytes induced by murine virus. It is only used in the treatment of condylomata acuminata. IFN gamma (choice E) is type II, since it binds only type 2 receptors. Natural killer cells (NK) and activated T cells produce this class of interferons. Type II interferons have modest antiviral activity, are more potent immunomodulators, and have a different range of immune functions, including macrophage activation.

Vignette 2 of 4

A 55-year-old African-American woman presents to her physician complaining of acute pain and redness in her right eye, with nausea and vomiting. She reports seeing halos around lights. On examination, Iacrimation, Iid edema, conjunctival injection, a steamy cornea, and a fixed mid-dilated pupil are seen. She has markedly elevated intraocular pressure (IOP), and a visual acuity of 20/200 in her right eye. She has no family history of eye diseases. She denies using cocaine or taking paroxetine. Which of the following is the most likely diagnosis? / / / / /

A. Acute closed-angle glaucoma B. Corneal laceration C. Ocular hypertension without glaucoma D. Open angle glaucoma E. Retinal detachment Explanation - Q: 2.1

Close

The correct answer is A. This patient has acute closed-angle glaucoma, as evidenced by the pain and redness of her eye, the steamy cornea, her description of halos around lights, and her markedly elevated intraocular

pressure. The fixed mid-dilated pupil indicates ischemia to the iris. Unlike open angle glaucoma, closed-angle glaucoma (also called narrow-angle or angle-closure glaucoma) is a medical emergency, and requires immediate diagnosis and treatment to prevent permanent visual impairment. The anterior and posterior chambers of the eye are filled with aqueous humor, which flows from the posterior chamber, through the pupil, into the anterior chamber, into the trabecular network, and then exits the eye via the canal of Schlemm. Individuals with a shallow or narrow anterior chamber or thickened lens may be predisposed to this type of glaucoma because their iris is in close opposition to the chamber angle and cornea (narrow angle). If the aqueous humor is produced at a greater rate than it can be drained, intraocular pressure rises. Any stimulus that causes pupillary dilation (e.g., anticholinergic medications, sympathomimetics, emotional upset, dim lighting) can precipitate an attack. The cornea may look hazy because of edema, but corneal laceration (choice B) is produced by trauma. There is no history of trauma in this patient and the symptoms exhibited by this patient strongly suggest acute closed-angle glaucoma Intraocular hypertension (choice C) is a condition in which IOP is higher than normal (greater than 21 mm Hg), but there is no damage to the optic nerve or visual loss. About 90% of people with elevated IOP never develop glaucoma. There is no associated pain or visual symptoms associated with this. Known as the "sneak thief of sight," open-angle glaucoma (choice D) has no early warning signs or symptoms, and accounts for 90% of all glaucoma cases. It is a slow progressive condition, usually affecting both eyes, in which there is an increase in resistance to the outflow of aqueous humor. This resistance is associated with a normal rate of production of the fluid and normal anterior chamber angle. Elevated IOP is the most common finding and age-related changes in the trabecular region are the most likely explanation of this disease. Blurred vision, appearance of colored halos around lights, problems with dark adaptation, and visual difficulties that new eye prescriptions do not help are characteristic for this disease. Risk factors for developing glaucoma are family history, age (increased risk with age), medical conditions (diabetes, myopia, hypertension), and an abnormally high IOP. African-Americans are three to four times more likely than Caucasians to develop glaucoma. Also, Asians and Eskimos are more likely to develop glaucoma than Caucasians. Retinal detachment (choice E) represents separation of the inner layers of retina from the retinal pigment epithelium, most commonly caused by a break in the retina. Symptoms include the sensation of flashing lights (photopsia),

decreased visual acuity, wavy distortion of objects, floaters, and visual field defects.

Which of the following medications might have precipitated this patient's condition? / / / / /

A. AIpha2-selective agonist B. Beta adrenergic antagonist C. Muscarinic antagonist D. Parasympathomimetic E. Prostaglandin analog Explanation - Q: 2.2

Close

The correct answer is C. Anticholinergics (parasympathetic antagonists) are a group of drugs that include alkaloids derived from the plant Atropa belladonna (atropine, hyoscyamine, scopolamine) and related, synthetic products (propantheline, dicyclomine, homatropine and eucatropine). They compete reversibly with acetylcholine at the level of muscarinic receptors. These agents block the cholinergic response of the ciliary muscle and the sphincter pupillae muscle, causing cycloplegia and mydriasis. Pupillary dilation causes an increased pupillary block that can precipitate acute closed-angle glaucoma in patients with occludable angles. Therefore, anticholinergics and other drugs with substantial anticholinergic activity should not be used in cases of suspected or overt glaucoma without previous gonioscopic angle evaluation. Alpha2 agonists (choice A) are indicated as second- or third-line agents in the therapy of glaucoma. Both alpha2-selective adrenergic agonists (apraclonidine and brimonidine) lower IOP effectively. But, because of high rates of allergic reactions and tachyphylaxis, apraclonidine is most useful for the short-term pressure control. Brimonidine (0.2%) is more lipophilic and alpha2-selective than apraclonidine, and is used for chronic treatment of glaucoma. Topical beta blockers (timolol, betaxolol, levobunol, choice B) can be used together with carbonic anhydrase inhibitors in the treatment of the acute attack. Systemic side effects of these drugs include bronchospasm, shortness of breath, depression, fatigue, confusion, impotence, bradycardia, but these effects are minimized by using topical preparations of these agents. Parasympathomimetics (choice D) such as pilocarpine help open the

drainage canals and increase the flow of aqueous humor out of the eye. In addition to miosis-induced increase in the outflow, contraction of the sphincter pupillae ms. causes increased tension on the scleral spur, thereby further reducing outflow resistance. Latanoprost is a selective prostanoid F2 alpha receptor agonist (choice E) that reduces the intraocular pressure by increasing the outflow of aqueous humor. Studies showed that the mechanism of this topical medication (0.005%) includes an increased uveoscleral outflow. A single daily application will effectively lower IOP for 24 hours. It has very few systemic side effects. The major ocular side effect is increased pigmentation of the iris and eyelid, and increased pigmentation and growth of the eyelashes.

Which of the following drugs should be included in this patient's treatment? / / / / /

A. Atropine B. Hydrochlorothiazide C. Pilocarpine D. Pirenzepine E. Propranolol Explanation - Q: 2.3

Close

The correct answer is C. The goal in the treatment of acute narrow angle glaucoma is to lower the intraocular pressure. This can be done in three different ways: blocking aqueous humor production (beta-blockers, carbonic anhydrase inhibitors, alpha-2 agonists), reducing vitreous volume (hyperosmotic agents), and increasing outflow of aqueous humor (parasympathomimetics, prostaglandin analogs). Pilocarpine is a muscarinic agonist, which causes pupillary miosis and constriction of the ciliary muscle. This leads to an increase in aqueous humor outflow via the canal of Schlemm, thus decreasing the intraocular pressure. Because this is an emergency situation, the goal is to lower intraocular pressure as quickly as possible. Therefore, several of these drugs may be used in combination. Atropine (choice A) is a cholinergic antagonist, which will cause mydriasis; this can precipitate or worsen acute narrow angle glaucoma. Hydrochlorothiazide (choice B) is a thiazide diuretic. It is not used in glaucoma. Carbonic anhydrase inhibitors are used instead. Pirenzepine (choice D) is a muscarinic (M1) antagonist. M3 receptors are present on the ciliary muscle and pupillary sphincter muscle, and this drug should theoretically have no effect. If it had any ability to block M1 receptors,

it would make this patient's situation worse. Propranolol (choice E) is a non-selective beta blocker, which is not used in glaucoma. However, beta blockers such as timolol and betaxolol are used.

Additional treatment might consist of which of the following agents? / / / / /

A. Furosemide B. Latanoprost C. Neostigmine D. Succinylcholine E. Terbutaline Explanation - Q: 2.4

Close

The correct answer is B. Latanoprost is a prostaglandin F2α analog that acts by increasing the outflow of aqueous humor; it is used topically. This drug and other similar drugs (unoprostone, travoprost, bimatoprost) permanently increases the brown pigment of the iris and may increase eyelash growth. Furosemide (choice A), a loop diuretic, is not used for glaucoma; however, carbonic anhydrase inhibitors such as dorzolamide (topical) and acetazolamide (parenteral) could be used for this purpose. These agents decrease aqueous humor production. Hyperosmotic agents such as mannitol and isosorbide are also used and act by creating an osmotic gradient between the ocular fluids and plasma. Neostigmine (choice C) is an acetylcholinesterase inhibitor. In theory, it could be helpful, however, it is positively charged and would not be able to gain access to the eye. It is useful in myasthenia gravis, which is a disease of the periphery, because it does not cross the blood-brain barrier. Physostigmine would be a better choice. Succinylcholine (choice D) is a depolarizing skeletal muscle relaxant and has no role in glaucoma therapy. Terbutaline (choice E) is a beta-2 agonist. This could worsen the situation by increasing aqueous humor production. Instead, beta antagonists such as timolol or betaxolol could be used to decrease aqueous humor production.

The patient is also given timolol eye drops and immediately becomes short of breath. This patient likely has which of the following?

/ / / / /

A. Asthma B. Bronchitis C. Cardiac arrhythmia D. Myocardial infarction E. Pericarditis Explanation - Q: 2.5

Close

The correct answer is A. Even topical β blockers can lead to fatal bronchoconstriction in asthmatics. β blockers are contraindicated in asthmatics. Bronchitis (choice B) is an inflammation of the bronchi from exogenous irritation or infection. There would have to be coexisting asthma to cause frank bronchoconstriction with the administration of a β blocker. β blockers are used as cardiac protectants in patients with arrhythmia (choice C) and myocardial infarction (choice D). β blockers should be avoided however in patients with a history of heart block Pericarditis (choice E) is inflammation of the pericardium from infection, inflammatory conditions or trauma. There is no real connection between pericarditis and β blocker administration.

Vignette 3 of 4

A 69-year-old man presents to the emergency department with painless vision loss of his right eye. He describes the visual loss as a gradual progression from blurry to total blackout over the past two hours. He has no history of prior visual problems. Past medical history is significant for a myocardial infarction three years ago. The patient takes 81 mg of aspirin daily. Vital signs are normaI. Physical examination reveals 20/20 vision of the left eye but no vision in the right eye.Extraocular muscles are intact. The neurologic examination is normaI. The cardiac examination reveals an S4 heart sound. The most likely cause of this unilateral blindness is which of the following? / A. Carotid artery dissection / B. Cavernous sinus thrombosis

/ C. Posterior cerebral artery occlusion / D. Posterior inferior cerebellar artery occlusion / E. Retinal artery occlusion

Explanation - Q: 3.1

Close

The correct answer is E. This patient has a sudden onset of complete, unilateral, painless loss of vision with intact extraocular muscle movement. This is consistent with occlusion of the retinal artery. The retina derives its blood supply from the internal carotid artery via the ophthalmic artery, from which branch the central retinal artery and ciliary arteries. The central retinal artery supplies the inner layers of retina, while ciliary arteries supply blood to the choriocapillaris and the outer retinal layers. Occlusion of small retinal arterioles produces "cotton wool" spots while total occlusion of the ophthalmic artery leads to total ischemia and blindness. This usually occurs secondary to atherosclerotic disease. The key here is recognizing that the loss of vision is an isolated finding with no other neurologic symptoms. Other deficits would be consistent with a stroke of a larger vascular territory such as the internal carotid artery. Carotid artery dissection (choice A) is less likely than retinal artery occlusion because there would likely be multiple associated neurologic findings in the case of dissection, as the internal carotid artery supplies a large portion of the anterior circulation of the brain. The most likely symptoms among a large cadre of possibilities is face and arm weakness contralateral to the dissection. Cavernous sinus thrombosis (choice B) would lead to facial pain and possible cranial nerve deficits. The optic nerve (cranial nerve II) extends anteriorly from the optic chiasm through the superior orbital fissure. Posterior cerebral artery occlusion (choice C) is less likely than retinal artery occlusion because there would likely be multiple associated neurologic findings. Also, the visual field loss resulting from a unilateral posterior cerebral artery occlusion would more likely be a homonymous hemianopia, it would not cause complete blindness in one eye. Other changes that may occur from posterior cerebral artery stroke are complex findings, such as the inability of recognize faces (prosopagnosia), failure to connect visual perception with conscious thought, color vision loss, and difficulty with reading (dyslexia or alexia). Memory problems and motor impairment are also common associated findings. Posterior inferior cerebellar artery occlusion (choice D) produces cerebellar symptoms such as ipsilateral clumsiness. The posterior inferior cerebellar artery supplies the cerebellar vermis and the ipsilateral cerebellar tonsil.

At the molecular leveI, which of the following components is essential for the first step of the visual cascade? / / / / /

A. 11-cis-retinal B. AII-cis-retinal C. AII-trans-retinal D. Meta-rhodopsin ll E. Rhodopsin Explanation - Q: 3.2

Close

The correct answer is A. The visual cascade: 11-cis-retinal + opsin -> rhodopsin + light -> meta-rhodopsin II. Meta-rhodopsin II dissociates after light exposure to form all-trans-retinal. 11-cis retinal and opsin are essential first steps in generating the photochemical visual cascade. All-cis-retinal (choice B) is not a part of the visual cascade. All-trans-retinal (choice C), meta-rhodopsin II (choice D), rhodopsin (choice E) is a later part of the visual cascade: 11-cis-retinal + opsin -> rhodopsin + light -> meta-rhodopsin II. Meta-rhodopsin II dissociates after light exposure to form all-trans-retinal. 11-cis retinal and opsin are essential first steps in generating the photochemical visual cascade.

This patient's condition affects both the rods and cones of the retina. Which of the following characteristics distinguishes rods from cones? / / / / /

A. Rods are not located within the fovea B. Rods detect color C. Rods have an on-center, off-surround organization of receptor fields D. Rods provide high visual acuity E. Rods recover sensitivity more rapidly after exposure to light Explanation - Q: 3.3

Close

The correct answer is A. The fovea only contains cones and no rods are present. The cones are responsible for color vision (choice B). There are three types of cone receptors: red, green, and blue.

Cones are smaller than rods and are organized into on-center, off-surround receptor fields (choice C). Cones are responsible for providing high visual acuity (choice D). Cones are more sensitive to light and recover their sensitivity about 5 times faster than rods after exposure to light (choice E). However, rods respond to lower light levels better than do cones.

Vignette 4 of 4

A 72-year-old woman with a history of hypertension and obesity presents to her primary care physician with a 3 day history of difficulty reading. She states that she is able to read the words on the left side of a page without difficulty, but has to turn her head to read the words on the right side of the page. Physical examination reveals that she has a right homonymous hemianopia. Otherwise she is without any other neurologic deficit. She is referred for a formal visual field evaluation, which confirms that she has a right homonymous hemianopia with macular sparing. A CT scan confirms that she had an ischemic stroke. Occlusion of which of the following blood vessels would most likely result in this patient's presentation? / / / / /

A. Left middle cerebral artery B. Left posterior cerebral artery C. Left posterior communicating artery D. Right anterior cerebral artery E. Right ophthalmic artery Explanation - Q: 4.1

Close

The correct answer is B. The posterior cerebral arteries are formed from the bifurcation of the basilar artery. They supply the midbrain, the posterior thalamus, and the occipital lobe, including the visual cortex. An occlusion of the posterior cerebral artery results in a contralateral homonymous hemianopia with macular sparing since the occipital pole subserving the macula can receive collateral blood supply from the middle cerebral artery. The left middle cerebral artery (choice A) arises from the internal carotid artery and makes up part of the anterior circle of Willis. The middle cerebral artery supplies the caudate nucleus, the putamen, the globus pallidus, and

the internal capsule through the lateral lenticulostriate arteries. As the middle cerebral artery travels distally it supplies the lateral convexity of the hemisphere and underlying insula. A left middle cerebral artery occlusion would result in a stroke that would result in a stroke presenting with a global aphasia and a contralateral hemiparesis. The left posterior communicating artery (choice C) arises from the carotid siphon and travels posteriorly to join the posterior cerebral artery. It supplies the optic chiasm and tract, the hypothalamus, the subthalamus, and a portion of the anterior thalamus. Any visual field deficit caused by a posterior communicating artery occlusion would most likely be accompanied by a contralateral motor deficit as well. The right anterior cerebral artery (choice D) arises from the internal carotid artery and makes up part of the anterior circle of Willis. The proximal anterior cerebral artery supplies the superior surface of the optic nerve, the optic chiasm, the anterior hypothalamus, the anterior commissure, and fornix via the medial lenticulostriate arteries. Just distal to the anterior communicating artery the medial distal striate artery (the recurrent artery of Heubner) arises to supply the head of the caudate, the anterior limb of the internal capsule, the anterior putamen and globus pallidus, and the inferior frontal lobe. The distal anterior cerebral artery is the usual location of occlusions by thrombotic emboli. This segment supplies the anterior 2/3 of the medial cortex. Occlusion of the distal right anterior cerebral artery would result in weakness in the left lower extremity because the leg is represented medially on the motor homunculus. The right ophthalmic artery (choice E) is the first intradural branch off the internal carotid artery. It travels with the optic nerve to supply the globe by way of the central retinal artery and the ciliary arteries. Occlusion of the right ophthalmic artery would result in a right monocular field deficit. If this is transient, it is called amaurosis fugax and is usually the result of a small fibrin embolus.

What is the most likely location of this patient's lesion? / / / / /

A. Banks of the calcarine sulcus B. Banks of the central sulcus C. Cingulate gyrus D. Middle frontal gyrus E. Superior temporal gyrus Explanation - Q: 4.2

Close

The correct answer is A. The primary visual cortex or striate cortex (area 17) receives its input from the geniculocalcarine fibers of the lateral geniculate body. The primary visual cortex is located in the banks of the calcarine fissure. The cuneus is the upper bank and the lingual gyrus is the lower bank. A lesion in this area results in contralateral hemianopia with macular sparing. The banks of the central sulcus (choice B) contain the precentral and postcentral gyrus. The precentral gyrus contains the primary motor cortex, while the postcentral gyrus contains the primary somatosensory cortex. The cingulate gyrus (choice C) contains a portion of the limbic association cortex and is involved in emotions. The middle frontal gyrus (choice D) contains the frontal eye fields (area 8). The frontal eye fields control the initiation of saccades. Stimulation of the frontal eye fields causes contralateral eye deviation. The superior temporal gyrus (choice E) contains Wernicke's speech area in the dominant hemisphere, as well as containing the primary and secondary auditory cortex.

If the patient instead presented with an inability of the left eye to perceive images in the left half of her visual field and of her right eye to perceive images in the right side of her right visual field, what would be the most likely location of her lesion?

/ / / / / / /

A. Lateral geniculate body B. Optic chiasm C. Optic nerve D. Optic tract E. Parietal geniculocalcarine radiations F. Primary visual cortex G. Temporal geniculocalcarine radiations Explanation - Q: 4.3

Close

The correct answer is B. This patient has bitemporal hemianopia, which occurs with lesions of the middle part of the optic chiasm. The decussating fibers of the optic nerves, which carry the visual impulses from the nasal halves of the retina, are blocked. Conditions such as pituitary adenoma and craniopharyngioma can cause this. (These patients would also present with

other symptoms, e.g., endocrine symptoms). A lateral geniculate body (choice A) would produce a contralateral homonymous hemianopia. An optic nerve lesion (choice C) can result in ipsilateral blindness. An optic tract lesion (choice D) could produce a contralateral homonymous hemianopia. A lesion of the parietal geniculocalcarine radiations (choice E), which project from the lateral geniculate body to the lower bank of the calcarine sulcus (lingual gyrus), could result in a contralateral lower quadrantanopia. A lesion of the primary visual cortex (choice F) could result in a contralateral hemianopia with macular sparing. A lesion of the temporal geniculocalcarine radiations (choice G), which project from the lateral geniculate body to the upper bank of the calcarine sulcus (lingual gyrus), could result in a contralateral upper quadrantanopia ("pie in the sky").

Vignette 1 of 5

A 5 year-old girl and her mother are referred for evaluation of bed-wetting. The mother states that her daughter has never fully gained control of her bladder. It took her an extended period of time to grow out of diapers and the girl has finally stopped having "accidents" during the day. However, she continues to wet her bed at night. The patient has developed normally in all other aspects and will be starting kindergarten in three months. Both her mother and father are extremely frustrated and have been losing sleep, as the girl wakes up at night 4 to 5 times per week. Physical examination is unremarkable. Which of the following will most likely be the result of further investigation? / / / / /

A. Large capacity bladder B. Learning disability C. NormaI "work-up" D. Urinary tract infection E. Urological cancer Explanation - Q: 1.1

Close

The correct answer is C. Enuresis was originally denoted as incontinence of urine, but now, it is a term restricted to those children who are over the age of 3 who wet the bed. Most children have achieved normal bladder control by that time, girls earlier than boys. At age six years, 10% have enuresis. It is important to rule out other possible causes of enuresis, but usually, investigation will reveal only normal results. It is believed that up to 50% of cases are caused by delayed maturation of the nervous system or intrinsic myoneurogenic bladder dysfunction. 30% are of psychologic origin and 20% are secondary to more obvious organic disease. The bladder capacity (choice A) of bed wetters and non-bed wetters is the same. The difference is that the non-bed wetters are able to hold their urine in their bladders throughout the night. When compared with normal children, the majority of children with bedwetting problems score just as well on tests of intelligence and school achievement. These children are no more likely to suffer from permanent neurological abnormalities, poor school achievement, low IQs, or severe learning disability (choice B). It is true that bed-wetting children are more likely to get urinary tract infections (choice D) if their personal hygiene is poor; it is not true that such infections are the main cause of bed-wetting. Malignancy (choice E) of the urinary tract is extremely rare in children and is

unlikely to be the cause of this child's problem. The condition persists, and the patient is placed on desmopressin acetate (DDAVP). This medication works via which of the following mechanisms? / / / / /

A. Decrease detrusor muscle tone B. Improves alertness of the patient during the sleep cycle C. Increase external sphincter contraction D. Increase salt reabsorption in the collecting tubules E. Increase water permeability and reabsorption in collecting tubules Explanation - Q: 1.2

Close

The correct answer is E. Desmopressin, or DDAVP, can be used intranasally in patients suffering from enuresis. This medication is a longacting synthetic analogue of vasopressin. The medication acts at V2 receptors, which are found on renal tubule cells, and mediates an antidiuretic effect by increasing water permeability and water reabsorption in the collecting tubules, leading to decreased urine output. It is effective in 70% of patients with increased nocturnal urine output. Detrusor muscle tone (choice A) is under control of the parasympathetic system. DDAVP does not affect this system. Some sympathomimetic drugs may cause enough wakefulness so that the child perceives the urge to void (choice B). Dextroamphetamine sulfate is one such medication. The external sphincter (choice C) is under somatic nervous control. This medication does not contribute to this system. Desmopressin works by increasing water permeability, not salt permeability in the collecting tubules (choice D).

Which of the following would be appropriate management of a child with this condition? / / / / /

A. Do not let the child spend the night away from home B. Have the child practice holding her urine during the day C. Let the child sleep through the night D. Punish the child for each accident E. Put night-Iights in the bedroom, bathroom, and hallway Explanation - Q: 1.3

Close

The correct answer is E. By putting lights in the bedroom, hallway, and bathroom, the child may feel safer getting up and walking to the bathroom during the night. You should make it as easy as possible for your child to spend the night away from home (choice A). If he or she goes to a slumber party, hide a disposable diaper in the bottom of the sleeping bag, and have the child slip it on under his or her pajamas. Having the child hold their urine during the day (choice B) i.e., create urinary retention, adds nothing to the effectiveness of treatment. Many of these children can already hold urine for a full school day, and often avoid going to the school bathroom because they are too shy or are uncomfortable going in the school toilet. This technique may be dangerous as it can result in reflux of urine towards the kidneys. The child must learn to wake up when he or she needs to urinate (choice C). If the child gets up for any reason, have him or her get out of bed and try to use the toilet. Never punish a child for wetting the bed (choice D). He or she cannot control it. Punishment can sometimes make bed-wetting worse by increasing the sense of shame and embarrassment the child feels.

If a biopsy of this patient's bladder were obtained, which histological cell type would be found in the superficial layer? / / / / /

A. Multiple nephrons B. Pseudostratified mucous membrane C. Smooth muscle fibers D. Squamous cells E. Transitional cells Explanation - Q: 1.4

Close

The correct answer is E. The mucosa of the bladder is composed of transitional epithelium. Beneath this is a well-developed submucosal layer formed largely of connective tissue and elastic tissue. The detrusor muscle lies external to the submucosa. It is made up of a mixture of smooth muscle fibers (choice C) arranged at random in a longitudinal, circular, and spiral manner.

The nephron (choice A) is the functioning unit of the kidney. It is composed of the glomerulus, which projects into Bowman's capsule, which, in turn, is continuous with the epithelium of the proximal convoluted tubule. This tubule then continues as the loop of Henle, distal convoluted tubule, and then the collecting duct. Together, these make up a nephron. Nephrons are not found in normal bladder epithelium. The seminal vesicles are made up of a mucous membrane that is pseudostratified (choice B). Squamous epithelium can be found lining that portion of the urethra that is contained within the glans penis. If squamous cells (choice D) are found in the bladder, it is considered an abnormality.

Vignette 2 of 5

A healthy 52 year-old man is evaluated during a routine physical examination. On rectal examination, he is found to have a nodule on the left apex of his prostate. His prostate specific antigen (PSA) is 6.7 ng/dL. He denies any nocturia, dysuria, hematuria, or urinary hesitancy. The patient undergoes a transrectal ultrasound with biopsy of the prostate gland. The pathology report is consistent with cancer of the prostate. Which of the following is the most common histological variant of prostate cancer? / / / / /

A. Adenocarcinoma B. Sarcoma C. Seminoma D. Squamous cell carcinoma E. Transitional cell carcinoma Explanation - Q: 2.1

Close

The correct answer is A. Prostate cancers are adenocarcinomas that arise from the prostatic acinar cells. In eighty-five percent of the cases, the adenocarcinoma is multifocal. Prostate cancers are graded according to their dysplasia, utilizing the Gleason scoring system. Sarcoma of the prostate (choice B) accounts for 0.1% of all malignant

prostatic tumors. Rhabdomyosarcoma is the most frequent mesenchymal tumor within the prostate, and is seen almost exclusively in childhood. Seminomas (choice C) are not seen within the prostate. Seminoma is a germ cell tumor of the testis. Primary squamous carcinoma of the prostate (choice D) is rare and is associated with a poor survival. More commonly, squamous differentiation occurs in the primary and metastatic deposits of adenocarcinomas after estrogen therapy. Transitional cell carcinoma (choice E) is the primary malignancy of the urothelial tract. While there is potential for the prostatic urethra to develop transitional cell carcinoma, it is unlikely that this will be diagnosed via a transrectal biopsy

Which of the following cell type(s) secretes prostate specific antigen (PSA)? / / / / /

A. Benign prostate epithelial cells only B. Malignant prostate epithelial cells only C. Benign and malignant prostate epithelial cells D. Prostate stromal cells E. Seminal vesicle epithelium Explanation - Q: 2.2

Close

The correct answer is C. Prostate-specific antigen (PSA), a glycoprotein with a MW of 34,000, is a biological marker used in the detection of prostate cancer. It is secreted from prostatic epithelial cells and is responsible for aiding in the liquefaction of semen. This marker, which is evaluated with a blood test, can be produced by both benign and malignant epithelial cells. The prostate stromal cells (choice D) do not produce PSA. The seminal vesicles (choice E) are responsible for providing a majority of semen volume, however they do not produce PSA.

This patient's cancer is most likely located in which of the following areas of the prostate? / A. Apex of the prostate / B. Base of the prostate / C. Central zone

/ D. Peripheral zone / E. Transitional zone

Explanation - Q: 2.3

Close

The correct answer is D. Approximately 60-70% of prostate cancers occur in the peripheral zone of the prostate. Fortunately, this is the portion of the prostate that is palpated during digital rectal examination. The prostate is often described as having an apex, middle area, and base. The apex is the most distal aspect and the area of narrowest diameter. The base of the prostate, the widest area, is the portion of the prostate adjacent to the bladder. On rectal examination, the apex is encountered initially and the base is the area located in further into the anal canal. These are descriptive anatomic terms and the frequency of prostate cancer is not differentiated between the two (choice A and B). 5-10% of prostate cancers occur in the central zone (choice C). This is the portion of the prostate that surrounds the ejaculatory ducts. 10-20% of prostate carcinomas develop in the transitional zone (choice E) of the prostate. This area is located just outside the urethra at the verumontanum and is composed of periurethral glands, which are responsible for benign prostatic hyperplasia (BPH).

The physician recommends surgery for removal of the prostate and associated lymph nodes. During this procedure, the obturator nerve is at risk of transection. If this occurs, the patient may suffer which of the following? / / / / /

A. Loss of erectile function B. Loss of sensation over the lateral aspect of the thigh C. Loss of sensation over the medial aspect of the thigh D. Loss of sensation over the posterior aspect of the thigh E. Weakness with lower extremity extension Explanation - Q: 2.4

Close

The correct answer is C. The obturator nerve, originating from L2, L3, and L4 of the lumbar plexus, is the nerve of the adductor muscles of the thigh. This nerve descends through the psoas major muscle, leaving its medial border at the brim of the pelvis. It pierces the psoas fascia, crosses the sacroiliac joint, passes lateral to the internal iliac vessels and ureter, and enters the pelvis minor. It leaves the pelvis through the obturator foramen

and enters the thigh. The obturator nerve supplies motor innervation to the obturator externus, which is responsible for laterally rotating the thigh. This nerve also provides a small cutaneous branch, which is responsible for sensation to the medial aspect of the thigh. The neural innervation responsible for erectile function (choice A) is complex, but the obturator nerve plays no role. The neurovascular bundles that contribute to erections travel on the lateral aspect of the prostate and may also be injured during prostatectomy. These bundles arise from the pelvic plexus, which is formed by parasympathetic visceral efferent preganglionic fibers that arise from the sacral center (S2-S4) and sympathetic fibers from the thoracolumbar center (T11-L2). The lateral femoral cutaneous nerve, originating from L2 and L3 of the lumbar plexus, is responsible for sensation to the lateral aspect of the thigh (choice B). The majority of the posterior aspect of the thigh (choice D) is innervated by branches from the posterior femoral cutaneous nerves, which originate from the sacral plexus. Extension of the thigh (choice E) is performed by many muscles. The anterior thigh muscles play a major role in this function. The muscles are innervated by the femoral nerve (originating from L2, L3, and L4).

The physician is concerned that this patient has metastatic prostate cancer. What is the imaging modality that is most Iikely to confirm his diagnosis of metastatic prostate cancer? / / / / /

A. Bone scan B. CT scan of the abdomen C. CT scan of the chest D. CT scan of the pelvis E. MRI of the brain Explanation - Q: 2.5 The correct answer is A. The most frequent site of metastatic prostate carcinoma is the lymphatic system. This is followed in frequency by bone metastases. Therefore, the imaging modality that is used to evaluate for metastatic prostate cancer must evaluate the skeletal system. Bone scintigraphy, a radionuclide bone scan (choice A), provides the most sensitive method for detecting bony metastases.

Close

Abdominal and pelvic imaging (choice B and D) may evaluate for local disease extension but are not routinely useful because of low sensitivity. These are very poor tests for evaluating for enlarged lymph nodes. Lung metastases may occur, however, however almost all cases have bone involvement as well. Therefore, bone scan will provide more information that a CT scan of the chest (choice C). It is rare for prostate cancer to metastasize to the brain, therefore an MRI of the brain (choice E) has little value.

If this patient ultimately develops metastatic prostate cancer, he may be treated with an agent such as leuprolide. Such medications work by which of the following mechanisms? / / / / /

A. BIocking cytochrome P450 B. BIocking testicular androgen production C. BIocking testosterone at the target tissue level D. Directly blocking the adrenal gland synthesis of androgens E. Indirectly suppressing LH and FSH secretion Explanation - Q: 2.6

Close

The correct answer is E. These medications, known as GnRH agonists (e.g., leuprolide, nafarelin, goserelin, buserelin), work by activating the GnRH receptors in the pituitary gland. This creates an initial increase in the release of LH and FSH, causing an initial increase in testosterone ("flare phenomenon"). With time, there is down-regulation of the pulsatile release of LH and FSH, and ultimately, suppression of LH and FSH due to inhibition of the hypothalamic-pituitary axis. This leads to a decrease in testosterone levels. Castrate levels of testosterone are reached within 30 days. Cytochrome P-450 (choice A) plays a role in the adrenal and gonadal synthesis of androgens. This can be inhibited by ketoconazole, which can produce castrate levels of testosterone within 8 hours. Directly inhibiting testicular androgen production (choice B) can be accomplished via bilateral orchiectomy. The activity of testosterone can be blocked at the tissue level by inhibiting the binding of testosterone and dihydrotestosterone to the intracellular androgen receptors (choice C). Medications that work via this mechanism are classified as nonsteroidal antiandrogens, e.g., flutamide and bicalutimide.

The only way to directly block the production of androgens from the adrenal gland (choice D) is bilateral adrenalectomy. This is no longer necessary because pharmacological androgen deprivation is now available.

Vignette 3 of 5

A 61-year-old white man complains of painless hematuria. The patient states that he noticed some blood in his urine approximately two months prior to presentation. However, the bleeding cleared on its own and he did not seek evaluation at that time. But, over the last 72 hours, the hematuria returned and has persisted, and now he is concerned. The patient denies dysuria, urethral discharge, fevers, chills, sensation of incomplete emptying, or weight loss. His history is significant for diet-controlled diabetes and hypertension. He has never had surgery. His only medication is amlodipine for hypertension. He admits to a 75 pack-year history of cigarette use. On examination, his vital signs are normaI. There is no palpable lymphadenopathy or abdominal masses. His urological examination, including digital rectal examination is normaI. Which of the following is the most likely diagnosis? / / / / /

A. Carcinoma of the bladder B. Carcinoma of the penis C. Carcinoma of the prostate D. Renal stone E. Urinary tract infection Explanation - Q: 3.1

Close

The correct answer is A. This patient has painless hematuria. The possibility of bladder cancer is very high, especially with his extensive smoking history. The possibility of cancer within the urinary tract must be considered in any patient who complains of painless, gross hematuria. The timing of the hematuria is very important. Initial hematuria usually arises from the urethra, typically the prostatic urethra. Total hematuria is most common and indicates that the bleeding is most likely coming from the bladder or upper urinary tracts. Terminal hematuria occurs at the end of micturition and is usually secondary to pathology in the area of the bladder neck. Carcinoma of the penis (choice B) is usually of squamous cell origin. The most common locations for this cancer are the glans penis and prepuce. It almost always arises in uncircumcised men. In order for penile cancer to cause hematuria it must obstruct the urethral meatus or invade the urethra.

Either scenario should be easily seen on physical examination. Hematuria is not a common complaint in patients with prostate cancer (choice C). Carcinoma of the prostate usually originates in the peripheral zone of the prostate, which does not communicate with the urethra. If a patient with prostate cancer complains of hematuria, they most likely have locally advanced or metastatic disease. Either of these should be appreciated on physical examination. Patients with renal stones (choice D) may have gross hematuria. However, they should have other symptoms as well: i.e., fever, flank pain, abdominal pain, dysuria, or urinary frequency. On examination, they may have costovertebral angle tenderness, and possibly an elevated body temperature if the stone obstructs the drainage of urine. If a patient with a urinary tract infection (choice E) has gross hematuria, they should have other complaints such as fever, chills, urinary frequency, or dysuria. It will be important to obtain a urine culture from this patient, but the history of tobacco use must make the physician concerned about a bladder tumor. Which of the following risk factors is associated with this patient's diagnosis? / / / / /

A. AIcohol abuse B. EIevated serum calcium levels C. Ingestion of artificial sweeteners D. Rubber tire factory workers E. Unprotected sexual intercourse Explanation - Q: 3.2

Close

The correct answer is D. Aromatic amines and azo dyes are known carcinogens. Beta-naphthylamine is one of the agents associated with an increased risk of cancer of the urothelium. People who work with dyes, petroleum, leather, printing, and the rubber industry are exposed to this carcinogen. After absorption, it is believed to be hydroxylated into an active form and then is detoxified by conjugation with glucuronic acid. When excreted in the urine, the nontoxic conjugate is split by the urinary enzyme glucuronidase to release the electrophilic reactant again, thus inducing bladder cancer. Alcohol (choice A) has been implicated in many different disease processes, but it has not been shown to be a risk factor for bladder cancer. Cigarette smoking is the agent most often implicated as a cause of bladder cancer. Elevated serum calcium (choice B) levels play a role in the formation of

renal stones, not bladder cancer. Artificial sweeteners (choice C) have been proposed as risk factors for bladder cancer, however multiple studies have failed to confirm this association. Unprotected sexual intercourse (choice E) puts a person at risk for a sexually transmitted disease such as gonorrhea. Gonorrhea is the cause of gonococcal urethritis.

During removal of this patient's bladder, the urachus is identified and ligated. Which of the following most appropriately describes the urachus? / / / / /

A. It connects the bladder to the rectum in early fetal life B. It is responsible for development of the renal parenchyma C. It is the remnant of the embryological allantois D. It is the remnant of the embryological cloaca E. It is the remnant of the embryological urogenital sinus Explanation - Q: 3.3

Close

The correct answer is C. Embryologically, the allantois connects the urogenital sinus with the umbilicus. Normally, the allantois is obliterated at birth and is represented by a fibrous cord, the urachus, which extends from the dome of the bladder to the navel. Urachal formation is related to bladder descent. Lack of descent is associated with a patent urachus. If the entire tract remains patent, then urine will constantly drain from the umbilicus. This will be identified in the first few days of life. Very early in fetal development, the blind end of the hindgut, caudal to the point of origin of the allantois, expands to form the cloaca, which is separated from the outside by a thin plate of tissue called the cloacal membrane. During the seventh week of development, the cloaca divides into a ventral and dorsal portion. The ventral aspect is renamed the urogenital sinus and the dorsal portion becomes the rectum (choices D and E). During the period of fetal life prior to division of the cloaca, the urinary and intestinal systems are connected. Although the allantois drains the cloaca, it is not responsible for keeping the two connected. When the cloaca is divided into the urogenital sinus and rectum, the allantois drains the urogenital sinus only (choice A). The renal parenchyma is formed from the metanephros. The metanephros is the final phase of development of the nephric system and originates from

both the intermediate mesoderm and the mesonephric duct. The ureteral bud is an outgrowth of the mesonephric duct, which grows cephalad and acquires a metanephric cap as it moves. During this move, the metanephric cap becomes progressively larger, and rapid internal differentiation takes place. Ultimately, this metanephric tissue will become the renal parenchyma (choice B).

Which of the following infectious agents is associated with another form of this patient's likely condition?

Explanation - Q: 3.4

Close

The correct answer is C. In Egypt, parts of Africa, and the Middle East, Schistosoma haematobium is the cause of bilharzial infection, which is associated with squamous cell cancer of the bladder. This type of bladder cancer is found in 60% of the bladder carcinomas in these parts of the world. In the US, chronic infection, vesical calculi, or chronic catheter use are associated with squamous cell cancer of the bladder. Schistosoma hematobium have a terminal spine and travel through the inferior mesenteric veins to settle in the veins of the pelvic organs, particularly the bladder. They may cause hematuria, dysuria and, an increased risk of bladder cancer. All of the parasites listed in the answer options are classified as trematodes, or flukes. The life cycle of the trematodes include snails as the intermediate host and humans as the definitive hosts. The Clonorchis and the Paragonimus are hermaphrodites and therefore have a second host, fish, and crabs, respectively. Clonorchis sinensis(choice A), also called the Oriental liver fluke, settles in the biliary ducts, where they grow into adults. Symptoms of patients with large worm loads include abdominal pain and weight loss. Paragonimus westermani(choice B) settles in the lung and causes symptoms such as a cough with bloody sputum. Schistosoma japonicum(choice D) has a very small lateral spine on its eggs. They also live in the mesenteric veins and may cause hepatosplenomegaly.

They are found in Japan and China. Schistosomes can be identified by their size and location of the spine on their eggs. Schistosoma mansoni(choice E) with a prominent lateral spine, affects the GI tract by living in the mesenteric veins and may cause massive hepatosplenomegaly.

Which of the following chemotherapeutic agents causes hematuria secondary to hemorrhagic cystitis? / / / / /

A. BIeomycin B. Cyclophosphamide C. Doxorubicin (Adriamycin) D. Hydroxyurea E. Vinblastine Explanation - Q: 3.5

Close

The correct answer is B. Cyclophosphamide is broken down to the metabolite acrolein. It is this metabolite that is responsible for the hemorrhagic cystitis that these patients develop. Patients given cyclophosphamide for treatment of other cancers, are also at an increased risk for developing bladder cancer. Pretreatment with MESNA (2mercaptoethanesulfonic acid) reduces the risk of hemorrhagic cystitis and bladder cancer. Bleomycin (choice A) is associated with pulmonary fibrosis, and patients should have pulmonary function tests prior to its use. Doxorubicin (choice C) may be cardiotoxic, but does not affect the bladder. Hydroxyurea (choice D) is associated with bone marrow depression. Vinblastine (choice E) is associated with bone marrow suppression and alopecia.

Vignette 4 of 5

A 61-year-old man presents to his family physician complaining that his urinary stream has become weaker over the past few months, and he finds himself waking up three to four times each night just to urinate. It is difficult for him to

generate a urine stream and he often feels that the bladder is still full even after he has completely urinated. He denies any burning or blood with urination and there is no urethral discharge or history of fevers. There has been no change in his bowel habits. He has never experienced anything similar in the past. His vital signs are normaI. Digital rectal examination reveals an enlarged prostate that is smooth, nontender, and without nodules. His prostate specific antigen (PSA) is 2.1 ng/dL (normal 100,000 colony forming units of Klebsiella pneumoniae. Which of the following most accurately describes this bacterium? / / / / /

A. Gram-negative diplococcus, oxidase-positive B. Gram-negative rod, motile C. Gram-negative rod with a capsule D. Gram-positive coccus, catalase-positive E. Gram-positive rod, spore forming Explanation - Q: 5.2

Close

The correct answer is C. Klebsiella pneumoniae is a gram-negative rod that has a capsule that helps to inhibit phagocytosis. This bacteria is also a

glucose and lactose fermenter and is nonmotile. Neisseria meningitidis is a gram-negative diplococcus that is oxidase-positive (choice A). Proteus is a gram-negative rod that is very motile (choice B). Staphylococcus aureus is a gram-positive coccus that is catalase-positive (choice D). Bacillus and Clostridia are both gram-positive rods that produce spores (choice E). Which of the following medications would be contraindicated in this patient? / / / / /

A. Doxazosin B. Finasteride C. Oxybutynin chloride D. Sildenafil E. Tamsulosin Explanation - Q: 5.3

Close

The correct answer is C. Oxybutynin chloride (trade name Ditropan) has a direct antispasmodic effect on smooth muscle and blocks cholinergic receptors that are on the bladder. This effect will inhibit detrusor muscle contraction. This will lead to an increase in bladder capacity, something that is already a problem with this patient. This patient's bladder needs all the help in can get when it comes time to contract. You do not want to give any medication which will inhibit these contractions. Doxazosin and tamsulosin (choices A and E), are alpha-1 blocking agents, that relax smooth muscle fibers of the prostate and bladder neck. Because of their alpha-1 blocking ability, they may also cause hypotension by blocking these receptors in the peripheral vascular system. These medications are not contraindicated in this patient. Tamsulosin is a newer generation of alpha-1 receptor blocking agents. Compared to doxazosin and the other alphablockers used in the treatment of benign prostatic hyperplasia, it is more highly selective for the alpha receptors of the prostate. For this reason its risk of hypotension is lower than the older generation of alpha-blockers. Finasteride (choice B) prevents the enzyme 5-alpha reductase from converting testosterone to dihydrotestosterone within the prostate. Decreased levels of dihydrotestosterone in the prostate help stop the growth

of the prostate due to benign prostatic hyperplasia. Sildenafil (choice D) inhibits the enzyme type 5 cyclic GMP phosphodiesterase, which is primarily located within the cavernosal smooth muscle of the penis. This leads to increased levels of cyclic GMP, causing decreased intracellular calcium, which leads to relaxation of cavernosal smooth muscle and therefore increased blood flow to the penis and an erection. It has no effect on the detrusor muscle. If this patient continues to have high residuals of urinary volume, the pressure in the ureters will increase and exert back-pressure on structures, which in the adult, form from which of the following embryological structures? / / / / /

A. Mesonephric duct B. Mesonephros C. Metanephric blastema D. Nephrotomes E. Urogenital sinus Explanation - Q: 5.4

Close

The correct answer is C. The back-pressure will be exerted on the kidneys. The development of the kidneys is as follows: The intermediate mesoderm on either side of the dorsal body wall gives rise to three successive nephric structures of increasingly advanced design. The first is a small group of transitory, nonfunctional, segmental nephrotomes (choice D), which develop in the cervical region. These structures represent a vestige of the pronephroi or primitive kidneys, which develop in some lower vertebrates. As the cranial nephrotomes regress in the fourth week, they are succeeded by a pair of elongated mesonephros (choice B), which develop in the thoracic and lumbar regions. These mesonephros are functional, having complete although simple nephrons. The mesonephros are drained by a pair of mesonephric (Wolffian) ducts (choice A), which grow caudally to open into the posterior wall of the primitive urogenital sinus. By the fifth week, a pair of ureteric buds sprout from the distal mesonephric ducts and induce the overlying sacral intermediate mesoderm to develop into the metanephric blastema (or definitive kidneys) (choice C). The ureters and the collecting duct system of the kidneys differentiate from the ureteric bud, whereas the nephrons differentiate from the metanephric blastema. The cloacal expansion of the hindgut is partitioned by the urorectal septum into an anterior primitive urogenital sinus (choice E) and a posterior rectum. This structure does not contribute to the definitive kidneys

A 16-year-old girl is brought to a pediatrician because her mother is concerned about her weight loss over the last year. When the pediatrician attempts to interview the girI, she is clearly hostile, denies that she has any medical problem, and states that she is just trying to "Iook good." The girI's height is 175 cm (5'9") and her weight is 41.4 kg (91 pounds). AIthough the girl is unwilling to discuss her weight with the physician, her mother reports that her daughter is convinced that she is too fat. The mother states that the girl is always finding excuses to not eat, or at the most, to eat only a few bites of fruit, even though she is still interested in cooking elaborate meals for her family. On one occasion, she discovered her daughter vomiting after a meaI, and believes that the vomiting was self-induced. Question 1 of 6

Which of the following is the normal range for body mass index (BMI)? / A. Below 16 / B. 16-19 / C. 19-25 / D. 25-35 / E. Over 35 Explanation - Q: 1.1

Close

The correct answer is C. Body mass index is coming into increasing use when evaluating weight problems, because it provides a simple calculation that allows individuals of a variety of heights and weights to be classified as underweight, overweight, or in the normal range. The normal range for BMI is 19 to 25. Individuals with BMI below 16 (choice A) are considered anorexic and those with BMI 16-19 (choice B) are considered underweight. Individuals with BMI 25-35 (choice D) are considered mildly to moderately obese and those with BMI greater than 35 (choice E) are considered severely obese.

Question 2 of 6

This girI's body mass index is which of the following? / A. 10.5 / B. 13.5 / C. 16.5 / D. 21.5 / E. 27.5 Explanation - Q: 1.2

Close

The correct answer is B. Body mass index was originally a European concept, and is calculated as the body weight in kilograms divided by the square of the height in meters. In this country, we need to convert from pounds to kilograms and from feet and inches to meters in order to perform the calculation (a calculator helps). This girl's body weight is 91 pounds, which is divided by the conversion factor 2.2, yielding 41.4 kilograms. Her height is 5 x 12 + 9 = 69 inches, which is divided by the conversion factor 39.4 to get the height in meters, in this case 1.75 meters. Now calculate the BMI as 41.4 kilograms/ (1.75 meters x 1.75 meters) = 13.5. This girl has a BMI in the anorexic range.

Question 3 of 6

Which of the following is the most likely diagnosis? / A. Anorexia nervosa / B. Binge eating disorder / C. Body dysmorphic disorder / D. Bulimia nervosa / E. Normal adolescent developmental issues / F. Orthorexia nervosa Explanation - Q: 1.3

Close

The correct answer is A. It is not uncommon for individuals with severe eating disorders to be very unwilling to seek help, as in this case. This markedly underweight girl, who is still convinced that she is fat and is dieting compulsively, most likely has anorexia nervosa. Anorexia nervosa is best understood as an eating disorder with a large psychological component in which an individual engages in the relentless pursuit of thinness. As many as 50% of anorexics also have binge and purge behavior. On a practical basis, the condition is usually diagnosed when the weight is 85% or less than what is expected for age and height. The psychological component usually includes a perception of being overweight despite evidence to the contrary, a terror of gaining weight, and a denial of the dangers of very low weight. Many theorists cite our society's preoccupation with slenderness as the premier standard for female beauty as a major contributing factor to this disease. While over 90% of patients with anorexia are young women who develop it in their teens to early 20's (peak age 16), the condition can also be seen in children, middle-aged adults, boys, and men. Binge eating disorder (choice B) refers to binge eating not followed by vomiting, and is often accompanied by obesity. Body dysmorphic disorder (choice C) is separated from anorexia nervosa by

psychiatrists because it is based on a preoccupation with "ugliness" rather than "fat"; patients with this condition are likely to make numerous statements like "my nose is terrible." In body dysmorphic disorder, the focus is generally on a part of the body (face, buttocks) rather than the body image as a whole. Bulimia nervosa (choice D) is characterized by episodes of binge eating followed by vomiting or purging with laxatives. There is some overlap with anorexia nervosa, but many patients have normal weight to mild obesity. Normal adolescent developmental issues (choice E) often involve concern for weight, self-image and sexual attractiveness. However, the case steps outside the normal range. The girl's overall weight, the hostility regarding discussing these issues, and the self-induced vomiting are all indications of something outside the range of normal. Orthorexia nervosa (choice F), while not a formally accepted medical classification, is a term in common use in the discussion of eating disorders, and refers to a pathologic fixation on eating "pure" or "superior" food. Question 4 of 6

This girl would most likely also have which of the following menstrual disorders as a complication of her condition? / A. Amenorrhea / B. Dysfunctional uterine bleeding / C. Premenstrual syndrome / D. Primary dysmenorrhea / E. Secondary dysmenorrhea Explanation - Q: 1.4

Close

The correct answer is A. Women with anorexia nervosa do not usually have menstrual periods. Many medical specialists require the presence of amenorrhea before making a diagnosis of anorexia nervosa in women. Depending upon the age at which the anorexia begins, they may either never have periods (primary amenorrhea) or may have periods that cease when they lose weight (secondary amenorrhea). The other conditions listed are not specifically related to anorexia nervosa. Dysfunctional uterine bleeding (choice B) is abnormal uterine bleeding (prolonged flow, irregular flow, or profuse flow) not associated with tumor, inflammation, or pregnancy. Premenstrual syndrome (choice C) refers to the presence of nervousness, irritability, emotional instability, anxiety, headaches, edema, or mastalgia

during the 7 to 10 days before the onset of menstruation. Dysmenorrhea is menstrual pain, and may be either primary (choice D) and unrelated to demonstrable reproductive pathology or secondary (choice E) to other reproductive disease.

Question 5 of 6

The most common cause of death in this condition is arrhythmia, which is usually related to which of the following electrolyte abnormalities? / A. High serum calcium / B. High serum iron / C. Low serum magnesium / D. Low serum potassium / E. Low serum sodium Explanation - Q: 1.5

Close

The correct answer is D. While many fluid and electrolyte abnormalities can be seen in patients with anorexia nervosa, low serum potassium is particularly common, and may cause a fatal arrhythmia. The low serum potassium is usually initially due to very low intake, but may be exacerbated by processes that increase body potassium losses, such as with vomiting, diuretic use, or heavy laxative use. Large numbers of mild cases of anorexia nervosa are never diagnosed and may not develop severe medical problems. Mortality rates among diagnosed cases are surprisingly high, and may be up to 10-20% among the most severe cases without effective intervention. In addition to arrhythmias, anorexics may develop potentially fatal liver or renal disease, teeth problems, esophageal rupture (in bulimics compulsively vomiting), a weakened immune system, anemia, malnutrition, impaired mentation, and salivary duct stones. Severe anorexics are initially treated with hospitalization to prevent death, suicide, and medical crises. It is of particular concern to the medical community serving these patients that our present system of health maintenance organizations often limits the hospitalization of these individuals to less than one month, when four or more months may be required to bring them away from the edge of death. Psychiatric help is usually only effective after a near normal weight has been medically achieved. The other answers listed in the choices are distracters.

Question 6 of 6

Over the next several years, in addition to the treatment of her current condition, this girl should also be monitored for the possible onset of which of the following? / A. Acute stress disorder / B. Agoraphobia / C. Conversion disorder / D. Depression / E. Panic disorder / F. Substance abuse Explanation - Q: 1.6

Close

The correct answer is D. Between 50% to 80% of patients diagnosed with anorexia nervosa also suffer from depression at some point in their lives. Acute stress disorder (choice A) is defined by mental reexperiencing of a traumatic event, avoidance of stimuli that remind the person of the event and a host of secondary symptoms including sleep disruption or excess, withdrawal from the world, impulsive behavior, headaches and somatic complaints. Symptoms must resolve in less than one month. If they do not, we use the diagnosis of posttraumatic stress disorder. There is no association between anorexia and acute stress disorder. Agoraphobia (choice B) suggests a fear of being open, vulnerable and exposed when out and about in the community. There is no link between this anxiety disorder and anorexia. Conversion disorder (choice C) is defined by the loss of physical functioning due to psychological etiology. Generally, the symptom is focused on the loss of the use of a limb or one of the primary senses. There is no correlation of this disorder with anorexia. Panic disorder (choice E) is characterized by the occurrence of three panic attacks in a three-week period. A panic attack is an overwhelming, sudden physiological arousal featuring tachycardia, profuse sweating, hyperventilation, and feelings of doom and dread. There is no association between anorexia and panic disorders. Substance abuse (choice F) is relatively uncommon with anorexic patients. They seem obsessively concerned with which types of substances they are willing to put into their bodies.

A 30-year-old woman has noticed a gradual increase in weight over the past eighteen months. The fat deposition has occurred mostly in the

face, neck, trunk, and abdomen. She has also noticed an increase in black facial hairs that has been accompanied by acne. Physical examination reveals the presence of purple abdominal striae, but generalized hyperpigmentation is absent. A poorly-healed laceration is observed on the right shin. BIood pressure is 155/100 mm Hg. The physician suspects Cushing disease. Question 1 of 5

Which of the following sets of hormone levels would be most consistent with this diagnosis?

Explanation - Q: 2.1

Close

The correct answer is D. Pituitary ACTH-dependent Cushing syndrome (Cushing disease) is the most frequent cause of Cushing syndrome. It is responsible for 70% of reported cases. It is characterized by increased plasma ACTH and increased serum cortisol. Another hallmark of Cushing disease is the absence of the normal diurnal rhythm in plasma cortisol. This rules out choice C, which shows a prominent decrease in PM cortisol levels. Plasma ACTH is typically only modestly increased. Hyperpigmentation is rare. Choice A is incorrect because serum cortisol levels are low. The increased plasma ACTH is consistent with primary adrenal insufficiency (Addison disease). Choice B is incorrect because serum cortisol levels are low. The lower-thannormal plasma ACTH is consistent with secondary (pituitary) adrenal insufficiency. Choice E shows increased serum cortisol and absence of diurnal rhythm, but plasma ACTH is below normal. These laboratory values would be consistent with Cushing syndrome caused by a cortisol-secreting adrenal adenoma

(primary hypercortisolism).

Question 2 of 5

ACTH measured in blood drawn from which of the following vessels would give the best indication of direct anterior pituitary secretion? / A. Inferior petrosal sinus / B. Internal jugular vein / C. Sigmoid sinus / D. Sphenoid sinus / E. Transverse sinus Explanation - Q: 2.2

Close

The correct answer is A. This question may seem difficult, but the key is the fact that the pituitary is located near the cavernous sinuses. Venous drainage from the anterior pituitary, although variable, can be expected to flow, via the nearby cavernous sinuses, posteriorly into the superior and inferior petrosal sinuses before draining, via the sigmoid sinuses, into the internal jugular veins. A skilled invasive radiologist can sample blood bilaterally from the inferior petrosal sinuses. If the ACTH concentration in the sinus blood is greater than twice the concentration in a peripheral vein, Cushing disease is likely. Bilateral sampling of blood from the inferior petrosal sinuses can also locate the tumor to one side or the other. The internal jugular vein (choice B) receives blood from a number of other veins in addition to the petrosal sinuses. Hence, the pituitary ACTH is diluted considerably by the time it reaches the jugular vein. The sigmoid sinus (choice C) receives drainage via the petrosal sinuses before flowing into the internal jugular vein, but also receives flow from the larger transverse sinuses (choice E), which dilutes the ACTH concentration. The transverse sinus receives drainage from a number of cerebral and cerebellar locations, which does not include flow from the anterior pituitary. The sphenoid sinus (choice D) is part of the nasal system. It is an air-filled cavity and does not carry blood.

Question 3 of 5

Which of the following is considered the regulated step in steroid hormone synthesis in the zona fasciculata? / A. Cholesterol to pregnenolone / B. Corticosterone to aldosterone / C. 11-Deoxycortisol to cortisol / D. Pregnenolone to progesterone

/ E. Progesterone to 17- -hydroxyprogesterone

Explanation - Q: 2.3

Close

The correct answer is A. The conversion of cholesterol to pregnenolone is considered the rate-limiting and regulated step in steroid biosynthesis in the zona fasciculata, which secretes the glucocorticoid, cortisol. ACTH binds to a membrane-associated receptor and increases cAMP formation. The subsequent activation of protein kinases and phosphorylation results in increased expression of LDL receptors and activation of cholesterol esterase. The increase in LDL receptors increases the delivery of cholesterol to the zona fasciculata. The activation of cholesterol esterase increases the liberation of intracellular cholesterol from cholesterol esters. ACTH also induces a mitochondrial protein called steroidogenic acute regulatory (StAR) protein. StAR functions to shuttle cholesterol from the outer mitochondrial membrane to the inner mitochondrial membrane, where the cholesterol side chain cleavage enzyme converts it to pregnenolone. Corticosterone is converted to aldosterone (choice B) in the zona glomerulosa, but not in the zona fasciculata. This is also a regulated step. Angiotensin II acts via a membrane-associated receptor to increase the activity of aldosterone synthase (a combination of two enzymes with 18βhydroxylase activity) and, hence, increase aldosterone synthesis and secretion. 11-deoxycortisol is converted to cortisol (choice C), primarily in the zona fasciculata, but also in the zona reticularis. The enzyme responsible is 11βhydroxylase, a mitochondrial enzyme. 11β-hydroxylase is not regulated by ACTH. Pregnenolone is converted to progesterone (choice D) in all steroid secreting tissues. The enzymes responsible are 3β-ol dehydrogenase and D4,5-isomerase. These enzymes are cytoplasmic, and are not regulated by ACTH. The conversion of progesterone to 17α-progesterone (choice E) is catalyzed by 17α-hydroxylase, which is cytoplasmic. This enzyme is not found in the zona glomerulosa, which explains why this layer of the adrenal cortex does not secrete cortisol or sex steroids. 17α-hydroxylase is not regulated by ACTH.

Question 4 of 5

MRI of the pituitary with gadolinium enhancement shows an 8 mm mass which is subsequently removed using microsurgery. Pathologic examination of the mass would most likely show which of the following? / A. Benign adenoma / B. Craniopharyngioma / C. Diffuse hyperplasia of corticotroph cells / D. Malignant adenocarcinoma / E. Metastatic breast carcinoma

Explanation - Q: 2.4

Close

The correct answer is A. A benign pituitary adenoma is present 90% of the time in patients with Cushing disease. The tumor is usually less than 10 mm in diameter. These microadenomas are not encapsulated, but are surrounded by a rim of compressed normal anterior pituitary cells. The microadenomas also typically exhibit bundles of perinuclear microfilaments surrounding the nucleus (Crooke's changes). Electron microscopy shows the presence of secretory granules (200-700 nm in diameter) within the cytoplasm of the adenoma cells. It should be noted that 10% of adults between 20 and 50 years of age have incidental pituitary tumors and no apparent symptoms. Craniopharyngioma (choice B) is the most common tumor that impairs hypothalamic-pituitary function in children and young adults. It may also occur in adults. These intracranial tumors are usually benign, but may become large before being detected. Endocrine abnormalities are common, but may be discovered only after other symptoms related to increased intracranial pressure become manifest. Growth hormone deficiency is the most common endocrine abnormality, but any of the anterior pituitary hormones may be diminished. Diffuse hyperplasia of corticotroph cells (choice C) of the anterior pituitary is rare. When it does occur, it is presumably due to excessive stimulation of the pituitary by corticotropin-releasing hormone (CRH) secreted by a benign hypothalamic gangliocytoma. Malignant adenocarcinoma (choice D) is rare in the anterior pituitary. These tumors are not functional and do not secrete ACTH. Breast tumor cells (choice E) have been known to metastasize to the anterior pituitary. These are space-filling growths that compromise pituitary function. They do not secrete ACTH. Question 5 of 5

Following the pituitary microsurgery, the patient complains of being confused, fatigued and nauseated. She also has hypoglycemia. Cold intolerance is not present. Treatment with which of the following is most likely indicated? / A. Estrogen / B. FIudrocortisone (mineralocorticoid) / C. Human growth hormone / D. Hydrocortisone (glucocorticoid) / E. Thyroxine Explanation - Q: 2.5

Close

The correct answer is D. The prolonged increase in cortisol due to the ACTH-secreting adenoma will have suppressed ACTH secretion from the uninvolved corticotrophs of the anterior pituitary. Following removal of the adenoma, the pituitary will not secrete enough ACTH at first, leading to adrenal insufficiency. The symptoms of the patient are all consistent with low blood levels of cortisol. Glucocorticoid replacement is indicated, and may be needed for up to a year before normal function of the pituitary returns. Depending on the extent of the pituitary microsurgery, a significant number of gonadotrophs might also be removed and lead to hypogonadotropic hypogonadism. In this case, estrogen (choice A) replacement may be necessary. However, the symptoms of this patient are not consistent with estrogen deficiency. Since aldosterone secretion from the zona glomerulosa of the adrenal cortex is primarily regulated by angiotensin II, removal of an ACTH-secreting pituitary adenoma has little effect on its blood level. Hence, mineralocorticoid (choice B) replacement is rarely required. Pituitary microsurgery also has the potential to remove somatotrophs. However, growth hormone deficiency is not likely, given the fact that about 50% of the cells in the anterior pituitary are somatotrophs. Furthermore, somatotrophs are located primarily in the lateral wings of the pituitary, while ACTH-secreting corticotrophs tend to be more medial. Hence, growth hormone (choice C) replacement is less likely to be required than cortisol replacement. Removal of thyrotrophs is also possible with pituitary microsurgery. However, the patients symptoms are not consistent with hypothyroidism, and thyroxine (choice E) replacement is not required.

A 35-year-old man presents to his physician with a 40-pound weight loss over the preceding 4 months. The man has not been dieting, but has been having abdominal pain, nausea, and diarrhea. He has also had a chronic cough. Sputum studies demonstrate Pneumocystis carinii, and esophagogastroduodenoscopy with biopsy demonstrates Candida and Herpes in the esophagus. Question 1 of 9

The pathogen which causes this patient's underlying disease is a member of which of the following groups of viruses? / A. Caliciviridae / B. Coronaviridae / C. FIaviviridae / D. Picornaviridae / E. Retroviridae Explanation - Q: 3.1

Close

The correct answer is E. This man with multiple infections probably has AIDS, in which "wasting" due to infection and weight loss is common. All three of this patient's secondary infections (Pneumocystis pneumonia, Candida esophagitis, and Herpes esophagitis) are considered to be "AIDSdefining illnesses," that indicate that the patient's HIV infection has progressed to the point of producing profound immunosuppression. HIV is a member of the Lentivirus subgroup of retroviruses. This viral family is characterized by prolonged infections, often with a latent period. These viruses are usually inadequately opposed by the host immune response, and often can infect the nervous system. The Retroviridae also include the oncovirus group that includes the human T-cell Leukemia/Lymphotropic virus. The Caliciviridae (choice A) includes the Norwalk virus and the virus that causes Hepatitis E. Coronaviruses (choice B) cause upper respiratory illnesses. Flaviviruses (choice C) include the Hepatitis C virus, yellow fever virus, dengue virus, and several encephalitis viruses. Picornaviruses (choice D) include poliovirus, coxsackieviruses, echoviruses, hepatitis A virus, and rhinoviruses.

Question 2 of 9

Approximately how many people worldwide are infected with the virus that causes this patient's underlying disease? / A. 430,000 / B. 750,000 / C. 1 million / D. 16 million / E. 33 million Explanation - Q: 3.2

Close

The correct answer is E. Currently, about 33 million people worldwide have HIV infection, and there have been 16 million (choice D) deaths. The vast majority of these cases are in developing countries in Africa and Southeast Asia. In the United States, we presently have approximately 1-2 million (choice C) individuals with asymptomatic HIV infection and nearly 750,000 individuals (choice B) with AIDS. Over 430,000 (choice A) of US patients have died of AIDS.

Question 3 of 9

Which of the following is the most frequent means of transmission worldwide for the virus that causes this patient's underlying disease? / A. Accidental needle exposures by health care workers / B. Infected blood products / C. Perinatal transmission / D. Sexual contact / E. Shared needles by drug abusers

Explanation - Q: 3.3

Close

The correct answer is D. Over 70% of HIV infection is acquired via sexual transmission. You may not be aware that worldwide HIV infection is more common in heterosexual men and women than in homosexual men. Most HIV cases in the US were initially in male homosexuals. However, more recently there has been a shift in the demographics and the majority of new cases of HIV infection are in the heterosexual population. Infection of health care workers through accidental needle exposures (choice A) is now rare in the United States because of the widespread use

of universal blood precautions. In this country, contaminated blood products (choice B) only rarely transmit the HIV virus. In contrast, in developing countries, contaminated blood products are an important problem, as many of these countries have high HIV rates and do not have the financial resources to adequately screen the blood. Most children who develop AIDS acquire the infection perinatally (choice C). Most of the individuals in the United States who acquire AIDS via a parenteral route are intravenous drug abusers who share needles (choice E).

Question 4 of 9

This virus has a particularly high affinity for which of the following? / A. CD4 T Iymphocytes and B cells / B. CD4 T Iymphocytes and monocytes. / C. CD8 T Iymphocytes and B cells / D. CD8 T Iymphocytes and CD4 T Iymphocytes / E. CD8 T Iymphocytes and monocytes Explanation - Q: 3.4

Close

The correct answer is B. HIV has a particular tendency to bind to CD4 T lymphocytes (helper T lymphocytes) and monocytes and then become internalized. Most other cells, include CD8 T lymphocytes (suppressor T cells) and B cells, are not particularly susceptible to the HIV virus.

Question 5 of 9

This virus uses which of the following enzymes to produce a DNA copy of itself? / A. Integrase / B. gp120 / C. p24 / D. Protease / E. Reverse transcriptase Explanation - Q: 3.5

Close

The correct answer is E. The viral enzyme reverse transcriptase is used to generate a DNA copy from RNA. This viral DNA is then inserted into the host DNA, which allows the virus to "hide" in the host DNA, and also permits more

copies to be made. Reverse transcriptase is a product of the gene Pol, which also codes for integrase (choice A, which integrates proviral dsDNA into host DNA) and protease (choice D, which cleaves polyprotein). The Env gene codes for gp120 (choice B, a surface protein that binds CD4 on the host cell and is responsible for tropism) and gp41 (a transmembrane protein for cell fusion). The Gag gene codes for group-specific antigens including p24 (choice C, a capsid protein), p7p9 (core nucleocapsid proteins), and p17 (matrix proteins that stabilize the envelope). The HIV genome also has several regulatory genes including LTR (integration and virus gene expression), Tat (transactivator of transcription that functions in upregulation), Rev (upregulates transport of transcripts to cytoplasm), and Nef (decreases major histocompatibility complex type I expression on infected T cells, thereby protecting the infected cells from the immune system).

Question 6 of 9

Which of the following is the typical period of latency between initial infection with this virus and development of clinically evident disease? / A. 1 year / B. 6 years / C. 11 years / D. 16 years / E. 21 years Explanation - Q: 3.6

Close

The correct answer is C. HIV infection remains a mostly fatal illness, since it still eventually causes a profound immunosuppression, which leaves individuals vulnerable to a variety of other infections. While there is some variation from case to case (dependent upon both the size of the initial inoculation and host resistance factors), it typically takes 11 years in untreated cases for HIV infection to progress to clinical AIDS. The introduction of regimens of multiple antiretroviral drugs and prophylaxis against opportunistic pathogens may be changing this pattern, but it is still too early to know to what degree.

Question 7 of 9

The patient is prescribed a non-nucleoside reverse transcriptase inhibitor. Which of the following agents was most likely prescribed? / A. Indinavir / B. Lamivudine / C. Lopinavir / D. Nevirapine / E. Zidovudine Explanation - Q: 3.7

Close

The correct answer is D. As we have developed a wider variety of methods of treating AIDS, the task of remembering the different drugs that can be used has become more complex. Most of the useful drugs with direct activity against the HIV virus block either reverse transcriptase or protease. Of the drugs listed in the choices, only nevirapine is classified as a non-nucleoside reverse transcriptase inhibitor. Indinavir (choice A) and lopinavir (choice C) and are classified as protease inhibitors. Lamivudine (choice B) and zidovudine (choice E) are thymidine analogues that block reverse transcriptase.

/ / / / /

A. Amphotericin B B. CIotrimazole C. Ganciclovir D. Pyrimethamine E. Trimethoprim/sulfamethoxazole Explanation - Q: 3.8

Close

The correct answer is E. The choices of drugs for the therapy of the wide variety of secondary infections AIDS patients develop is also of considerable concern to medical personnel. Pneumocystis pneumonia is most often treated with trimethoprim/sulfamethoxazole (Bactrim), with an alternative drug for patients allergic to trimethoprim/sulfamethoxazole being pentamidine. Early steroid therapy may also be helpful in hospitalized patients with Pneumocystis pneumonia who have a PaO2 less than 70 mm Hg. Amphotericin B (choice A) is a systemic antifungal agent typically used in

AIDS patients to treat cryptococcal meningitis. Clotrimazole (choice B) is used in AIDS patients to treat oral candidiasis. Ganciclovir (choice C) is used in AIDS patients to treat CMV infections. Pyrimethamine (choice D) is used in AIDS patients to treat toxoplasmosis.

Question 9 of 9

Individuals with this patient's underlying disease also have an increased chance of developing which of the following cancers? / A. BIadder cancer / B. Breast cancer / C. Cervical cancer / D. Prostate cancer / E. Testicular cancer Explanation - Q: 3.9

Close

The correct answer is C. Individuals with AIDS also have an increased cancer rate of tumors known to have viral associations, including Kaposi's sarcoma (related to infection with Human Herpes Virus 8); Burkitt's, immunoblastic, and primary CNS lymphomas (related to Ebstein-Barr virus infection); and invasive cervical cancer (related to HPV infection). Cancers of the bladder, breast, prostate, and testes (choices A, B, D, and E) are not related to any known viral infection.

A 3-year-old boy is evaluated by a pediatric endocrinologist because of excessive weight gain. The boy has had problems since birth. He was noted to have neonatal central hypotonia, which has improved somewhat since 1 year of age. He was also noted as a baby to have an unusually long head (dolichocephaly) with almond-shaped eyes and a small mouth. His testes were undescended and he had a small penis. Initially, he had feeding problems with poor weight gain in infancy, but since 1 year of age he has shown excessive weight gain. As a baby, he was lethargic and had a weak cry. His motor milestones and speech development have been delayed.

Question 1 of 5

Which of the following is the most likely diagnosis? / A. Angelman syndrome / B. Down syndrome / C. Prader-Willi syndrome / D. Triple X syndrome / E. Turner syndrome Explanation - Q: 4.1

Close

The correct answer is C. In general, when you encounter a child who has had problems since infancy and who has a distinctive facial appearance, you should consider the possibility that the child has a genetic abnormality (often that involves all or a significant portion of a chromosome). Excessive weight gain is a specific cue to think of Prader-Willi syndrome, although you should be aware that, as illustrated in this case, this may not develop until 1 to 6 years of age. The other conditions listed in the answer options do not have a marked tendency to develop obesity. Angelman syndrome (choice A) is often considered together with PraderWilli syndrome for genetic reasons (see question 3 below), but has a very different clinical presentation with severe mental retardation, gait irregularities, and a tendency to uncontrolled laughter ("happy puppet" syndrome). Down syndrome (choice B) is characterized by mental retardation, abnormal facies (with a flat hypoplastic face with short nose and small low set ears), a tendency to develop lenticular opacities, heart disease, leukemia, and Alzheimer disease. Triple X syndrome (choice D) produces phenotypic females that are usually apparently normal. Turner syndrome (choice E) produces phenotypic females with short stature, hypogonadism, amenorrhea, webbed neck, and a predisposition for coarctation of the aorta.

Question 2 of 5

This patient's probable genetic disorder most likely involves which of the following chromosomes? / A. 15 / B. 18 / C. 21

/ D. X / E. Y

Explanation - Q: 4.2

Close

The correct answer is A. Prader-Willi syndrome is due to a defect in the long arm of chromosome 15 in the 15q11-q13 region. Associate chromosome 18 (choice B) with Edwards syndrome (trisomy 18). Associate chromosome 21 (choice C) with Down syndrome (trisomy 21). Associate the X chromosome (choice D) with Turner syndrome (XO), triple X syndrome (XXX), and Klinefelter syndrome (XXY). Associate the Y chromosome (choice E) with Klinefelter syndrome (XXY).

Question 3 of 5

This patient's probable genetic disorder is often cited as an example of which of the following genetic mechanisms? / A. Genetic imprinting / B. Monosomy / C. Robertsonian translocation / D. Trinucleotide repeats / E. Trisomy Explanation - Q: 4.3

Close

The correct answer is A. Genetic imprinting refers to the finding that certain genes must come from a specific parent in order to function. The molecular basis of the phenomenon appears to be that male and female germ cells apparently add methyl groups to different areas of DNA as a posttranslational modification. These areas of methylation then affect when and how the affected genes are transcribed. (This is part of a broader phenomenon that determines how cells differentiate, based on which genes are "turned on" or turned off.") There has been a great deal of research interest in both Prader-Willi syndrome and Angelman syndrome (see brief description in the discussion of question 1) since it was discovered that these two very different conditions both seemed to be due to a deletion in the same area of chromosome 15. The initial understanding was that if the deletion was in the father's chromosome, then Prader-Willi syndrome occurred, while if it was in the mother's, Angelman syndrome occurred. In practice, for the USMLE, this is probably the level of detail you will need to remember. (What

actually turns out to be the case is that in this region of the genome there are at least two genes that require paternal imprinting and whose absence causes Prader-Willi syndrome, and there is at least one gene requiring maternal imprinting, whose absence causes Angelman syndrome.) With respect to the other choices listed in the question, associate monosomy X (choice B) with Turner syndrome (monosomies involving autosomal chromosomes are fatal in utero). Robertsonian translocations (choice C) are a specialized form of translocation in which the centromeres of two acrocentric chromosomes appear to have fused to make a new chromosome containing both long arms of the original chromosomes; this can which behave like a trisomy, and accounts for some cases of Down syndrome. Trinucleotide repeats (choice D) are highly repeated short segments of DNA that tend to be seen in autosomal dominant diseases that show progression in severity with succeeding generations, such as Fragile X syndrome and Huntington disease. Common trisomies (choice E) include Down syndrome, Edwards syndrome, and Patau syndrome.

Question 4 of 5

If this child goes on to develop morbid obesity, which of the following complications would be most likely to occur? / A. Aortic dissection / B. Crohn disease / C. Diabetes mellitus / D. Pheochromocytoma / E. Zollinger-EIIison syndrome Explanation - Q: 4.4

Close

The correct answer is C. The Prader-Willi patients have markedly increased appetite secondary to hypothalamic dysfunction and a much less than normal caloric need (secondary primarily to low muscle mass, which may be somewhat ameliorated by growth hormone supplementation during childhood and adolescence). To maintain a normal weight, they require a daily caloric intake about half of those of normal individuals, only around 1000 calories per day. If they do not have this severe of a dietary restriction, they will eventually develop morbid obesity, which is often complicated by diabetes mellitus type II. Other complications to which they are particularly vulnerable include osteoporosis often with scoliosis, hypertension, right-sided heart failure, a tendency to pick at skin lesions, dental problems, and a tendency to

have an exaggerated response to sedatives and anesthetics. The other choices listed in the question are distracters and are not associated with Prader-Willi syndrome.

Question 5 of 5

Over the next decade, this child has progression of his disease. When he is seen as a teenager, he is noted to be mildly mentally retarded, and shows multiple behaviors distressing to those around him. He spends a great deal of time ravenous and being pre-occupied with food to the point of breaking windows to get into the locked kitchen and storeroom. He almost always sleeps poorly, and has temper tantrums, violent outbursts, obsessive-compulsive behavior, a tendency to be argumentative, and he steals (most often food or things to get money for food) and lies (most often about food). He is late going through puberty. The physiologic basis of the feeding and sleep problems that dominate this clinical picture is thought to primarily involve dysfunction of which of the following parts of the brain? / A. Cerebellum / B. Cerebral cortex / C. Hypothalamus / D. Medulla / E. Pons Explanation - Q: 4.5

Close

The correct answer is C. The genetics of Prader-Willi syndrome are fascinating, and the disease is often taught to medical students by geneticists. While this can leave a medical student with a great appreciation of how far we have progressed in our ability to sort through the genetic basis of disease, it has the disadvantage that the clinical side of the picture is often comparatively neglected. It tends to be forgotten that Prader-Willi syndrome is a disease with a fairly high incidence of 1:12,000-15,000 (affecting both sexes and all races), which may have a devastating impact on the patient and family. This question illustrates some of the problems that are seen. Severe hypothalamic dysfunction leading to chronic insatiable hunger and chronic sleep deprivation (which may be aggravated by obesity-related sleep apnea) are thought to be a major part of the underlying cause of most of these patients' sometimes striking behavior problems, and are believed to be related to severe dysfunction of the hypothalamus. Added to their unhappiness are the problems that come with the irritability this produces, particularly when it affects interpersonal relationships with those around them. The families of these patients are also usually very unhappy as well, secondary to concern about the patient, constant emotional turmoil, the financial pressures of chronic illness, and often nearly irreconcilable differences among family members about how to fairly allocate financial and

time resources between the affected child and other family members. (Often what evolves is that one parent becomes the affected child's principal advocate and caregiver, while the other parent becomes the advocate and caregiver for the remainder of the family.) The other choices listed in the question are distracters.

Normal Lab Values

Close

* = Included in the Biochemical Profile (SMA-12)

BLOOD, PLASMA, SERUM * Alanine aminotransferase (ALT, GPT at 30°C) Amylase, serum * Aspartate aminotransferase (AST, GOT at 30°C) Bilirubin, serum (adult) Total // Direct * Calcium, serum (Total) Cholesterol, serum Cortisol, serum

REFERENCE RANGE

SI REFERENCE INTERVALS

8-20 U/L

8-20 U/L

25-125 U/L

25-125 U/L

8-20 U/L

8-20 U/L

0.1-1.0 mg/dL // 0.0-0.3 mg/dL 8.4-10.2 mg/dL < 200 mg/dL 0800 h: 5-23 µg/dL // 1600 h: 3-15 µg/dL

2-17 µmol/L // 0-5 µmol/L 2.1-2.8 mmol/L < 5.2 mmol/L 138-635 nmol/L // 82413 nmol/L Fraction of 0800 h: ≤0.50

2000 h: ≤ 50% of 0800 h Creatine kinase, serum (at 30°C) ambulatory

Male: 25-90 U/L

Female: 10-70 U/L * Creatinine, serum 0.6-1.2 mg/dL Electrolytes, serum Sodium 135-145 mEq/L Chloride 95-105 mEq/L * Potassium 3.5-5.0 mEq/L Bicarbonate 22-28 mEq/L Magnesium (mg 2+) 1.5-2.0 mEq/L Estriol (E3) total, serum (in pregnancy) 30-170 ng/mL // 60-280 24-28 wks // 32-36 wks ng/mL 40-220 ng/mL // 80-350 28-32 wks // 36-40 wks ng/mL Ferritin, serum Male: 15-200 ng/mL Female: 12-150 ng/mL 12-150 µg/L Follicle-stimulating hormone, Male: 4-25 mIU/mL serum/plasma Female: -- premenopause 430 mIU/mL -- midcycle peak 10-90 mIU/mL --postmenopause 40-250 mIU/mL Gases, arterial blood (room air) pH 7.35-7.45 PCO2 33-45 mm Hg PO2 75-105 mm Hg Glucose, serum Fasting: 70-110 mg/dL

25-90 U/L 10-70 U/L 53-106 µmol/L 136-145 mmol/L 95-105 mmol/L 3.5-5.0 mmol/L 22-28 mmol/L 1.5-2.0 mmol/L 104-590 // 208-970 nmol/L 140-760 // 280-1210 nmol/L 15-200 µg/L 4-25 U/L 4-30 U/L 10-90 U/L 40-250 U/L [H+] 36-44 nmol/L 4.4-5.9 kPa 10.0-14.0 kPa 3.8-6.1 mmol / L

Growth hormone - arginine stimulation

2-h postprandial: < 120 mg/dL

< 6.6 mmol/L

Fasting: < 5 ng/mL

< 5 µg/L

provocative stimuli: 7 ng/mL > 7 µg/L Immunoglobulins, serum IgA IgE IgG IgM Iron Lactate dehydrogenase, serum Luteinizing hormone, serum/plasma

Osmolality, serum Parathyroid hormone, serum, Nterminal Phosphatase (alkaline), serum (pNPP at 30°C) Phosphorus (inorganic), serum Prolactin, serum (hPRL) Proteins, serum Total (recumbent) Albumin Globulins Thyroid-stimulating hormone, serum or plasma Thyroidal iodine (123 I) uptake Thyroxine (T4), serum Triglycerides, serum Triiodothyronine (T3), serum (RIA) Triiodothyronine (T3) resin uptake Urea nitrogen, serum (BUN) Uric acid, serum

76-390 mg/dL 0-380 IU/mL 650-1500 mg/dL 40-345 mg/dL 50-170 µg/dL 45-90 U/L Male: 6-23 mIU/mL Female: --follicular phase 530 mIU/mL --midcycle 75-150 mIU/mL --postmenopause 30-200 mIU/mL 275-295 mOsmol/kg

0.76-3.90 g/L 0-380 kIU/mL 6.5-15 g/L 0.4-3.45 g/L 9-30 µmol/L 45-90 U/L 6-23 U/L

230-630 pg/mL

230-630 ng/L

20-70 U/L

20-70 U/L

3.0-4.5 mg/dL < 20 ng/mL

1.0-1.5 mmol/L < 20 µg/L

6.0-7.8 g/dL 3.5-5.5 g/dL 2.3-3.5 g/dL

60-78 g/L 35-55 g/L 23-35 g/L

0.5-5.0 µU/mL

0.5-5.0 mU/L

8-30% of administered dose/24 h 5-12 µg/dL 35-160 mg/dL 115-190 ng/dL 25-35% 7-18 mg/dL 3.0-8.2 mg/dL

5-30 U/L 75-150 U/L 30-200 U/L 275-295 mOsmol/kg

0.08-0.30/24 h 64-155 nmol/L 0.4-1.81 mmol/L 1.8-2.9 nmol/L 0.25-0.35 1.2-3.0 mmol urea/L 0.18-0.48 mmol/L

CEREBROSPINAL FLUID Cell count Chloride Gamma globulin Glucose Pressure Proteins, total

0-5 cells/mm3 118-132 mEq/L 3-12% total proteins 40-70 mg/dL 70-180 mm H2O < 40 mg/dL

0-5 x 106/L 118-132 mmol/L 0.03-0.12 2.2-3.9 mmol/L 70-180 mm H2O < 0.40 g/L

HEMATOLOGIC Bleeding time (template)

2-7 minutes

2-7 minutes

Erythrocyte count Erythrocyte sedimentation rate (Westergren) Hematocrit Hemoglobin A1C Hemoglobin, blood Hemoglobin, plasma Leukocyte count and differential Leukocyte count Segmented neutrophils Band forms Eosinophils Basophils Lymphocytes Monocytes Mean corpuscular hemoglobin Mean corpuscular hemoglobin concentration Mean corpuscular volume Partial thromboplastin time (activated) Platelet count Prothrombin time Reticulocyte count Thrombin time Volume Plasma Red cell

SWEAT Chloride URINE Calcium Chloride Creatinine clearance Estriol, total (in pregnancy) 30 wks 35 wks 40 wks 17-Hydroxycorticosteroids

Male: 4.3-5.9 million/mm3 4.3-5.9 x 1012/L 3 Female: 3.5-5.5 million/mm 3.5-5.5 x 1012/L Male: 0-15 mm/h

0-15 mm/h

Female: 0-20 mm/h Male: 41-53% Female: 36-46% ≤ 6% Male: 13.5-17.5 g/dL Female: 12.0-16.0 g/dL 1-4 mg/dL

0-20 mm/h 0.41-0.53 0.36-0.46 ≤ 0.06% 2.09-2.71 mmol/L 1.86-2.48 mmol/L 0.16-0.62 µmol/L

4500-11,000/mm3 54-62% 3-5% 1-3% 0-0.75% 25-33% 3-7% 25.4-34.6 pg/cell

4.5-11.0 x 109/L 0.54-0.62 0.03-0.05 0.01-0.03 0-0.0075 0.25-0.33 0.03-0.07 0.39-0.54 fmol/cell

31-36% Hb/cell

4.81-5.58 mmol Hb/L

.80-100 µm3

80-100 fl

25-40 seconds

25-40 seconds

150,000-400,000/mm3 11-15 seconds 0.5-1.5% of red cells < 2 seconds deviation from control

150-400 x 109/L 11-15 seconds 0.005-0.015 < 2 seconds deviation from control

Male: 25-43 mL/kg Female: 28-45 mL/kg Male: 20-36 mL/kg Female: 19-31 mL/kg

0.025-0.043 L/kg 0.028-0.045 L/kg 0.020-0.036 L/kg 0.019-0.031 L/kg

0-35 mmol/L

0-35 mmol/L

100-300 mg/24 h Varies with intake Male: 97-137 mL/min Female: 88-128 mL/min

2.5-7.5 mmol/24 h Varies with intake

6-18 mg/24 h 9-28 mg/24 h 13-42 mg/24 h Male: 3.0-10.0 mg/24 h Female: 2.0-8.0 mg/24 h

21-62 µmol/24 h 31-97 µmol/24 h 45-146 µmol/24 h 8.2-27.6 µmol/24 h 5.5-22.0 µmol/24 h

17-Ketosteroids, total Osmolality Oxalate Potassium Proteins, total Sodium Uric acid

Male: 8-20 mg/24 h Female: 6-15 mg/24 h 50-1400 mOsmol/kg 8-40 µg/mL Varies with diet < 150 mg/24 h Varies with diet Varies with diet

28-70 µmol/24 h 21-52 µmol/24 h 90-445 µmol/L Varies with diet < 0.15 g/24 h Varies with diet Varies with diet

View more...

Comments

Copyright ©2017 KUPDF Inc.
SUPPORT KUPDF